Sie sind auf Seite 1von 507

ENGINEERING MATHEMATICS I

First Version

D.W. Ddumba & H.W-Kayondo


Department of Mathematics
Makerere University
Contents

1 Functions 1
1.1 Interval Notation . . . . . . . . . . . . . . . . . . . . . . . . . . . . . . . . 1
1.1.1 Open Intervals . . . . . . . . . . . . . . . . . . . . . . . . . . . . . 1
1.1.2 Closed Intervals . . . . . . . . . . . . . . . . . . . . . . . . . . . . . 1
1.1.3 Half-open Intervals . . . . . . . . . . . . . . . . . . . . . . . . . . . 2
1.2 Infinite Intervals . . . . . . . . . . . . . . . . . . . . . . . . . . . . . . . . . 2
1.2.1 Open Infinite Intervals . . . . . . . . . . . . . . . . . . . . . . . . . 2
1.2.2 A closed-infinite interval [a, ∞). . . . . . . . . . . . . . . . . . . . . 3
1.2.3 Infinite Half Open Interval (−∞, a]. . . . . . . . . . . . . . . . . . . 3
1.2.4 An Infinite-Infinite Interval (−∞, ∞). . . . . . . . . . . . . . . . . . 3
1.3 Absolute values in calculus . . . . . . . . . . . . . . . . . . . . . . . . . . . 4
1.3.1 Sketching absolute functions . . . . . . . . . . . . . . . . . . . . . . 10
1.4 Functions . . . . . . . . . . . . . . . . . . . . . . . . . . . . . . . . . . . . 12
1.4.1 Domain and Range as a set of Ordered Pairs. . . . . . . . . . . . . 12
1.4.2 Function Notation . . . . . . . . . . . . . . . . . . . . . . . . . . . 17
1.4.3 Evaluation of Functions . . . . . . . . . . . . . . . . . . . . . . . . 17
1.5 Types of functions . . . . . . . . . . . . . . . . . . . . . . . . . . . . . . . 19
1.5.1 Equal Functions . . . . . . . . . . . . . . . . . . . . . . . . . . . . . 19
1.5.2 Identity Function . . . . . . . . . . . . . . . . . . . . . . . . . . . . 19
1.5.3 Constant Function . . . . . . . . . . . . . . . . . . . . . . . . . . . 19
1.5.4 One-to-One Function . . . . . . . . . . . . . . . . . . . . . . . . . . 20
1.5.5 Many-to-One Function . . . . . . . . . . . . . . . . . . . . . . . . . 22
1.5.6 Onto-Functions . . . . . . . . . . . . . . . . . . . . . . . . . . . . . 22
1.5.7 Bijective Function . . . . . . . . . . . . . . . . . . . . . . . . . . . . 23
1.5.8 Even Functions . . . . . . . . . . . . . . . . . . . . . . . . . . . . . 24
1.5.9 Odd Functions . . . . . . . . . . . . . . . . . . . . . . . . . . . . . 25
1.5.10 Inverse of functions . . . . . . . . . . . . . . . . . . . . . . . . . . . 25
1.5.11 Operations of functions . . . . . . . . . . . . . . . . . . . . . . . . . 27
1.5.12 Other functions . . . . . . . . . . . . . . . . . . . . . . . . . . . . . 35

2 Limits of functions 39
2.1 Informal definition of a limit of a function . . . . . . . . . . . . . . . . . . 39
2.2 Computation of limits . . . . . . . . . . . . . . . . . . . . . . . . . . . . . 44
CONTENTS

2.2.1 Substitution Method . . . . . . . . . . . . . . . . . . . . . . . . . . 44


2.2.2 Numerator Factorisation [Non-analytic Technique] . . . . . . . . . . 45
2.2.3 Infinity . . . . . . . . . . . . . . . . . . . . . . . . . . . . . . . . . . 47
2.2.4 Using La’Hopital rule . . . . . . . . . . . . . . . . . . . . . . . . . . 53
2.2.5 Indeterminate Forms and La’Hopital’s Rule . . . . . . . . . . . . . 57
2.3 Properties of limits . . . . . . . . . . . . . . . . . . . . . . . . . . . . . . . 63

3 Continuity of Functions 68
3.1 Informal definition of Continuity of a function . . . . . . . . . . . . . . . . 68
3.1.1 Removable discontinuity . . . . . . . . . . . . . . . . . . . . . . . . 88
3.1.2 Jump discontinuity . . . . . . . . . . . . . . . . . . . . . . . . . . . 90
3.1.3 Essential discontinuity . . . . . . . . . . . . . . . . . . . . . . . . . 91
3.1.4 Formal definition of continuity of function f (x) at x=a . . . . . . 92
3.1.5 Continuity at end points of domain . . . . . . . . . . . . . . . . . . 92
3.2 Intermediate Value Theorem, IVT . . . . . . . . . . . . . . . . . . . . . . . 94
3.3 Fixed point Theorem . . . . . . . . . . . . . . . . . . . . . . . . . . . . . . 95
3.4 Questions with Solutions . . . . . . . . . . . . . . . . . . . . . . . . . . . . 98
3.4.1 Questions . . . . . . . . . . . . . . . . . . . . . . . . . . . . . . . . 98
3.4.2 Solutions . . . . . . . . . . . . . . . . . . . . . . . . . . . . . . . . . 99

4 Differentiation 101
4.1 Derivative of a function . . . . . . . . . . . . . . . . . . . . . . . . . . . . 101
4.2 Continuity Versus Differentiability . . . . . . . . . . . . . . . . . . . . . . . 127
4.3 Differentiation Theorems . . . . . . . . . . . . . . . . . . . . . . . . . . . . 136
4.4 Other techniques of differetiation . . . . . . . . . . . . . . . . . . . . . . . 141
4.4.1 Chain Rule - Composite differentiation . . . . . . . . . . . . . . . . 141
4.4.2 Differentiation of implicit functions . . . . . . . . . . . . . . . . . . 147
4.4.3 Parametric equations . . . . . . . . . . . . . . . . . . . . . . . . . . 152
4.4.4 Logarithmic differentiation . . . . . . . . . . . . . . . . . . . . . . . 156
4.5 Applications of differentiation . . . . . . . . . . . . . . . . . . . . . . . . . 169
4.5.1 Maxima and Minima . . . . . . . . . . . . . . . . . . . . . . . . . . 169
4.5.2 Mean Value Theorem MVT . . . . . . . . . . . . . . . . . . . . . . 175
4.5.3 Second Derivative Test and Concavity . . . . . . . . . . . . . . . . 181
4.5.4 Approximation of functions and Rates of change . . . . . . . . . . . 202
4.5.5 Curve Sketching . . . . . . . . . . . . . . . . . . . . . . . . . . . . . 208

5 The Definite Integral 231


5.1 Area of plane regions . . . . . . . . . . . . . . . . . . . . . . . . . . . . . . 231
5.2 Approximating Sums: Riemann sums . . . . . . . . . . . . . . . . . . . . . 232
5.2.1 Introduction . . . . . . . . . . . . . . . . . . . . . . . . . . . . . . . 232
5.2.2 Riemann sums . . . . . . . . . . . . . . . . . . . . . . . . . . . . . 233
5.3 Boundedness Theorem . . . . . . . . . . . . . . . . . . . . . . . . . . . . . 253
5.4 Fundamental theorem of Integration . . . . . . . . . . . . . . . . . . . . . . 254

H.W-Kayondo & D.W-Ddumba, Engineering Math I- Lecture Notes


CONTENTS

5.4.1 First Fundamental theorem of integral calculus . . . . . . . . . . . . 254


5.4.2 Second Fundamental Theorem of the Integral Calculus . . . . . . . 255
5.5 Mean Value Theorem (MVT)of Integration . . . . . . . . . . . . . . . . . . 258
5.6 Some applications of definite integrals . . . . . . . . . . . . . . . . . . . . . 261

6 Antiderivatives 265
6.1 The indefinite integral . . . . . . . . . . . . . . . . . . . . . . . . . . . . . 265
6.2 You can always check the answer . . . . . . . . . . . . . . . . . . . . . . . 266
6.3 About “+C” . . . . . . . . . . . . . . . . . . . . . . . . . . . . . . . . . . 267

7 Techniques of Integration 269


7.1 Standard Integrals . . . . . . . . . . . . . . . . . . . . . . . . . . . . . . . 269
7.2 Function and its derivative . . . . . . . . . . . . . . . . . . . . . . . . . . . 273
7.3 Trigonometric functions . . . . . . . . . . . . . . . . . . . . . . . . . . . . 276
7.3.1 Even powers of trigonometric functions . . . . . . . . . . . . . . . . 276
7.3.2 Odd powers of trigonometric functions . . . . . . . . . . . . . . . . 277
7.3.3 Factor formula . . . . . . . . . . . . . . . . . . . . . . . . . . . . . 277
7.3.4 The double angle trick . . . . . . . . . . . . . . . . . . . . . . . . . 277
7.4 Method of substitution . . . . . . . . . . . . . . . . . . . . . . . . . . . . . 279
7.5 Logarithmic integration . . . . . . . . . . . . . . . . . . . . . . . . . . . . . 284
7.6 Integration by Parts . . . . . . . . . . . . . . . . . . . . . . . . . . . . . . 286
7.7 Reduction Formulas . . . . . . . . . . . . . . . . . . . . . . . . . . . . . . . 292
7.8 Partial Fraction Expansion . . . . . . . . . . . . . . . . . . . . . . . . . . . 299
7.8.1 Reduce to a proper rational function . . . . . . . . . . . . . . . . . 299
7.8.2 Partial Fraction Expansion: The Easy Case . . . . . . . . . . . . . 300
7.8.3 Partial Fraction Expansion: The General Case . . . . . . . . . . . . 302
7.9 Inverse Trigonometric Functions . . . . . . . . . . . . . . . . . . . . . . . . 314
7.10 t-substitution . . . . . . . . . . . . . . . . . . . . . . . . . . . . . . . . . . 318
7.11 Trignometric Substitutions . . . . . . . . . . . . . . . . . . . . . . . . . . . 319
7.12 Miscellaneous and Mixed Integrals . . . . . . . . . . . . . . . . . . . . . . . 320

8 Improper Integrals 342


8.1 Introduction . . . . . . . . . . . . . . . . . . . . . . . . . . . . . . . . . . . 342
8.2 Improper Integrals of First Kind . . . . . . . . . . . . . . . . . . . . . . . . 342
8.3 Convergence of integrals . . . . . . . . . . . . . . . . . . . . . . . . . . . . 344
8.3.1 Comparison test . . . . . . . . . . . . . . . . . . . . . . . . . . . . . 344
8.4 Improper Integrals of Second Kind . . . . . . . . . . . . . . . . . . . . . . 345
8.5 Applications of improper integrals. . . . . . . . . . . . . . . . . . . . . . . 359

9 Applications of definite integrals 364


9.1 Area Between Curves . . . . . . . . . . . . . . . . . . . . . . . . . . . . . . 364
9.2 Volumes of Solids of revolution . . . . . . . . . . . . . . . . . . . . . . . . 376
9.3 Lengths of Curves . . . . . . . . . . . . . . . . . . . . . . . . . . . . . . . . 381
9.4 Area of Surface of Revolution . . . . . . . . . . . . . . . . . . . . . . . . . 387

H.W-Kayondo & D.W-Ddumba, Engineering Math I- Lecture Notes


CONTENTS

9.5 Examples . . . . . . . . . . . . . . . . . . . . . . . . . . . . . . . . . . . . 391

10 Matrix 396
10.1 Definitions . . . . . . . . . . . . . . . . . . . . . . . . . . . . . . . . . . . . 396
10.2 Special Types of matrices . . . . . . . . . . . . . . . . . . . . . . . . . . . 398
10.2.1 Diagonal Matrix . . . . . . . . . . . . . . . . . . . . . . . . . . . . 398
10.2.2 Tri diagonal Matrix . . . . . . . . . . . . . . . . . . . . . . . . . . . 399
10.2.3 Triangular matrix . . . . . . . . . . . . . . . . . . . . . . . . . . . . 399
10.2.4 Idempotent matrix . . . . . . . . . . . . . . . . . . . . . . . . . . . 401
10.2.5 Invertible or non-singular matrix . . . . . . . . . . . . . . . . . . . 402
10.3 Operations of Matrices . . . . . . . . . . . . . . . . . . . . . . . . . . . . . 402
10.3.1 Addition and Scalar Multiplication of matrices . . . . . . . . . . . . 402
10.3.2 Matrix Multiplication . . . . . . . . . . . . . . . . . . . . . . . . . . 406
10.4 Properties of matrices . . . . . . . . . . . . . . . . . . . . . . . . . . . . . 414
10.5 Determinants . . . . . . . . . . . . . . . . . . . . . . . . . . . . . . . . . . 415
10.5.1 Permutations and inversion of a permutation . . . . . . . . . . . . . 415
10.5.2 Matrix Adjoint,Minors,Cofactors . . . . . . . . . . . . . . . . . . . 417
10.5.3 Properties of determinants . . . . . . . . . . . . . . . . . . . . . . . 422
10.6 Test your self . . . . . . . . . . . . . . . . . . . . . . . . . . . . . . . . . . 423
10.7 Matrix Inverses . . . . . . . . . . . . . . . . . . . . . . . . . . . . . . . . . 432
10.7.1 Direct method . . . . . . . . . . . . . . . . . . . . . . . . . . . . . . 432
10.7.2 Adjoint method . . . . . . . . . . . . . . . . . . . . . . . . . . . . . 432
10.7.3 Method of elementary row operation ; Gauss-Jordan elimination
method . . . . . . . . . . . . . . . . . . . . . . . . . . . . . . . . . 435
10.7.4 Properties of inverses . . . . . . . . . . . . . . . . . . . . . . . . . . 441

11 Linear System of Equations 449


11.1 Row reduction to Echelon form . . . . . . . . . . . . . . . . . . . . . . . . 450
11.2 Existence of a solution to a linear system . . . . . . . . . . . . . . . . . . . 450
11.2.1 No solution . . . . . . . . . . . . . . . . . . . . . . . . . . . . . . . 451
11.2.2 Unique solution . . . . . . . . . . . . . . . . . . . . . . . . . . . . . 452
11.2.3 Infinitely many solutions . . . . . . . . . . . . . . . . . . . . . . . 453
11.3 Crammer’s rule . . . . . . . . . . . . . . . . . . . . . . . . . . . . . . . . . 458
11.4 Homogeneous systems of linear equations . . . . . . . . . . . . . . . . . . . 459
11.4.1 Solution of a homogeneous system . . . . . . . . . . . . . . . . . . . 459
11.4.2 Nature of Solution of a homogeneous system Ax = 0 . . . . . . . . 460
¯
11.5 Sets of linear systems . . . . . . . . . . . . . . . . . . . . . . . . . . . . . . 462

12 Eigen Values and Eigen Vectors 465


12.1 Finding eigen values & corresponding eigen vectors. . . . . . . . . . . . . . 465
12.2 Similar Matrices . . . . . . . . . . . . . . . . . . . . . . . . . . . . . . . . . 470
12.2.1 Properties of similarity . . . . . . . . . . . . . . . . . . . . . . . . . 473
12.3 Diagonisable matrix . . . . . . . . . . . . . . . . . . . . . . . . . . . . . . . 473

H.W-Kayondo & D.W-Ddumba, Engineering Math I- Lecture Notes


CONTENTS

13 Complex Variable Algebra 485


13.1 Complex Numbers . . . . . . . . . . . . . . . . . . . . . . . . . . . . . . . 485
13.1.1 Addition and Multiplication . . . . . . . . . . . . . . . . . . . . . . 485
13.1.2 Subtraction and Division . . . . . . . . . . . . . . . . . . . . . . . . 486
13.1.3 Complex plane . . . . . . . . . . . . . . . . . . . . . . . . . . . . . 487
13.1.4 Complex Conjugate Numbers . . . . . . . . . . . . . . . . . . . . . 487
13.2 Polar Form of Complex Numbers, Powers and Roots . . . . . . . . . . . . 488
13.2.1 Triangle inequality . . . . . . . . . . . . . . . . . . . . . . . . . . . 489
13.2.2 Generalized triangle inequality: . . . . . . . . . . . . . . . . . . . . 490
13.2.3 Multiplication and Division in Polar Form . . . . . . . . . . . . . . 490
13.2.4 Integer powers and De Moivre’s formula . . . . . . . . . . . . . . . 491
13.3 The basic Transcendental Functions . . . . . . . . . . . . . . . . . . . . . . 494
13.3.1 Exponential Function . . . . . . . . . . . . . . . . . . . . . . . . . . 494
13.3.2 Trigonometric Functions . . . . . . . . . . . . . . . . . . . . . . . . 496
13.4 Real, imaginary parts and Absolute value. . . . . . . . . . . . . . . . . . . 497
13.5 Hyperbolic Functions . . . . . . . . . . . . . . . . . . . . . . . . . . . . . . 499

H.W-Kayondo & D.W-Ddumba, Engineering Math I- Lecture Notes


Chapter 1

Functions

1.1 Interval Notation


In the definitions of Open, closed and Half closed intervals we shall assume that a and b
are real numbers such that a < b.

1.1.1 Open Intervals


An Open Interval, written in interval notation as (a, b), is defined as the set of all numbers
x such that a < x < b : where a < b that is x lies between a and b. This can be represented
in set-builder notation as (a, b) = {x : a < x < b}. The points a and b are themselves not
included.

Example 1.1.1 Express the following open intervals in set-builder notation:

(a) (2, 4)

(b) (−1, 3)

These open intervals can be expressed in set-builder notation as:

(a) (2, 4) = {x : 2 < x < 4}

(b) (−1, 3) = {x : −1 < x < 3}

1.1.2 Closed Intervals


A Closed Interval, written in interval notation as [a, b] is defined as the set of all numbers
x such that a ≤ x ≤ b. Closed intervals can be represented in set-builder notation as:
[a, b] = (x : a ≤ x ≤ b) x lies between and includes a and b.

Example 1.1.2 Express the following closed intervals in set builder notation.

(a) [m, n]

1
CHAPTER 1. FUNCTIONS

(b) [−1, 3]

These closed intervals can be expressed in set builder notation as:

(a) [m, n] = {x : m ≤ x ≤ n}

(b) [−1, 3] = {x : −1 ≤ x ≤ 3}

1.1.3 Half-open Intervals


There are two types of half open intervals. The left half open interval (a, b] and the right
half open interval [a, b).

The left half open interval is defined as the set of numbers x such that a < x ≤ b. This
can be represented in set-builder notation as

(a, b] = {x : a < x ≤ b}

where a < b.

The right half open interval is defined as the set of number x such that a ≤ x < b, where
a < b. This can be represented in set-builder notation as

[a, b) = {x : a ≤ x < b}

Example 1.1.3 Express the following half open intervals in set-builder notation.

(a) (−2, 5]

(b) [−a, a)

The half open intervals above can be expressed in set builder notation as:

(a) (−2, 5] = {x : −2 < x ≤ 5}

(b) [−a, a) = {x : −a ≤ x < a}

1.2 Infinite Intervals


We have four types of infinite intervals.

1.2.1 Open Infinite Intervals


Let a be a real number. The Open Infinite Interval (a, ∞) is the set of numbers x such that
a < x < ∞. This is represented in set - builder notation by (a, ∞) = {x : a < x < ∞}.
This can be displayed on the real line graph by the space between a and infinity but not

H.W-Kayondo & D.W-Ddumba, Engineering Math I- Lecture Notes 2


CHAPTER 1. FUNCTIONS

including the point a.

Similarly, the open interval (−∞, a), is the set of numbers x such that ∞ < x < a. This
is represented in set builder notation by (−∞, a) = {x : −∞ < x < a). On the real
line graph, this can be displayed by the space between negative infinity and a, but not
including a.

1.2.2 A closed-infinite interval [a, ∞).


These are the set of numbers x such that a ≤ x < ∞. This is expressed in set-builder
notation by (a, ∞) = {x : a ≤ x < ∞}. This can be displayed on the real line graph by
the space between a and infinity including the point a.

1.2.3 Infinite Half Open Interval (−∞, a].


This is the set of numbers x such that −∞ < x ≤ a. This is represented in set-builder
notation by (−∞, a] = (x : −∞ < x ≤ a). This can be displayed on the real line graph
by the space between negative infinity and a including the point a.

1.2.4 An Infinite-Infinite Interval (−∞, ∞).


This is the set of numbers x such that −∞ < x < ∞. This is represented in set builder
notation by (−∞, ∞) = {x : −∞ < x < ∞}. This can be displayed on the number line
by all this points between negative infinity and positive infinity. This represents the entire
number line.

Example 1.2.1 Express the following intervals in set - builder notation.

(a) (2, 4)

(b) [2, 4)

(c) (−4, 3]

The above intervals can be expressed in set-builder notation as

(a) (2, 4) = {x : 2 < x < 4}

(b) [2, 4) = {x : 2 ≤ x < 4}

(c) (−4, 3] = {x : −4 < x ≤ 3}

Example 1.2.2 Write the following sets in interval notation:

(a) A = {x : −∞ < x < 2}

(b) B = {x : −1 < x < 4}

H.W-Kayondo & D.W-Ddumba, Engineering Math I- Lecture Notes 3


CHAPTER 1. FUNCTIONS

(c) C = {x : −∞ < x ≤ 3}
(d) D = {x : 2 ≤ x < ∞}
These sets can be represented in interval notation as follows:-
(a) The set of points do not include 2, but continues to the left of 2 up to negative
infinity, that is (−∞, 2).
(b) The set of points that lies between −1 and −4, that is, (−1, 4).
(c) The set of points that includes 3 and continues to the left of 3 up to negative infinity,
that is, (−∞, 3]
(d) The set of points that includes 2 and continues to the right of 2 up to infinity that
is, [2, ∞).

1.3 Absolute values in calculus


The absolute value of a real number x is denoted by |x| and is given by


x, x≥0
|x| =
−x , x<0

The absolute value of a real number x is a measure of how far the real number x is from
0, the origin of the number line. For this reason it is always a positive quantity and
sometimes it is referred to as the magnitude
√ of the number.
√ Alternatively, the absolute
value of x may be defined as |x| = x2 . (Note that x stands for the positive square
root of x). The distance between two real numbers a and b is the number |a − b| = |b − a|.

Example 1.3.1 Simplify the norm (abolute value) |x − 5|


 (x − 5) , x≥5
|x − 5| =
−(x − 5) , x<5

Example 1.3.2 Solve the inequality |x + 3| = 4. We know that


 (x + 3) , x ≥ −3
|x + 3| =
−(x + 3) , x < −3

x + 3 = 4, ⇒ x = 1 or −(x + 3) = 4, ⇒ x = −7

H.W-Kayondo & D.W-Ddumba, Engineering Math I- Lecture Notes 4


CHAPTER 1. FUNCTIONS

Theorem 1.3.1 The inequality |x| ≤ a is equivalent to the double inequality −a ≤ x ≤ a.


The inequality |y| ≥ b holds for y ≤ −b or y ≥ b.
Theorem 1.3.2 For any real numbers x and y,
(a) |x| ≥ 0,
(b) |xy| = |x||y|,
(c) |x + y| ≤ |x| + |y|.
Note 1.3.1 Think of absolute as in two parts.
Example 1.3.3 Solve the inequality
|x + 2| < 3
Method I:
|x + 2| < 3 ⇒ −3 < x + 2 < 3
x+2<3 ⇒ x<1
x + 2 > −3 ⇒ x > −5
x ∈ (−5, 1)
Method II: Splitting the absolute into two, this is the more general one, and more reliable.
|x + 2| < 3

(x + 2) 3 : x ≥ −2
<
−(x + 2) 3 : x < −2

(x + 2) < 3 ⇒ x < 1
−(x + 2) < 3 ⇒ x > −5
x ∈ (−5, 1)
Example 1.3.4 Solve the inequality
|2x + 3| ≥ 8
Splitting the absolute into two for |x + 2| ≥ 8

3
(2x + 3) 8 : x≥−
2

3
−(2x + 3) 8 : x<−
2
H.W-Kayondo & D.W-Ddumba, Engineering Math I- Lecture Notes 5
CHAPTER 1. FUNCTIONS

5
(2x + 3) ≥ 8 ⇒ x ≥
2
11
−(2x + 3) ≥ 8 ⇒ x ≤ −
2
   
11 5
−∞, − ∪ ,∞
2 2

Example 1.3.5 Solve the inequality

| − 2x − 4| ≥ 9

   
13 5
−∞, − ∪ ,∞
2 2

Example 1.3.6 Solve the inequality

x + 2 < |x2 − 4|

This can be rewritten as |x2 − 4| > x + 2


Splitting the absolute into two for |x2 − 4| > x + 2

(x2 − 4) x + 2 : x ∈ (−∞, −2] ∪ [2, ∞)


>
2
−(x − 4) x + 2 : x ∈ (−2, 2)

(x2 − 4) > x + 2 ⇒ x2 − x − 6 > 0 ⇒ (x + 2)(x − 3) > 0 ⇒ (−∞, −2) ∪ (3, ∞)


−(x2 − 4) > x + 2 ⇒ x2 + x − 2 < 0 ⇒ (x + 1)(x − 2) < 0 ⇒ (−1, 2)

[(−∞, −2] ∪ [2, ∞) ∩ (−∞, −3) ∪ (3, ∞)] or [(−2, 2) ∩ (−1, 2)]
⇒ (−∞, −2) ∪ (−1, 2) ∪ (3, ∞)

Example 1.3.7 Solve the equation

4|2x − 1| − 2 = 10

4|2x − 1| − 2 = 10
4|2x − 1| = 12
|2x − 1| = 3

H.W-Kayondo & D.W-Ddumba, Engineering Math I- Lecture Notes 6


CHAPTER 1. FUNCTIONS

Splitting the absolute into two for |2x − 1| = 3

1
(2x − 1) 3 : x≥
2
=
1
−(2x − 1) 3 : x<
2

(2x − 1) = 3 ⇒ x = 2
−(2x − 1) = 3 ⇒ x = −1

x = −1, x = 2

Example 1.3.8 Solve for the inequality

|3x + 1| < 2|x − 6|

Whenever we have absolutes, we represent it as into two parts,

(3x + 1)x≥− 1 2(x − 6)x≥6


3
<
−(3x + 1)x<− 1 −2(x − 6)x<6
3

Taking all possible combinations of the terms,

[(3x + 1)] < [2(x − 6)] ⇒ x < −13 : for region x ≥ 6 ⇒ no solution
 
11 1 1 11
[(3x + 1)] < [−2(x − 6)] ⇒ x< : in region x ∈ [− , 6) ⇒ x ∈ − ,
5 3 3 5

11
[−(3x + 1)] < [2(x − 6)] ⇒ x> : in no region ⇒ no solution
5
 
1 1
[−(3x + 1)] < [−2(x − 6)] ⇒ x > −13 : (for region) x < − ⇒ x ∈ −13, −
3 3

   
1 11 1
x∈ − , ∪ −13, −
3 5 3

Example 1.3.9 Solve the equation

|x − 2| + |x + 5| ≤ 0

H.W-Kayondo & D.W-Ddumba, Engineering Math I- Lecture Notes 7


CHAPTER 1. FUNCTIONS

Splitting the absolutes into two for;


(x − 2)x≥2 (x + 5)x≥−5
+ ≤0
−(x − 2)x<2 −(x + 5)x<−5
Taking all possible combinations of the terms,

3
[(x − 2)] + [(x + 5)] ≤ 0 ⇒ x ≤ − : in region x ≥ 2 ⇒ no solution
2
[(x − 2)] + [−(x + 5)] ≤ 0 ⇒ no solution : in no region ⇒ no solution
[−(x − 2)] + [(x + 5)] ≤ 0 ⇒ no solution : in region x ∈ [−5, 2) ⇒ no solution

3
[−(x − 2)] + [−(x + 5)] ≤ 0 ⇒ x ≥ − : in region (for) x < −5 ⇒ no solution
2

No solution for the problem above.


Example 1.3.10 Solve the equation
|2x − 2| = x + 1

1
x = 3, x =
3

Example 1.3.11 Solve for|x − 5| ≤ 2



 (x − 5) , x≥5
|x − 5| =
−(x − 5) , x<5

(i) (x − 5) ≤ 2 and x ≥ 5, ⇒ x ≤ 7 and x ≥ 5, ⇒ 5 ≤ x ≤ 7


(ii) −(x − 5) ≤ 2 and x < 5, ⇒ x ≥ 3 and x < 5, ⇒ 3 ≤ x ≤ 4
(iii) Form (i) and also (ii) (the union), we realise 3 ≤ x ≤ 7
Alternatively, |x − 5| ≤ 2 ⇒ − 2 ≤ (x − 5) ≤ 2 ⇒ 3 ≤ x ≤ 7 by adding a 5 everywhere
Example 1.3.12 Solve for|x + 3| ≤ 4

 (x + 3) , x ≥ −3
|x + 3| =
−(x + 3) , x < −3

H.W-Kayondo & D.W-Ddumba, Engineering Math I- Lecture Notes 8


CHAPTER 1. FUNCTIONS

(i) (x + 3) ≤ 4 and x ≥ −3, ⇒ x ≤ 1 and x ≥ −3, ⇒ −3 ≤ x ≤ 1


(ii) −(x + 3) ≤ 4 and x < −3, ⇒ x ≥ −7 and x < −3, ⇒ −7 ≤ x < −3
(iii) Form (i) and also (ii) (the union), we realise −7 ≤ x ≤ 1
Alternatively, |x + 3| ≤ 4 ⇒ − 4 ≤ (x + 3) ≤ 4
−7 ≤ x ≤ 1
Example 1.3.13 Solve |3 − 2x| ≥ 1.
 3
 (3 − 2x) , x≤ 2
|3 − 2x| =
3
−(3 − 2x) , x>

2

(i) (3 − 2x) ≥ 1 and x ≤ 32 , ⇒ x ≤ 1 and x ≤ 23 , ⇒ x ≤ 1


(ii) −(3 − 2x) ≥ 1 and x < 23 , ⇒ x ≥ 2 and x > 32 , ⇒ x ≥ 2
(iii) Form (i) and also (ii) (the union), we realise x ∈ (−∞, 1] ∪ [2, ∞)
Alternatively, Either 3 − 2x ≥ 1 or 3 − 2x ≤ −1. That is, either x ≤ 1 or x ≥ 2.
x ∈ (−∞, 1] ∪ [2, ∞)
Example 1.3.14 Solve |2x + 3| < 6

9 3
− <x<
2 2

Example 1.3.15 Solve |2x − 3| > 5.


x < −1 or x > 4
Example 1.3.16 Find the absolute-value inequality statement that corresponds to the
inequality
−2 < x < 4
I first look at the endpoints. Negative two and four are six units apart. Half of six is
three. So I want to adjust this inequality so that it relates to −3 and 3, instead of to −2
and 4. To accomplish this, I will adjust the ends by subtracting 1 from all three ”sides”:
−2 < x < 4
−2 − 1 < x − 1 < 4 − 1
−3 < x − 1 < 3
Since the last line above is in the ”less than” format, the absolute-value inequality will
be of the form ”absolute value of something is less than 3”. I can convert this nicely to
|x − 1| < 3

H.W-Kayondo & D.W-Ddumba, Engineering Math I- Lecture Notes 9


CHAPTER 1. FUNCTIONS

Example 1.3.17 Find the absolute-value inequality statement that corresponds to the
inequalities
x < 19 or x > 24
I first look at the endpoints. Nineteen and 24 are five units apart. Half of five is 2.5. So
I want to adjust the inequality so it relates to −2.5 and 2.5, instead of relating to 19 and
24. Since 19 − (−2.5) = 21.5 and 24 − 2.5 = 21.5, I need to subtract 21.5 all around:
x < 19 or x > 24
x − 21.5 < 19 − 21.5 or x − 21.5 > 24 − 21.5
x − 21.5 < −2.5 or x − 21.5 > 2.5
Since the last line above is the ”greater than” format, the absolute-value inequality will
be of the form ”absolute value of something is greater than or equal to 2.5”. I can convert
this nicely to:
|x − 21.5| > 2.5
Exercise 1.1 Solve for x in the following:

1. 2x + 7 > 4x − 5 5. |x − 8| = 2
2. −4 ≤ 2(x + 2) < 12
6. |x + 4| − |x − 1| < 4
3. (x + 8)(4x − 6) > 0
4. x2 + x < 0 7. |x + 2| + |x − 5| ≥ 10.

1.3.1 Sketching absolute functions


Example 1.3.18 Graph the curve

|x + 2|x2
f (x) =
|x|

Now its not only one point to think of, but now both x = −2 and x = 0. We need to
have different functions for
x < −2
−2 ≤ x ≤ 0
x>2

−(x+2)x2


 −(x)
, if x < −2




(x+2)x2
f (x) = −(x)
, if − 2 ≤ x ≤ 0




(x+2)x2

, if x > 0


(x)

H.W-Kayondo & D.W-Ddumba, Engineering Math I- Lecture Notes 10


CHAPTER 1. FUNCTIONS

and sketch those function in the different ranges.


Example 1.3.19 Sketch the graph

f (x) = |x − 2||x − 4|

Example 1.3.20 Graph the function

x (|4x − 3| − |x + 6| + |x|)
f (x) =
3|x|

Hence or otherwise, solve the inequality

x (|4x − 3| − |x + 6| + |x|)
>0
3|x|

Example 1.3.21 Let



 x+2 , x≥0
g(x) =
x−2 , x<0

Sketch the graph of g(x).

Figure 1.1: A piecewise function sketch

Does lim g(x) exist? Justify your answer.


x→0

H.W-Kayondo & D.W-Ddumba, Engineering Math I- Lecture Notes 11


CHAPTER 1. FUNCTIONS

1.4 Functions
Definition 1.4.1 A function is a set of ordered pairs of elements such that not two or-
dered pairs of a set have the same first element.

Functions can be considered as mappings. A function is a special kind of relation in which


all ordered pairs have unique first elements. Every element of the first set is assigned to
an element of the second set. Usually subsets of the set of real numbers R are considered.

A function involves two sets and a rule of correspondence between them. The rule of
correspondence specifies how to pair the elements of the one set with those in the other.

A function is a relation in which for each member of the first set there is a single corre-
sponding member of the second set. A rule exists between two quantities.

A function has to satisfy The Vertical Line Test.

1.4.1 Domain and Range as a set of Ordered Pairs.


The domain (D) is the set of all values that the first elements, in the ordered pair can
take. The Range (R) is the set of all values that the second elements, in the ordered pair,
can take.
Example 1.4.1 The following table defines a function.

x −3 5 6 7
y −2 8 9 5

(a) List the ordered pairs of the function


(b) State the domain of the function
(c) State the range of the function
(a) The ordered pairs of the function are: (−3, −2); (5, 8); (6, 9)and(7, 5).
(b) The Domain are the set of the first elements of the ordered pairs. D = {−3, 5, 6, 7}
(c) The Range are the set of the second elements of the ordered pairs. R = {−2, 8, 9, 5}
Example 1.4.2 State the domain and range of the function f = {(−2, 4)(−1, 4)(0, 4)(2, 4)}

The Domain D = {−2, −1, 0, 2}, are the first elements in the ordered pairs. The Range
R = {4}, is the second element in the ordered pairs.

H.W-Kayondo & D.W-Ddumba, Engineering Math I- Lecture Notes 12


CHAPTER 1. FUNCTIONS

Example 1.4.3 A mapping defined by


{(−1, 2), (4, 6), (5, 0), (4, 3)}
is not a function, since the first element 4 appears twice (more than once).
Note 1.4.1 A function can be given in the form of ordered pairs such as
{(−1, 1), (4, 11), (5, 13), (1, 5)}
or by a formula i.e f (x) = 2x + 3
Definition 1.4.2 The domain of a function is the set of ”input” or argument values for
which the function is defined. That is, the function provides an ”output” or ”value” for
each member of the domain.
Definition 1.4.3 The range is the codomain or the image of the function
Example 1.4.4 Determine the domain and range of the given function:

(x2 + x − 2)
y=
(x2 − x − 2)

The domain is all the values that x is allowed to take on. The only problem I have with
this function is that I need to be careful not to divide by zero. So the only values that x
can not take on are those which would cause division by zero. So I’ll set the denominator
equal to zero and solve; my domain will be everything else.
x2 − x − 2 = 0
(x − 2)(x + 1) = 0
x = 2 or x = −1
Then the domain is ”all x not equal to −1 or 2”.

Since the graph will eventually cover all possible values of y, then the range is ”all real
numbers”.
Example 1.4.5 Determine the domain and range of the given function:

y = − −2x + 3
The domain is all values that x can take on. The only problem I have with this function
is that I cannot have a negative inside the square root. So I’ll set the insides greater-
than-or-equal-to zero, and solve. The result will be my domain:
−2x + 3 ≥ 0
−2x ≥ −3
2x ≤ 3
3
x ≤ = 1.5
2
H.W-Kayondo & D.W-Ddumba, Engineering Math I- Lecture Notes 13
CHAPTER 1. FUNCTIONS

Then the domain is ”all x ≤ 32 ”.

The range requires a graph. I need to be careful when graphing radicals:The range is
”y < 0”.
Example 1.4.6 Determine the domain and range of the given function:

y = −x4 + 4

This is just a garden-variety polynomial. There are no denominators (so no division-by-


zero problems) and no radicals (so no square-root-of-a-negative problems). There are no
problems with a polynomial. There are no values that I can’t plug in for x. When I have
a polynomial, the answer is always that the domain is ”all x”.

The range will vary from polynomial to polynomial, and they probably won’t even ask,
but when they do, I look at the picture: The graph goes only as high as y = 4, but it will
go as high as I like. Then: The range is ”all y ≥ 4”.
Example 1.4.7 Find the domain of function f defined by

1
f (x) =
(x − 1)

The domain x is such that x 6= 1, that is

x ∈ (−∞, 1) ∪ (1, +∞)

Example 1.4.8 what is the domain of the function



f (x) = x2 − 1 ?

Well, what could go wrong here? No division is indicated at all, so there is no risk of
dividing by 0. But we are taking a square root, so we must insist that x2 − 1 ≥ 0 to avoid
having complex numbers come up. That is, a preliminary description of the ‘domain’ of
this function is that it is the set of real numbers x so that x2 − 1 ≥ 0.
But we can be clearer than this: we know how to solve such inequalities. Often it’s
simplest to see what to exclude rather than include: here we want to exclude from the
domain any numbers x so that x2 − 1 < 0 from the domain.
We recognize that we can factor

x2 − 1 = (x − 1)(x + 1) = (x − 1) (x − (−1))

This is negative exactly on the interval (−1, 1), so this is the interval we must prohibit in
order to have just the domain of the function. That is, the domain is the union of two
intervals:
(−∞, −1] ∪ [1, +∞)

H.W-Kayondo & D.W-Ddumba, Engineering Math I- Lecture Notes 14


CHAPTER 1. FUNCTIONS

Exercise 1.2 Find the domain of the function

x−2
f (x) =
x2 +x−2

That is, find the largest subset of the real line on which this formula can be evaluated
meaningfully.
Exercise 1.3 Find the domain of the function

x−2
f (x) = √
x2 + x − 2

Exercise 1.4 Find the domain of the function


p
f (x) = x(x − 1)(x + 1)

Example 1.4.9 Compute the domain of the fuction

|x − 3.2| + |x − 5.2| = 2

Note 1.4.2 The domain is where the function lives (are the x values), and the range is
what the function can be (is the f (x)). To get domain, make sure the denominator is not
equal to zero, and cannot have a square root of a negative number.
Example 1.4.10 Find the domain and range of the following function
(i)
3
f (x) =
x2 −1

3 3
f (x) = =
x2 − 1 (x − 1)(x + 1)

so function defined every where otherthan at x = 1 and x = −1. Thus the domain,
Df = R − {−1, 1}, the domain are all possible solutions, which is all numbers, i.e
Rf = R

(ii) f (x) = x − 4, We know that we want only positive entries in the square root sign,
that is, x − 4 ≥ 0, meaning the domain is only values of x, such that x ≥ 4

(iii) Find the domain of √


f (x) = x2 − 1
(−∞, −1) ∪ (1, +∞)

H.W-Kayondo & D.W-Ddumba, Engineering Math I- Lecture Notes 15


CHAPTER 1. FUNCTIONS

(iv)
r
2x
f (x) =
x+2

This is just a mix of the two conditions, make sure you dont have zero below, and
2x
no negative numbers in the square root. That is x 6= −2, and x−2 ≥ 0, x ≥ −2

Example 1.4.11 Find the domain and range of the function

s
x(x − 2)
f (x) =
(4 − 3x)(6 − 2x)

The domain is
4 x(x − 2)
x 6= , x 6= 3, ≥0
3 (4 − 3x)(6 − 2x)

The range is R

Example 1.4.12 Find the range of values of the given function

x
f (x) =
|x|

if x 6= 0 while f (0) = 0

Example 1.4.13 Let

7x
f (x) =
x2 − 16

g(x) = x

Find (f og) (x) and give its domain.


7 x
f og(x) = , Domain := R+ − 16
x − 16

H.W-Kayondo & D.W-Ddumba, Engineering Math I- Lecture Notes 16


CHAPTER 1. FUNCTIONS

1.4.2 Function Notation


The notation f (x) represents the second element in the ordered pair that has x as its first
element. This ordered pair can be represented as (x, f (x)). We read f (x) as ”f of x” or
”f at x” since f (x) gives the value of f at x. This relation can also be represented as:

x ⇒ f (x)

Often we replace f (x) by y, also called the dependent variable, and write y = f (x). With
this notation, x is called the independent variable. For example f (x) = 3x + 2 may be
written as y = 3x + 2.

1.4.3 Evaluation of Functions


The evaluation of a function is illustrated by examples. A rule describing a function
can be represented by an equation or formula, a table or a graph, which are recorded as
ordered pairs. The evaluation of a function is to find a number that is paired with a given
number.

Example 1.4.14 To evaluate the function f (x) = 3x + 2 at x = 2, we need the number


f (2). Since f (x) = 3x + 2, replace x by 2 in the expression for the function and simplify.
Thus f (2) = 3 × 2 + 2 = 6 + 2 = 8. Therefore the ordered pair is (2, 8). We write f (2) = 8
or 2 → 8. This is the y value in the expression by y = 3x + 2 when x = 2.

Example 1.4.15 Let f (x) = x2 + 6x + 2 Find the value of

(a) f (0) (c) f (1) (e) f (n − 2)


(b) f (−2) (d) f (x + 1)

(a) f (0) = 02 + 6(0) + 2 = 2. Thus 0 → 2 or as an ordered pair, (0, 2).

(b) f (−2) = (−2)2 + 6(−2) + 2 = −6. Thus −2 → −6 or as an ordered pair, (−2, −6).

(c) f (1) = 12 + 6(1) + 2 = 9. Thus 1 → 9 or as an ordered pair, (1, 9).

(d)

f (x + 1) = (x + 1)2 + 6(x + 1) + 2
= (x2 + 2x + 1 + 6x + 6 + 2)
= x2 + 8x + 9.

Thus (x + 1) → x2 + 8x + 9 or as an ordered pair, ((x + 1), (x2 + 8x + 9)).

H.W-Kayondo & D.W-Ddumba, Engineering Math I- Lecture Notes 17


CHAPTER 1. FUNCTIONS

(e)

f (n − 2) = (n − 2)2 + 6(n − 2) + 2
= n2 − 4n + 4 + 6n − 12 + 2
= n2 + 2n − 6 or as an ordered pair, ((n − 2), (n2 + 2n − 6)

Thus (n − 2) → n2 + 2n − 6

f (x+h)−f (x)
Example 1.4.16 Let f (x) = 2x2 − 1. Let h > 0. Find the value of h
.

f (x + h) = 2(x + h)2 − 1
= 2(x2 + 2hx + h2 ) − 1
= 2x2 + 2hx + 2h2 − 1
f (x + h) − f (x) = (2x2 + 4hx + 2h2 − 1) − (2x2 − 1)
= 4hx + 2h2

f (x + h) − f (x) 4hx + 2h2


Therefore, =
h h
= 4x + 2h

Example 1.4.17 Given the function f (x) = 2x + 3 and a = 2. Compute and simplify
f (x)−f (a)
the value of x−a
, with x 6= a

Since f (x) = 2x + 3 and a = 2, f (a) = f (2) = 7. In addition (x − a) = (x − 2) and


therefore
f (x) − f (a) 2x + 3 − 7 2x − 4 (x − 2)
= = =2 =2
x−a x−2 x−2 (x − 2)

H.W-Kayondo & D.W-Ddumba, Engineering Math I- Lecture Notes 18


CHAPTER 1. FUNCTIONS

1.5 Types of functions


1.5.1 Equal Functions
Let f1 (x) and f2 (x) be functions that are defined on the same domain, D. If for each
element x of D, f1 (x) = f2 (x), then the two functions are equal and we write f1 (x) = f2 (x).
Two functions f1 (x) and f2 (x) are equal if and only if f1 (x) and f2 (x) have the same
domains and f1 (x) = f2 (x) for all x in this common domain.

Example 1.5.1 Prove that the functions:

(a) f1 (x) = 2x + 1 and f2 (x) = 4x


2
+ 1 are equal, where the domain for both functions
is the set of real numbers R.

π
(b) f1 (x) = sin x and f2 (x) = cos 2
−x are equal, where the domain of for both

functions is the set of real numbers.

(a) Let a be a real number. Then f1 (a) = 2a + 1 = 42 a + 1 = 4a


2
+ 1 = f2 (a). Since a was
arbitrary, f1 (x) = f2 (x)for all real numbers. The two functions are therefore equal.

     
π π π
(b) Let a be a real number. Then f2 (a) = cos 2
−a = cos 2
cos a+sin 2
sin a =
 
π
0 × cos a + 1 × sin a = sin a. Here we have used the fact that cos 2
= 0 and
 
π
sin 2
= 1.

1.5.2 Identity Function


A function f that associates each member of the domain with itself is called the identity
function. The identity function is defined by the equation f : x → x or f (x) = x. The
domain and the range of the identity function is the set of real numbers.

The identity function is a special kind of one-to-one and onto function.

1.5.3 Constant Function


A function f that associates each real number in a set A the same fixed number k in
the set B is called a constant function. The constant function is defined by the formula
f : x → k : f (x) = k(a single fixed number).

H.W-Kayondo & D.W-Ddumba, Engineering Math I- Lecture Notes 19


CHAPTER 1. FUNCTIONS

1.5.4 One-to-One Function


If f is a function of A into B is said to be one-to-one (1 − 1) if no two elements of A
correspond to the same element in B. Each element of the domain has a different image
in the Range.

A function is one-to-one if and only if each element in the domain is mapped into a unique
element of the co-domain (range).

A one to one function is a function in which every element in the range of the function
corresponds with one and only one element in the domain.

For a 1 − 1 function, If x1 6= x2 then f (x1 ) 6= f (x2 )


Example 1.5.2 If f (a) = f (b) implies that a = b, then f is 1 − 1, show whether or not
g(x) = 3x − 2 is one-on-one?

see if g(a) = g(b) implies a = b


3a − 2 = 3b − 2
3a = 3b
a = b

Thus g is 1 − 1.
Example 1.5.3 The function f : R → R defined by f (x) = 2x + 1 is injective.
Example 1.5.4 The function g : R → R defined by g(x) = x2 is not injective, because
(for example) g(1) = 1 = g(−1). However, if g is redefined so that its domain is the
non-negative real numbers [0, +∞), then g is injective.

A one-to-one function is one in which each x has only one y and each y has at most one
x to form ordered pairs.
Example 1.5.5 Let the function f : R → R be defined by the equation f (x) = 3x + 2
where R is the set of real numbers. Each real number will be mapped onto a unique image
by the function f (x) = 3x + 2. Hence f is a one-to-one function.
Example 1.5.6 Let the function f : R → R be defined by the formula f (x) = 3x2 + 2
Verify whether or not f is a one-to-one function.

The negative values of R are mapped onto the same elements as the corresponding positive
elements. For example, when x = −2, f (−2) = 14 and f (2) = 14. The images of two real
numbers −2 and 2 are the same number equal to 14. It follows that f is NOT a one-to-one
function.
Definition 1.5.1 A one to one function is also called an injective function.

H.W-Kayondo & D.W-Ddumba, Engineering Math I- Lecture Notes 20


CHAPTER 1. FUNCTIONS

Example 1.5.7 Which functions below are one to one ?


Function #1 {(2, 27), (3, 28), (4, 29), (5, 30)}
Function #2 {(11, 14), (12, 14), (16, 7), (18, 13)}
Function #3 {(3, 12), (4, 13), (6, 14), (8, 1)}
Relation #1 and Relation #3 are both one-to-one functions.
Example 1.5.8 A diagramatical example of 1 − 1

r 4
h
o 2
q 5
A
"One-to-One" B
NOT "One-to-One"

Figure 1.2: 1-1 and not 1-1 respectively

Theorem 1.5.1 The Horizontal Line Test: If a function is one to one, then the function
not only passes the vertical line test, but it also passes the horizontal line test.
Definition 1.5.2 The Horizontal Line Test : If a horizontal line only intersects with the
graph of a function once, then this function is one-to-one. If a horizontal line intersects
the graph of the function more than once, then this function is not one to one.

Fail the test (not injective)


Passes the test (injective)

Figure 1.3: A 1-1 and not 1-1 by Horizontal test

Example 1.5.9 Which functions below are one to one ?


Function #1 {(2, 1), (4, 5), (6, 7), (8, 9)}, Function #2 {(3, 4), (8, 5), (6, 7), (22, 4)}
Function #3 {(−3, 4), (21, −5), (0, 0), (8, 9)}, Function #4 {(9, 19), (34, 5), (6, 17), (8, 19)}
Relation #1 and Relation #3 are both one-to-one functions.

H.W-Kayondo & D.W-Ddumba, Engineering Math I- Lecture Notes 21


CHAPTER 1. FUNCTIONS

1.5.5 Many-to-One Function


If f is a relation of A into B. If more than one element of A is mapped into the same
element of B then f is a many-to-one function.

1.5.6 Onto-Functions
If f is a function of A into B. Sometimes the range, f (A) does not exhaust all the elements
of the set B called the co-domain. The range is therefore a subset of the co-domain. If
each member of the co-domain is an image of at least one member of A then f is a function
of A ONTO B and is therefore an onto function.
Definition 1.5.3 A function f from A to B is called onto if for all b in B there is an a
in A such that f (a) = b. All elements in B are used. Such functions are referred to as
surjective.

r 4 r 4
h 8
o 2 h
2
q 5 1
q 5
A B
A B
"Onto"
NOT "Onto"
(all elements in B are used)
(the 8 and 1 in Set B are not used)

Figure 1.4: An ”onto” and ”Not onto” functions

A function is said to be onto if all in range is an image (is a result of the transformation
- mapping)
Example 1.5.10 Is f (x) = 3x − 4 onto where f : R → R?

The function is onto; as you progress along


the line, every possible y-value is used.

Figure 1.5: An onto function

Example 1.5.11 Is g(x) = x2 − 2 onto where g : R → R??

H.W-Kayondo & D.W-Ddumba, Engineering Math I- Lecture Notes 22


CHAPTER 1. FUNCTIONS

Not onto; Values less than -2 on the y-axis are never


used. Since possible y-values belong to the set of ALL
Real numbers, not ALL possible y-values are used.

Figure 1.6: A Not onto function

1.5.7 Bijective Function


A Bijective, is a function that is both 1 − 1 and onto.
Example 1.5.12 Example (1.5.10) is 1 − 1 and Surjective, thus Bijective.
Example 1.5.13 Example (1.5.11) is not 1 − 1 and not ”onto”
Definition 1.5.4 Bijections are functions that are both injective and surjective.

Example 1.5.14 Let f : [0, ∞) → [0, ∞) be defined by f (x) = x. This function is an
injection and a surjection and so it is also a bijection.
Example 1.5.15 Describe the four functions below as injective, onto or bijective.

Figure 1.7: Mixed functions

(i) the function is neither injective nor surjective.


(ii) is a surjection, but not an injection

H.W-Kayondo & D.W-Ddumba, Engineering Math I- Lecture Notes 23


CHAPTER 1. FUNCTIONS

(iii) is an injection, but not a surjection.

(iv) is both a surjection and an injection, and therefore a bijection.

Example 1.5.16 Prove that the function f : N → N be defined by f (n) = n2 is injective.

Proof: Let a, b ∈ N be such that f (a) = f (b). This implies a2 = b2 by the definition of f .
Thus a = b or a = −b. Since the domain of f is the set of natural numbers (positive
integers), both a and b must be non-negative. Thus a = b. This shows

∀a, ∀b, [f (a) = f (b) ⇒ a = b]

which shows f is injective.

Example 1.5.17 Prove that the function g : N → N, defined by g(n) = n3 , is surjective.

Proof: Let n ∈ N. Notice that g(3n) = 3n3


= n.
Since 3n ∈ N, this shows n is in the range of g. Hence g is surjective.

Example 1.5.18 Prove that the function g : N → N, defined by g(n) = 0, is not


injective.

Proof: The numbers 1 and 2 are in the domain of g and are not equal, but g(1) = g(2) = 0.
Thus g is not injective.

Example 1.5.19 Prove that the function f : N → N be defined by f (n) = n2 , is not


surjective.

Proof: The number 3 is an element of the codomain, N. However, 3 is not the square of
any integer. Therefore, there is no element of the domain that maps to the number 3, so
f is not surjective.

1.5.8 Even Functions


A function f is called an even function if it maps the negative of x in the domain to the
same image as x. This can be defined as f (−x) = f (x) for all x in the domain.

Example 1.5.20 The function f (x) = x2 + 2 is even since

f (−x) = (−x)2 + 2 = x2 + 2 = f (x)

H.W-Kayondo & D.W-Ddumba, Engineering Math I- Lecture Notes 24


CHAPTER 1. FUNCTIONS

1.5.9 Odd Functions


A function f is called an Odd function if it maps the negative of x to the negative of f (x)
for all x in the domain. That is, if f (−x) = −f (x).
Example 1.5.21 The function f (x) = x3 + x is odd since

f (−x) = (−x)3 + (−x) = −x3 − x = −(x3 + x) = −f (x)

Example 1.5.22 Determine whether the function f (x) = x2 − x is even, odd or neither.

f (−x) = (−x)2 − (−x)


= x2 + x

f (−x) is neither equal to f (x) nor equal to −f (x). Therefore the function f is neither
even nor odd.
Example 1.5.23 The function f (x) = x2 is even, g(x) = x3 is an odd function but
h(x) = x + 1 is neither odd nor even.

1.5.10 Inverse of functions


An inverse of a relation that satisfies the definition of a function is called an inverse
function.

The inverse of a function f is denoted by f −1 .


Note 1.5.1 An inverse exists if the function is one-to-one and onto.

Let A and B be any sets and f be a function from A into B, that is, f : A → B. The
inverse function maps elements in B into those in A, that is f −1 : B → A. The domain
of the function f is the range of f −1 and the range of f is the domain of f −1 .
Definition 1.5.5 Suppose f : A → B is a bijection. Then the inverse of f , denoted

f −1 : B → A

is the function defined by the rule

f −1 (y) = x if and only if f (x) = y

Finding the Inverse Function


If we are given a function that is one-to-one and onto then we can find its inverse. If a
relation is defined by an equation, the inverse can be obtained by
1. interchanging x and y in the equation,

H.W-Kayondo & D.W-Ddumba, Engineering Math I- Lecture Notes 25


CHAPTER 1. FUNCTIONS

2. solving the new equation with y as the subject of the equation, that is, y = ... and

3. replacing y by f −1 (x).

This gives the inverse function.

Example 1.5.24 Find the inverse of the function f (x) = 2x + 4 if it exists.

This function is surely one-to-one and onto (verify this). It is therefore possible to find
its inverse. The function f (x) = 2x + 4 can be written as y = 2x + 4. Now, interchanging
x and y gives x = 2y + 4. Making y the subject of the equation gives y = x−4
2
. The inverse
function f −1 (x) = x−4
2
= 1
2
x − 2.

Example 1.5.25 Suppose f : R − (2) → R − (1) is defined by

x
f (x) =
x−2

Then the function


2x
f −1 (x) =
x−1

is the inverse of f . Show.

Exercise 1.5 Find the inverse of the function f : R − (2) → R − (1) defined by

x−1
f (x) =
x+2

Exercise 1.6 Find the inverse of the function f : R → (−∞, 1) defined by f (x) = 1−ex .

Theorem 1.5.2 Let A and B are any sets and the function f : A → B be one-to-one
and onto (the inverse function exists) (f −1 of ) : A → A is the identity function on A
similarly the composite function (f of −1 ) : B → B is the identity function on B.

Proof: Let a ∈ A. Then (f −1 of )(a) = f −1 (f (a)) = f −1 (b) for some b ∈ B. Now we


have f (a) = b which implies that f −1 (b) = a. It follows that (f −1 of )(a) = f −1 (f (a)) =
f −1 (b) = a and therefore that (f −1 of ) : A → A is the identity function on A.

Similarly if b ∈ B, then (f of −1 )(b) = f (f −1 (b)) = f (a) for some a satisfying f −1 (b) = a


and f (a) = b. Hence (f of −1 )(b) = f (f −1 (b)) = f (a) = b. Therefore (f of −1 ) is the identity
function on B.

Theorem 1.5.3 If a function is a bijection, then its inverse is also a bijection.

H.W-Kayondo & D.W-Ddumba, Engineering Math I- Lecture Notes 26


CHAPTER 1. FUNCTIONS

Example 1.5.26 The function f (x) = x2 does not have an inverse, since it is not a
bijection. Whenever required to compute an inverse, need to first check whether it is
bijective.
Exercise 1.7 Compute the inverse of the bijective function h(x) : R → R defined by

3x+2

x−1
, x 6= 1
h(x) =
3, x=1

1.5.11 Operations of functions


(1) Sums of functions
Given two functions g(x), f (x), their sum denoted by (f + g)(x) is defined as (f +
g) (x) = f (x) + g(x)
or (g + f ) (x) = g(x) + f (x)

Example 1.5.27 Given that f (x) = 3x and g(x) = x2 , then


(f + g) (x) = f (x) + g(x) = 3x + x2
or (g + f ) (x) = g(x) + f (x) = x2 + 3x

Example 1.5.28 Given that f (x) = 2x and g(x) = 3x, then

(f + g) (x) = f (x) + g(x) = 2x + 3x = 5x


or (g + f ) (x) = g(x) + f (x) = 3x + 2x = 5x

Example 1.5.29 Given f1 = 5x + 2 and f2 = x2 + 4. Find (f1 + f2 )(x).


(f1 + f2 )(x) = f1 (x) + f2 (x)
= 5x + 2 + x2 + 4
= x2 + 5x + 6

(2) Difference of functions


Given two functions g(x), f (x), their difference denoted by (f − g)(x) is defined as
(f − g) (x) = f (x) − g(x)
or (g − f ) (x) = g(x) − f (x)

Example 1.5.30 Given that f (x) = 3x and g(x) = x2 , then


(f − g) (x) = f (x) − g(x) = 3x − x2
or (g − f ) (x) = g(x) − f (x) = x2 − 3x

Example 1.5.31 Given that f (x) = 2x and g(x) = 3x, then

(f − g) (x) = f (x) − g(x) = 2x − 3x = −x


or (g − f ) (x) = g(x) − f (x) = 3x − 2x = x

H.W-Kayondo & D.W-Ddumba, Engineering Math I- Lecture Notes 27


CHAPTER 1. FUNCTIONS

1
Example 1.5.32 Let f (x) = 3x2 + 5 and f2 (x) = x
+ 2. Evaluate (f1 − f2 )(x).

(f1 − f2 )(x) = f1 (x) − f2 (x)


 
 2  1
= 3x + 5 − +2
x

1
= 3x2 − +3
x

(3) Product of functions


Given two functions g(x), f (x), their product denoted by (f.g)(x) or simplfy (f g)(x)
is defined as (f.g) (x) = f (x).g(x)
or (g.f ) (x) = g(x).f (x)

Example 1.5.33 Given that f (x) = 3x and g(x) = x2 , then


(f.g) (x) = f (x).g(x) = 3x.x2 = 3x3
or (g.f ) (x) = g(x).f (x) = x2 .3x = 3x3

Example 1.5.34 Given that f (x) = 2x and g(x) = 3x, then

(f.g) (x) = f (x).g(x) = 2x.3x = 6x2


or (g.f ) (x) = g(x).f (x) = 3x.2x = 6x2

Example 1.5.35 If f1 (x) = x2 − 4 and f2 (x) = x − 2 find (f1 · f2 )(x).

(f1 · f2 )(x) = f1 (x) · f2 (x)


= (x2 − 4)(x − 2)
= x3 − 2x2 − 4x + 8

(4) Quotient of functions


Given two functions g(x), f (x), their quotient denoted by (f /g)(x) is defined as
(f /g) (x) = f (x)/g(x)
or (g/f ) (x) = g(x)/f (x)

Example 1.5.36 Given that f (x) = 3x and g(x) = x2 , then


(f /g) (x) = f (x)/g(x) = 3x/x2 = 3/x
or (g.f ) (x) = g(x)/f (x) = x2 /3x = x/3

Example 1.5.37 Given that f (x) = 2x and g(x) = 3x, then

(f /g) (x) = f (x)/g(x) = 2x/3x = 2/3


or (g/f ) (x) = g(x)/f (x) = 3x/2x = 3/2 The quotient is sometimes referred to as a
rational function.

H.W-Kayondo & D.W-Ddumba, Engineering Math I- Lecture Notes 28


CHAPTER 1. FUNCTIONS

Example 1.5.38 If f1 (x) = x2 − 4 and f2 (x) = x − 2. Evaluate (f1 /f2 )(x).


(f1 /f2 )(x) = f1 (x) ÷ f2 (x)
= (x2 − 4) ÷ (x − 2)

(x + 2)(x − 2)
=
(x − 2)

= (x + 2)
The domain of this function in the set of real numbers excludes x = 2 since for
x = 2 we have f2 (x) = 0.
(5) Composite functions
Given two functions g(x), f (x), their composite function denoted by f og(x) is de-
fined as f og (x) = f (g(x))
or gof (x) = g (f (x))

Example 1.5.39 Given that f (x) = 3x and g(x) = x2 , then


f og (x) = f (x2 ) = 3x2
gof (x) = g (3x) = (3x)2 = 9x2

Example 1.5.40 Given that h(x) = 2x and g(x) = 5x, then


hog (x) = h (5x) = 2(5x) = 10x
goh (x) = g (2x) = 5(2x) = 10x

Example 1.5.41 Let f1 (x) = x2 + 3x + 2 and f2 (x) = x + 3. Compute and simplify


(if possible) (f2 of1 )(x) and (f1 of2 )(x).
(f2 of1 )(x) = f2 (f1 (x))
= f2 (x2 + 3x + 2)
= (x2 + 3x + 2) + 3
= x2 + 3x + 5
To compute (f1 of2 )(x), substitute f2 into f1
(f1 of2 )(x) = f1 (f2 (x))
= f1 (x + 3)
= (x + 3)2 + 3(x + 3) + 2
= x2 + 6x + 9 + 3x + 9 + 2
= x2 + 9x + 20
Note that in this case commutativity does not hold that is,
(f2 of1 )(x) 6= (f1 of2 )(x).

H.W-Kayondo & D.W-Ddumba, Engineering Math I- Lecture Notes 29


CHAPTER 1. FUNCTIONS

1
Example 1.5.42 Let f1 (x) = x
and f2 (x) = x2 . Compute and simplify (if possible)
(f2 of1 )(x) and (f1 of2 )(x).

(f2 of1 )(x) = f2 (f1 (x))


 
1
= f2
x

1
=
x2
whereas (f1 of2 )(x) = f1 (f2 (x))
= f1 (x2 )

1
= .
x2

Note that in this case the composite functions commute, that is,

(f2 of1 )(x) = (f1 of2 )(x)

Theorem 1.5.4 The composition of two injective functions is injective.

Domain of Composite, Difference, addition and product of func-


tions
Definition 1.5.6 If we denote a domain as D, we define the domains

(i) Df +g = Df −g = Df g = Df ∩ Dg

(ii) Df /g = (Df ∩ Dg ) \{x : g(x) = 0}

(iii) Df og = {x ∈ Dg : g(x) ∈ Df }. Since g(x) is a domain of f og, then domain of g is


domain of the composite, where g(x) is in domain of f og.

Note 1.5.2 Finding the domain of a composite function consists of two steps:

Step 1. Find the domain of the ”inside” (input) function. If there are any restrictions on
the domain, keep them.

Step 2. Construct the composite function. Find the domain of this new function. If
there are restrictions on this domain, add them to the restrictions from Step 1.
If there is an overlap, use the more restrictive domain (or the intersection of the
domains). The composite may also result in a domain unrelated to the domains
of the original functions.

H.W-Kayondo & D.W-Ddumba, Engineering Math I- Lecture Notes 30


CHAPTER 1. FUNCTIONS


Example 1.5.43 Let function f (x) = x + 1, function g(x) = x1 , and function h(x) =
x + 3. Find an equation defining each function and state the domain.
(1) f + g

√ 1
f +g = x+1+
x
Df : x ≥ −1 , Dg : x 6= 0 Df +g : [−1, 0) ∪ (0, +∞)

(2) f − g

√ 1
f −g = x+1−
x
Df : x ≥ −1 , Dg : x 6= 0 Df −g : [−1, 0) ∪ (0, +∞)

(3) f · g

  √
√ 1 x+1
f ·g = x+1· =
x x

Df : x ≥ −1 , Dg : x 6= 0 Df ·g : [−1, 0) ∪ (0, +∞)

(4) f /h


x+1
f /h =
x+3

Df : x ≥ −1 , Dh : <, h(x) 6= 0 → x 6= −3 Df /h : [−1, +∞)

Example 1.5.44 Given



f (x) = x2 + 2 and g(x) = 3−x

The composite functions

√ 2
f og(x) = f (g(x)) = 3−x +2=5−x
p √
gof (x) = g (f (x)) = 3 − (x2 + 2) = 1 − x2

The domain for g(x) = 3 − x is x ≤ 3.
The domain for f (g(x)) = 5 − x is all real numbers, but you must keep the domain of the

H.W-Kayondo & D.W-Ddumba, Engineering Math I- Lecture Notes 31


CHAPTER 1. FUNCTIONS

inside function. So the domain for the composite function is also x ≤ 3.

The domain for f (x) = x2 + 2 is all real numbers. √


The domain for the composite function g(f (x)) = 1 − x2 is −1 ≤ x ≤ 1. The input
function f (x) has no restrictions, so the domain of g(f (x)) is determined only by the
composite function. So the domain is −1 ≤ x ≤ 1.
Example 1.5.45 Find f og and gof and the domain of each, where

1−x 1
f (x) = and g(x) =
3x 1 + 3x

f og: Step 1. What is the domain of the inside function g(x) x 6= −1/3 Keep this!!
Step 2. The composite
1 3x

1 − 1+3x 1+3x
f (g(x)) = 1
 = 3 =x
3 1+3x 1+3x

This function puts no additional restrictions on the domain, so the composite domain is
x 6= −1/3.

gof : Step 1. What is the domain of the inside function f (x)? x 6= 0 Keep this!!
Step 2. The composite
1 1
g(f (x)) = 1−x = 1 = x
1 + 3 3x x

This function puts no additional restrictions on the domain, so the composite domain is
x 6= 0.
Example 1.5.46 Find f og and gof and the domain of each, where

3x 2
f (x) = and g(x) =
x−1 x

f og: Step 1. What is the domain of the inside function g(x) x 6= 0 Keep this!!
Step 2. The composite

2 6

3 x x 6
f (g(x)) = 2
 = 2−x = Domain : x 6= 2
x
−1 x
2−x

Combine this domain with the domain from Step 1: the composite domain is x 6= 0 and
x 6= 2

H.W-Kayondo & D.W-Ddumba, Engineering Math I- Lecture Notes 32


CHAPTER 1. FUNCTIONS

gof : Step 1. What is the domain of the inside function f (x)? x 6= 1 Keep this!!
Step 2. The composite

2 2(x − 1)
g(f (x)) = 3x = Domain : x 6= 0
x−1
3x

Combine this domain with the domain from Step 1: the composite domain is x 6= 1 and
x 6= 0
Example 1.5.47 Find f og and gof and the domain of each, where
√ √
f (x) = x − 2 and g(x) = x2 − 1
f og: Step 1. What is the domain of the inside function g(x) x ≥ 1 or x ≤ −1 Keep this!!
Step 2. The composite

q
f (g(x)) = x2 − 1 − 2
The domain of this function is where
√ √ √
x2 − 1 ≥ 2 ⇒ x2 − 1 ≥ 4 ⇒ x2 ≥ 5 ⇒ x ≥ 5 or x ≤ − 5
This function has a more restrictive domain than g(x), so the composite domain is
√ √
Df og := x ≥ 5 or x ≤ − 5
We can also show that
Dgof := x ≥ 3
Exercise 1.8 Find f og and gof and the domain of each for the following functions.

1. f (x) = x + 3 g(x) = 9 − x2

f og(x) = 9 − x2 + 3 Domain : − 3 ≤ x ≤ 3

gof (x) = −x − 6x
2 Domain : − 6 ≤ x ≤ 0

2. f (x) = x + 3 g(x) = 2x − 5

f og(x) = 2x − 2 Domain : x ≥ 1

gof (x) = 2 x + 3 − 5 Domain : x ≥ −3
−3 x
3. f (x) = x
g(x) = x−2

3(x − 2)
f og(x) = − Domain : x 6= 2 and x 6= 0
x
3
gof (x) = Domain : x 6= 0 and x 6= −3/2
3 + 2x

H.W-Kayondo & D.W-Ddumba, Engineering Math I- Lecture Notes 33


CHAPTER 1. FUNCTIONS


4. f (x) = x2 + 2 g(x) = x−5
f og(x) = x − 3 Domain : x ≥ 5
√ √ √
gof (x) = x2 − 3 Domain : x ≥ 3 or x ≤ − 3
2 5
5. f (x) = x−3
g(x) = x+2

2(x + 2)
f og(x) = − Domain : x 6= −2 and x 6= −1/3
3x + 1

5(x − 3)
gof (x) = Domain : x 6= 3 and x 6= 2
2x − 4

√ √
Example 1.5.48 Let f (x) = x + 3 and g(x) = 16 − x2 , find
(i) Df = [−3, ∞)
(ii) Dg = [−4, 4]
(iii) Df ∩ Dg = [−3, ∞) ∩ [−4, 4] = [−3, 4]
(iv) Df +g = Df −g = Df g = [−3, 4]
(v) Df /g = (Df ∩ Dg ) \{x : g(x) = 0} = [−3, 4]\{−4, 4} = [−3, 4)

Example 1.5.49 Given the functions f (x) = x and g(x) = x2 + 5

(i) (f og)(x) = f (g(x)) = f (x2 + 5) = x2 + 5
√ √
(ii) (gof )(x) = g (f (x)) = g( x) = ( x)2 + 5 = x + 5
(iii) Df og = {x ∈ Dg : g(x) ∈ Df }. Since Dg = R and g(x) ∈ Df , so Df og = R
(iv) Dgof = {x ∈ Df : f (x) ∈ Dg }. Since Df = x ≥ 0 = R+ = [0, ∞) and f (x) ∈ Dg , so
Df og = [0, ∞)
Example 1.5.50 Find (f og)(−2) given
f (x) = −3x + 2, g(x) = |x − 4|
Ans= -16 [Note that we only take |x − 4| = −(x − 4) since x = −2 < 4]
Example 1.5.51 Find (f og)(x) and the domain of f og, given

(x − 1) (x + 1)
f (x) = , g(x) =
(x + 2) (x − 2)

Ans: (f og)(x) = 3/(3x − 3). The domain of f og is: (−∞, 1) ∪ (1, 2) ∪ (2, +∞)

H.W-Kayondo & D.W-Ddumba, Engineering Math I- Lecture Notes 34


CHAPTER 1. FUNCTIONS


Example 1.5.52 Let function f (x) = x + 1, function g(x) = x1 , and function h(x) =
x + 3. Find gof oh and state the domain.

1
gof oh(x) = √ ; x > −4
x+4

1.5.12 Other functions


Definition 1.5.7 A function is said to be piecewise defined if its defined by applying
different formulas to the different parts of its domain.
Example 1.5.53 An example of a piecewise function in t,


 t − 1 , t ≤ −3
g(t) = 2t3 , −3 < t ≤ 9
4 − 6t, t > 9

Example 1.5.54 An example of a piecewise function in x,


x2 + 1 , x≥0
f (x) =
−x − 9 , x<0

Definition 1.5.8 A function f is said to be a power function if it has the form


f (x) = xn , for some n ∈ N.
Definition 1.5.9 A function f is said to be a quadratic function if it can be written
in the form
f (x) = Ax2 + Bx + C
Definition 1.5.10 A polynomial function f is one that can be written in the form

f (x) = an xn + an−1 xn−1 + · · · + a1 x + a0

where a0 , a1 , . . . , an are constants and n is a positive integer.


This integer n is then called the degree of the polynomial, provided an 6= 0.
Definition 1.5.11 A function g is said to be a rational function if it is of the type

f (x)
g(x) =
h(x)

where both f and h are polynomial functions.

H.W-Kayondo & D.W-Ddumba, Engineering Math I- Lecture Notes 35


CHAPTER 1. FUNCTIONS

Definition 1.5.12 A function f is said to be periodic, with period T , if

f (x + nT ) = f (x)

for all x in its domain and for n = 1, 2, . . .

That means the function repeats itself as the rate of T intervals of x. For example the
function f (x) = sin x is periodic with period 2π since sin(x + 2nπ) = sin x for all n = 1,
2, . .

Definition 1.5.13 A function f is said to be strictly monotonic increasing in the


interval (a, b) if, for all pairs of numbers x1 , x2 in (a, b),

f (x1 ) < f (x2 ) when x1 < x2

Definition 1.5.14 The floor function or greatest integer function is the function
defined as follows: for a real number x, the floor of x, denoted bxc, is the greatest integer
less than or equal to x.

The floor of x is sometimes referred to as the integer part or integral value of x. (Is the
integer just below - on the left)

Example 1.5.55

(i) b5c = 5 (iii) b0c = 0 (v) b−3.5c = −4

(ii) b1.78c = 1 (iv) b−1.6c = −2 (vi) b−99.9c = −100

Example 1.5.56
b1.7c = 1, bπc = 3 and b−3.2c = −4
The floor of x satisfies the following inequality.

bxc ≤ x < bxc + 1

Definition 1.5.15 The ceiling function or smallest integer function is the function
defined as follows: for a real number x, the ceiling of x, denoted dxe, is the smallest integer
not less than x. (The integer just above - on the right)

Example 1.5.57

(i) d5e = 5 (iii) d0e = 0 (v) d−3.5e = −3

(ii) d1.78e = 2 (iv) d−1.6e = −1 (vi) d−99.9e = −99

H.W-Kayondo & D.W-Ddumba, Engineering Math I- Lecture Notes 36


CHAPTER 1. FUNCTIONS

Example 1.5.58
d1.7e = 2, dπe = 4 and d−3.2e = −3
The ceiling of x satisfies the following inequality.
x ≤ dxe < x + 1
Definition 1.5.16 The sign function is the function, denoted sgn, defined as

 −1, x<0
sgn(x) = 0, x=0
1, x>0

Note that for any real number x = sgn(x)|x|. So if x 6= 0,

x
sgn(x) =
|x|

The sgn function is usually referred to as the signum function.


Example 1.5.59 Graph the curve

|x − 3|
f (x) = , x∈R
|x − 1|

Note that, we cannot solve and play around any mathematical expression with norms
(absolute, modulus), so we remove it by considering the positive and negative options for
each modulus

+(x − 3) (x − 3)
f1 (x) = : x ≥ 3, x ≥ 1 ⇒ f1 (x) = : x ∈ [3, ∞)
+(x − 1) (x − 1)

+(x − 3) (x − 3)
f2 (x) = : x ≥ 3, x < 1 ⇒ f2 (x) = − : x DNE
−(x − 1) (x − 1)

−(x − 3) (x − 3)
f3 (x) = : x < 3, x ≥ 1 ⇒ f3 (x) = − : x ∈ [1, 3)
+(x − 1) (x − 1)

−(x − 3) (x − 3)
f4 (x) = : x < 3, x < 1 ⇒ f4 (x) = : x ∈ (−∞, 1)
−(x − 1) (x − 1)

We sketch each function f1 , f2 , f3 , f4 within its region/domain. For example, for

(x − 3) x 3 4 5 6
f1 (x) = : x ∈ [3, ∞) ⇒ 1 2 3
(x − 1) f1 (x) 0 3 4 5

H.W-Kayondo & D.W-Ddumba, Engineering Math I- Lecture Notes 37


CHAPTER 1. FUNCTIONS

Sketching the functions f1 , f2 , f3 , f4 will result into the graph

f3 (x)
f4 (x)

f1 (x)

|x−3|
Figure 1.8: A graph of f (x) = |x−1|

Exercise 1.9 Graph the curve


|x + 6|
f (x) =
|x + 2|

Exercise 1.10 Graph the curve

|x + 3| − 4|x − 1|
f (x) =
|x − 2|

Exercise 1.11 Compute the domain of the function f (x) where


a x+b
f (x) =
x+c

where a, b, c ∈ R

H.W-Kayondo & D.W-Ddumba, Engineering Math I- Lecture Notes 38


Chapter 2

Limits of functions

Introduction
In this lecture, we try to analyze the behavior of a function around and near a point.
Examining the function as it approaches a point, from the left and from the right.

2.1 Informal definition of a limit of a function


Definition 2.1.1 We say that a number L is a limit of a function f (x) as x approaches
a number 0 a0 from either direction if limx→a f (x) from one direction is the same as
limx→a f (x) from the other direction. The statement can be abbreviated and written
as,
lim− f (x) = lim+ f (x) = lim f (x)
x→a x→a x→a

lim f (x) = L (2.1)


x→a

The words either direction in the statement are important.

Note 2.1.1 the number L depends only on the behavior of f (x) near x = a but not on
the functional value f (a). In fact we shall later learn that L may exist and yet f (a) is not
defined. This is an important property of limits of functions. Later you will learn that
for functions called continuous functions,

lim f (x) = f (a)


x→a

This is not true for any other function.

39
CHAPTER 2. LIMITS OF FUNCTIONS

Example 2.1.1 Determine the limit of the function



x2 + 1 , x<1
f (x) =
2x , x≥1

lim− f (x) = lim (x2 + 1) = 2 and lim f (x) = lim (2x) = 2


x→1 x→1 x→1+ x→1

Since
lim f (x) = 2 = lim+ f (x)
x→1− x→1

The limit exists


Example 2.1.2 Establish the limit of the function

x−3

x2
, x≥3
f (x) =
5x + 7 , x<3

x−3
 
lim f (x) = lim (5x + 7) = 22 and lim f (x) = lim =0
x→3− x→3 x→3+ x→3 x2

Since
lim f (x) 6= lim+ f (x)
x→3− x→3

The limit DNE


Example 2.1.3 Given
 2
 x +k x<1
f (x) =
x3 x≥1

Find the value of k such that lim f (x) exists and find it.
x→1

Since lim− f(x) = lim (x2 + k)


x→1 x→1

= 12 + k
= k+1
and lim+ f(x) = lim x3 = 13 = 1
x→1 x→1

H.W-Kayondo & D.W-Ddumba, Engineering Math I- Lecture Notes 40


CHAPTER 2. LIMITS OF FUNCTIONS

But for the limx→1 f (x) to exist


lim f (x) = lim f (x)
x→1− x→1+

i.e, k + 1 = 1
⇒ k = 1−1=0

and when k = 0, then lim f (x) = 1


x→1

Example 2.1.4 Show that the limit of the function h(x) = |c| doesn’t exist at x → 0.

Since

c, c≥0
|c| =
−c , c<0

Since lim− h(x) = lim (−c) = −c


x→0 x→0

and lim+ h(x) = lim (c) = c


x→0 x→0

But for the limx→0 h(x) to exist


lim h(x) = lim h(x)
x→0− x→0+

but, −c 6= c
thus limit doesnt exist (DNE).
Example 2.1.5 Determine the limit of the piecewise function

2x2 − 9 , x<3
f (x) =
3x , x≥3

From left of 3, the function, the road is f (x) = 2x2 − 9, and from the right of 3, we use
f (x) = 3x
lim f (x) = lim (2x2 − 9) = 9 and lim f (x) = lim (3x) = 9
x→3− x→3 x→3+ x→3

Since
lim f (x) = 9 = lim+ f (x)
x→3− x→3

The limit exists

H.W-Kayondo & D.W-Ddumba, Engineering Math I- Lecture Notes 41


CHAPTER 2. LIMITS OF FUNCTIONS

Example 2.1.6 Establish the limit of the function


2x − 3 , x≥2
f (x) = 2x
4+x2
, x<2

2x
From left of 2, the function, the road is f (x) = 4+x2
, and from the right of 2, we use
f (x) = 2x − 3

 
2x 1
lim− f (x) = lim = and lim f (x) = lim (2x − 3) = 1
x→2 x→2 4 + x2 2 x→2+ x→2

Since
lim f (x) 6= lim+ f (x)
x→2− x→2

The limit DNE

Example 2.1.7 Investigate the limit of the following function


x2 + 1 , x>0
f (x) =
4x , x≤0

From left of 0, the function, the road is f (x) = 4x, and from the right of 0, we use
f (x) = x2 + 1

lim− f (x) = lim (4x) = 0 and lim+ f (x) = lim (x2 + 1) = 1


x→0 x→0 x→0 x→0

Since
lim f (x) 6= lim+ f (x)
x→0− x→0

The limit DNE

Example 2.1.8 Find the limit of the mapping below at x = 0

|x|
f (x) =
x

x

x
, x≥0
f (x) =
− xx , x<0

H.W-Kayondo & D.W-Ddumba, Engineering Math I- Lecture Notes 42


CHAPTER 2. LIMITS OF FUNCTIONS

Which implies that



1, x≥0
f (x) =
−1 , x<0

whose limit we want to get as x tends to zero.

lim f (x) = lim −1 = −1 and lim f (x) = lim 1 = 1


x→0− x→0 x→0+ x→0

Since
lim f (x) 6= lim+ f (x)
x→0− x→0

The limit DNE

Example 2.1.9 Compute the limit of the function as x → −2

x2 − 1
f (x) =
x+2

Since the function is not defined at x = −2, the limit does not exist.

Example 2.1.10 Find


|x|
lim
x→0 x

We define

x, x≥0
|x| =
−x , x<0

Thus
lim x x≥0

= lim 1 = 1
x→0+ x x→0



|x| 
lim =
x→0 x 
 −x
 lim = lim −1 = −1 x<0


x→0 x x→0

|x|
Though this limit is finite, it is not unique (one and only one) We note that the limx→0 x
does not exist, since the left hand limit (−1) is not equal to the right hand limit (1)

H.W-Kayondo & D.W-Ddumba, Engineering Math I- Lecture Notes 43


CHAPTER 2. LIMITS OF FUNCTIONS

2.2 Computation of limits


2.2.1 Substitution Method
If a function remain analytic (’defined’ or ”denominator 6= 0” ) at x = a, we just substitute
in x as a.

Example 2.2.1 Compute the limits of the following functions

(i)

lim (4x2 − 1) = 4.22 − 1 = 15


x→2

we have simply substituted because 4x2 −1 is a polynomial function and polynomials


are analytic over the entire number line.

(ii)


π 2
limπ sin x = sin =
x→ 4 4 2

sin x is also analytic on R

(iii)
x2 + 2x + 1 11 + 2.1 + 1
   
4
lim = = =2
x→1 x+1 1+1 2

2
(In fact the rational function x +2x+1
x+1
is defined-analytic at all points except at the
poles but x = 1 is not a pole of this rational function,thus we merely substitute to
get the limit 2).

(iv)
x2 + 2 42 + 2
   
lim = =8
x→4 x−3 4−3

Example 2.2.2 Compute the following limits

(i) lim (4x + 6) = 6 (iii) lim cos 2x = 1


x→0 x→π

2x x−2
 
(ii) lim x+1
=1 (iv) lim x−3
DNE
x→1 x→3

H.W-Kayondo & D.W-Ddumba, Engineering Math I- Lecture Notes 44


CHAPTER 2. LIMITS OF FUNCTIONS

2.2.2 Numerator Factorisation [Non-analytic Technique]


When the functional value f (a) is not defined, not analytic,
or ”denominator = 0”.
we first factorise the function, cancel out terms, so that it becomes analytic and we just
substitute [(A)].

Example 2.2.3 Find the


x2 − 1
 
lim
x→1 x−1

Point x = 1 is a pole of this rational function. Hence function not defined at x = 1

x2 − 1 (x + 1)(x − 1)
 
therefore lim = lim
x→1 x−1 x→1 x−1

= lim (x + 1)
x→1

= 1+1=2

Example 2.2.4 Find the


x2 + 2x − 15
 
lim
x→3 x−3

The point x = 3 is a pole of this rational function. Hence function not defined - non
analytic, a need to first factorise the numerator.

x2 + 2x − 15 (x − 3)(x + 5)
 
therefore lim = lim
x→3 x−3 x→3 x−3

= lim (x + 5)
x→3

= 3+5=8

Example 2.2.5 Find the


x2 − 2x
 
lim
x→2 x−2

H.W-Kayondo & D.W-Ddumba, Engineering Math I- Lecture Notes 45


CHAPTER 2. LIMITS OF FUNCTIONS

The point x = 2 is a pole of this rational function.

x2 − 2x x(x − 2)
 
therefore lim = lim
x→2 x−2 x→2 x − 2

= lim (x)
x→2

= 2
Example 2.2.6 Find the
1
lim
x→1 x − 1

lim 1 does not exist Since 1


= 1
is not defined.
x→1 x−1 1−1 0

Example 2.2.7

x2 − 4 (x − 2)(x + 2)
 
lim = lim = lim (x + 2) = 4
x→2 x−2 x→2 (x − 2) x→2

Example 2.2.8

x2 − 5x + 6 (x − 2)(x − 3)
 
lim = lim = lim (x − 2) = 1
x→3 x−3 x→2 (x − 3) x→2

Example 2.2.9

x2 + 5x
 
x(x + 5)
lim = lim = lim (x + 5) = 5
x→0 x x→0 x x→0

Example 2.2.10

x2 − 1 (x − 1)(x + 1) (x + 1) 2
lim = lim = lim =
x→1 (x + 2)(x − 1) x→1 (x + 2)(x − 1) x→1 (x + 2) 3

Example 2.2.11 Find the limit below

(x2 − 5x + 6)
lim
x→4 (x − 4)

The limit DNE by methods of substitution and non-analytic functions.

H.W-Kayondo & D.W-Ddumba, Engineering Math I- Lecture Notes 46


CHAPTER 2. LIMITS OF FUNCTIONS

2.2.3 Infinity
Limits at infinity and infinite limits

Limits at infinity
When asked the limits at infinity, we first divide through the function by the highest
power of x, then find the limit of the function. [Recall limx→∞ xan = 0 where a,n are
constants] A function f (x) may approach a finite number L as x goes to infinity i.e as x
becomes very big but positive or negative.

Definition 2.2.1 we say that limx→∞ f (x) = L1 (finite number) if the value of f (x)
approaches L1 as x in creases beyond bound (becomes very big)
We also say that limx→−∞ f (x) = L2 (a finite number) means that the value of f (x)
approaches L2 as x decreases beyond bound (i.e becomes big in absolute value but negative
in sign.

Example 2.2.12 Find


x+1
(i) lim x
x→+∞

x+1
(ii) lim x
x→−∞

By dividing through by the highest power of x

(i)
 
x+1 1
lim = lim 1 +
x→+∞ x x→+∞ x

= 1+0=1

The idea is that we divide by the variable with the highest power to get the dominant
terms.

(ii)

 
x+1 1
lim = lim 1 +
x→−∞ x x→−∞ x

= 1+0=1

H.W-Kayondo & D.W-Ddumba, Engineering Math I- Lecture Notes 47


CHAPTER 2. LIMITS OF FUNCTIONS

Example 2.2.13 Find


x2 + 3x + 1
lim
x→∞ 3x2 + x + 2

The idea is to divide all through by x2 (variable with highest power)

3 1
x2 + 3x + 1 1+ x
+ x2
therefore lim = lim 1 2
x→∞ 3x2 + x + 2 x→∞ 3 + +
x x2

1+0+0
=
3+0+0
1
=
3

The interpretation is that as x blows beyond bound (becomes very big) the function

x2 + 3x + 1
f (x) =
3x2 + x + 2

1 1
approaches a finite number 3
or behaves like the straight line f (x) = 3

Example 2.2.14 Compute the

x2 + 4x + 1
 
lim
x→∞ −4x2 − 6

!
x2
x2 + 4x + 1 + 4x + x12
 
x2 x2
lim = lim −4x2
x→∞ −4x2 − 6 x→∞
x2
− x62

1 + x4 + x12
 
= lim
x→∞ −4 − x62
 
1+0+0
=
−4 − 0

1
= −
4

H.W-Kayondo & D.W-Ddumba, Engineering Math I- Lecture Notes 48


CHAPTER 2. LIMITS OF FUNCTIONS

Since

1 1 1 1 1 1
lim = , , , , ≈0
x→∞ x 10 1000 10000 100000000 1000000000000000
1 1 1 1 1
lim = , , , ≈0
x→∞ x2 100 1000000 100000000 100000000000000000000
1 −1 −1 −1 −1 −1
lim = , , , , ≈0
x→−∞ x 10 1000 10000 100000000 1000000000000000
1 −1 −1 −1 −1
lim 3
= , , , ≈0
x→−∞ x 1000 10000000 10000000000000 1000000000000000000

Example 2.2.15 Compute the

4x2 − 2x
 
lim
x→−∞ 7x3 + 8x2

!
4x2
4x2 − 2x − 2x
 
x3 x3
lim = lim 7x3 8x2
x→−∞ 7x3 + 8x2 x→−∞
x3
+ x3
4
− x22

x
= lim
x→−∞ 7 + x8

0−0
 
=
7+0

= 0
Example 2.2.16 Compute the

5 − 9x4
 
lim
x→∞ 2x + x3

4
!
5
5 − 9x4 − 9x
 
x4 x4
lim = lim 3
x→∞ 2x + x3 x→∞ 2x
x4
+ xx4

5
−9 0−9
   
x4
= lim 2 =
x→∞
x3
+ x1 0+0

= DNE

H.W-Kayondo & D.W-Ddumba, Engineering Math I- Lecture Notes 49


CHAPTER 2. LIMITS OF FUNCTIONS

Example 2.2.17 Compute the

x2 + 1
 
lim
x→∞ x

x2 + 1
   
1
lim = lim x +
x→∞ x x→∞ x

= (∞ + 0)
= ∞
DNE

Infinite limits
A function f (x) can blow beyond bound as x approaches a finite number.

Definition 2.2.2 We define

lim f (x) = +∞
x→a

If f (x) increases without bound as x approaches the number a We say that

lim f (x) = −∞
x→a

If f (x) decreases without bound as x approaches the number a. We say that

lim f (x) = +∞
x→+∞

If f (x) increases without bound as x increases without bound,


and the

lim f (x) = −∞
x→−∞

If f (x) decreases without bound as x decreases without bound.


In such cases we say that the limit does not exist.

H.W-Kayondo & D.W-Ddumba, Engineering Math I- Lecture Notes 50


CHAPTER 2. LIMITS OF FUNCTIONS

Example 2.2.18 Given the function

f (x) = tan x

Use the knowledge of curve sketching (asymptotes)

Figure 2.1: Graph of y = tan x

to find that

(i) lim
π−
f (x) = +∞
x→ 2

(ii) lim f (x) = −∞


x→− π2 +

Note 2.2.1 The curve above indicates that, some limits can actually be infinity, do not
exist.

Exercise 2.1 Evaluate the limit


x2 + 9
lim
x→∞ x+1

H.W-Kayondo & D.W-Ddumba, Engineering Math I- Lecture Notes 51


CHAPTER 2. LIMITS OF FUNCTIONS

Example 2.2.19 Given

x2
f (x) = ,
1 − x2

graph the function and hence find

(i) lim− f (x) (iii) lim f (x)


x→1 x→−1+

(ii) lim+ f (x) (iv) lim f (x)


x→1 x→−1−

The knowledge of Curve sketching is assumed. However, later in the lectures you will
learn how to sketch such curves of rational functions. Here we only present the sketch
without detail.

x2
Figure 2.2: Graph of y = 1−x2

x2 x2
(i) lim− 1−x2
= +∞ (iii) lim + 1−x2
= +∞
x→1 x→−1

x2 x2
(ii) lim+ 1−x2
= −∞ (iv) lim − 1−x2
= −∞
x→1 x→−1

H.W-Kayondo & D.W-Ddumba, Engineering Math I- Lecture Notes 52


CHAPTER 2. LIMITS OF FUNCTIONS

2.2.4 Using La’Hopital rule


The La’Hopital rule states that, if

f (x) 0 f (x) ∞
lim = or lim =±
x→a g(x) 0 x→a g(x) ∞

then
f (x) f 0 (x)
lim = lim 0
x→a g(x) x→a g (x)

Example 2.2.20 Compute

sin x 0 cos x 1
lim = [⇒ La’Hopital] = lim = =1
x→0 x 0 x→0 1 1

Example 2.2.21 Compute

x2 − x 0 2x − 1 −1
lim = [⇒ La’Hopital] = lim = = −1
x→0 x3 + x 0 x→0 3x2 + 1 1

Example 2.2.22 Compute the

(x2 − 9) 0 2x
lim = [⇒ La’Hopital] = lim =6
x→3 (x − 3) 0 x→3 1

Example 2.2.23 Use the La’Hopital technique to compute

(2x2 − 2) 0 4x
lim = [⇒ La’Hopital] = lim =4
x→1 (x − 1) 0 x→1 1

Example 2.2.24 Find the limit

x3 0 3x2 6x
lim = [⇒ La’Hopital] = lim = lim =0
x→0 4x2 0 x→0 8x x→0 8

You differentiate again and again, whenever La’Hopital applies, till cannot apply the the-
orem anymore.

H.W-Kayondo & D.W-Ddumba, Engineering Math I- Lecture Notes 53


CHAPTER 2. LIMITS OF FUNCTIONS

Example 2.2.25 Find the limit

 
sin x 0 h cos x i 1
lim = [⇒ La’Hopital] = lim =
x→0 2x 0 x→0 2 2

Exercise 2.2 By La’Hopital rule, show the following

sin2 x
lim = 0
x→0 x
1 − cos x
lim = 0
x→0 x

Example 2.2.26

sin πx sin y
lim = lim
x→0 πx y→0 y

cos y
= lim
y→0 1
= 1

Example 2.2.27 Applying the rule three times

2 sin x − sin 2x 2 cos x − 2 cos 2x


   
0
lim = [⇒ La’Hopital] = lim
x→0 x − sin x 0 x→0 1 − cos x

−2 sin x + 4 sin 2x
 
0
= [⇒ La’Hopital] = lim
0 x→0 sin x

−2 cos x + 8 cos 2x
 
0
= [⇒ La’Hopital] = lim
0 x→0 cos x

−2 + 8
=
1
= 6

Example 2.2.28

d 2
2 ln x 0 dx
(2 ln x) x
lim = [⇒ La’Hopital] = lim d
= lim =2
x→1 x − 1 0 x→1 (x − 1) x→1 1
dx

H.W-Kayondo & D.W-Ddumba, Engineering Math I- Lecture Notes 54


CHAPTER 2. LIMITS OF FUNCTIONS

Example 2.2.29

d
ex − 1 (ex − 1) ex
 
0 dx
lim = [⇒ La’Hopital] = lim d
= lim =∞
x→0 x2 0 x→0
dx
(x2 ) x→0 2x

Example 2.2.30

ex ∞ ex
lim = [⇒ La’Hopital] = lim =∞
x→∞ x ∞ x→∞ 1

Sometimes it is necessary to use La’Hopital’s Rule several times in the same problem:
Example 2.2.31

1 − cos x
 
0 sin x cos x 1
lim 2
= [⇒ La’Hopital] = lim = lim =
x→0 x 0 x→0 2x x→0 2 2

Occasionally, a limit can be re-written in order to apply La’Hopital’s Rule:


Example 2.2.32 LaHopital when it is ∞/∞

x3 + 3 ∞ 3x2
   
lim = [⇒ La’Hopital] = lim
x→∞ x2 + ex ∞ x→∞ 2x + ex

∞ 6x
= [⇒ La’Hopital] = lim
∞ x→∞ 2 + ex

∞ 6
= [⇒ La’Hopital] = lim x = 0
∞ x→∞ e

Example 2.2.33 Solve for the limit



x+2 −2
lim
x→2 x−2

This is usual La’Hopital rule 0/0.


x+2 −2 0 1
lim = [⇒ La’Hopital] = lim √
x→2 x−2 0 x→2 2 x+2

1
=
4

H.W-Kayondo & D.W-Ddumba, Engineering Math I- Lecture Notes 55


CHAPTER 2. LIMITS OF FUNCTIONS

Example 2.2.34

−x ∞ −1
lim −x
= [⇒ La’Hopital] = lim =0
x→−∞ e ∞ x→−∞ −e−x

Example 2.2.35

x2 + 2e−x ∞ 2x − 2e−x
lim = [⇒ La’Hopital] = lim
x→∞ 3x2 ∞ x→∞ 6x
∞ 2 + 2e−x
= [⇒ La’Hopital] = lim
∞ x→∞ 6
2
=
6

Example 2.2.36 Here is another example involving 0/0:

ex − 1 − x 0 ex − 1 0 ex 1
lim 2
= [⇒ La’Hopital] = lim = [⇒ La’Hopital] = lim =
x→0 x 0 x→0 2x 0 x→0 2 2

Example 2.2.37 This example involves ∞/∞. Assume n is a positive integer. Then

xn nxn−1 xn−1
lim xn e−x = lim = lim = n lim
x→∞ x→∞ ex x→∞ ex x→∞ ex

Repeatedly apply La’Hopital’s rule until the exponent is zero to conclude that the limit
is zero.

The most important thing to learn about La’Hopital’s rule is when it should not be
used:
(1). When the limits of the two parts are not both 0, or both infinite. In this case the
rule is likely to give a wrong answer! Example:

(cos x)
lim+
x→0 x

is positive infinity, because the numerator approaches 1 while the denominator ap-
proaches 0. If we incorrectly apply La’Hopital’s rule, we get

(− sin x)
lim+ =0
x→0 1

(2). When there is a better way to get the answer.

H.W-Kayondo & D.W-Ddumba, Engineering Math I- Lecture Notes 56


CHAPTER 2. LIMITS OF FUNCTIONS

2.2.5 Indeterminate Forms and La’Hopital’s Rule


With indeterminate forms, for limit evaluation cases (by substitution) there are competing
interests or rules and it’s not clear which will win out (thus answer not known).

Other than the


0 ∞
,
0 ∞

some other types of indeterminate forms are,


00 , 1∞ , ∞0 , ∞ − ∞ , (0)(∞)
These all have competing interests or rules that tell us what should happen and it’s just
not clear which, if any, of the interests or rules will win out. The topic of this section is
how to deal with these kinds of limits.

We deal with this kind of limits by introducing in a La’Hopital’s rule conditions, a quotient
function by
(i) creating in a denominator
(ii) having one denominator by LCM
(iii) taking logarithms (natural log) if function has powers in x (exponents).
Example 2.2.38 Evaluate the following limit.
lim x ln x
x→0+

Note that we really do need to do the right-hand limit here. We know that the natural
logarithm is only defined for positive x and so this is the only limit that makes any sense.

Now, in the limit, we get the indeterminate form (0)(∞). La’Hopital’s Rule won’t work
on products, it only works on quotients. However, we can turn this into a fraction if we
rewrite things a little.
ln x
lim+ x ln x = lim+
x→0 x→0 1/x

The function is the same, just rewritten, and the limit is now in the form ∞
and we can
now use La’Hopital’s Rule.

ln x 1/x
lim+ x ln x = lim+ = lim+ = lim (−x) = 0
x→0 x→0 1/x x→0 −1/x2 x→0+

H.W-Kayondo & D.W-Ddumba, Engineering Math I- Lecture Notes 57


CHAPTER 2. LIMITS OF FUNCTIONS

Example 2.2.39 Evaluate the following limit

lim xex
x→−∞

By using method of analytic at x = a (substitution), we get the indeterminate form


(−∞)(0). Thus we force in a La’Hopital’s rule. This means that we’ll need to write it as
a quotient. Moving the x to the denominator worked in the previous example so let’s try
that with this problem as well.

x ex ex ex ex
lim xe = lim = lim = lim = lim = ···
x→−∞ x→−∞ 1/x x→−∞ −1/x2 x→−∞ 2/x3 x→−∞ −1/x4

Hummmm ..... This doesn’t seem to be getting us anywhere. With each application
of La’Hopital’s Rule we just end up with another 0/0 indeterminate form and in fact
the derivatives seem to be getting worse and worse. Also note that if we simplified the
quotient back into a product we would just end up with either (−∞)(0) or (0)(∞) and
so that won’t do us any good.

This does not mean however that the limit can’t be done. It just means that we moved
the wrong function to the denominator. Let’s move the exponential function instead.

x x
lim xex = lim x
= lim −x
x→−∞ x→−∞ 1/e x→−∞ e

The quotient is now an indeterminate form of −∞/∞ and use La’Hopital’s Rule gives,

x x 1
lim xex = lim = lim −x = lim =0
x→−∞ x→−∞ 1/e x x→−∞ e x→−∞ −e−x

Example 2.2.40 Evaluate the following limit


1
lim x x
x→∞

In the limit this is the indeterminate form . We’re actually going to spend most of this
problem on a different limit. Let’s first define the following.
1
y = xx

Now, if we take the natural log of both sides we get,

1 ln x
ln y = ln x =
x x
H.W-Kayondo & D.W-Ddumba, Engineering Math I- Lecture Notes 58
CHAPTER 2. LIMITS OF FUNCTIONS

Let’s now take a look at the following limit.

ln x 1/x
lim ln y = lim = lim =0
x→∞ x→∞ x x→∞ 1

This limit was just a La’Hopital’s Rule problem and we know how to do those. So, what
did this have to do with our limit? Well first notice that,
1
lim x x = lim y = lim eln y
x→∞ x→∞ x→∞

We can now use the limit above to finish this problem.

1 lim ln y
lim x x = lim y = lim eln y = ex→∞ = e0 = 1
x→∞ x→∞ x→∞

Example 2.2.41 To evaluate a limit involving ∞ − ∞, convert the difference of two


functions to a quotient:

x ln x − x + 1
 
x 1 0
lim − = lim = [⇒ La’Hopital] (2.2)
x→1 x − 1 ln x x→1 (x − 1) ln x 0
x
ln x + x
−1 ln x
= lim = lim (2.3)
x→1 (x−1) + ln x x→1 (x−1) + ln x
x x

x ln x 0
= lim = [⇒ La’Hopital] (2.4)
x→1 (x − 1) + x ln x 0

ln x + xx 1 + ln x
= lim x = lim (2.5)
x→1 1 + ln x + x→1 1 + 1 + ln x
x

1 + ln x 1+0
= lim =
x→1 2 + ln x 2+0
1
=
2

where La’Hopital’s rule was applied in going from (2.2) to (2.3) and then again in going
from (2.4) to (2.5).

Note 2.2.2 La’Hopital’s rule can be used on indeterminate forms involving exponents
by using logarithms to ”move the exponent down”.

H.W-Kayondo & D.W-Ddumba, Engineering Math I- Lecture Notes 59


CHAPTER 2. LIMITS OF FUNCTIONS

Example 2.2.42 Here is an example involving the indeterminate form 00 :

x lim (x ln x)
lim+ xx = lim+ eln x = lim+ ex ln x = ex→0+
x→0 x→0 x→0

It is valid to move the limit inside the exponential function because the exponential
function is continuous. Now the exponent x has been ”moved down” But (using La’Hopital
rule)
lim+ x ln x = 0
x→0

Thus
lim xx = e0 = 1
x→0+

Example 2.2.43 Although La’Hopital’s rule is a powerful way of evaluating otherwise


hard-to-evaluate limits, it is not always the easiest way. Consider

1
lim x sin
|x|→∞ x

This limit may be evaluated using La’Hopital’s rule:

1 sin x1
lim x sin = lim
|x|→∞ x |x|→∞ 1/x

−x−2 cos x1
= lim
|x|→∞ −x−2

1
= lim cos
|x|→∞ x
 
1
= cos lim
|x|→∞ x

= 1.
It is valid to move the limit inside the cosine function because the cosine function is
continuous.

Another way to evaluate this limit is to use a substitution. y = 1/x. As |x| approaches
infinity, y approaches zero. So,

1 sin y
lim x sin = lim =1
|x|→∞ x y→0 y

H.W-Kayondo & D.W-Ddumba, Engineering Math I- Lecture Notes 60


CHAPTER 2. LIMITS OF FUNCTIONS

The example may be evaluated using La’Hopital’s rule or by noting that it is the definition
of the derivative of the sine function at zero.

Still another way to evaluate this limit is to use a Taylor series expansion:
 
1 1 1 1
lim x sin = lim x − 3
+ − ···
|x|→∞ x |x|→∞ x 3! x 5! x5

1 1
= lim 1 − 2
+ − ···
|x|→∞ 3! x 5! x4
 
1 1 1
= 1 + lim − + − ···
|x|→∞ x 3! x 5! x3

For |x| = 1, the expression in parentheses is bounded, so the limit in the last line is zero.

The other better and easier way, is to apply Sandwich theorem (to be seen in the next
section).
Example 2.2.44

4x3 +2x 4x3 +2x


   
lim
lim e x3 −1 = e x→∞ x3 −1
= e4
x→∞

Note 2.2.3 The lim and the exponential e, can always change positions.
Example 2.2.45 Compute the limit
 3x
1
lim 1 +
x→∞ 2x

This is an indeterminate form 1∞ . Since it is of the the exponent form (with powers in
x), we wil take the natural log
 3x
1
let y = 1+
2x
 
1
take logs ln y = 3x ln 1 +
2x
 
1
take lim lim ln y = lim 3x ln 1 +
x→∞ x→∞ 2x

= (∞)(0)

H.W-Kayondo & D.W-Ddumba, Engineering Math I- Lecture Notes 61


CHAPTER 2. LIMITS OF FUNCTIONS

Still an indeterminate form, but now without exponents, meaning we can force in a
La’Hopital by creating in a denominator

 
1
lim ln y = lim 3x ln 1 +
x→∞ x→∞ 2x
1

ln 1 + 2x 0
= 3 lim 1 = [⇒ La’Hopital]
x→∞
x
0

− 2x12 / 1 + 1

2x
= 3 lim
x→∞ − x12
 
1 1
= 3 lim 1+
x→∞ 2 2x

3
=
2

We were not looking for lim ln y, but for lim y, we get it by taking exponentials to
x→∞ x→∞

remove the natural log.

lim y = lim eln y


x→∞ x→∞

lim ln y
= e x→∞
3
= e2

Thus
 3x
1 3
lim 1 + = e2
x→∞ 2x

Exercise 2.3 Use La’Hopital’s Rule (for indeterminate forms) to evaluate the limit
(a)
2
lim (ln x) x
x→∞

(b)
1
lim (sin x) x
x→0

H.W-Kayondo & D.W-Ddumba, Engineering Math I- Lecture Notes 62


CHAPTER 2. LIMITS OF FUNCTIONS

2.3 Properties of limits


If

lim f (x) = L1 (a finite number)


x→a

and lim g(x) = L2 (a finite number),


x→a

then

(i) lim [f (x) + g(x)] = L1 + L2


x→a

(ii) lim [f (x) − g(x)] = L1 − L2


x→a

(iii) lim αf (x) = αL1 (α is a constant)


x→a

(iv) lim f (x)g(x) = L1 L2


x→a

f (x) L1
(v) lim = L2 6= 0
x→a g(x) L2

Example 2.3.1 Illustrate the given properties given

Let f(x) = x2 ,
g(x) = x2 − 1
therefore lim f (x) = 22 = 4 = L1
x→2

and lim g(x) = 22 − 1 = 3 = L2


x→2

Therefore

part(i)

lim [f (x) + g(x)] = lim (x2 + x2 − 1)


x→2 x→2

= lim (2x2 − 1)
x→2

= 8−1=7
= 4 + 3 = L1 + L2

H.W-Kayondo & D.W-Ddumba, Engineering Math I- Lecture Notes 63


CHAPTER 2. LIMITS OF FUNCTIONS

part(ii)
lim [f (x) − g(x)] = lim (x2 − x2 + 1)
x→2 x→2

= 4 − 3 = L1 − L2

part(iii) lim αf (x) = lim αx2 = 4α = αL1


x→2 x→2

part(iv)
lim f (x)g(x) = lim x2 (x2 − 1)
x→2 x→2

= lim (x4 − x2 )
x→2

= 16 − 4 = 12 = 4.3 = L1 L2

f (x) x2 4 L1
part(v) lim = lim 2 −1 = =
x→2 g(x) x→2 x 3 L2

Example 2.3.2 Illustrate the properties of limits (even for limits at infinity) if

2x − 1 3x
f (x) = , g(x) =
x x+1

2x − 1
therefore lim f (x) = lim
x→∞ x→∞ x
 
1
= lim 2 − = 2 = L1
x→∞ x

3x
and lim g(x) = lim
x→∞ x→∞ x + 1

 
3
= lim = 3 = L2
x→∞ 1 + x1

H.W-Kayondo & D.W-Ddumba, Engineering Math I- Lecture Notes 64


CHAPTER 2. LIMITS OF FUNCTIONS

part(i)

2x − 1
  
3x
lim [f (x) + g(x)] = lim +
x→∞ x→∞ x x+1

(x + 1)(2x − 1) + 3x2
 
= lim
x→∞ x(x + 1)

2x2 − x + 2x − 1 + 3x2
 
= lim
x→∞ x2 + x

5x2 + x − 1
 
= lim
x→∞ x2 + x

5 + x1 − 1
 
x2
= lim
x→∞ 1 + x1

= 5
= 2 + 3 = L1 + L2

part(ii)

2x − 1
 
3x
lim [f (x) − g(x)] = lim −
x→∞ x→∞ x x+1

(2x − 1)(x + 1) − 3x2


 
= lim
x→∞ x(x + 1)

2x2 + x − 1 − 3x2
 
= lim
x→∞ x2 + x

−x2 + x − 1
 
= lim
x→∞ x2 + x

−1 + x1 − 1
 
x2
= lim
x→∞ 1 + x1

= −1 = 2 − 3 = L2 − L3

H.W-Kayondo & D.W-Ddumba, Engineering Math I- Lecture Notes 65


CHAPTER 2. LIMITS OF FUNCTIONS

part(iii)

2x − 1
 
lim αf (x) = lim α
x→∞ x→∞ x
 
1
= α lim 2−
x→∞ x

= 2α = αL1

part(iv)

2x − 1 3x 6x − 3
   
lim . = lim
x→∞ x x+1 x→∞ x+1
3
6− x
= lim 1
x→∞ 1 +
x

= 6 = 2.3 = L1 L2

part(v)

!
  2x−1
f (x) x
lim = lim 3x
x→∞ g(x) x→∞
x+1

 2x−1 
x
= lim x+1
x→∞
3x

2x2 + x − 1
 
= lim
x→∞ 3x2
1 1
2+ x
− x2
= lim
x→∞ 3
2 L1
= =
3 L2

H.W-Kayondo & D.W-Ddumba, Engineering Math I- Lecture Notes 66


CHAPTER 2. LIMITS OF FUNCTIONS

Example 2.3.3 Given that

lim g(x) = 4
x→2

lim f (x) = 5
x→2

Compute

(i)

lim 4f (x)g(x) = 4 lim f (x)g(x)


x→2 x→2

h i
= 4 lim f (x) · lim g(x)
x→2 x→2

= 4(5)(4)
= 80

(ii)

lim [f (x) + g(x)] = lim f (x) + lim g(x)


x→2 x→2 x→2

= 5+4
= 9

(iii)


f (x)
 lim f (x)
x→2
lim =
x→2 3 + g(x) lim [3 + g(x)]
x→2

lim f (x)
x→2
=
lim 3 + lim g(x)
x→2 x→2

5
=
3+4
5
=
7

H.W-Kayondo & D.W-Ddumba, Engineering Math I- Lecture Notes 67


Chapter 3

Continuity of Functions

3.1 Informal definition of Continuity of a function


Definition 3.1.1 We start by stating what we call an informal definition of continuity
of a function. This clearly presents what exactly is meant by a function f (x) being
Continuous at the point x = a.
We say that a function f (x) is continuous at x = a if

(a) the lim f (x) exists ie


x→a

lim f (x) = lim+ f (x)


x→a− x→a

(b) the functional value f (a) exists.

(c) lim f (x) = f (a)


x→a

From the three conditions, it is sufficient to say that a function f (x) is continuous at
x = a if

lim f (x) = f (a)


x→a

However, if f (x) is not Continuous at x = a we say that it is discontinuous at x = a.

Note 3.1.1

1. Polynomial functions are continuous at all points on the real axis R

2. Rational functions are Continuous on the entire axis R except at the poles.

Note 3.1.2 If one of the conditions above fails, then the function is not continuous at
that point.

68
CHAPTER 3. CONTINUITY OF FUNCTIONS

Example 3.1.1 Show that the polynomial function f (x) = x2 + 2x + 1 is continuous at


x=2

Since

(a) limx→2 (x2 + 2x + 1) = 22 + 2(2) + 1 = 9 i.e exists

(b) f (2) = 22 + 2(2) + 1 = 9 i.e exists

(c) limx→2 f (x) = f (2) = 9

Hence f (x) = x2 + 2x + 1 is continuous at x = 2. In fact being a polynomial function, it


is continuous at all points on R

Example 3.1.2 Show that the rational function,

x+2
f (x) =
x−1

is continuous at x = 3

Since x = 3 is not a pole of the rational function for all substitution of 3 in the function,
the denominator does not go to zero. Checking through the conditions of Continuity,

(a)
x+2 3+2 5
lim = =
x→3 x − 1 3−1 2

(b)
3+2 5
f (3) = =
3−1 2

(c)
5
lim f (x) = f (3) =
x→3 2

x+2
Therefore f (x) = x−1
is continuous at x = 3, indeed f (x) is continuous at all points R
except x = 1.

H.W-Kayondo & D.W-Ddumba, Engineering Math I- Lecture Notes 69


CHAPTER 3. CONTINUITY OF FUNCTIONS

Example 3.1.3 Check whether the function


x2 − 1, x<3
f (x) =
2x + 2, x≥3

is continuous at x = 3
(a) lim f (x) ??
x→3

lim f (x) = lim (x2 − 1) = 8


x→3− x→3

lim f (x) = lim (2x + 2) = 8


x→3+ x→3

Thus the limit exists and equal to 8, that is

lim f (x) = 8
x→3

(b) f (3) = 2(3) + 2 = 8

(c)
lim f (x) = f (3) = 8
x→3

Therefore, the function is continuous.


Example 3.1.4 Check whether the function f (x) below is continuous at x = 1.


2x + 1, x ≥ 1
f (x) =
4x, x<1

(a) lim f (x)??


x→3

lim f (x) = lim (4x) = 4 & lim f (x) = lim (2x + 1) = 3


x→1− x→1 x→1+ x→1

Thus the limit does not exists.

(b) No need to check for f (1) = 2(1) + 1 = 3


Therefore, the function is not continuous, since one of the properties fails.

H.W-Kayondo & D.W-Ddumba, Engineering Math I- Lecture Notes 70


CHAPTER 3. CONTINUITY OF FUNCTIONS

Example 3.1.5 Given the function,



 x3 , x≤2
f (x) =
α − x, x>2

Find the scalar α for which f (x) is continuous at x = 2.

f (x) is continuous at x = 2 if lim f (x) exists and limx→2 f (x) = f (2). But for the lim f (x)
x→2 x→2

to exist,
lim f (x) = lim f (x)
x→2− x→2+

lim x3 = lim (α − x)
x→2 x→2

⇒8 = α−2
⇒ α = 10
Therefore when α = 10,
lim f (x) = f (2)
x→2

Implying that f (x) is continuous at x = 2


Example 3.1.6 Determine the value of k such that,
 2
 x − k2, x≤2
f (x) =
kx + 5, x>2

f (x) is continuous at x = 2.

f (x) is continuous at x = 2 if the lim f (x) to exist,


x→2

lim f (x) = lim f (x)


x→2− x→2+

lim x2 − k 2 = lim kx + 5
x→2 x→2

4 − k 2 = 2k + 5
⇒ k = −1

H.W-Kayondo & D.W-Ddumba, Engineering Math I- Lecture Notes 71


CHAPTER 3. CONTINUITY OF FUNCTIONS

Example 3.1.7 Modify the definition of f (x) such that it is continuous at the point
x = a if
x2 − 1
f (x) = (a 6= 1)
x−1

To modify is to redefine a function, so that it is defined everywhere, this is done by


defining the function, as its limits where was initially undefined.

x2 −1

 x−1
, a 6= 1
f (x) =
2, a=1

Note that the limit 2 has been got by finding the limit of f (x) by La’Hopital rule.
Example 3.1.8 Show that the function f (x) below is discontinuous at x = −2.

x3 + x − 2
f (x) = 3
x − x2 − 6x

We realise that, f (x) is not defined at x = −2, thus function is not continuous.
Example 3.1.9 Show that the function f (x) = x2 + 2x + 1 is continuous at x = 2

It is enough to show that it satisfies the three conditions of continuity.


Example 3.1.10 Determine if the following function is continuous at x = 1.

 3x − 5, if x 6= 1
f (x) =
7, if x = 1

(a) The limit


lim f (x) = lim (3x − 5) = −2
x→1 x→1

i.e.,
lim f (x) = −2
x→1

(b) But f (1) = 7, the function f (x) is defined at x = 1.


(c) Since lim f (x) 6= f (1), then condition (c) is not satisfied and function f is not con-
x→1

tinuous at x = 1.

H.W-Kayondo & D.W-Ddumba, Engineering Math I- Lecture Notes 72


CHAPTER 3. CONTINUITY OF FUNCTIONS

Example 3.1.11 Determine if the following function is continuous at x = −2.


 2
 x + 2x, if x ≤ −2
f (x) =
x3 − 6x, if x > −2

(a) The left-hand limit


lim − f (x) = lim − (x2 + 2x) = 0
x→−2 x→−2

The right-hand limit


lim f (x) = lim + (x3 − 6x) = 4
x→−2+ x→−2

Since the left-hand and right-hand limits are not equal, lim f (x) does not exist.
x→−2

(b) Although the function f is defined at x = −2 since


f (−2) = (−2)2 + 2(−2) = 4 − 4 = 0

Thus, function f is not continuous at x = −2 as the first condition failed.


Example 3.1.12 Determine if the following function is continuous at x = 0.

x−6

 x−3
,
if x < 0
f (x) = 2, if x = 0
 √ 2
4 + x , if x > 0

(a) The left-hand limit


x−6 −6
lim− f (x) = lim− = =2
x→0 x→0 x−3 −3

The right-hand limit


√ p
lim+ f (x) = lim+ 4 + x2 = 4 + (0)2 = 2
x→0 x→0

Thus, lim f (x) exists with lim f (x) = 2.


x→0 x→0

(b) The function f is defined at x = 0 sincef (0) = 2,


(c) Since lim f (x) = 2 = f (0), all three conditions satisfied ⇒ f is continuous at x = 0.
x→0

H.W-Kayondo & D.W-Ddumba, Engineering Math I- Lecture Notes 73


CHAPTER 3. CONTINUITY OF FUNCTIONS

Example 3.1.13 Check the following function for continuity at x = 3 and x = −3.

x3 −27


 x2 −9
, if x 6= 3
f (x) =
 9
 , if x = 3
2

Continuity at x = 3
(a) The limit (since function not a piecewise, we compute to test existence of a limit)

x3 − 27 ” ”
0
lim f (x) = lim =
x→3 x→3 x −9
2 ” 0”

(Circumvent this indeterminate form by factoring the numerator and the denomina-
tor).
x3 − 27 x3 − 33
lim f (x) = lim = lim
x→3 x→3 x2 − 9 x→3 x2 − 32

Recall that A2 − B 2 = (A − B)(A + B) and A3 − B 3 = (A − B)(A2 + AB + B 2 )

x3 − 33 (x − 3)(x2 + 3x + 9) x2 + 3x + 9 (3)2 + 3(3) + 9 9


lim f (x) = lim = lim = lim = =
x→3 x→3 x −3
2 2 x→3 (x − 3)(x + 3) x→3 x+3 (3) + 3 2

Could even have applied the La’Hopital rule to compute the limit above. i.e.,

9
lim f (x) =
x→3 2

9
(b) The function f is defined at x = 3 sincef (3) =
2

(c) Since,
9
lim f (x) = = f (3)
x→3 2

all three conditions are satisfied, and f is continuous at x = 3. Now, check for continuity
at x = −3.
Continuity at x = −3
Function f is not defined at x = −3 because of division by zero. Thus, f (−3) does not
exist, condition (b) is violated, and thus f is not continuous at x = −3.

H.W-Kayondo & D.W-Ddumba, Engineering Math I- Lecture Notes 74


CHAPTER 3. CONTINUITY OF FUNCTIONS

Example 3.1.14 Show that the function f (x) = sin x is continuous at all numbers x.

First, f is defined for all x ∈ R. Let a be an arbitrary real number. Then

lim f (a + h) = lim sin(a + h)


h→0 h→0

= lim [sin a cos h + cos a sin h]


h→0

   
= (sin a) lim cosh + cos a lim sin h
h→0 h→0

= (sin a)(1) + cos a(0)


= sin a
= f (a)

Since a was arbitrary f is continuous on R.

x2 + 3x + 5
Example 3.1.15 For what values of x is the function f (x) = continuous?
x2 + 3x − 4

Functions y = x2 + 3x + 5 and y = x2 + 3x − 4 are continuous for all values of x since


x2 + 3x + 5
both are polynomials. Thus, the quotient of these two functions, f (x) = is
x2 + 3x − 4
continuous for all values of x where the denominator, y = x2 + 3x − 4 = (x − 1)(x + 4)
does NOT equal zero. Since (x − 1)(x + 4) = 0 for x = 1 and x = −4, function f is
continuous for all values of x except x = 1 and x = −4.
Example 3.1.16 For what values of x is the function g(x) = (sin(x20 +5))1/3 continuous?

1
First describe function g using functional composition. Let f (x) = x 3 , h(x) = sin x, and
k(x) = x20 + 5. Function k is continuous for all values of x since it is a polynomial, and
functions f and h are well-known to be continuous for all values of x. Thus, the functional
compositions
h(k(x)) = sin(k(x)) = sin(x20 + 5)
and
f (h(k(x))) = (h(k(x)))1/3 = (sin(x20 + 5))1/3
are continuous for all values of x. Since

g(x) = (sin(x20 + 5))1/3 = f (h(k(x)))

function g is continuous for all values of x.

H.W-Kayondo & D.W-Ddumba, Engineering Math I- Lecture Notes 75


CHAPTER 3. CONTINUITY OF FUNCTIONS


Example 3.1.17 For what values of x is the function f (x) = x2 − 2x continuous ?

First describe function f using functional composition. Let g(x) = x2 −2x and h(x) = x
Function g is continuous for all values of x since it is a polynomial, and function h is well-
known to be continuous for x ≥ 0. Since g(x) = x2 − 2x = x(x − 2), it follows easily that
g(x) ≥ 0 for x ≤ 0 and x ≥ 2. Thus, the functional composition

p √
h(g(x)) = g(x) = x2 − 2x

is continuous for x ≤ 0 and x ≥ 2. Since



f (x) = x2 − 2x = h(g(x))

function f is continuous for x ≤ 0 and x ≥ 2.

x − 1
Example 3.1.18 For what values of x is the function f (x) = ln continuous?
x+2

x−1
First describe function f using functional composition. Let g(x) = and h(x) = ln x
x+2
Since g is the quotient of polynomials y = x − 1 and y = x + 2, function g is continuous for
all values of x except where x + 2 = 0, i.e., except for x = −2. Function h is well-known
x−1
to be continuous for x > 0. Since g(x) = , it follows easily that g(x) > 0 for x < −2
x+2
and x > 1. Thus, the functional composition

x − 1
h(g(x)) = ln (g(x)) = ln
x+2

is continuous for x < −2 and x > 1. Since

x − 1
f (x) = ln = h(g(x))
x+2

function f is continuous for x < −2 and x > 1

[The ln is not defined at negative values. But also the quotient is always taken seriously
with the denominator]

H.W-Kayondo & D.W-Ddumba, Engineering Math I- Lecture Notes 76


CHAPTER 3. CONTINUITY OF FUNCTIONS

esin x
Example 3.1.19 For what values of x is the function f (x) = √ continuous?
4 − x2 − 9

First describe function f using functional composition. Let g(x) = sin x and h(x) = ex ,
both of which are well-known to be continuous for all values of x. Thus, the numerator
y = esin x = h(g(x)) is continuous (the functional composition
√ of continuous functions) for
all values of x. Now consider the denominator y = 4 − x2 − 9. Let

g(x) = 4, h(x) = x2 − 9, and k(x) = x

Functions g and h are continuous for all values of x since both are polynomials, and it is
well-known that function k is continuous for x ≥ 0. Since h(x) = x2 −9 = (x−3)(x+3) = 0
when√x = 3 or x = −3, it follows easily that h(x) ≥ 0 for x ≥ 3 and x ≤ −3, so that
y = x2 − 9 = k(h(x)) is continuous (the functional composition√ of continuous functions)
for x ≥ 3 and x ≤ −3. Thus, the denominator y = 4 − x2 − 9 is continuous (the
difference of continuous functions) for x ≥ 3 and x ≤ −3.

There is one other important consideration. We must insure that the denominator is
never zero. If √
y = 4 − x2 − 9 = 0
then √
4= x2 − 9
Squaring both sides, we get
16 = x2 − 9
so that
x2 = 25
when
x = 5 or x = −5
Thus, the denominator is zero if x = 5 or x = −5. Summarizing, the quotient of these
esin x
continuous functions, f (x) = √ , is continuous for x ≥ 3 and x ≤ −3, but not
4 − x2 − 9
for x = 5 and x = −5.

H.W-Kayondo & D.W-Ddumba, Engineering Math I- Lecture Notes 77


CHAPTER 3. CONTINUITY OF FUNCTIONS

Example 3.1.20 For what values of x is the following function continuous ?

x−1

 √ if x > 1
x−1







f (x) = 5 − 3x, if − 2 ≤ x ≤ 1




 6


if x < −2

x−4

Consider separately the three component functions which determine f . Function

x−1
y=√
x−1

is continuous for x > 1 since it is the quotient of continuous functions and the denominator
is never zero.

Function y = 5 − 3x is continuous for −2 ≤ x ≤ 1 since it is a polynomial.

Function
6
y=
x−4

is continuous for x < −2 since it is the quotient of continuous functions and the denomi-
nator is never zero.

Now check for continuity of f where the three components are joined together, i.e., check
for continuity at x = 1 and x = −2.

For x = 1:

(a) The right-hand limit


” ”
x−1 0
lim+ f (x) = lim+ √ =” ”
x→1 x→1 x−1 0

Circumvent this indeterminate form one of two ways. Either factor the numerator as
the difference of squares, or multiply by the conjugate of the denominator over itself.

√ √ √
( x)2 − (1)2 ( x − 1)( x + 1) √ √
= lim+ √ = lim+ √ = lim+ ( x + 1) = ( 1 + 1) = 2
x→1 x−1 x→1 x−1 x→1

H.W-Kayondo & D.W-Ddumba, Engineering Math I- Lecture Notes 78


CHAPTER 3. CONTINUITY OF FUNCTIONS

or applying the La’Hopital rule.

The left-hand limit

lim f (x) = lim− (5 − 3x) = 5 − 3(1) = 2


x→1− x→1

Thus,
lim f (x) = 2
x→1

(b) function f is defined since f (1) = 5 − 3(1) = 2.

(c) Since
lim f (x) = 2 = f (1)
x→1

all three conditions are satisfied, and function f is continuous at x = 1.

For x = −2:

(a) The right-hand limit

lim f (x) = lim + (5 − 3x) = 5 − 3(−2) = 11


x→−2+ x→−2

The left-hand limit

6 6 6
lim − f (x) = lim − = = = −1
x→−2 x→−2 x−4 (−2) − 4 −6

Since the left- and right-hand limits are different,

lim f (x)
x→−2

does not exist,

(b) Although the function f is defined at x = −2 since f (−2) = 5 − 3(−2) = 11

condition (a) is violated, and function f is not continuous at x = −2.

Summarizing, function f is continuous for all values of x except x = −2.

H.W-Kayondo & D.W-Ddumba, Engineering Math I- Lecture Notes 79


CHAPTER 3. CONTINUITY OF FUNCTIONS

Example 3.1.21 Determine all values of the constant A so that the following function
is continuous for all values of x.
 2
 A x − A, if x ≥ 3
f (x) =
4, if x < 3

First, consider separately the two components which determine function f .

Function y = A2 x − A is continuous for x ≥ 3 for any value of A since it is a polynomial.

Function y = 4 is continuous for x < 3 since it is a polynomial.

Now determine A so that function f is continuous at x = 3.


(a) The right-hand limit

lim f (x) = lim+ (A2 x − A) = 3A2 − A


x→3+ x→3

The left-hand limit


lim f (x) = lim− 4 = 4
x→3− x→3

For the limit to exist, the right- and left-hand limits must exist and be equal. Thus,

lim f (x) = 3A2 − A = 4 ⇒


x→3

4
3A2 − A − 4 = 0 ⇒ (3A − 4)(A + 1) = 0 ⇒ A = or A = −1
3

(b) The function is defined at x = 3

f (3) = A2 (3) − A = 3A2 − A

For either choice of A,

(c)
lim f (x) = 4 = f (3)
x→3

all three conditions are satisfied, and f is continuous at x = 3. Therefore, function f is


4
continuous for all values of x if A = or A = −1
3

H.W-Kayondo & D.W-Ddumba, Engineering Math I- Lecture Notes 80


CHAPTER 3. CONTINUITY OF FUNCTIONS

Example 3.1.22 Determine all values of the constants A and B so that the following
function is continuous for all values of x.


 Ax − B, if x ≤ −1



f (x) = 2x2 + 3A + B, if − 1 < x ≤ 1




4, if x > 1

First, consider separately the three components which determine function f .

Function y = Ax − B is continuous for x ≤ −1 for any values of A and B since it is a


polynomial.

Function y = 2x2 + 3Ax + B is continuous for −1 < x ≤ 1 for any values of A and B
since it is a polynomial.

Function y = 4 is continuous for x > 1 since it is a polynomial.

Now determine A and B so that function f is continuous at x = −1 and x = 1.

Continuity at x = −1:

(a) The left-hand limit

lim f (x) = lim − (Ax − B) = A(−1) − B = −A − B


x→−1− x→−1

The right-hand limit

lim f (x) = lim + (2x2 + 3Ax + B) = 2(−1)2 + 3A(−1) + B = 2 − 3A + B


x→−1+ x→−1

For the limit to exist, the right- and left-hand limits must exist and be equal. Thus,

lim f (x) = −A − B = 2 − 3A + B
x→−1

so that
2A − 2B = 2
or

A−B = 1 (3.1)

H.W-Kayondo & D.W-Ddumba, Engineering Math I- Lecture Notes 81


CHAPTER 3. CONTINUITY OF FUNCTIONS

(b) The function will be defined at x = −1 as f (−1) = A(−1) − B = −A − B

Now consider Continuity at x = 1:

(a) The left-hand limit

lim f (x) = lim− (2x2 + 3Ax + B) = 2(1)2 + 3A(1) + B = 2 + 3A + B


x→1− x→1

The right-hand limit


lim f (x) = lim+ 4 = 4
x→1+ x→1

For the limit to exist, the right- and left-hand limits must exist and be equal. Thus,

lim f (x) = 2 + 3A + B = 4
x→1

or

3A + B = 2 (3.2)

(b) The function will be defined at x = 1 since f (1) = 2(1)2 + 3A(1) + B = 2 + 3A + B

For continuity at both x = −1 and x = 1, we solve Equations (3.1) and (3.2) simultane-
ously. Thus,

3
A =
4
1
B = −
4

For this choice of A and B it can easily be shown that

lim f (x) = 4 = f (1) and


x→1

1
lim f (x) = − = f (−1)
x→−1 2

so that all three conditions are satisfied at both x = 1 and x = −1, and function f is
continuous at both x = 1 and x = −1. Therefore, function f is continuous for all values
of x if
3 1
A= and B = −
4 4

H.W-Kayondo & D.W-Ddumba, Engineering Math I- Lecture Notes 82


CHAPTER 3. CONTINUITY OF FUNCTIONS

Example 3.1.23 Show that the following function is continuous for all values of x.

 −1
 e x2 , if x 6= 0
f (x) =
0, if x = 0

First describe f using functional composition. Let g(x) = − x12 and h(x) = ex . Function
h is well-known to be continuous for all values of x.

Function g is the quotient of functions continuous for all values of x, and is therefore
continuous for all values of x except x = 0, that x which makes the denominator zero.
Thus, for all values of x except x = 0,

1
f (x) = h(g(x)) = eg(x) = e− x2

is a continuous function (the functional composition of continuous functions).

Now check for continuity of f at x = 0. Function f is defined at x = 0 since


(a) The limit
−1 −1
lim = = −∞
x→0 x2 0+

The numerator approaches −1 and the denominator is a positive number approaching


zero.

so that
” ” ” ”
2 1 1
lim f (x) = lim e−1/x = e−∞ = ∞
= =0
x→0 x→0 e ∞

i.e.,
lim f (x) = 0
x→0

(b)
f (0) = 0

(c)
lim f (x) = 0 = f (0)
x→0

all three conditions are satisfied, and f is continuous at x = 0. Thus, f is continuous for
all values of x.

H.W-Kayondo & D.W-Ddumba, Engineering Math I- Lecture Notes 83


CHAPTER 3. CONTINUITY OF FUNCTIONS

Example 3.1.24 Assume that

x2 sin x1
 
, x 6= 0
f (x) =
0, x=0

Show that f is continuous at x = 0.

Recall that function f is continuous at x = 0 if

(a) lim f (x) exists, (b) f (0) is defined (exists), and


x→0

(c) lim f (x) = f (0).


x→0

First note that it is given that

(a) Use the Squeeze Principle to compute

lim f (x)
x→0

For x 6= 0 we know that


1
−1 ≤ sin ≤ +1
x

so that
1
2 2
−x ≤ x sin ≤ x2
x

Since
lim (−x2 ) = 0 = lim x2
x→0 x→0

it follows from the Squeeze Principle that

1
2
lim f (x) = lim x sin =0
x→0 x→0 x

(b) f (0) = 0.

(c) Finally, lim f (x) = 0 = f (0),


x→0

confirming that function f is continuous at x = 0.

H.W-Kayondo & D.W-Ddumba, Engineering Math I- Lecture Notes 84


CHAPTER 3. CONTINUITY OF FUNCTIONS

Example 3.1.25 Determine whether the following function is continuous


 3x − 5 ; x 6= 1
f (x) =
2 ; x=1

at x = 1

lim = lim (3x − 5) = −2


x→1 x→1

f (1) = 2

Since are not the same, the function is not continuous.


Example 3.1.26 Determine the values of constants a, b so that the function f (x)



 2ax + b ; x < 3



f (x) = ax + 3b ; x > 3




10 ; x=3

is continuous at x = 3

The limits at a point should be equal and equal to the value of the function at that point.

lim = lim (2ax + b) = 6a + b


x→3− x→3

lim = lim (ax + 3b) = 3a + 3b


x→3+ x→3

f (3) = 10

All the three equations above should be equal, i.e

6a + b = 10
3a + 3b = 10

Solving simultaneously gives

4
a =
3
b = 2

H.W-Kayondo & D.W-Ddumba, Engineering Math I- Lecture Notes 85


CHAPTER 3. CONTINUITY OF FUNCTIONS

Example 3.1.27 Find the values of aand b for which the function



 3x − 6a , x<1



f (x) = 2ax − b , 1 ≤ x ≤ 3




x − 2b , 3<x

is continuous at 1 and 3

To be continuous at x = 1

lim f (x) = lim f (x)


x→1− x→1+

lim (3x − 6a) = lim (2ax − b)


x→1 x→1

3 − 6a = 2a − b (3.3)

To be continuous at x = 3

lim f (x) = lim f (x)


x→3− x→3+

lim (2ax − b) = lim (x − 2b)


x→3 x→3

6a − b = 3 − 2b (3.4)

Solving the two equations (3.3) and (3.4) simultaneously

8a − b = 3
6a + b = 3

to have

3
a =
7
3
b =
7

H.W-Kayondo & D.W-Ddumba, Engineering Math I- Lecture Notes 86


CHAPTER 3. CONTINUITY OF FUNCTIONS

Example 3.1.28 Determine the values of constants a, b so that the function f (x)


 a + bx ; x > 2



f (x) = 3 ; x=2




b − ax2 ; x < 2

is continuous at x = 2

The limits at a point should be equal and equal to the value of the function at that point.
lim = lim (b − ax2 ) = b − 4a
x→2− x→2

lim = lim (a + bx) = a + 2b


x→2+ x→2

f (2) = 3
All the three equations above should be equal, i.e
b − 4a = 3
a + 2b = 3
Solving simultaneously gives
1 5
a=− , b=
3 3

x2 + 1
Example 3.1.29 Determine if the function h(x) = is continuous at x = −1
x3 + 1

Function h is not defined at x = −1 since it leads to division by zero. Thus,


h(−1) does not exist, condition (b) is violated, and function h is not continuous at x = −1.
Theorem 3.1.1 Let f be continuous at a point x = a in the domain of f and let g be
continuous at a point f (a) in its domain. Then the composite function gof is continuous
at x = a.
Theorem 3.1.2 Let f be defined on an open interval containing the number a. f is
continuous at a if and only if
lim f (a + h) = f (a)
x→a

Because of the if and only if this statement can be used as an alternative definition of
continuity.

H.W-Kayondo & D.W-Ddumba, Engineering Math I- Lecture Notes 87


CHAPTER 3. CONTINUITY OF FUNCTIONS

3.1.1 Removable discontinuity


There are discontinuities which can be removed by redefining the function.

Definition 3.1.2 If L− and L+ at x0 exist, are finite, and are equal to L = L− = L+ .


Then, if f (x0 ) is not equal to L, x0 is called a removable discontinuity. This discontinuity
can be ”removed to make f continuous at x0 ”.

Removable discontinuity ≡ A hole in a graph. That is, a discontinuity that can be


”repaired” by filling in a single point. In other words, a removable discontinuity is a point
at which a graph is not connected but can be made connected by filling in a single point.

Example 3.1.30 Show that the function

x2 − 9
f (x) =
x−3

Is discontinuous at x = 3

x2 −9 0
Checking through conditions of continuity, we have for f (x) = x−3
⇒ f (3) = 0
is not
defined . Hence f (x) must be discontinuous at x = 3.
But since the

x2 − 9 (x + 3)(x − 3)
 
lim f (x) = lim = lim = 6 exists
x→3 x→3 x−3 x→3 x−3

and for continuity of f (x) at x = 3

lim f (x) = f (3)


x→3

Thus if we redefine f (x) at x = 3 we can remove the discontinuity at x = 3. In fact if


f (x) = 6 at x = 3, then the function becomes continuous at x = 3 i.e

x2 −9

 x−3
, x 6= 3
f (x) =
6, x=3

is continuous at x = 3

Such discontinuity which can be removed by redefining the function at the discontinuity
are called removable discontinuities

H.W-Kayondo & D.W-Ddumba, Engineering Math I- Lecture Notes 88


CHAPTER 3. CONTINUITY OF FUNCTIONS

Example 3.1.31 Redefine the function

1 − cos2 x
f (x) =
sin x

so that it is continuous at x = 0

Since

1 − cos2 x sin2 x
 
lim = lim = lim (sin x) = 0
x→0 sin x x→0 sin x x→0

f (x) to be continuous at x = 0,
lim f (x) = f (x0 ) ⇒ f (0) = lim f (x)
x→x0 x→0

Therefore if we redefine f (0) = 0 i.e


 1−cos2 x
 ( sin x ), x 6= 0
f (x) =
0, x=0

then f (x) is continuous at x = 0.


Example 3.1.32 Redefine the function

x2 − 5x + 6
f (x) =
(x − 2)

so that it is continuous at x = 2.

The function is not defined at x = 2. The function is continuous if

x2 − 5x + 6 (x − 2)(x − 3)
f (2) = lim f (x) = lim = lim = lim (x − 3) = −1
x→2 x→2 (x − 2) x→2 (x − 2) x→2

Thus the redefined continuous function is

x2 −5x+6

 (x−2)
, if x 6= 2
f (x) =
−1,

if x = 2

Such discontinuity is termed as removable discontinuity.

H.W-Kayondo & D.W-Ddumba, Engineering Math I- Lecture Notes 89


CHAPTER 3. CONTINUITY OF FUNCTIONS

Example 3.1.33 Consider the function



 x2 for x < 1



f (x) = 0 for x = 1




2−x for x > 1

Then, the point x0 = 1 is a removable discontinuity.

3.1.2 Jump discontinuity


Definition 3.1.3 The limits L− and L+ exist and are finite, but not equal. Then, x0
is called a jump discontinuity or step discontinuity. For this type of discontinuity, the
function f may have any value in x0 .

Example 3.1.34 Consider the function


 x2 for x < 1
f (x) = 0 for x = 1
2 − (x − 1)2 for x > 1

Then, the point x0 = 1 is a jump discontinuity.

Figure 3.1: The function in Example 3.1.34, with a jump discontinuity at x0 = 1

H.W-Kayondo & D.W-Ddumba, Engineering Math I- Lecture Notes 90


CHAPTER 3. CONTINUITY OF FUNCTIONS

3.1.3 Essential discontinuity


Definition 3.1.4 One or both of the limits L− and L+ does not exist or is infinite.
Then, x0 is called an essential discontinuity, or infinite discontinuity.

Example 3.1.35 Consider the function

5


 sin x−1 for x < 1



f (x) = 0 for x = 1




 0.1
x−1
for x > 1

Then, the point x0 = 1 is an essential discontinuity (sometimes called infinite discontinu-


ity).

For it to be an essential discontinuity, it would have sufficed that only one of the two
one-sided limits did not exist or were infinite.

However, given this example the discontinuity is also an essential discontinuity for the
extension of the function into complex variables.

Figure 3.2: The function in example 3.1.35, is an essential discontinuity

H.W-Kayondo & D.W-Ddumba, Engineering Math I- Lecture Notes 91


CHAPTER 3. CONTINUITY OF FUNCTIONS

3.1.4 Formal definition of continuity of function f (x) at x = a


Definition 3.1.5 A function f (x) is continuous at x = a if and only if given  > 0 there
exists δ > 0 such that |f (x) − f (a)| <  whenever |x − a| < δ

Example 3.1.36 Prove that


f (x) = (3x + 5)
is continuous at x = 10. Begin by letting  > 0 be given. Find δ > 0 (which depends on
) so that if |x − 10| ≤ δ, then |f (x) − 35| ≤ .

Begin with |f (x) − 35| and ”solve for” |x − 10|. Then

|f (x) − 35| = |(3x + 5) − 35|


= |3x − 30|
= |3(x − 10)|
≤ |3||(x − 10)|
≤ 3|x − 10|
≤ 

iff |x − 10| ≤
3

Now choose δ = 3 .

Thus if |x − 10| ≤ 3 , that is δ ≤ 3 , it follows that |f (x) − 35| ≤ . This completes the
proof.

3.1.5 Continuity at end points of domain


We say that a function is continuous at a left endpoint α of its domain if,

lim f (x) = f (α)


x→α+

Likewise we say that a function f (x) is continuous at a right endpoint β of its domain if

lim f (x) = f (β)


x→β −

H.W-Kayondo & D.W-Ddumba, Engineering Math I- Lecture Notes 92


CHAPTER 3. CONTINUITY OF FUNCTIONS

Exercise 3.1 Why is the function

x2 −9

 x−3
, x 6= 3
f (x) =
8, x=3

not continuous at x = 3? Redefine the function f to make it continuous at x = 3.

Example 3.1.37 Given that

 2
 x + 1; if x < 2,
f (x) =
3x − 1; if x > 2,

(a) Sketch the graph of f (x).

(b) Does lim f (x) exist? Justify your answer.


x→2

(c) Why is f (x) NOT continuous at x = 2? Re-define f (x) to make it continuous at x = 2

The redefined continuous function is


 2
 x + 1; if x < 2,
f (x) = 5; if x = 2,
3x − 1; if x > 2

Example 3.1.38 Redefine the function

x2 − 3x + 2
f (x) =
x−1

such that it is continuous at x = 1.

The continuous redefined function is

x2 −3x+2

 x−1
; if x 6= 1,
f (x) =
−1; if x = 1

H.W-Kayondo & D.W-Ddumba, Engineering Math I- Lecture Notes 93


CHAPTER 3. CONTINUITY OF FUNCTIONS

3.2 Intermediate Value Theorem, IVT


Theorem 3.2.1 Let f be continuous on the closed interval [a, b] with f (a) 6= f (b). Let
d be any number between f (a) and f (b). Then there exists at least one number c ∈ (a, b)
with f (c) = d.

All that this theorem says is that a continuous function cannot skip any number in pasing
from any of its values to another. The IV T is an existence theorem. It guarantees the
existence of the number c, though it does not say how to find it.
It also guarantees that a continuous function cannot change sign without becoming zero
at some point, that is, if a continuous function g has positive and negative values for some
two points on an interval, then there exists a point c in the interval at which g(c) = 0.
Example 3.2.1 Show that the function f (x) = x2 − 4 has a roots between −3 and −1
and also between 1 and 3.
A root of f is a number x for which f (x) = 0. Now f (−3) = 5 and f (−1) = −3. Since
0 ∈ [−3, 5], by the IVT there exists a number c ∈ (−3, −1) such that f (c) = 0.
Similarry A root of f is a number x for which f (x) = 0. Now f (1) = −3 and f (3) = 5.
Since 0 ∈ [−3, 5], by the IVT (We can use the theorem since f (x) is a continuous function
everywhere) there exists a number c ∈ (1, 3) such that f (c) = 0.
Example 3.2.2 Show that the function f (x) = 4x3 − 6x2 + 3x − 2 has a root between
1 and 2.
A root of f is a number x for which f (x) = 0. Now f (1) = −1 and f (2) = 12. Since
0 ∈ [−1, 12], by the IVT there exists a number c ∈ (1, 2) such that f (c) = 0.
Example 3.2.3 If a child grows from 1m to 1.5m between the ages of 2 years and 6
years, then, at some time between 2 years and 6 years of age, the child’s height must have
been 1.25m.
Example 3.2.4 Show that the function f (x) = ln(x) − 1 has a solution in [2, 3].
Example 3.2.5 Show that the function f (x) = x5 + 2x3 + x − 5 has only one real
solution. Hint: Use x = 1 and x = 2
Example 3.2.6 Use the Intermediate Value Theorem to show that there is a positive
number c such that c2 = 2.
Let f (x) = x2 . Then f is continuous and f (0) = 0 < 2 < 4 = f (2). By the IVT there is
c ∈ (0, 2) such that c2 = f (c) = 2.
Example 3.2.7 If f (x) = x3 − x2 + x, show that there is c ∈ R such that f (c) = 10.
But f (1) = 1 and f (3) = 33 − 32 + 3 = 27 − 9 + 3 = 21, so f (0) < 10 < f (3). Since f is
continuous everywhere, there must be c ∈ R such that f (c) = 10.
Example 3.2.8 Let f be a continuous function on [0, 1]. Show that if −1 ≤ f (x) ≤ 1
for all x ∈ [0, 1] then there is c ∈ [0, 1] such that [f (c)]2 = c.
If f (x) is continuous on [0, 1] then so is [f (x)]2 . Set g(x) = [f (x)]2 − x. Then g is also
continuous on [0, 1]. Now g(0) = [f (0)]2 − 0 = [f (0)]2 ≥ 0 and g(1) = [f (1)]2 − 1 ≤ 0, so
by IVT there is c ∈ [0, 1] such that g(c) = 0. Then [f (c)]2 − c = 0 or [f (c)]2 = c.

H.W-Kayondo & D.W-Ddumba, Engineering Math I- Lecture Notes 94


CHAPTER 3. CONTINUITY OF FUNCTIONS

3.3 Fixed point Theorem


Theorem 3.3.1 Let f : [0, 1] → [0, 1] be a continuous function. Then there is at least
one number c that is fixed by f , that is, for which

f (c) = c

Definition 3.3.1 In mathematics, a fixed point (sometimes shortened to fix point, also
known as an invariant point) of a function is a point that is mapped to itself by the
function. A set of fixed points is sometimes called a fixed set.

Example 3.3.1 For example, if f is defined on the real numbers by

f (x) = x2 − 3x + 4,

then 2 is a fixed point of f , because f (2) = 2.

f (c) = c
2
c − 3c + 4 = c
c2 − 4c + 4 = 0
p
4± 42 − 4(1)(4)
c = =2
2(1)

The point c = 2 is the fixed point for the function f (x) = x2 − 3x + 4

Example 3.3.2 Find the fixed points for the continuous function

(x + 1)
f (x) =
2

in the interval [−1, 1]

For a fixed point

f (c) = c

(c + 1)
= c
2
c + 1 = 2c
c = 1

Since c = 1 ∈ [−1, 1], c = 1 is a fixed point.

H.W-Kayondo & D.W-Ddumba, Engineering Math I- Lecture Notes 95


CHAPTER 3. CONTINUITY OF FUNCTIONS

Example 3.3.3 Find all the fixed points for the function

f (x) = x2 − 6

in the closed interval [−4, 4]

For a fixed point

f (c) = c
2
c −6 = c
c2 − c − 6 = 0
p
1± 1 + 4(1)(6)
c = = −2, 3
2(1)

Thus the points c = −2 and c = 3 are the fixed points of f (x) = x2 −6 since −2, 3 ∈ [−4, 4]

Example 3.3.4 Find all the fixed points for the function

f (x) = x2 − 2x + 2

in the closed interval [−6, 1]

For a fixed point

f (c) = c
c2 − 2c + 2 = c
c2 − 3c + 2 = 0
c = 1, 2

Thus the point c = 1 is a fixed point but c = 2 is not a fixed point of f (x) = x2 − 2x + 2
since 1 ∈ [−6, 1] but 2 6∈ [−6, 1]

Example 3.3.5 A continuous function that maps [0, 1] into itself has a fixed point.

Example 3.3.6 A continuous function that maps a disk into itself has a fixed point.

Example 3.3.7 A continuous function that maps a spherical ball into itself necessarily
has a fixed point.

H.W-Kayondo & D.W-Ddumba, Engineering Math I- Lecture Notes 96


CHAPTER 3. CONTINUITY OF FUNCTIONS

Exercise 3.2

1. Find values of x for which the following functions are discontinuous.


2 +2

(i) xx−1  2 − x, x≤2
(v) f (x) =
(ii) cos
x2
x
x − 2, x > 2

x+1
(iii) x2 −1
x x4 −9x2
(iv) x3 −1
(vi) x4 −3x3

2. Redefine f (x) so that it is continuous at the given points


x2 −2 1−cos2 x sin x
(i) x−2
at x = 1 (iii) sin2 x
at x = 0 (v) x
at x = 0

x2 −1 1−cos2 x
(ii) x−1
at x = 1 (iv) sin x cos x
at x = 0

3. Given

 4 − x2 , x ≤ −1
f (x) =
x + 1, x > −1

Discuss the continuity of f (x) at x = −1

4. Find the constant k that will make the function f continuous at x = 1 if

x3 −3x2 +2


 x2 −1
, f or x 6= 1

f (x) =

 k, f or x = 1

5. Given

 sin x, if 2nπ < x < 2(n + 1)π for n even
f (x) =
cos x, if 2nπ < x < 2(n + 1)π for n odd

(a) Sketch f (x)


(b) Find f (π), f (2π) and f (3π)
(c) Find limx→2π f (x) if it exists
(d) Find limx→3π f (x) if it exists

H.W-Kayondo & D.W-Ddumba, Engineering Math I- Lecture Notes 97


CHAPTER 3. CONTINUITY OF FUNCTIONS

3.4 Questions with Solutions


3.4.1 Questions
[Limits & Continuity]

(a) State the definition of a limit of a function f (x) as x → a.

(b) Compute the following limits.


2x2 +5x−3 x3 −1
(i) lim x+3
(iii) lim
x→−3 x→1 x−1

√ 7x2 −9x3 +x
(ii) lim 2x2 + 1 (iv) lim 3 2
x→1 x→∞ −18x −5x −x

(c) Find λ such that lim exists where


x→−2


6 − x, x ≤ −2
f (x) =
λx2 , x > −2

|x|
(d) Does the lim 4x
exist ? Give reasons for your answer.
x⇒0

e(i) Define what is meant by the function f (x) being continuous at x = a.

(ii) Let
x2 − 9
f (x) =
x−3

(ii1) Show that f (x) is discontinuous at x = 3


(ii2) Reduce f (x) so that it is continuous at x = 3

(f) Check whether the function,

 3
 x, x≤2
f (x) =
10x, x>2

is continuous at x = 2.

H.W-Kayondo & D.W-Ddumba, Engineering Math I- Lecture Notes 98


CHAPTER 3. CONTINUITY OF FUNCTIONS

3.4.2 Solutions
[Limits & Continuity]

(a) We say that L is the limit of f (x) as x approaches a if for every  > 0 (however
small but positive) there exists a corresponding δ > 0 also dependent on  such that
|f (x) − L| <  whenever |x − a| < δ.

(b) (i)

2x2 + 5x − 3 0
lim = thus by La’Hopitals’ rule
x→−3 x+3 0
4x + 5
= lim = −7
x→−3 1

(ii)

√ p √
lim 2x2 + 1 = 2(1)2 + 1 = 3
x→1

(iii)

x3 − 1 (x − 1)(x2 + x + 1)
lim = lim = lim (x2 + x + 1) = 3
x→1 x − 1 x→1 x−1 x→1

(iv)

7
7x2 − 9x3 + x x
− 9 + x12 −9 1
lim = lim 5 1 = =
x→∞ −18x − 5x − x
3 2 x→∞ −18 − − x2 −18 2
x

(c) For the limit to exist

lim f (x) = lim f (x)


x→−2− x→−2+

8 = 4λ
2 = λ

H.W-Kayondo & D.W-Ddumba, Engineering Math I- Lecture Notes 99


CHAPTER 3. CONTINUITY OF FUNCTIONS

(d) The limit does not exist since for

x
− 14 ,
 
− 4x , x≤0 x≤0
f (x) = x = 1
4x
, x≥0 4
, x≥0

 
1 1
since lim f (x) 6= lim f (x) that is − 6=
x→0− x→0+ 4 4

e(i) We say that a function f (x) is continuous at x = a if


(i) the lim f (x) exists ie lim− f (x) = lim+ f (x)
x→a x→a x→a

(ii) the functional value f (a) exists.


(iii) limx→a f (x) = f (a)
OR
A function f (x) is continuous at x = a if and only if given  > 0 there exists δ > 0
such that |f (x) − f (a)| <  whenever 0 ≤ |x − a| < δ
x2 −9
(ii) Let f (x) = x−3

(ii1) Since f (x) is not defined at x = 3, then is discontinuous at that point.


(ii2) Using limits

x2 − 9 (x − 3)(x + 3)
lim = lim = lim (x + 3) = 6
x→3 x − 3 x→3 x−3 x→3

x2 −9

 x−3
, x 6= 3
f (x) =
6, x=3

(f)
lim f (x) = 23 = 8
x→2−

and lim = 10(2) = 20


x→2+

since lim f (x) 6= lim


x→2− x→2+

The limit does not exist, thus the function is not continuous.
Exercise 3.3 Suppose that 6x − x2 ≤ f (x) ≤ x2 − 6x + 18 for all x. Find lim f (x)
x→3

H.W-Kayondo & D.W-Ddumba, Engineering Math I- Lecture Notes 100


Chapter 4

Differentiation

4.1 Derivative of a function


Definition 4.1.1 A derivative of a function f (x) denoted as f 0 (x) is said to exists if

f (x + h) − f (x)
lim exists
h→0 h

ie if

f (x + h) − f (x) f (x + h) − f (x)
lim− = lim+ (4.1)
h→0 h h→0 h

The derivative of a function f (x) denoted by f 0 (x) if exists, is given by

f (x + h) − f (x)
f 0 (x) = lim (4.2)
h→0 h

Definition 4.1.2 A derivative of a function f (x) at a point x = a denoted as f 0 (a) is


said to exists if

f (a + h) − f (a)
f 0 (a) = lim (4.3)
h→0 h

Definition 4.1.3 A derivative of a function f (x) at a point x = a denoted as f 0 (a) is


said to exists if

f (x) − f (a)
lim exists ⇒
x→a x−a

101
CHAPTER 4. DIFFERENTIATION

f (x) − f (a) f (x) − f (a)


lim− = lim+ (4.4)
x→a x−a x→a x−a

The derivative of a function f (x) at a point x = a denoted as f 0 (a) if exists, is given by

f (x) − f (a)
f 0 (a) = lim (4.5)
x→a x−a

This definition of the derivative is illustrated by the following figure.

Figure 4.1: Illustration of the derivative of f (x0 )

The slope of the chord AB is

∆y f (x0 + h) − f (x0 )
=
h h

and indeed as h → 0, this quotient tends to the slope of the tangent to the curve at x = x0
which is f 0 (x0 ).

Example 4.1.1 Suppose f (x) = α ( constant). Use the definition of a derivative to


compute f 0 (x)

By definition,

f (x + h) − f (x) α−α 0
f 0 (x) = lim = lim = lim = 0
h→0 h h→0 h h→0 h

Note 4.1.1 This result shows that the derivative of a constant is 0 . Thus

d d d
(10) = (a) = (90) = 0
dx dx dx

H.W-Kayondo & D.W-Ddumba, Engineering Math I- Lecture Notes 102


CHAPTER 4. DIFFERENTIATION

Example 4.1.2 Suppose f (x) = x2 , use the definition of a derivative to find f 0 (x)

f (x + h) − f (x)
f 0 (x) = lim
h→0 h

but f(x + h) = (x + h)2

(x + h)2 − x2
therefore f 0 (x) = lim
h→0 h

x2 + 2xh + h2 − x2
= lim
h→0 h

2xh + h2
= lim
h→0 h
= lim (2x + h) = 2x
h→0

d 2
⇒ (x ) = 2x
dx

Example 4.1.3 Let us start with the function f (x) = x2 + 1. We have

f (x + h) − f (x)
f 0 (x) = lim
h→0 h

[(x + h)2 + 1] − [x2 + 1]


= lim bracketing f (x + h) and f (x)
h→0 h

[x2 + 2xh + h2 + 1] − [x2 + 1]


= lim the f (x) terms have to cancel
h→0 h

2xh + h2
= lim which of the five techniques of finding limits to use
h→0 h
(2x + h)(h)
= lim
h→0 h
= lim 2a + h = 2x can use method I now, of substitution
h→0

H.W-Kayondo & D.W-Ddumba, Engineering Math I- Lecture Notes 103


CHAPTER 4. DIFFERENTIATION

Example 4.1.4 For f (x) = xn where n ≥ 1 integer. Use the definition of a derivative
d
to compute dx (xn ).

f (x + h) − f (x)
f 0 (x) = lim
h→0 h
since f(x + h) = (x + h)n
and f(x) = xn

(x + h)n − xn
⇒ f 0 (x) = lim
h→0 h

We use Binomial theorem to expand (x + h)n

n(n−1)xn−2 (h)2
!
xn + nxn−1 h + + · · · + hn − xn
⇒ f 0 (x) = lim 2!
h→0 h

hn(n − 1)xn−2
 
n−1
= lim nx + + ... + hn−1 ⇒
h→0 2!

d n
(x ) = nxn−1
dx

This result is an important differentiation formula . The formula is valid for all n ∈ R
(real numbers)
Example 4.1.5 Suppose f (x) = x, using the definition of a derivative find f 0 (x)

f (x + h) − f (x)
f 0 (x) = lim
h→0 h
x+h−x
= lim
h→0 h
h
= lim = lim 1 = 1
h→0 h h→1

The result shows that


d
(x) = 1
dx

H.W-Kayondo & D.W-Ddumba, Engineering Math I- Lecture Notes 104


CHAPTER 4. DIFFERENTIATION

Example 4.1.6 Consider the function


f (x) = 1/x
for x 6= 0. We have

f (x + h) − f (x)
f 0 (x) = lim
h→0 h
1 1
0 x+h
− x
f (x) = lim
h→0 h
x − (x + h)
f 0 (x) = lim
h→0 hx(x + h)

−h
f 0 (x) = lim
h→0 hx(x + h)

−1 1
= lim =− 2
h→0 x(x + h) x

Exercise 4.1 Use the limit definition to compute the derivative, f 0 (x), for

1
f (x) = (x + 1) 3

f (x + h) − f (x)
f 0 (x) = lim
h→0 h
1 1
0 [(x + 1) + h] 3 − (x + 1) 3
f (x) = lim Binomial expansion, fractional powers
h→0 h
−5
  h
1 −2 3 h2
i
1 1 −2 (x+1) 1
(x + 1) 3 + 3 (x + 1) 3 h + 3 3 2! + · · · − (x + 1) 3
f 0 (x) = lim
h→0 h
−5
1 −2
1 −2 (x+1) 3 h2
(x + 1) 3 h+ 3 3
+ ···
f 0 (x) = lim 3 2!
h→0 h
−5
" #
1 −2
0 1 −2
3 3
(x + 1) 3 h
f (x) = lim (x + 1) 3 +
h→0 3 2!

1 −2
= (x + 1) 3
3

H.W-Kayondo & D.W-Ddumba, Engineering Math I- Lecture Notes 105


CHAPTER 4. DIFFERENTIATION

Note 4.1.2 For any value of n, whether positive, negative, integer or non-integer, the
value of the nth power of a binomial is given by:

nan−1 b n(n − 1)an−2 b2 n(n − 1)(n − 2)an−3 b3


(a + b)n = an + + + + · · · + bn
1! 2! 3!

d
Example 4.1.7 Using the definition of a derivative, find dx
(sin x)

f (x + h) − f (x)
f 0 (x) = lim
h→0 h
sin(x + h) − sin(x)
sin0 (x) = lim
h→0 h
sin x cos h + cos x sin h − sin x
= lim
h→0 h
sin x(cos h − 1) + cos x sin h
= lim
h→0 h
sin x(cos h − 1) cos x sin h
= lim + lim
h→0 h h→0 h
(cos h − 1) sin h
= sin x lim + cos x lim
h→0 h h→0 h

(− sin h) cos h
= sin x lim + cos x lim
h→0 1 h→0 1
= sin x(0) + cos x(1)
= cos x

Example 4.1.8 Let a function f at a point x = 2 be defined by f (x) = (x + 3)10 x. Find


df
dx x=2

The function f (x) given to you is at only x = 2, but we do not have the function as
x → 2+ or the f (x) as x → 2− , so we cannot compute the derivative since the function is
not known.

H.W-Kayondo & D.W-Ddumba, Engineering Math I- Lecture Notes 106


CHAPTER 4. DIFFERENTIATION

Example 4.1.9 Show that the function y = |x| is not differentiable at x = 0.

Recall that

 x, x≥0
|x| =
−x, x < 0

with the graph of y = |x| on the left

Now check for differentiability at x = 0, i.e., compute f 0 (0). Then

f (x) − f (a)
f 0 (a) = lim
x→a x−a

f (x) − f (0)
f 0 (0) = lim
x→0 x−0

Finding a limit for a piecewise function, we check from left and from right, if equal, that
is the limit, otherwise, the limit does not exist.

From the right of point x = 0

f (x) − f (0) (x) − (0) x


lim+ = lim = lim = lim 1 = 1
x→0 x−0 x→0 x x→0 x x→0

From the left of point x = 0

f (x) − f (0) (−x) − (0) −x


lim− = lim = lim = lim −1 = −1
x→0 x−0 x→0 x x→0 x x→0

f (x) − f (0)
Since the one-sided limits exist but are not equal though finite, f 0 (0) = lim
x→0 x−0
does not exist, and f is not differentiable at x = 0. This implies that the derivative of
f (x) = |x| does not exist at x = 0

H.W-Kayondo & D.W-Ddumba, Engineering Math I- Lecture Notes 107


CHAPTER 4. DIFFERENTIATION

Example 4.1.10 Show that the derivative of the function f (x) = x|x| is given by.
The function is also given by
 2
 x, x≥0
f (x) =
−x2 , x < 0

Convince your self that




 2x, x > 0



0
f (x) = 0, x = 0 = 2|x|




−2x, x < 0

Example 4.1.11 Use the definition of derivatives to compute f 0 (x) given f (x) = mx + c
Example 4.1.12 Use the limit definition to compute the derivative, f 0 (x), for

1 3
f (x) = x −
2 5

f (x + h) − f (x)
f 0 (x) = lim
h→0 h
1
(x + h) − 53 − 1 3
 
0 2 2
x − 5
f (x) = lim
h→0 h
1
x + 12 h − 35 − 12 x + 3
f 0 (x) = lim 2 5
h→0 h

Algebraically and arithmetically simplify the expression in the numerator.

1
0 2
h
f (x) = lim
h→0 h

The term h now divides out and the limit can be calculated.

1
f 0 (x) = lim
h→0 2

1
=
2

H.W-Kayondo & D.W-Ddumba, Engineering Math I- Lecture Notes 108


CHAPTER 4. DIFFERENTIATION

Example 4.1.13 Use the limit definition to compute the derivative, f 0 (x), for

f (x) = 5x2 − 3x + 7

f (x + h) − f (x)
f 0 (x) = lim
h→0 h

(5(x + h)2 − 3(x + h) + 7) − (5x2 − 3x + 7)


f 0 (x) = lim
h→0 h

5x2 + 10xh + 5h2 − 3x − 3h + 7 − 5x2 + 3x − 7


f 0 (x) = lim
h→0 h

Algebraically and arithmetically simplify the expression in the numerator.

10xh + 5h2 − 3h
f 0 (x) = lim
h→0 h

The term h now divides out and the limit can be calculated.

f 0 (x) = lim (10x + 5h − 3)


h→0

= 10x − 3

Example 4.1.14 Use the limit definition to compute the derivative, f 0 (x), for

f (x) = 4 − x + 3

f (x + h) − f (x)
f 0 (x) = lim
h→0 h
 p  √ 
4 − (x + h) + 3 − 4 − x + 3
f 0 (x) = lim
h→0 h
√ p
x + 3 − (x + h) + 3
f 0 (x) = lim
h→0 h

Eliminate the square root terms in the numerator of the expression by multiplying by the

H.W-Kayondo & D.W-Ddumba, Engineering Math I- Lecture Notes 109


CHAPTER 4. DIFFERENTIATION

conjugate of the numerator divided by itself


√ √ √ √
0 x+3− x+h+3 x+3+ x+h+3
f (x) = lim √ √
h→0 h x+3+ x+h+3

(x + 3) − (x + h + 3)
f 0 (x) = lim √ √ 
h→0 h x+3+ x+h+3

−h
f 0 (x) = lim √ √ 
h→0 h x+3+ x+h+3

The term h now divides out and the limit can be calculated.

−1 −1 −1
f 0 (x) = lim √ √ = √ √ = √
h→0 x+3+ x+h+3 x+3+ x+3 2 x+3

Example 4.1.15 Use the limit definition to compute the derivative, f 0 (x), for

x+1
f (x) =
2−x

(x+h)+1 x+1
0 f (x + h) − f (x) 2−(x+h)
− 2−x
f (x) = lim = lim
h→0 h h→0 h
(x + h + 1)(2 − x) − (x + 1)(2 − x − h) 1
f 0 (x) =
(2 − x − h)(2 − x) h

Algebraically and arithmetically simplify the expression in the numerator. It is important


to note that the denominator of this expression should be left in factored form so that
the term h can be easily eliminated later.

2x + 2h + 2 − x2 − xh − x − {2x − x2 − xh + 2 − x − h}
f 0 (x) = lim
h→0 (2 − x − h)(2 − x)h

3h
f 0 (x) = lim
h→0 (2 − x − h)(2 − x)h

The term h now divides out and the limit can be calculated

3 3 3
f 0 (x) = lim = =
h→0 (2 − x − h)(2 − x) (2 − x)(2 − x) (2 − x)2

H.W-Kayondo & D.W-Ddumba, Engineering Math I- Lecture Notes 110


CHAPTER 4. DIFFERENTIATION

Example 4.1.16 Use the limit definition to compute the derivative, f 0 (x), for

2
f (x) = x 3
This problem may be more difficult than it first appears.

2 2
0 f (x + h) − f (x) (x + h) 3 − x 3
f (x) = lim = lim
h→0 h h→0 h

Note that A − B can be written as the difference of cubes,


x3 − y 3 = (x − y)(x2 + xy + y 2 )
so that
 1 3  1 3  1 1
 2 1 1 2

A − B = A3 − B 3 = A3 − B 3 A3 + A3 B 3 + B 3

 1 1
 A−B
⇒ A −B
3 3 =  2 1 1 2

A +A B +B
3 3 3 3

This will help explain the next step.

2 2 1 1
0 (x + h) 3 − x 3 {(x + h)2 } 3 − {x2 } 3
f (x) = lim = lim
h→0 h h→0 h

(x + h)2 − x2
= lim n 4 2 2 4
o
h→0
h (x + h) 3 + (x + h) 3 x 3 + x 3

(x2 + 2xh + h2 ) − x2
= lim n 4 2 2 4
o
h→0
h (x + h) 3 + (x + h) 3 x 3 + x 3

h(2x + h)
= lim n 4 2 2 4
o
h→0
h (x + h) 3 + (x + h) 3 x 3 + x 3

2x + h
= lim n 4 2 2 4
o
h→0
(x + h) + (x + h) x + x
3 3 3 3

2x 2x 2
= n 4 2 2 4
o= 4 = 1
x3 + x3 x3 + x3 3x 3 3x 3

H.W-Kayondo & D.W-Ddumba, Engineering Math I- Lecture Notes 111


CHAPTER 4. DIFFERENTIATION

Example 4.1.17 Use the limit definition to compute the derivative, f 0 (x), for
f (x) = cos 3x

f (x + h) − f (x)
f 0 (x) = lim
h→0 h
cos 3(x + h) − cos 3x
f 0 (x) = lim
h→0 h
cos(3x + 3h) − cos 3x
f 0 (x) = lim
h→0 h
{cos 3x cos 3h − sin 3x sin 3h} − cos 3x
f 0 (x) = lim
h→0 h
cos 3x(cos 3h − 1) − sin 3x sin 3h
f 0 (x) = lim
h→0 h

Since
cos(A + B) = cos A cos B − sin A sin B
Recall the following two well-known trigonometry limits (La’Hopital rule):

cos h − 1 sin h sin 3h


lim = 0 , lim = 1 ⇒ lim =3
h→0 h h→0 h h→0 h

cos 3x(cos 3h − 1) − sin 3x sin 3h


f 0 (x) = lim
h→0 h
cos 3x(cos 3h − 1) sin 3x sin 3h
f 0 (x) = lim − lim
h→0 h h→0 h
(cos 3h − 1) sin 3h
f 0 (x) = cos 3x lim − sin 3x lim
h→0 h h→0 h

Terms without h were factored out, as


lim αf (x) = α lim f (x)
x→a x→a

(cos 3h − 1) sin 3h
f 0 (x) = cos 3x lim − sin 3x lim
h→0 h h→0 h

f 0 (x) = cos 3x (0) − sin 3x (3)


f 0 (x) = −3 sin 3x

H.W-Kayondo & D.W-Ddumba, Engineering Math I- Lecture Notes 112


CHAPTER 4. DIFFERENTIATION

Example 4.1.18 Use the limit definition to compute the derivative, f 0 (x), for

x−1
f (x) =
x2 + 3x

f (x + h) − f (x)
f 0 (x) = lim
h→0 h
(x + h) − 1 x−1
2
− 2
(x + h) + 3(x + h) x + 3x
= lim
h→0 h

Get a common denominator for the expression in the numerator.

(x3 + 3x2 + x2 h + 3xh − x2 − 3x − x3 − 2x2 h − xh2 − 3x2 − 3xh + x2 + 2xh + h2 + 3x + 3h) 1


= lim
h→0 (x2 + 2xh + h2 + 3x + 3h)(x2 + 3x) h

Algebraically and arithmetically simplify the expression in the numerator. The terms
x3 , 2x2 , −3x, and 3xh will subtract out. It is important to note that the denominator of
this expression should be left in factored form so that the term h can be easily eliminated
later.
−x2 h + 2xh + h2 + 3h
= lim
h→0 (x2 + 2xh + h2 + 3x + 3h)(x2 + 3x)h

Factor h from the numerator.

h(−x2 + 2x + h + 3)
= lim
h→0 (x2 + 2xh + h2 + 3x + 3h)(x2 + 3x)h

The term h now divides out and the limit can be calculated.

−x2 + 2x + h + 3
f 0 (x) = lim
h→0 (x2 + 2xh + h2 + 3x + 3h)(x2 + 3x)

−x2 + 2x + 3
f 0 (x) =
(x2 + 3x)(x2 + 3x)

2x + 3 − x2
f 0 (x) =
(x2 + 3x)2

H.W-Kayondo & D.W-Ddumba, Engineering Math I- Lecture Notes 113


CHAPTER 4. DIFFERENTIATION

Example 4.1.19 Use the limit definition to compute the derivative, f 0 (x), for

f (x) = x3 − x

p √
0 f (x + h) − f (x) (x + h)3 − (x + h) − x3 − x
f (x) = lim = lim
h→0 h h→0 h

Eliminate the square root terms in the numerator of the expression by multiplying by the
conjugate of the numerator divided by itself.
p √ p √
(x + h)3 − (x + h) − x3 − x (x + h)3 − (x + h) + x3 − x
= lim p √
h→0 h (x + h)3 − (x + h) + x3 − x

Algebraically and arithmetically simplify the expression in the numerator. It is important


to note that the denominator of this expression should be left in factored form so that
the term h can be easily eliminated later.

[(x + h)3 − (x + h)] − (x3 − x)


f 0 (x) = lim p √
h→0 h (x + h)3 − (x + h) + x3 − x

[x3 + 3xh2 + 3x2 h + h3 − x − h] − (x3 − x)


f 0 (x) = lim p √
h→0 h (x + h)3 − (x + h) + x3 − x

3xh2 + 3x2 h + h3 − h
f 0 (x) = lim p √
h→0 h (x + h)3 − (x + h) + x3 − x

Factor h from the numerator.

h [3xh + 3x2 + h2 − 1]
f 0 (x) = lim p √
h→0 h (x + h)3 − (x + h) + x3 − x

The term h now divides out and the limit can be calculated.

3xh + 3x2 + h2 − 1
f 0 (x) = lim p √
h→0 (x + h)3 − (x + h) + x3 − x

3x2 − 1
f 0 (x) = √ √
x3 − x + x 3 − x

3x2 − 1
f 0 (x) = √
2 x3 − x

H.W-Kayondo & D.W-Ddumba, Engineering Math I- Lecture Notes 114


CHAPTER 4. DIFFERENTIATION

Example 4.1.20 Assume a piecewise function f (x) defined as


x, if x ≥ 1


 2 +
f (x) =
 1 x + 5 , if x < 1

2 2

Show whether or not f (x) is differentiable at x = 1, i.e., use the limit definition of the
derivative to compute f 0 (1).

To compute f 0 (1): Lets first compute f (1) = 2 + 1 = 3, then

f (x) − f (a)
f 0 (a) = lim
x→a x−a

f (x) − f (1)
f 0 (1) = lim
x→1 x−1

Finding a limit for a piecewise function, we check from left and from right, if equal, that
is the limit, otherwise, the limit does not exist.

From the right of point x = 1


f (x) − f (1) (2 + x ) − (3)
lim = lim
x→1+ x−1 x→1 x−1
√ √
x−1 ( x − 1)
= lim = lim √ √
x→1 x−1 x→1 ( x − 1)( x + 1)

1 1
= lim √ =
x→1 ( x + 1) 2

From the left of point x = 1

 
1 5
x+ − (3) x−1
f (x) − f (1) 2 2 2 1 1
lim− = lim = lim = lim =
x→1 x−1 x→1 x−1 x→1 x − 1 x→1 2 2

f (x) − f (1) 1
Since the one-sided limits exists and are equal, f 0 (1) = lim = does exist,
x→1 x−1 2
and thus f is differentiable at x = 1.

H.W-Kayondo & D.W-Ddumba, Engineering Math I- Lecture Notes 115


CHAPTER 4. DIFFERENTIATION

Example 4.1.21 Assume that

 1
2
 x sin
 , if x 6= 0
f (x) = x

0, if x = 0

Show that f is differentiable at x = 0, i.e., use the limit definition of the derivative to
compute f 0 (0).

To have from right and from left, we use the Squeeze law to create functions from left
and right.

 1   x2 , if x > 0
x2 sin =
x
−x2 , if x < 0

The derivative at x = 0 is
f 0 (0) = 0
Show the solution above.
Remark 4.1.1 What follows is a common incorrect attempt to solve this problem using
1
2
another method. Since f (x) = x sin for x 6= 0, it follows, using the product rule
x
and chain rule, that

 1 n −1 o 1 1 1


f 0 (x) = x2 cos + 2x sin = − cos + 2x sin
x x2 x x x

for x 6= 0. Then
f 0 (0) = lim f 0 (x)
x→0

n 1  1 o
= lim − cos + 2x sin
x→0 x x
1
Because the term − cos oscillates between 1 and −1 as h approaches zero, this limit
x
does not exist.

An incorrect conclusion would be that f 0 (0) does not exist, i.e., f is not differentiable at
x = 0. If f 0 were continuous at x = 0, this would be a valid method to compute f 0 (0).

H.W-Kayondo & D.W-Ddumba, Engineering Math I- Lecture Notes 116


CHAPTER 4. DIFFERENTIATION

Example 4.1.22 Use the limit definition to compute the derivative, f 0 (x), for a piecewise
function
f (x) = |x2 − 3x|
First rewrite f (x). That is,

 2
 (x − 3x), x ∈ (−∞, 0] ∪ [3, ∞)
f (x) = |x2 − 3x| =
−(x2 − 3x), 0 < x < 3

In other words, the region for positive, +(x2 − 3x)

(x2 − 3x) ≥ 0 ⇒ x(x − 3) ≥ 0 ⇒


either (both positives) x ≥ 0 & (x − 3) ≥ 0 or (both negatives) x ≤ 0 & (x − 3) ≤ 0
either x ≥ 0 & x ≥ 3 or x ≤ 0 & x ≤ 3
either x ≥ 3 or x ≤ 0
⇒ x ∈ (−∞, 0] ∪ [3, ∞)

and the region for negative, −(x2 − 3x)

(x2 − 3x) < 0 ⇒ x(x − 3) < 0 ⇒


either (one negative) x > 0 & (x − 3) < 0 or (the other negative) x < 0 & (x − 3) > 0
either x > 0 & x < 3 or x < 0 & x > 3
either (0, 3) or no solution
⇒ x ∈ (0, 3)

This can be summarized as


 2

 (x − 3x), if x ≤ 0



f (x) = −(x2 − 3x), if 0 < x < 3




 2
(x − 3x), if x ≥ 3

1. Check for differentiability at x = 0, i.e., compute f 0 (0). Then

f (x) − f (a)
f 0 (a) = lim
x→a x−a

f (x) − f (0)
f 0 (0) = lim
x→0 x−0

H.W-Kayondo & D.W-Ddumba, Engineering Math I- Lecture Notes 117


CHAPTER 4. DIFFERENTIATION

Finding a limit for a piecewise function, we check from left and from right, if equal,
that is the limit, otherwise, the limit does not exist.

From the right of point x = 0

f (x) − f (0) (3x − x2 ) − (0) 3x − x2


lim+ = lim = lim = lim 3 − x = 3
x→0 x−0 x→0 x x→0 x x→0

From the left of point x = 0

f (x) − f (0) (x2 − 3x) − (0) x2 − 3x


lim− = lim = lim = lim x − 3 = −3
x→0 x−0 x→0 x x→0 x x→0

f (x) − f (0)
Since the one-sided limits exist but are not equal, f 0 (0) = lim does not
x→0 x−0
exist, and f is not differentiable at x = 0.

2. Now check for differentiability at x = 3, i.e., compute f 0 (3): f (3) = (3)2 − 3(3) = 0
Then

f (x) − f (a)
f 0 (a) = lim
x→a x−a

f (x) − f (3)
f 0 (3) = lim
x→3 x−3

Finding a limit for a piecewise function, we check from left and from right, if equal,
that is the limit, otherwise, the limit does not exist.

From the right of point x = 3

f (x) − f (3) (x2 − 3x) − (0) x2 − 3x


lim+ = lim = lim = lim x = 3
x→3 x−3 x→3 x−3 x→3 x−3 x→3

From the left of point x = 3

f (x) − f (3) (3x − x2 ) − (0) 3x − x2


lim− = lim = lim = lim −x = −3
x→3 x−3 x→3 x−3 x→3 x−3 x→3

H.W-Kayondo & D.W-Ddumba, Engineering Math I- Lecture Notes 118


CHAPTER 4. DIFFERENTIATION

f (x) − f (3)
Since the one-sided limits exist but are not equal, f 0 (3) = lim does not
x→3 x−3
exist, and f is not differentiable at x = 3. Since the one-sided limits exist but are not
f (x) − f (3)
equal, f 0 (3) = lim does not exist, and f is not differentiable at x = 3.
x→3 x−3

3. Assume that x < 0. Then

f (x + h) − f (x) (x + h)2 − 3(x + h) − (x2 − 3x)


f 0 (x) = lim = lim
h→0 h h→0 h

x2 + 2xh + h2 − 3x − 3h − x2 + 3x
= lim
h→0 h

2xh + h2 − 3h h(2x + h − 3)
= lim = lim = lim (2x + h − 3) = 2x − 3.
h→0 h h→0 h h→0

4. Assume that x > 3. Then it is also true (the same function of f (x) = (x2 − 3x)) that
f 0 (x) = 2x − 3

5. Assume that 0 < x < 3. Then

f (x + h) − f (x) 3(x + h) − (x + h)2 − (3x − x2 )


f 0 (x) = lim = lim
h→0 h h→0 h

3x + 3h − (x2 + 2xh + h2 ) − 3x + x2 3x + 3h − x2 − 2xh − h2 − 3x + x2


= lim = lim
h→0 h h→0 h

3h − 2xh − h2 h(3 − 2x − h)
= lim = lim = lim (3 − 2x − h) = 3 − 2x.
h→0 h h→0 h h→0

6. Summarizing, the derivative of f (the function f is not differentiable at x = 0 or x = 3)


is


 2x − 3, if x<0




DNE, if x=0







f 0 (x) = 3 − 2x, if 0<x<3




DNE, if x=3








2x − 3, if x>3

H.W-Kayondo & D.W-Ddumba, Engineering Math I- Lecture Notes 119


CHAPTER 4. DIFFERENTIATION

Example 4.1.23 Assume that


1 3 1 2
 4 x − 2 x , if x ≥ 2



f (x) =

 −3x + 6

 , if x < 2
x2 + 2

Determine if f is differentiable at x = 2, i.e., determine if

f (x) − f (2)
f 0 (2) = lim exists
x→2 x−2

From right, for region x > 2

n o
1 3 1 2
f (x) − f (2) 4
x − 2
x −0 1 2
− 2)
x (x 1
lim+ = lim = lim 4
= lim x2 = 1
x→2 x−2 x→2 x−2 x→2 (x − 2) x→2 4

From left, for region x < 2

n o
−3x+6
f (x) − f (2) x2 +2
−0 −3x + 6 −3(x − 2) 1
lim− = lim = lim = lim =−
x→2 x−2 x→2 x−2 x→2 (x − 2)(x + 2)
2 x→2 (x − 2)(x + 2)
2 2

Since the derivatives are not equal, the derivative does not exist.

Remark 4.1.2 : Use of the limit definition of the derivative of f at x = 2 also leads to
a correct solution to this problem.

Remark 4.1.3 : What follows is a common incorrect attempt to solve this problem
using another method.

For x > 2
3
f 0 (x) = x2 − x
4

For x < 2

0 (x2 + 2)(−6) − (−6x − 6)(2x) 6x2 + 12x − 12


f (x) = =
(x2 + 2)2 (x2 + 2)2

H.W-Kayondo & D.W-Ddumba, Engineering Math I- Lecture Notes 120


CHAPTER 4. DIFFERENTIATION

Then
n3 o 3
lim+ f 0 (x) = lim+ x2 − x = (2)2 − 2 = 1
x→2 x→2 4 4

and
6x2 + 12x − 12 6(2)2 + 12(2) − 12
lim− f 0 (x) = lim− = =1
x→2 x→2 (x2 + 2)2 ((2)2 + 2)2

An incorrect conclusion would be that f 0 (2) = 1. If f 0 were continuous at x = 2, this


would be a valid method to compute f 0 (2).

Example 4.1.24 Given a piecewise function

 2

 x, if x < 0



f (x) = 2, if 0 ≤ x ≤ 3




4 − x, if x > 3

Show that (using the definitions of differentiability)



 2x, if x < 0



f 0 (x) = 0, if 0 ≤ x < 3




−1, if x > 3

Exercise 4.2 Consider the function f : R → R defined by:

 1+x

 2
, if x < 1



f (x) = 1, if x = 1


 √


x, if x > 1

(a) Find f 0 (1)

(b) Find f 00 (1)

H.W-Kayondo & D.W-Ddumba, Engineering Math I- Lecture Notes 121


CHAPTER 4. DIFFERENTIATION

Example 4.1.25 Show that the piecewise function

 2
 x, x≥0
f (x) =
−x, x < 0

is not differentiable at x = 0. [0 6= −1]


It can also be seen from the graph, having a sharp turn at x = 0.

Figure 4.2: Sharp turn at x = 0, not differentiable

Example 4.1.26 Find the derivative of

y = |5x − 2|

The function is given by


 2
 (5x − 2), x≥ 5
y=
2
−(5x − 2), x <

5

Note that, we have not been asked f 0 25 [at a point], but f 0 (x) [everywhere], but since


we have a special point x = 52 , to differentiate everywhere, it is to differentiate at x = 25 ,


x > 52 and x < 25

H.W-Kayondo & D.W-Ddumba, Engineering Math I- Lecture Notes 122


CHAPTER 4. DIFFERENTIATION

(1). Differentiable at x = 25 :
Derivative is given by

f (x) − f (a)
f 0 (a) = lim
x→a x−a
2

f (x) − f
 
0 2 5
f = lim2
5 x→ 5 x − 25

Since finding limit of a piecewise function, we use the informal definition of limits:

2
From the right of point x = 5

2

f (x) − f 5 (5x − 2) − 0
lim+ = lim2
x→ 25 x − 25 x→ 5 x − 25

5 x − 25 − 0

= lim2
x − 25

x→ 5

= lim2 5 = 5
x→ 5

2
From the left of point x = 5

2

f (x) − f 5 −(5x − 2) − 0
lim− = lim2
x→ 25 x − 25 x→ 5 x − 52

−5 x − 25 − 0

= lim2
x − 52

x→ 5

= lim2 − 5 = −5
x→ 5

since

2 2 2
  
f (x) − f f (x) − f f (x) − f
 
5 5 5 0 2
lim+ 6= lim− ⇒ lim2 =f DNE
x→ 52 x − 25 x→ 25 x − 25 x→ 5 x − 25 5

the function is not differentiable at x = 52 .

H.W-Kayondo & D.W-Ddumba, Engineering Math I- Lecture Notes 123


CHAPTER 4. DIFFERENTIATION

(2). Differentiability for x > 52 :

For the region x > 25 , f (x) = y = (5x − 2)

f (x + h) − f (x)
f 0 (x) = lim
h→0 h
[5(x + h) − 2] − [(5x − 2)]
= lim
h→0 h
5h
= lim = lim 5 = 5
h→0 h h→0

(3). Differentiability for x < 25 :

For the region x < 25 , f (x) = y = −(5x − 2)

f (x + h) − f (x)
f 0 (x) = lim
h→0 h
[− {5(x + h) − 2}] − [−(5x − 2)]
= lim
h→0 h
−5h
= lim = lim −5 = −5
h→0 h h→0

In summary, the derivative of the function y denoted as y 0 is given by

 2

 5, x> 5



0 2
y = DNE, x = 5



 2
−5, x<

5

H.W-Kayondo & D.W-Ddumba, Engineering Math I- Lecture Notes 124


CHAPTER 4. DIFFERENTIATION

Example 4.1.27 Given a function

|4x − 3|
y=
4x − 3

Find the derivative of y


 (4x−3) 3
 3
 4x−3
, x≥ 4  1, x≥ 4
y= =
−(4x−3) 3
3 −1, x <
 
4x−3
, x< 4 4

(1). Differentiability at x = 34 :

f (x) − f ( 34 ) f (x) − f ( 34 ) f (x) − f ( 34 )


lim− 6
= lim ⇔ lim3 DNE
x→ 34 x − 34 x→ 43
+ x − 34 x→ 4 x − 43

y0 3

4
does not exist.
(2). Differentiability in region x > 34 : consider f (x) = 1
y0 = 0
3
(3). Differentiability in region x < 4
: consider f (x) = −1
y0 = 0

 3

 0, x> 4



y0 = DNE, x = 3
4



 3
0, x<

4

Exercise 4.3 Using the definition of derivative, calculate the derivative of


2
f (x) = (3x + 2) 3
Apply Binomial expansion at a certain point.
Exercise 4.4 Using the definition of derivative, calculate the derivative of

5
f (x) =
3x + 4

Exercise 4.5 Find the derivative of the function x at the point x = 1. f 0 (1) = 1
2

H.W-Kayondo & D.W-Ddumba, Engineering Math I- Lecture Notes 125


CHAPTER 4. DIFFERENTIATION

Example 4.1.28 Given


 2
 x + 1, x ≥ 0
f (x) =
cos x, x<0

Show that f (x) is differentiable at x = 0, and f 0 (0) = 0

Remark 4.1.4 Even if f does have a derivative, it may not have a second derivative.
For example, let

 +x2 , if x ≥ 0
f (x) =
−x2 , if x < 0

Calculation shows that f is a differentiable function whose derivative is


 +2x, if x ≥ 0
f 0 (x) = = 2|x|
−2x, if x < 0

an absolute function which does not have a derivative, thus f 00 (x) does not exist.

H.W-Kayondo & D.W-Ddumba, Engineering Math I- Lecture Notes 126


CHAPTER 4. DIFFERENTIATION

4.2 Continuity Versus Differentiability


Theorem 4.2.1 Let f be differentiable at x0 . Then f is continuous at x0 .

Proof: What is known is that f is differentiable at x0 . That is

f (x) − f (x0 )
lim exists say ξ (4.6)
x→x0 x − x0

Now that known, we need to prove that the function f (x) is continuous. A function is
said to be continuous if the limit exists and equal to function at that point.

Lets compute the limit at x0 but using known information Equation (4.6)

lim f (x) = lim [f (x) − f (x0 ) + f (x0 )]


x→x0 x→x0

= lim [f (x) − f (x0 )] + lim f (x0 )


x→x0 x→x0

[f (x) − f (x0 )] (x − x0 )
= lim + lim f (x0 )
x→x0 x − x0 x→x0

f (x) − f (x0 )
 
= lim lim (x − x0 ) + lim f (x0 )
x→x0 x − x0 x→x0 x→x0

= ξ · [0] + lim f (x0 )


x→x0

= 0 + lim f (x0 )
x→x0

= lim f (x0 )
x→x0

= f (x0 )

Hence
lim f (x) = f (x0 )
x→x0

so f is continuous at x = x0 .

Remark 4.2.1 A differentiable function is a continuous function but the reverse is not
true

H.W-Kayondo & D.W-Ddumba, Engineering Math I- Lecture Notes 127


CHAPTER 4. DIFFERENTIATION

Example 4.2.1 Show that the function f (x) = |x| is continuous but not differentiable
at x = 0. Check whether the function

 x, x≥0
|x| =
−x, x < 0

is continuous at x = 0

i). f (0) = 0

ii). lim f (x)?? Since a piecewise function, to compute the limit, we use the informal
x→0

definition of limits.

From the left : lim f (x) = lim (−x) = 0


x→0− x→0

From the right : lim f (x) = lim (x) = 0


x→0+ x→0

Thus the limit exists and equal to 0, that is lim f (x) = 0


x→0

iii).
lim f (x) = f (0) = 0
x→0

Therefore, the function is continuous at x = 0.

Show that the function y = |x| is not differentiable at x = 0.

Recall that

 x, x≥0
|x| =
−x, x < 0

Now check for differentiability at x = 0, i.e., compute f 0 (0). Then

f (x) − f (a)
f 0 (a) = lim
x→a x−a

f (x) − f (0)
f 0 (0) = lim
x→0 x−0

H.W-Kayondo & D.W-Ddumba, Engineering Math I- Lecture Notes 128


CHAPTER 4. DIFFERENTIATION

Finding a limit for a piecewise function, we check from left and from right, if equal, that
is the limit, otherwise, the limit does not exist.

From the right of point x = 0

f (x) − f (0) (x) − (0) x


lim+ = lim = lim = lim 1 = 1
x→0 x−0 x→0 x x→0 x x→0

From the left of point x = 0

f (x) − f (0) (−x) − (0) −x


lim− = lim = lim = lim −1 = −1
x→0 x−0 x→0 x x→0 x x→0

f (x) − f (0)
Since the one-sided limits exist but are not equal though finite, f 0 (0) = lim
x→0 x−0
does not exist, and f is not differentiable at x = 0. This implies that the derivative of
f (x) = |x| does not exist at x = 0 as seen by the sharp curve at the point.

Figure 4.3: Graph of y = |x|

Remark 4.2.2 Sometimes a derivative may fail to exist at a point. In general, there are
three reasons why a derivative at a point may not exist.

1. The graph of the function has a sharp turn or a cusp, e.g. f (x) = |x| at x = 0.
x2 +x
2. The graph is not continuous at the point, e.g. g(x) = x
at x = 0.

Note 4.2.1 So what is the derivative, after all? The derivative measures the steepness
of the graph of a function at some particular point on the graph

H.W-Kayondo & D.W-Ddumba, Engineering Math I- Lecture Notes 129


CHAPTER 4. DIFFERENTIATION

Example 4.2.2 A function is defined by the following formula:



 x2 + 2, x≤1
f (x) =
1
 
a x− + b, x > 1

x

Find a and b such that f is continuous and differentiable. Plot the function, if possible.
(a) To be continuous at x = 1
i) f (1) = 12 + 2 = 3
ii) lim f (x)??
x→1

lim− f (x) = lim (x2 + 2) = 3


x→1 x→1

    
1
lim f (x) = lim a x − +b =b
x→1+ x→1 x

iii) For continuity, lim f (x) = f (1) ⇒ b = 3


x→1

(b) To be differentiable at x = 1

f (x) − f (a) f (x) − f (1)


f 0 (a) = lim exists ⇒ f 0 (1) = lim exists
x→a x−a x→1 x−1

Finding a limit for a piecewise function, we check from left and from right, and to be
equal since a derivative exists.
i) From the left of point x = 1

f (x) − f (1) (x2 + 2) − (3) x2 − 1


lim− = lim = lim = lim x + 1 = 2
x→1 x−1 x→1 x−1 x→1 x − 1 x→1

ii) From the right of point x = 1

1
a x − x1 + 3 − 3
   
f (x) − f (1) a x− x
+b−3
lim+ = lim = lim
x→1 x−1 x→1 x−1 x→1 x−1
a(x2 −1)
a x − x1
 
x a (x + 1)
= lim = lim = lim = 2a
x→1 x−1 x→1 x−1 x→1 x
limits to equal, 2 = 2a ⇒ a = 1

H.W-Kayondo & D.W-Ddumba, Engineering Math I- Lecture Notes 130


CHAPTER 4. DIFFERENTIATION

Example 4.2.3 We wish to determine the values of the parameters k and m for which
the function below is differentiable at x = 3:
 √
 k x + 1, 0 ≤ x ≤ 3
f (x) =
5 − mx, 3<x≤5

For a function to be differentiable at a domain value,


(i) the function must be continuous there.

(i) the derivative exists (the pieces must match with the same slope).

(a) To be continuous at x = 3

i) f (3) = k 3 + 1 = 2k
ii) lim f (x) exists, and since a piecewise function,
x→3

 √ 
lim− f (x) = lim k x + 1 = 2k
x→3 x→3

lim f (x) = lim (5 − mx) = 5 − 3m


x→3+ x→3

iii) The limits should be equal and equal to f (3) to be continuous.

5 − 3m = 2k (4.7)

(b) To be differentiable at x = 3

f (x) − f (3)
f 0 (3) = lim exists
x→3 x−3

Finding a limit for a piecewise function, we check from left and from right, and to be
equal since a derivative exists.

i) From the left of point x = 3


f (x) − f (3) (k x + 1) − (2k) 0
lim− = lim = ⇒ La’Hopital
x→3 x−3 x→3 x−3 0
1
1
2
k(x + 1)− 2 1
k 1
= lim = lim √ 2 = k
x→3 1 x→3 x+1 4

H.W-Kayondo & D.W-Ddumba, Engineering Math I- Lecture Notes 131


CHAPTER 4. DIFFERENTIATION

ii) From the right of point x = 3

f (x) − f (3) (5 − mx) − (2k) (5 − 3m) − 2k


lim+ = lim =
x→3 x−3 x→3 x−3 0

0 : Eqn (4.7)
= ⇒ La’Hopital
0
−m
= lim = −m
x→3 1

iii) For derivative to exist, the limits must be equal,

1
k = −m (4.8)
4

To be differentiable, it has to be continuous and derivative eists. Thus solving the simul-
taneous equations (4.7) and (4.8)

5 − 3m = 2k
k
= −m
4

⇒ k = 4, m = −1

Example 4.2.4 Let


 1
2
 x cos
 , if x 6= 0
f (x) = x

0, if x = 0

Show that f is continuous for all values of x. Show that f is differentiable for all values
of x, but that the derivative, f 0 , is not continuous at x = 0.

First show that f is continuous for all values of x. Describe f using functional composition.
Let
1
g(x) = , h(x) = cos x, and k(x) = x2
x

Function h is well-known to be continuous for all values of x.

H.W-Kayondo & D.W-Ddumba, Engineering Math I- Lecture Notes 132


CHAPTER 4. DIFFERENTIATION

Function k is a polynomial and is therefore continuous for all values of x.

Function g is the quotient of functions continuous for all values of x, and is therefore
continuous for all values of x except x = 0, that x which makes the denominator zero.
Thus, for all values of x except x = 0

1
2 2
f (x) = k(x)h(g(x)) = x cos (g(x)) = x cos
x

is a continuous function (the product and functional composition of continuous functions).


Continuity of f at x = 0. Function f is defined at x = 0 since

1 1
(a) The limit lim cos does not exist since the values of cos oscillate between −1
x→0 x x
and +1 as x approaches zero. However, for x 6= 0

1
−1 ≤ cos ≤ +1
x

so that
1
−x2 ≤ x2 cos ≤ x2 .
x

Since
lim (−x2 ) = 0 = lim x2 ,
x→0 x→0

it follows from the Squeeze Principle that

1
lim f (x) = lim x2 cos = 0.
x→0 x→0 x

(b) The function is defined at x = 0


f (0) = 0.

(c)
lim f (x) = 0 = f (0),
x→0

all three conditions are satisfied, and f is continuous at x = 0. Thus, f is continuous for
all values of x.

H.W-Kayondo & D.W-Ddumba, Engineering Math I- Lecture Notes 133


CHAPTER 4. DIFFERENTIATION

Now show that f is differentiable for all values of x. For x 6= 0 we can differentiate f
using the product rule and the chain rule. That is, for x 6= 0 the derivative of f is

n  1 o 1
f 0 (x) = x2 D cos + D{x2 } cos
x x
n  1  n 1 oo 1
2
= x − sin D + {2x} cos
x x x
 1 n −1 o 1
= −x2 sin + 2x cos
x x2 x
1 1
= sin + 2x cos
x x

Use the limit definition of the derivative to differentiate f at x = 0. Then

f (0 + h) − f (0)
f 0 (0) = lim
h→0 h
f (h) − 0
= lim
h→0 h
1
(h)2 cos
= lim h
h→0 h
1
= lim h cos
h→0 h

Use the Squeeze Principle to evaluate this limit. For h 6= 0

1
−1 ≤ cos ≤ +1.
h

If h > 0, then
1
−h ≤ h cos ≤ h.
h

If h < 0, then
1
−h ≥ h cos ≥ h.
h

H.W-Kayondo & D.W-Ddumba, Engineering Math I- Lecture Notes 134


CHAPTER 4. DIFFERENTIATION

In either case,
lim (−h) = 0 = lim h,
h→0 h→0

and it follows from the Squeeze Principle that

1
0
f (0) = lim h cos = 0.
h→0 h

Thus, f is differentiable for all values of x. Check to see if f 0 is continuous at x = 0. The


function f 0 is defined at x = 0 since

(a) However,
 1  1 
0
lim f (x) = lim sin + 2x cos
x→0 x→0 x x
1
does not exist since the values of sin oscillate between −1 and +1 as x approaches
x
zero.

(b)
f 0 (0) = 0

Thus, condition (a) is violated, and the derivative, f 0 , is not continuous at x = 0.

Note 4.2.2 The continuity of function f for all values of x also follows from the fact
that f is differentiable for all values of x.

H.W-Kayondo & D.W-Ddumba, Engineering Math I- Lecture Notes 135


CHAPTER 4. DIFFERENTIATION

4.3 Differentiation Theorems


Let f (x) and g(x) be differentiable and α a scalar, then αf (x), (f + g)(x), (f g)(x), ( fg )(x)
are all differentiable functions such that

(a) Constant rule: if f (x) is constant, then

f 0 (x) = 0

(b) The scalar multiplication


(αf )0 (x) = αf 0 (x)

(c) The sum rule


(f + g)0 (x) = f 0 (x) + g 0 (x)

(d)
(f g)0 (x) = f 0 (x).g(x) + g 0 (x)f (x)
This is popularly known as the product rule

(e)
 0
f f 0 (x)g(x) − g 0 (x)f (x)
(x) =
g [g(x)]2

This is popularly known as the quotient rule of differentiation

Example 4.3.1 Prove that

(f + g)0 (x) = f 0 (x) + g 0 (x)

(f + g)(x + h) − (f + g)(x)
(f + g)0 (x) = lim
h→0 h
f (x + h) + g(x + h) − f (x) − g(x)
= lim
h→0 h
f (x + h) − f (x) g(x + h) − g(x)
= lim + lim
h→0 h h→0 h

= f 0 (x) + g 0 (x)

Exercise 4.6 Prove that (f − g)(x) = f 0 (x) − g 0 (x)

Example 4.3.2 Let f (x) and g(x) be differentiable and α a scaler, prove that,

H.W-Kayondo & D.W-Ddumba, Engineering Math I- Lecture Notes 136


CHAPTER 4. DIFFERENTIATION

(i) (αf )0 (x) = αf 0 (x)


(ii) (f g)0 (x) = f 0 (x)g(x) + g 0 (x)f (x)

(f g)(x + h) − (f g)(x)
(f g)0 (x) = lim
h→0 h
f (x + h)g(x + h) − f (x)g(x)
= lim
h→0 h
f (x + h)g(x + h) − f (x)g(x + h) + f (x)g(x + h) − f (x)g(x)
= lim
h→0 h
g(x + h) [f (x + h) − f (x)] + f (x) [g(x + h) − g(x)]
= lim
h→0 h
g(x + h) [f (x + h) − f (x)] f (x) [g(x + h) − g(x)]
= lim + lim
h→0 h h→0 h
[f (x + h) − f (x)] [g(x + h) − g(x)]
= lim g(x + h) lim + lim f (x) lim
h→0 h→0 h h→0 h→0 h

= g(x)f 0 (x) + f (x)g 0 (x)

Exercise 4.7 Show that


 0
f f 0 (x)g(x) − g 0 (x)f (x)
(x) =
g [g(x)]2

d 2
Example 4.3.3 Use the product rule -part(c) of the theorem to find dx
(ex sin x)

2 2
Let f(x) = ex ⇒ f 0 (x) = 2xex
and g(x) = sin x ⇒ g 0 (x) = cos x
⇒ By the theorem, we have that,
(f g)0 (x) = f 0 (x)g(x) + g 0 (x)f (x)
2 2
= 2xex sin x + cos xex
2
= ex (2x sin x + cos x)
Example 4.3.4 Use the quotient rule of the theorem to compute

sin2 x
 
d
dx 1 − e−x

H.W-Kayondo & D.W-Ddumba, Engineering Math I- Lecture Notes 137


CHAPTER 4. DIFFERENTIATION

f (x) = sin2 x ⇒ f 0 (x) = 2 sin x cos x


g(x) = 1 − e−x ⇒ g 0 (x) = e−x

By the theorem, we have that,

 0
f f 0 (x)g(x) − g 0 (x)f (x)
(x) =
g [g(x)]2

2 sin x cos x(1 − e−x ) − e−x sin2 x


=
(1 − e−x )2

Example 4.3.5

d d
· (x − 2) − 1 · dx (x − 2) 0 · (x − 2) − 1 · 1 −1
 
d 1 dx
1
= = =
dx x−2 (x − 2) 2 (x − 2) 2 (x − 2)2

Example 4.3.6

x−1 (x − 1)0 (x − 2) − (x − 1)(x − 2)0 1 · (x − 2) − (x − 1) · 1


 
d
= =
dx x−2 (x − 2) 2 (x − 2)2

(x − 2) − (x − 1) −1
= =
(x − 2) 2 (x − 2)2

Example 4.3.7

5x3 + x (5x3 + x)0 · (2 − x7 ) − (5x3 + x) · (2 − x7 )


 
d
=
dx 2 − x7 (2 − x7 )2

(15x2 + 1) · (2 − x7 ) − (5x3 + x) · (−7x6 )


=
(2 − x7 )2

and there’s hardly any point in simplifying the last expression, unless someone gives you
a good reason. In general, it’s not so easy to see how much may or may not be gained in
‘simplifying’, and we won’t make ourselves crazy over it.

H.W-Kayondo & D.W-Ddumba, Engineering Math I- Lecture Notes 138


CHAPTER 4. DIFFERENTIATION

Note 4.3.1 One way that the product rule can be useful is in postponing or eliminating
a lot of algebra. For example, to evaluate

d
(x3 + x2 + x + 1)(x4 + x3 + 2x + 1)

dx

we could multiply out and then take the derivative term-by-term as we did with several
polynomials above. This would be at least mildly irritating because we’d have to do a bit
of algebra. Rather, just apply the product rule without feeling compelled first to do any
algebra:
d
(x3 + x2 + x + 1)(x4 + x3 + 2x + 1)

dx

= (x3 + x2 + x + 1)0 (x4 + x3 + 2x + 1) + (x3 + x2 + x + 1)(x4 + x3 + 2x + 1)0


= (3x2 + 2x + 1)(x4 + x3 + 2x + 1) + (x3 + x2 + x + 1)(4x3 + 3x2 + 2)
Now if we were somehow still obliged to multiply out, then we’d still have to do some
algebra. But we can take the derivative without multiplying out, if we want to, by using
the product rule.
For that matter, once we see that there is a choice about doing algebra either before
or after we take the derivative, it might be possible to make a choice which minimizes our
computational labor. This could matter.
Example 4.3.8 Suppose we want to differentiate y = x2 cos 3x.

dy
= x(−3x sin 3x + 2 cos 3x)
dx

Example 4.3.9 Suppose we want to differentiate


y = x3 (4 − x)1/2

f (x) = x3 ⇒ f 0 (x) = 3x2

1
g(x) = (4 − x)1/2 ⇒ g 0 (x) = − (4 − x)−1/2
2

(f g)0 (x) = f 0 (x).g(x) + g 0 (x)f (x)

dy x3
= (3x2 )(4 − x)1/2 −
dx 2(4 − x)1/2

dy (4 − x)1/2 · 3x2 2(4 − x)1/2 x3 6x2 (4 − x) − x3 x2 (24 − 7x)


= · − = =
dx 1 2(4 − x)1/2 2(4 − x)1/2 2(4 − x)1/2 2(4 − x)1/2

H.W-Kayondo & D.W-Ddumba, Engineering Math I- Lecture Notes 139


CHAPTER 4. DIFFERENTIATION

Example 4.3.10 Suppose we want to differentiate y = (1 − x3 )e2x .

dy
= (1 − x3 ) × 2e2x + e2x × (−3x2 )
dx

= e2x (2 − 3x2 − 2x3 )

Example 4.3.11 Find the derivative of each of the following:

a) x tan x f) x−2 (1 + x2 )1/2


2
x sec x + tan x
b) x2 e−x −x−3 ((1 + x2 )−1/2 (2 + x2 )
x(2 − x)e−x
c) 5e−2x sin 3x
g) xex sin x
−2x
5e (3 cos 3x − 2 sin 3x)

d) 3x1/2 cos 2x ex [(1 + x) sin x + cos x]

3 −1/2
x (cos 2x − 4x sin 2x)
2 h) 7x3/2 e−4x cos 2x

e) 2x6 (1 + x)5
7 1/2 −4x
2x5 (1 + x)4 (6 + 11x) x e (3 cos 2x − 8x cos 2x − 4x sin x)
2

Example 4.3.12 Compute the derivative of y = (2x2 + 6x)(2x3 + 5x2 )

y 0 = (2x2 + 6x)(6x2 + 10x) + (2x3 + 5x2 )(4x + 6) = 20x4 + 88x3 + 90x2


d
Exercise 4.8 Find dx
(f (x)) given that

1
(i) f (x) = 2x 2 − x3 + 2

(ii) f (x) = αx3 + βx2 + λx + θ where α, β, λ and θ are constants.

H.W-Kayondo & D.W-Ddumba, Engineering Math I- Lecture Notes 140


CHAPTER 4. DIFFERENTIATION

4.4 Other techniques of differetiation


Other than the sum, difference, product, quotient or constant differentiation, other forms
of differentitaion include

(i) Chain rule (iii) Parametric differentiation

(ii) Implicit differentiation (iv) Logarithmic differentiation

4.4.1 Chain Rule - Composite differentiation


Theorem 4.4.1 Let g(x) be differentiable at x and h(g) be differentiable at g(x), then
hog(x) = h(g(x)) is differentiable at x and if f (x) = h(g(x)), then

d dh dg
f 0 (x) = [h(g(x))] = .
dx dg dx

f 0 (x) = h0 (g(x)) · g 0 (x)


Proof

d h(g(x + h)) − h(g(x))


[h(g(x))] = lim
dx h→0 h
h(g(x + h)) − h(g(x)) g(x + h) − g(x)
= lim .
h→0 g(x + h) − g(x) h

Let ∆g = g(x + h) − g(x)

h(g + h) − h(g) g(x + h) − g(x)


lim ∆g = 0 = lim . lim
h→0 h→0 ∆g h→0 h

Note 4.4.1 The chain rule can only be used when you can express a function f given
as a composite of two functions h and g.
Example 4.4.1 Using the chain rule find f 0 (x) for

1
f (x) =
(4x2 − x)5

This can be decomposed as the composite of two functions:


f (x) = h(g(x))
g(x) = 4x2 − x,

1
h(g) =
g5

H.W-Kayondo & D.W-Ddumba, Engineering Math I- Lecture Notes 141


CHAPTER 4. DIFFERENTIATION

Their derivatives are:

dg dh −5
= 8x − 1 , = −5g −6 = 6
dx dg g

The derivative function is therefore:

df dh dg −5 5(8x − 1)
= · = 6 · (8x − 1) = −
dx dg dx g (4x2 − x)6

Example 4.4.2 Differentiate


f (x) = (3x + 1)2
In a short form, its
f 0 (x) = 2(3x + 1)(3)
f 0 (x) = 6(3x + 1)
To use the chain rule, this can be decomposed as the composite of two functions:
f (x) = h(g(x))
g(x) = 3x + 1,
h(g) = g 2
Their derivatives are:

dg dh
= 3, = 2g
dx dg

The derivative function is therefore:

df dh dg
= · = 2g(3) = 6g = 6(3x + 1)
dx dg dx

Example 4.4.3 Differentiate



y= 13x2 − 5x + 8

1 1
y0 = (13x2 − 5x + 8)− 2 (26x − 5)
2
26x − 5
y0 = √
2 13x2 − 5x + 8

H.W-Kayondo & D.W-Ddumba, Engineering Math I- Lecture Notes 142


CHAPTER 4. DIFFERENTIATION

Example 4.4.4 Differentiate

y = (1 − 4x + 7x5 )30

y 0 = 30(1 − 4x + 7x5 )29 (−4 + 35x4 )


y 0 = 30(35x4 − 4)(1 − 4x + 7x5 )29

Example 4.4.5 Differentiate


1
y = (4x + x−5 ) 3

1 2
y0 = (4x + x−5 )− 3 (4 − 5x−6 )
3

Example 4.4.6 Differentiate


− 45
8x − x6

y=
x3

First, begin by simplifying the expression before we differentiate it

− 45
y = 8x−2 − x3

4 − 9
y0 = − 8x−2 − x3 5 (−16x−3 − 3x2 )
5

Example 4.4.7 The derivative of

f (x) = x4 + sin(x2 ) − ln(x)ex + 7

d (x2 ) d (ln x) x d (ex )


f 0 (x) = 4x(4−1) + cos(x2 ) − e − ln x +0
dx dx dx
1
= 4x3 + 2x cos(x2 ) − ex − ln(x)ex .
x

Here the second term was computed using the chain rule and third using the product
rule. The known derivatives of the elementary functions x2 , x4 , sin x, ln(x) and ex , as well
as the constant 7, were also used.

H.W-Kayondo & D.W-Ddumba, Engineering Math I- Lecture Notes 143


CHAPTER 4. DIFFERENTIATION

Example 4.4.8 For concreteness, consider the function


2
f (x) = esin x
This can be decomposed as the composite of three functions:
f (x) = h(g(p(x)))
p(x) = x2 ,
g(p) = sin p,
h(g) = eg
Their derivatives are:

dp dg dh
= 2x, = cos p, = eg
dx dp dg

The derivative function is therefore:

df dh dg dp
= · ·
dx dg dp dx

df 2
= esin x · cos x2 · 2x
dx

Exercise 4.9 Differentiate


y = sin(5x)
Exercise 4.10 Differentiate
2 +7x−13
y = e5x
Exercise 4.11 Differentiate √
y = 3 tan x

0 3 sec2 x
y = √
2 x

Exercise 4.12 Differentiate


y = cos2 (x3 )
Example 4.4.9 Use the chain rule to differentiate
f (x) = (x3 + 5x)7

f (x) = h(g(x))
3
g(x) = x + 5x, h(g) = g 7
f 0 (x) = 7(x3 + 5x)6 · (3x2 + 5)

H.W-Kayondo & D.W-Ddumba, Engineering Math I- Lecture Notes 144


CHAPTER 4. DIFFERENTIATION

Example 4.4.10 Use the chain rule to differentiate



f (x) = 5 cos x

f (x) = h(g(x))
1
g(x) = 5 cos x, h(g) = g 2

1 1
f 0 (x) = (5 cos x)− 2 · 5(− sin x)
2

Example 4.4.11 Use the chain rule to differentiate


2 −5
f (x) = 7ex

f (x) = h(g(x))
2
g(x) = x − 5, h(g) = 7eg
2 −5
f 0 (x) = 7ex · (2x)

Example 4.4.12 Use the chain rule to differentiate

f (x) = −3 tan(5x4 )

f (x) = h(g(x))
g(x) = 5x4 , h(g) = −3 tan g
0
f (x) = −3 sec2 (5x4 ) · (20x3 )

Example 4.4.13 Use the chain rule to differentiate

8
f (x) =
4 + sin x

f (x) = h(g(x))

8
g(x) = 4 + sin x, h(g) =
g

f 0 (x) = −8(4 + sin x)−2 · cos x

H.W-Kayondo & D.W-Ddumba, Engineering Math I- Lecture Notes 145


CHAPTER 4. DIFFERENTIATION

Example 4.4.14 Find the derivative of f (x) = sin(5x) using the chain rule.

h i
f 0 (x) = 5 · cos(5x) = 5 cos(5x)

Example 4.4.15 Find the derivative of

 2
2 2
f (t) = t − 3
t

We will use the Chain rule. Set f = y(u(t))

2
u = t2 − 3
and y = u2
t

The Chain rule implies


df du dy
=
dt dt du

Since
du 6 dy
= 2t + 4 and = 2u
dt t du

we get
    
df 6 6 2 2
= 2t + 4 2u = 2 2t + 4 t − 3
dt t t t

Exercise 4.13 Use the chain rule to find f 0 (x) for

1 1
(a) f (x) = (4 − x) 2 h(g) = g 2

(b) f (x) = (7x2 − 5x)3 h(g) = g 3

(c) f (x) = 1
(3x−2)
h(g) = g −1 = 1
g

(d) f (x) = 5 + cos3 x



(e) f (x) = 3 1 + tan x

H.W-Kayondo & D.W-Ddumba, Engineering Math I- Lecture Notes 146


CHAPTER 4. DIFFERENTIATION

4.4.2 Differentiation of implicit functions


Implicit differentiation is nothing more than a special case of the well-known chain rule
for derivatives. The majority of differentiation problems in first-year calculus involve
functions y written explicitly as functions of x. For example, if

y = 3x2 − sin(7x + 5)

then the derivative of y is


y 0 = 6x − 7 cos(7x + 5)
However, some functions y are written implicitly as functions of x. A familiar example of
this is the equation
x2 + y 2 = 25
which represents a circle of radius five centered at the origin. Suppose that we wish to
find the slope of the line tangent to the graph of this equation at the point (3, −4)

How could we find the derivative of y in this instance ? One way is to first write y
explicitly as a function of x. Thus,

x2 + y 2 = 25 ⇒ y 2 = 25 − x2

and √
y = ± 25 − x2
where the positive square root represents the top semi-circle and the negative square root
represents the bottom semi-circle. Since the point (3, −4) lies on the bottom semi-circle
given by √
y = − 25 − x2
i.e,
x
y0 = √
25 − x2

Thus, the slope of the line tangent to the graph at the point (3, −4) is

3 3
m = y0 = p =
25 − (3)2 4

Unfortunately, not every equation involving x and y can be solved explicitly for y

With Implicit differentiation, we differentiate both sides with respect to x, and make y 0
the subject.

H.W-Kayondo & D.W-Ddumba, Engineering Math I- Lecture Notes 147


CHAPTER 4. DIFFERENTIATION

Example 4.4.16 Differentiate


x2 + y 2 = 25

x2 + y 2 = 25
D(x2 ) + D(y 2 ) = D(25)
2x + 2yy 0 = 0
2yy 0 = −2x

−2x −x
y0 = =
2y y

Thus, the slope of the line tangent to the graph at the point (3, −4) is

−x −(3) 3
m = y0 = = =
y (−4) 4

Example 4.4.17 Assume that y is a function of x. Find y 0 = dy/dx for

x3 + y 3 = 4

Begin with x3 + y 3 = 4. Differentiate both sides of the equation, getting

x3 + y 3 = 4
D(x3 + y 3 ) = D(4)
D(x3 ) + D(y 3 ) = D(4)

Remember to use the chain rule on D(y 3 )

3x2 + 3y 2 y 0 = 0

so that (Now solve for y 0 )


3y 2 y 0 = −3x2
and
−x2
y0 =
y2

Example 4.4.18 Assume that y is a function of x. Find y 0 = dy/dx for

(x − y)2 = x + y − 1

H.W-Kayondo & D.W-Ddumba, Engineering Math I- Lecture Notes 148


CHAPTER 4. DIFFERENTIATION

Begin with (x − y)2 = x + y − 1. Differentiate both sides of the equation, getting

(x − y)2 = x+y−1
D(x − y)2 = D(x + y − 1)
D(x − y)2 = D(x) + D(y) − D(1)
2(x − y)D(x − y) = 1 + y0 − 0
2(x − y)(1 − y 0 ) = 1 + y0

Now solve for y 0

y 0 [−2(x − y) − 1] = 1 − 2(x − y)

1 − 2(x − y) 2y − 2x + 1
y0 = =
−2(x − y) − 1 2y − 2x − 1

Example 4.4.19 Assume that y is a function of x. Find y 0 = dy/dx for

y = sin(3x + 4y)

Begin with y = sin(3x + 4y). Differentiate both sides of the equation, getting

y = sin(3x + 4y)
D(y) = D (sin(3x + 4y))
y0 = cos(3x + 4y)D (3x + 4y)
y0 = cos(3x + 4y) (3 + 4y 0 )

Now solve for y 0

y 0 [1 − 4 cos(3x + 4y)] = 3 cos(3x + 4y)

3 cos(3x + 4y)
y0 =
1 − 4 cos(3x + 4y)

Example 4.4.20 Assume that y is a function of x. Find y 0 = dy/dx for

y = x2 y 3 + x3 y 2

Begin with y = x2 y 3 + x3 y 2 . Differentiate both sides of the equation, getting

y = x2 y 3 + x3 y 2
D x2 y 3 + x3 y 2

D(y) =
y0 = D(x2 y 3 ) + D(x3 y 2 )
y0 = x2 (3y 2 y 0 ) + (2x)y 3 + x3 (2yy 0 ) + (3x2 )y 2
y0 = 3x2 y 2 y 0 + 2xy 3 + 2x3 yy 0 + 3x2 y 2

H.W-Kayondo & D.W-Ddumba, Engineering Math I- Lecture Notes 149


CHAPTER 4. DIFFERENTIATION

Now solve for y 0

2xy 3 + 3x2 y 2
y0 =
1 − 3x2 y 2 − 2x3 y

Example 4.4.21 Assume that y is a function of x. Find y 0 = dy/dx for

exy = e4x − e5y

4e4x − yexy
y0 =
xexy + 5e5y

Example 4.4.22 Assume that y is a function of x. Find y 0 = dy/dx for

cos2 x + cos2 y = cos(2x + 2y)

[cos x sin x − sin(2x + 2y)]


y0 =
[sin(2x + 2y) − cos y sin y]

Example 4.4.23 Assume that y is a function of x. Find y 0 = dy/dx for

p
x= x2 + y 2

p
0 x2 + y 2 − x
y =
y

Example 4.4.24 Assume that y is a function of x. Find y 0 = dy/dx for

x − y3
=x+2
y + x2

1 − y − 3x2 − 4x
y0 =
3y 2 + x + 2

H.W-Kayondo & D.W-Ddumba, Engineering Math I- Lecture Notes 150


CHAPTER 4. DIFFERENTIATION

Example 4.4.25 Assume that y is a function of x. Find y 0 = dy/dx for


(x2 + y 2 )3 = 8x2 y 2

0 16xy 2 − 6x(x2 + y 2 )2
y =
6y(x2 + y 2 )2 − 16x2 y

dy
Example 4.4.26 For the function, x2 + y 3 = 5y, find dx

Since x2 + y 3 = 5y

d 2 d d
⇒ (x ) + (y 3 ) = (5y)
dx dx dx
dy dy
⇒ 2x + 3y 2 = 5
dx dx
dy
⇒ (3y 2 − 5) = −2x
dx
−2x
 
dy
⇒ =
dx 3y 2 − 5

dy
Example 4.4.27 Find dx
for x2 y − 2x3 y 2 = 4

We differentiate term by term with respect to x i.e

d 2 d d
⇒ (x y) − 2 (x3 y 2 ) = (4)
dx dx dx
dy dy
⇒ 2xy + x2 − 2(3x2 y 2 + 2x3 y ) = 0
dx dx
dy dy
⇒ 2xy + x2 − 6x2 y 2 − 4x3 y = 0
dx dx
dy 2
⇒ (x − 4x3 y) = 6x2 y 2 − 2xy
dx
dy 6x2 y 2 − 2xy
therefore =
dx x2 − 4x3 y

6xy 2 − 2y
=
x − 4x2 y

H.W-Kayondo & D.W-Ddumba, Engineering Math I- Lecture Notes 151


CHAPTER 4. DIFFERENTIATION

4.4.3 Parametric equations


To differentiate parametric equations, we must use the chain rule.

The equations of a plane curve f (x, y) = 0 may be given by equations of the type x = x(t)
and y = y(t), where t is the variable called a parameter. These equations are called
parametric equations of the curve.

Example 4.4.28 Consider the parametric equations

x = cos t, y = sin t for 0 ≤ t ≤ 2π

dy
dy dy dt dt cos t
= = dx
= = −cot t
dx dt dx dt
− sin t

Example 4.4.29 If x = 2at2 and y = 4at, find dy/dx and dx/dy

dy dy dt 1 1
= · = (4a) · =
dx dt dx 4at t
dx dy dt 1
= · = (4at) · =t
dy dt dy 4a

Example 4.4.30 Finding the second derivative is a little trickier.

d2 y
     
d dy d dy dt d dy dt
2
= = · =
dx dx dx dx dt dx dt dx dx

or Example (4.4.29)

d2 y −1
     
d dy dt d 1 dt 1
2
= = = 2
dx dt dx dx dt t dx t 4at

Exercise 4.14 Given


x = t3 − t y = 4 − t2
Compute

(a) dx/dy (c) (dy/dx)2

(b) dy/dx (d) d2 y/dx2

H.W-Kayondo & D.W-Ddumba, Engineering Math I- Lecture Notes 152


CHAPTER 4. DIFFERENTIATION

Example 4.4.31 Find the equation of a Curve whose parametric equations are
(i) x = t and y = 5t + 6

(ii) x = α cos θ and y = β sin θ


To solve, just eliminate the parameter
(i) We eliminate the parameter t from the equations i.e y = 5x + 6

(ii)

x2 = α2 cos2 θ
x2
⇒ 2
= cos2 θ (4.9)
α

and y 2 = β 2 sin2 θ

y2
⇒ = sin2 θ (4.10)
β2

Adding equations (4.9) to (4.10) we have

x2 y2
2
+ 2
= cos2 θ + sin2 θ
α β

x2 y2
⇒ + = 1
α2 β 2

Which is an ellipse

Derivatives of parametrically defined curves


If we let x = x(t) and y = y(t) be parametric equations of f (x, y) = 0 this means that,

dx dy
= x0 (t), = y 0 (t)
dt dt
dy dy dt
but = . (Chain rule )
dx dt dx
dy
y 0 (x)
= dt
dx
= 0
provided (x0 (t) 6= 0)
dt
x (t)

H.W-Kayondo & D.W-Ddumba, Engineering Math I- Lecture Notes 153


CHAPTER 4. DIFFERENTIATION

Example 4.4.32 Compute dy/dx for the following parametric equations


(i) x = t2 and y = 4t2 + 5
t3 1
(ii) x = t and y = 3
+ 7t2

(iii) x = et cos t and y = et sin t (0 ≤ t ≤ π)

(i) Since x(t) = t2 ⇒ x0 (t) = 2t

and y = 4t2 + 5 ⇒ y 0 (t) = 8t

dy y 0 (t) 8t
therefore = 0 = =4
dx x (t) 2t

(ii) x = t ⇒ x0 (t) = 1

t3 1 −2 2
and y = + t ⇒ y 0 (t) = t2 − t−3
3 7 7
dy y 0 (t) 2
therefore = 0 = t2 − 3
dx x (t) 7t

(iii)

Since x = et cos t, y = et sin t (0 ≤ t ≤ π)

x0 (t) = et cos t − et sin t


y 0 (t) = et sin t + et cos t

dy et (cos t + sin t)
⇒ = t
dx e (cos t − sin t)

(cos t + sin t)
=
(cos t − sin t)

This holds only when cos t 6= sin t


Example 4.4.33 Find the tangent line(s) to the parametric curve given by

x = t5 − 4t3 , y = t2

dy 2
= 3
dx 5t − 12t

H.W-Kayondo & D.W-Ddumba, Engineering Math I- Lecture Notes 154


CHAPTER 4. DIFFERENTIATION

Example 4.4.34 Given


x(t) = t3 , y(t) = t4

dy 4 d2 y 4
= t 2
= 2
dx 3 dx 9t

Example 4.4.35
x = t + cos t, y = sin t

dy cos t
=
dx 1 − sin t

d2 y − sin t + 1 1
= =
dx 2 (1 − sin t)3 (1 − sin t)2

Example 4.4.36 Find the second derivative if x = t − t2 , y = t − t3

d2 y 1 − 3t2
=
dx2 (1 − 2t)2

Exercise 4.15 Given an equation

y = sin 2p
x = cosp

Compute the ordinary differential equations

(a) dx/dy

(b) (dy/dx)2

H.W-Kayondo & D.W-Ddumba, Engineering Math I- Lecture Notes 155


CHAPTER 4. DIFFERENTIATION

4.4.4 Logarithmic differentiation


Logarithmic differentiation is a powerful technique for differentiating functions . However,
the method is ”uneconomical ” for simple functions like polynomial functions.

This is done by

(i). Let y, be the function to differentiate

(ii). Take logs on both sides, take loge = ln

(iii). Differentiate both sides

(iv). Make dy/dx the subject

(v). Substitute back the y value.

We here present some common suitable forms for the logarithmic differentiation.

(a)
y = u(x)v(x)
where u(x) and v(x) are quite big expressions .On differentiating, we take logarithms
to base e on both sides i.e

ln y = ln u(x) + ln v(x)

1 dy 1 1 0
⇒ = .u0 (x) + v (x)
y dx u(x) v(x)

dy u0 (x) v 0 (x)
⇒ = y( + )
dx u(x) v(x)

u0 (x) v 0 (x)
= v(x)u(x)( + )
u(x) v(x)

= v(x)u0 (x) + u(x)v 0 (x)

(b)
u(x)v(x)
y=
h(x)g(x)

H.W-Kayondo & D.W-Ddumba, Engineering Math I- Lecture Notes 156


CHAPTER 4. DIFFERENTIATION

Taking logs to both sides, we have

ln y = ln u(x) + ln v(x) − ln h(x) − ln g(x)

1 dy u0 (x) v 0 (x) h0 (x) g 0 (x)


= + − −
y dx u(x) v(x) h(x) g(x)

dy u0 (x) v 0 (x) h0 (x) g 0 (x)


⇒ = y[( + − − )].
dx u(x) v(x) h(x) g(x)

(c)
y = (u(x))v(x)
Taking logs on both sides we have,

ln y = v(x) ln u(x)

1 dy u0 (x)
= v 0 (x) ln u(x) + v(x)
y dx u(x)

Example 4.4.37 Find the derivative of

(x2 + 1)3 (x + 1)4


p p
x (x − 1) (x + 3)

Using logarithmic differentiation,

(x2 + 1)3 (x + 1)4


Let y = p p
x (x − 1) (x + 3)

1 1
⇒ ln y = 3 ln(x2 + 1) + 4 ln(x + 1) − ln x −
ln(x − 1) − ln(x + 3)
2 2
 
1 dy 6x 4 1 1 1
⇒ = + − − −
y dx x2 + 1 x + 1 x 2(x − 1) 2(x + 3)
 
dy 6x 4 1 1 1
therefore = + − − − y
dx x2 + 1 x + 1 x 2(x − 1) 2(x + 3)

(x2 + 1)3 (x + 1)4


 
6x 4 1 1 1
= + − − −
x2 + 1 x + 1 x 2(x − 1) 2(x + 3) x (x − 1) (x + 3)
p p

H.W-Kayondo & D.W-Ddumba, Engineering Math I- Lecture Notes 157


CHAPTER 4. DIFFERENTIATION

Example 4.4.38 Given


y = (sin x)cos x
find the derivative dy/dx

Since y = (sin x)cosx


⇒ ln y = cos x ln(sin x)

1 dy cos2 x
⇒ = − sin x ln sin x +
y dx sin x
 2 
dy cos x
⇒ = − sin x ln sin x (sin x)cos x
dx sin x

Note 4.4.2 In this case it is only logarithmic differentiation which is applicable

Example 4.4.39 Differentiate


y = xx
Because a variable is raised to a variable power in this function, the ordinary rules of
differentiation do not apply ! The function must first be revised before a derivative can
be taken. Begin with
y = xx
Apply the natural logarithm to both sides of this equation getting

ln y = ln xx
ln y = x ln x

Differentiate both sides of this equation. The left-hand side requires the chain rule since
y represents a function of x. Use the product rule on the right-hand side. Thus, differen-
tiating, we get
1 0 1
y = x + (1) ln x = 1 + ln x
y x

Multiply both sides of this equation by y (making y 0 the subject), getting

y 0 = y(1 + ln x) = xx (1 + ln x)

H.W-Kayondo & D.W-Ddumba, Engineering Math I- Lecture Notes 158


CHAPTER 4. DIFFERENTIATION

Example 4.4.40 Differentiate the function

(x + 2)(x − 6)3 (x + 4)2


y=
(x − 3)

(x + 2)(x − 6)3 (x + 4)2


 
ln y = ln = ln(x + 2) + ln(x − 6)3 + ln(x + 4)2 − ln(x − 3)
(x − 3)
dy
dx 1 3(x − 6)2 2(x + 4) 1
= + + −
y (x + 2) (x − 6)3 (x + 4) 2 (x − 3)

1 3 2 1
= + + −
(x + 2) (x − 6) (x + 4) (x − 3)
 
dy 1 3 2 1
= y + + −
dx (x + 2) (x − 6) (x + 4) (x − 3)

(x + 2)(x − 6)3 (x + 4)2


  
1 3 2 1
= + + −
(x − 3) (x + 2) (x − 6) (x + 4) (x − 3)

Example 4.4.41 Differentiate


x)
y = x(e
Because a variable is raised to a variable power in this function, the ordinary rules of
differentiation DO NOT APPLY ! The function must first be revised before a derivative
can be taken.

Apply the natural logarithm to both sides of this equation getting


x
ln y = ln x(e ) = ex ln x
Differentiate both sides of this equation. The left-hand side requires the chain rule since
y represents a function of x. Use the product rule on the right-hand side.

1 0 x 1
n o
y = e + ex ln x
y x

1 0 ex n x o x ex xex ln x ex + xex ln x ex (1 + x ln x)
y = + e ln x = + = =
y x x x x x x

Multiply both sides of this equation by y, getting (by combining the powers of x)

ex (1 + x ln x) x
x e (1 + x ln x) x
y0 = y = x(e ) 1
= x(e −1) ex (1 + x ln x)
x x

H.W-Kayondo & D.W-Ddumba, Engineering Math I- Lecture Notes 159


CHAPTER 4. DIFFERENTIATION

Example 4.4.42 Differentiate


1
y = (3x2 + 5) x
Because a variable is raised to a variable power in this function, the ordinary rules of
differentiation do not apply ! The function must first be revised before a derivative can
be taken.

Apply the natural logarithm to both sides of this equation getting

ln(3x2 + 5)
 
2 1/x 1
ln y = ln(3x + 5) = ln(3x2 + 5) =
x x

Differentiate both sides of this equation. The left-hand side requires the chain rule since
y represents a function of x. Use the quotient rule and the chain rule on the right-hand
side. Thus,
n 1 o
2
1 0 x 3x2 + 5 (6x) − ln(3x + 5)(1)
y =
y x2

Get a common denominator and combine fractions in the numerator.

6x2 2
n 3x2 + 5 o
1 0 2
− ln(3x + 5)
y = 3x + 5 3x2 + 5
2
y x
1

Dividing by a fraction is the same as multiplying by its reciprocal.

1 0 6x2 − (3x2 + 5) ln(3x2 + 5) 1


y =
y 3x2 + 5 x2

1 0 6x2 − (3x2 + 5) ln(3x2 + 5)


y =
y x2 (3x2 + 5)

Multiply both sides of this equation by y, getting

6x2 − (3x2 + 5) ln(3x2 + 5) 2 2 2


1 6x − (3x + 5) ln(3x + 5)
y0 = y = (3x 2
+ 5) x
x2 (3x2 + 5) x2 (3x2 + 5)1

Combine the powers of (3x2 + 5)

(3x 2
+ 5) ( x1 −1) 6x2 − (3x2 + 5) ln(3x2 + 5)
y0 =
x2

H.W-Kayondo & D.W-Ddumba, Engineering Math I- Lecture Notes 160


CHAPTER 4. DIFFERENTIATION

Example 4.4.43 Differentiate


3
y = (sin x)x
Because a variable is raised to a variable power in this function, the ordinary rules of
differentiation do not apply ! The function must first be revised before a derivative can
be taken.

Apply the natural logarithm to both sides of this equation getting


3
ln y = ln(sin x)x = x3 ln(sin x)

Differentiate both sides of this equation. The left-hand side requires the chain rule since
y represents a function of x. Use the product rule and the chain rule on the right-hand
side.
1 0 n 1 o
y = x3 cos x + (3x2 ) ln(sin x)
y sin x

Get a common denominator and combine fractions on the right-hand side.

1 0 x3 cos x n sin x o x3 cos x + 3x2 sin x ln(sin x)


y = + 3x2 ln(sin x) =
y sin x sin x sin x

Multiply both sides of this equation by y, getting

x3 cos x + 3x2 sin x ln(sin x) 3 2


3 x cos x + 3x sin x ln(sin x)
y0 = y = (sin x)x
sin x (sin x)1

Combine the powers of (sin x).

3 −1)
y 0 = (sin x)(x x3 cos x + 3x2 sin x ln(sin x)


H.W-Kayondo & D.W-Ddumba, Engineering Math I- Lecture Notes 161


CHAPTER 4. DIFFERENTIATION

Example 4.4.44 Differentiate


x
y = 7x(cos x) 2
Because a variable is raised to a variable power in this function, the ordinary rules of
differentiation do not apply ! The function must first be revised before a derivative can
be taken.

Apply the natural logarithm to both sides of this equation and use the algebraic properties
of logarithms, getting

 x
 x
x
ln y = ln (7x)(cos x) = ln(7x) + ln(cos x) = ln(7x) +
2 2 ln(cos x)
2

Differentiate both sides of this equation. The left-hand side requires the chain rule since
y represents a function of x . Use the product rule and the chain rule on the right-hand
side.
 
1 0 n 7 o x n 1 o 1 1 x sin x ln(cos x)
y = + (− sin x) + ln(cos x) = − +
y 7x 2 cos x 2 x 2 cos x 2

Get a common denominator and combine fractions on the right-hand side.

1 0 2 cos x − x2 sin x + x cos x ln(cos x)


y =
y 2x cos x

Multiply both sides of this equation by y, getting

2 cos x − x2 sin x + x cos x ln(cos x)


y0 = y
2x cos x
2 cos x − x2 sin x + x cos x ln(cos x)
 
x
y0 = 7x(cos x) 2
2x cos x

2 cos x − x2 sin x + x cos x ln(cos x)


 
0 x/2
y = 7(cos x)
2(cos x)1
 
7 x
0
(cos x)( 2 −1) 2 cos x − x2 sin x + x cos x ln(cos x)

y =
2

H.W-Kayondo & D.W-Ddumba, Engineering Math I- Lecture Notes 162


CHAPTER 4. DIFFERENTIATION

Example 4.4.45 Differentiate


√ √ x x2
y= x e
Because a variable is raised to a variable power in this function, the ordinary rules of
differentiation do not apply ! The function must first be revised before a derivative can
be taken.

Apply the natural logarithm to both sides of this equation and use the algebraic properties
of logarithms, getting

√ √ 
x x2
ln y = ln x e
√ √   2
x
= ln x + ln ex
√ √
= x ln( x) + x2 ln(e)
√ √
= x ln( x) + x2 (1)
√ √
= x ln( x) + x2

Differentiate both sides of this equation. The left-hand side requires the chain rule since
y represents a function of x. Use the product rule and the chain rule on the right-hand
side.

1 0 √ n 1 o −1/2 −1/2
√ 1 ln( x)
y = x √ (1/2)x + (1/2)x ln( x) + 2x = √ + √ + 2x
y x 2 x 2 x

Get a common denominator and combine fractions on the right-hand side.

√ n 2√x o 1 + ln(√x) + 4x1+1/2 √


0 1 ln( x) 1 + ln( x) + 4x3/2
y = √ + √ + 2x √ = √ = √
2 x 2 x 2 x 2 x 2 x

Multiply both sides of this equation by y, getting

√ √
0 1 + ln( x) + 4x3/2 √ √x x2 1 + ln( x) + 4x3/2
y =y √ = x e √ 1
2 x 2 x


Combine the powers of x.

√ (√x−1) x2  √
y 0 = (1/2) x e 1 + ln( x) + 4x3/2

H.W-Kayondo & D.W-Ddumba, Engineering Math I- Lecture Notes 163


CHAPTER 4. DIFFERENTIATION

Example 4.4.46 Differentiate


y = xln x (sec x)3x
Because a variable is raised to a variable power in this function, the ordinary rules of
differentiation do not apply ! The function must first be revised before a derivative can
be taken.

Apply the natural logarithm to both sides of this equation and use the algebraic properties
of logarithms, getting
 
ln y = ln xln x (sec x)3x

= ln x(ln x) + ln(sec x)3x


= (ln x)(ln x) + 3x ln(sec x)
= (ln x)2 + 3x ln(sec x)
Differentiate both sides of this equation. The left-hand side requires the chain rule since
y represents a function of x. Use the product rule and the chain rule on the right-hand
side.
1 0 n1o n 1 o
y = 2(ln x) + 3x (sec x tan x) + (3) ln(sec x)
y x sec x

Divide out a factor of sec x.

1 0 2 ln x
y = + 3x tan x + 3 ln(sec x)
y x

Get a common denominator and combine fractions on the right-hand side.

1 0 2 ln x nxo nxo
y = + 3x tan x + 3 ln(sec x)
y x x x

2 ln x + 3x2 tan x + 3x ln(sec x)


=
x

Multiply both sides of this equation by y, getting

2 ln x + 3x2 tan x + 3x ln(sec x) 2 ln x + 3x2 tan x + 3x ln(sec x)


y0 = y = xln x (sec x)3x
x x1

Combine the powers of x.


y 0 = x(ln x−1) (sec x)3x 2 ln x + 3x2 tan x + 3x ln(sec x)


H.W-Kayondo & D.W-Ddumba, Engineering Math I- Lecture Notes 164


CHAPTER 4. DIFFERENTIATION

Example 4.4.47 Consider the function

x5 ex (4x + 3)
f (x) =
5ln x (3 − x)2

Find an equation of the line tangent to the graph of f at x = 1.

First note that


(1)5 e1 (4(1) + 3) 7 1
f (1) = = e
5ln 1 (3 − 1)2 4

so that the tangent line passes through the point


 
7
(x, y) = 1, e1
4

Then getting the tangent, by differentiating


ln f (x) = 5 ln x + x + ln(4x + 3) − (ln 5) ln x − 2 ln(3 − x)

1 0 5 4 ln 5 2
f (x) = +1+ − +
f (x) x 4x + 3 x 3−x
 
0 5 4 ln 5 2
f (x) = f (x) +1+ − +
x 4x + 3 x 3−x

The slope of the line tangent to the graph of f at x = 1 is


 
0 5 4 ln 5 2
f (1) = f (1) +1+ − +
1 4+3 1 3−1
 
7 1 4
= e 7 + − ln 5
4 7
 
7 1 53
= e − ln 5
4 7

Thus, the equation of the line tangent to the graph of f at x = 1 is

y − 47 e1
 
7 1 53
= e − ln 5
x−1 4 7
 
7 7 1 53
y = + e − ln 5 (x − 1)
4 4 7

H.W-Kayondo & D.W-Ddumba, Engineering Math I- Lecture Notes 165


CHAPTER 4. DIFFERENTIATION

Example 4.4.48 Consider the function


1
f (x) = π 2 + 2x + x2 + x x

Determine the slope of the line perpendicular to the graph of f at x = 1.


1
In this function the only terms that requires logarithmic differentiation is x x and 2x

ln x
ln y =
x
1 0 1 − ln x
y =
y x2
1
0 x x (1 − ln x)
y =
x2

1
Now return to the original function f (x) = π 2 + 2x + x2 + x x . Differentiating, we get

1
0 x x x (1 − ln x)
f (x) = (0) + 2 ln 2 + 2x +
x2
1
x x x (1 − ln x)
= 2 ln 2 + 2x +
x2

The slope of the line tangent to the graph of f at x = 1 is

1
0 (1) (1) 1 (1 − ln 1)
f (1) = 2 ln 2 + 2(1) +
12
= 3 + ln 4

Thus, the slope of the line perpendicular to the graph of f at x = 1 is

−1
m=
3 + ln 4

H.W-Kayondo & D.W-Ddumba, Engineering Math I- Lecture Notes 166


CHAPTER 4. DIFFERENTIATION

(x4 ) )
Example 4.4.49 Differentiate y = x(x

(x4 )
y = x(x )
4
ln y = x(x ) ln x
 4 
(x )
ln(ln y) = ln x ln x

4
= ln x(x ) + ln(ln x) = x4 ln x + ln(ln x)

Differentiate both sides of this equation.

ln(ln y) = x4 ln x + ln(ln x)
       
1 1 0 4 1 3
 1 1
y = x + 4x ln x +
ln y y x ln x x
  
1 1 1
y0 = x3 + 4x3 ln x +

ln y y x ln x

x4 (1 + 4 ln x) ln x + 1
  
1 1
y0 =
ln y y x ln x

(x4 ) +x4 −1)


y 0 = x(x x4 (1 + 4 ln x) ln x + 1


Example 4.4.50 Differentiate


(ln x)x
y=
23x+1

ln y = x ln(ln x) − (3x + 1) ln 2

1 0 1
y = + ln(ln x) − ln 23
y ln x

(ln x)(x−1) {1 + (ln x) ln(ln x) − (ln 8) ln x}


y0 =
23x+1

H.W-Kayondo & D.W-Ddumba, Engineering Math I- Lecture Notes 167


CHAPTER 4. DIFFERENTIATION

Example 4.4.51 Differentiate

x2x (x − 1)3
y=
(3 + 5x)4

ln y = (2x) ln x + 3 ln(x − 1) − 4 ln(3 + 5x)

1 0 3 20 2(ln x)(x − 1)(3 + 5x) + 10x2 − 9x + 23


y = 2 + 2 ln x + − =
y x − 1 3 + 5x (x − 1)(3 + 5x)

x2x (x − 1)2 {2(ln x)(x − 1)(3 + 5x) + 10x2 − 9x + 23}


y0 =
(3 + 5x)5

Exercise 4.16 Differentiate


x
(i) y = xx (ii) y = 2x (iii) y = 2cot x

dy
Exercise 4.17 Let y = xx , find dx
.

Exercise 4.18 Differentiate the following functions with respect to x


1
(i) xx−1 (iii) (x − 1)ln x (v) (x2 −1)3 (x−1) 2
(x−1)3

xex 1
(ii) xsin x (iv) sin x
(vi) (x+1)6

dy
Exercise 4.19 Find dx
given that,
1
(i) x = t3 − 2, y = t2 + 2 (iii) x = t2
y = 4t3 + 8

√ √
(ii) x = cos t, y = 6 sin t (iv) x = 2 + t, y = 2 − t

Exercise 4.20 Find y 0 (x) given that

(i) xy 2 − 3x2 y = 10 (iii) cot y = 2x3 + cot(x + y)

1 1
(ii) (yx) 2 + y 2 = 0 (iv) (x + y 2 )3 + x2 y = α2

H.W-Kayondo & D.W-Ddumba, Engineering Math I- Lecture Notes 168


CHAPTER 4. DIFFERENTIATION

4.5 Applications of differentiation


4.5.1 Maxima and Minima
When using mathematics to model the physical world in which we live, we frequently
express physical quantities in terms of variables. Then, functions are used to describe
the ways in which these variables change. A scientist or engineer will be interested in the
ups and downs of a function, its maximum and minimum values (optimization problems),
its turning points. Drawing a graph of a function using a graphical calculator or computer
graph plotting package will reveal this behavior, but if we want to know the precise
location of such points we need to turn to algebra and differential calculus.
Definition 4.5.1 A function f is said to have a local maximum at x0 (or a relative
maximum at x0 ) if for all x in some interval containing x0 we have

f (x0 ) ≥ f (x)

Similarly, a function f is said to have a local minimum or relative minimum at x0


iff (x0 ) ≤ f (x)
Note 4.5.1 A local maximum at one point can have a higher value of f than a local
maximum at another point.
Definition 4.5.2 A function f defined on a domain D is said to have a maximum
(or an absolute maximum or a global maximumat x0 ) at x0 ∈ D if

f (x0 ) ≥ f (x) ∀ x ∈ D

Similarly, a function f defined on a domain D is said to have a minimum (or an


absolute minimum or a global minimumat x0 ) at x0 ∈ D if f (x0 ) ≤ f (x) ∀ x ∈ D
Note 4.5.2 The number f (x0 ) is called the maximum value of f on the domain D. The
maximum and minimum values of f are called extreme values of f .

A
local
maximum

local
minimum

H.W-Kayondo & D.W-Ddumba, Engineering Math I- Lecture Notes 169


CHAPTER 4. DIFFERENTIATION

Example 4.5.1 For the function

f (x) = sin 4x, on − 2π ≤ x ≤ 2π

with the curve

(i) f (−1) is a local and an abso-


lute (global) minimum

(ii) f (1) is a local and a global


maximum

Example 4.5.2 For the function

f (x) = x2 , on − 4 ≤ x ≤ 4

which is represented by the graph

(i) f (0) = 0 is an absolute


(global) minimum

(ii) f (x) has no global maximum

H.W-Kayondo & D.W-Ddumba, Engineering Math I- Lecture Notes 170


CHAPTER 4. DIFFERENTIATION

Example 4.5.3 For the function

f (x) = 3x4 − 16x3 + 18x2 , on − 1 ≤ x ≤ 4

which can be graphed as

(i) f (−1) = 37 > f (4) = 32 is


an absolute (global) maximum
for the interval −1 ≤ x ≤ 4

(ii) f (3) = −27 is a local and a


global minimum

(iii) f (0) = 0 is a local minimum


on interval [−1, 1]

(iv) f (1) = 5 is a local maximum


on the interval [0, 2]

Example 4.5.4 Sketch the graph on [−2, 4] where

(i) f (−2) is an absolute minimum

(ii) f (0) = 0, and f (0) is a relative


maximum for interval [−1, 2)

(iii) f (2) is a relative minimum on


interval [0, 3]

(iv) f (3) = 0 ⇒ cutting the


x-axis

(v) f (4) is an absolute maximum

H.W-Kayondo & D.W-Ddumba, Engineering Math I- Lecture Notes 171


CHAPTER 4. DIFFERENTIATION

Theorem 4.5.1 Let f be a continuous function on a closed finite interval [a, b]. Then
f has both a maximum and minimum.

Definition 4.5.3 Let f be a function defined on interval I and c be an interior point of


I. A critical number for f is a number c for which f 0 (c) = 0 or f 0 (c) fails to exist.

Note 4.5.3 A critical number is a Stationary point or Turning point.

Theorem 4.5.2 Suppose f is differentiable in (a, b) and x0 ∈ (a, b). If f has a local
extremum (local maxima or minima) at x0 , then f 0 (x0 ) = 0.

Note 4.5.4 A point of inflection is not an extrema point.

Note 4.5.5 If point c is a critical point or an extrema, then f 0 (c) = 0, but if f 0 (c) = 0,
it does not necessarily mean that point c is an extrema, it could be an inflection point.

Example 4.5.5 Find all the critical numbers and the maximum and minimum values
for f = 41 x4 − 2x2 on the given interval −2 ≤ x ≤ 2

1 4
f (x) = x − 2x2
4

⇒ f 0 (x) = x3 − 4x = x(x2 − 4)

For the critical values

f 0 (x) = 0 ⇒ x(x2 − 4) = 0
x(x − 2)(x + 2) = 0

The critical values are x = 0, −2, 2.


The extremas are

f (−2) = −4 an absolute minimum


f (0) = 0 a relative maximum on − 2 ≤ x ≤ 2
f (2) = −4 an absolute minimum

H.W-Kayondo & D.W-Ddumba, Engineering Math I- Lecture Notes 172


CHAPTER 4. DIFFERENTIATION

Theorem 4.5.3 First Derivative Test: Let c be a critical number of f and f contin-
uous at c. If there exists a δ > 0 such that
(a) f 0 (x) < 0 for all x ∈ (c − δ, c) and f 0 (x) > 0 for all x ∈ (c, c + δ) the f (c) is a local
minimum.
(b) f 0 (x) > 0 for all x ∈ (c − δ, c) and f 0 (x) < 0 for all x ∈ (c, c + δ) the f (c) is a local
maximum.
(c) f 0 (x) has the same sign on (c − δ, c) ∪ (c, c + δ) then f (c) is neither a local maximum
nor minimum.
Example 4.5.6 Think about what happens to the gradient of the graph as we travel
through the minimum turning point, from left to right, that is as x increases. Study
Figure (4.4) to help you do this.

dy dy
is negative is positive
dx dx

dy
is zero
dx

Figure 4.4: Minima

Notice that to the left of the minimum point, dy/dx is negative because the tangent
has negative gradient. At the minimum point, dy/dx = 0. To the right of the minimum
point dy/dx is positive, because here the tangent has a positive gradient.
Example 4.5.7 Now think about what happens to the gradient of the graph as we
travel through the maximum turning point, from left to right, that is as x increases.
Study Figure (4.5) to help you do this.
dy
is zero
dx

dy dy
is positive is negative
dx dx

Figure 4.5: Maxima

Notice that to the left of the maximum point, dy/dx is positive because the tangent
has positive gradient. At the maximum point, dy/dx = 0. To the right of the maximum
point dy/dx is negative, because here the tangent has a negative gradient. So, dy/dx goes
from positive, to zero, to negative as x increases.

H.W-Kayondo & D.W-Ddumba, Engineering Math I- Lecture Notes 173


CHAPTER 4. DIFFERENTIATION

Example 4.5.8 Sketch the graph on [−3, 8] such that

(i) f (4) = 0 (iii) f 0 (5) = 0 (v) f 0 (5.02) > 0


(ii) f (6) = 0 (iv) f 0 (4.88) < 0

Using the first derivative test, we realise that the critical point, x = 5 is a local minima
since on left, the derivative is negative, and on right of 5, the derivative is positive. But
the curve has to cut the x-axis at 4 and 6.

Example 4.5.9 Describe the type of extremas for a function f (x) by

f (x)

f (0) ≡ Relative maxima in (−1, 1)


f (−4) ≡ Absolute maxima
f (3) ≡ Local minima in (2, 4)
f (2) ≡ local maxima in (−3, 4)
1 3 4
f (−1) ≡ Minima
−5 −4 −3 −2 −1 2
x f (1) ≡ Relative minima in (0, 4)
f (−3) ≡ ??
f (−2) ≡ ??

H.W-Kayondo & D.W-Ddumba, Engineering Math I- Lecture Notes 174


CHAPTER 4. DIFFERENTIATION

4.5.2 Mean Value Theorem MVT


Theorem 4.5.4 Mean Value Theorem: Let

(a) f be continuous on [a, b]

(b) f be differentiable in (a, b) and

then there exists at least one number c ∈ (a, b) such that

f (b) − f (a)
f 0 (c) = (4.11)
b−a

Mean Value Theorem - Geometrical


Look at the secant line through (a, f (a)) and (b, f (b)). We expect that somewhere between
a and b there is a point c where the tangent is parallel to this secant.

y Tangent parallel
to chord.
Slope of tangent:

f ′(c) B

Slope of chord: That is, the slopes of these two


f (b) − f ( a ) lines are equal. This is formalized
b−a in the MVT.
A

x
0 a c b
y = f ( x) →

Proof of Mean Value Theorem


To prove the MVT, we apply the Rolle’s theorem

Theorem 4.5.5 Rolle’s Theorem: Let

(a) f be continuous on [a, b]

(b) f be differentiable in (a, b) and

(c) f (a) = f (b)

then there exists at least one number c ∈ (a, b) such that f 0 (c) = 0

H.W-Kayondo & D.W-Ddumba, Engineering Math I- Lecture Notes 175


CHAPTER 4. DIFFERENTIATION

Back to proof of MVT: We define a function

x−a
 
h(x) = f (x) − f (a) + [f (b) − f (a)]
b−a

represents the difference in height between the curve y = f (x) and the line joining its end
points.

Clearly, by the nature of our function h(x)


(a) h(x) be continuous on [a, b] by sum of continuous functions
(b) h(x) be differentiable in (a, b) by the sum of differentiable functions, and
(c) h(a) = h(b) by substitution
satisfying all the conditions of Rolle’s Theorem. Hence there is some c ∈ (a, b) such that
h0 (c) = 0 (Rolle’s Theorem)

f (b) − f (a)
f 0 (c) − = 0
b−a

Thus, the proof


f (b) − f (a)
f 0 (c) =
b−a

Corollary 4.5.1 Let f 0 (x) = 0 for all x ∈ (a, b), then f is constant on (a, b).

Proof: Let x1 and x2 be any two numbers in (a, b) with x1 < x2 . Since f is differentiable in
(a, b) - the derivative given- it is differentiable in (x1 , x2 ) and is continuous on [x1 , x2 ] - all
differentiable functions, are continuous. Then by MVT, there exists a number c ∈ (x1 , x2 )
such that

f (x2 ) − f (x1 )
f 0 (c) =
x2 − x1

f (x2 ) − f (x1 )
0 =
x2 − x1

Since we were given f 0 (x) = 0 for all x ∈ (a, b).


f (x1 ) = f (x2 )
for any arbitrarily chosen numbers x1 , x2 ∈ (a, b). Hence f is constant on (a, b).

H.W-Kayondo & D.W-Ddumba, Engineering Math I- Lecture Notes 176


CHAPTER 4. DIFFERENTIATION

Corollary 4.5.2 Let f 0 (x) > 0 for all x ∈ (a, b), then f is increasing on (a, b).
Corollary 4.5.3 Let f 0 (x) < 0 for all x ∈ (a, b), then f is decreasing on (a, b).
Example 4.5.10 For a function f (x) = −x2 + 6x − 6, find a c on [1, 3] that satisfies the
Mean Value Theorem.

Since f (x) is continuous and differentiable (because a polynomial), it satisfies the hy-
potheses of MVT, a = 1, b = 3, f 0 (x) = −2x + 6, therefore,

f (b) − f (a)
f 0 (c) =
b−a

f (3) − f (1)
−2c + 6 =
3−1
−2c + 6 = 2
And a c ∈ (1, 3) is such that, f 0 (c) = −2c + 6 = 2 this implies that c = 2.
Example 4.5.11 For a function f (x) = x3 − x find a c on [0, 2] that satisfies the mean
value theorem.

The function f (x) is differentiable in (0, 2) and continuous on [0, 2] and f 0 (x) = 3x2 − 1,
therefore,

f (b) − f (a)
f 0 (c) =
b−a
6−0
3c2 − 1 =
2−0

3c2 − 1 = 3
0 2

And a c ∈ (0, 2) is such
√ that, f (c) = 3c − 1 this implies that c = ±2/ 3. So the c in
the interval is c = 2/ 3
Exercise 4.21 Using Intermediate Value Theorem, show that x3 − x + 1 has only one
real root on [−2, −1] and that other two roots are complex.
Example 4.5.12 Determine a and b for the function:

ax − 3, x<4
f (x) = 2
−x + 10x − b, x ≥ 4

If it satisfies the hypothesis of Mean Value Theorem on the interval [2, 6]. Hint: To
satisfies the MVT, it has to be continuous on [2, 6] and differentiable in (2, 6) [a,b] =
[2,19]

H.W-Kayondo & D.W-Ddumba, Engineering Math I- Lecture Notes 177


CHAPTER 4. DIFFERENTIATION

Example 4.5.13 Determine a c that satisfys the MVT for the function


x + 1, x < 1
f (x) =
x − 1, x ≥ 1

on the interval [0, 3].

The MVT hypotheses not satisfied, since function not continuous at x = 1.

Example 4.5.14 Determine a c that satisfys the MVT for the function f (x) = |x| on
[−2, 5]

x, x≥0
f (x) =
−x, x < 0

The MVT hypotheses not satisfied, since function not differentiable at x = 0.

Example 4.5.15 Let f (x) = |x2 − x − 2|. Determine if the Mean Value Theorem applies
to f on the interval [a, b] = [0, 3]. If the MVT does not apply, state the hypothesis that
is not satisfied. If the MVT does apply, identify all numbers c in the interval where
f 0 (c) = f (b)−f
b−a
(a)
. In either case, include a graph that supports your conclusion.

Example 4.5.16 Use the mean value theorem to prove that for any two real numbers
a and b,

| cos a − cos b| ≤ |a − b|
The function cos x is continuous and differentiable for all real numbers. Using the mean
value theorem, using 2 real numbers a and b to write

[cos a − cos b]
(cos x)0 =
[a − b]

0
[cos a − cos b]
|(cos x) | =

[a − b]

[cos a − cos b]
≤ 1
[a − b]

| cos a − cos b|
≤ 1
|a − b|

| cos a − cos b| ≤ |a − b|

H.W-Kayondo & D.W-Ddumba, Engineering Math I- Lecture Notes 178


CHAPTER 4. DIFFERENTIATION

Example 4.5.17 Given the piecewise defined function



 x, x<0
f (x) =
x2 − 1, x ≥ 0

(i) determine if the function satisfies the Mean Value Theorem in (1, 3) and (−1, 1)

The function f (x) = x2 − 1 is continuous and differentiable in the interval [1, 3]


and therefore satisfies the initial conditions (hypotheses) of the MVT in the interval
(1, 3).

The function f (x) is neither continuous nor differentiable in the interval at a point
x = 0 and therefore f (x) does not satisfy the initial conditions (hypotheses) of the
MVT in the interval (−1, 1).
(ii) plot the function

(iii) find all values of c that satisfy the conclusion of the Mean Value Theorem.

f (b) − f (a)
f 0 (c) =
b−a

f (3) − f (1)
2c =
3−1
8−0
2c = =4
3−1
c = 2

The function, f (x), is not continuous at x = 0 and therefore is not continuous in


[−1, 1]. The MVT does not apply in this interval.

H.W-Kayondo & D.W-Ddumba, Engineering Math I- Lecture Notes 179


CHAPTER 4. DIFFERENTIATION

Example 4.5.18 Sketch the graph where: f 0 (x) > 0, 1 ≤ x < 5 and f 0 (x) < 0, 5 < x ≤ 7

Exercise 4.22 For the numbers 1-7, find a number c which satisfies the MVT.
x−1
1). x+1
on [0, 1] 3). |5 − x2 | on [−2, 2] 6). x3 on [0, 4]
4). tan x on [0, π4 ]
2
2). x 5 on [0, 8] 5). x2 on [0, 3] 7). x3 − 2x2 + 3x + 1 on [0, 2]

8). Is Rolle’s theorem applicable to the function f (x) = |x − 1| on the interval [0, 2]?
9). Determine if the function f (x) = x − x3 satisfies the conditions of Rolle’s theorem on
the interval [−1, 0] and [0, 1]. In the affirmative case, determine the values of c.
10). Does the function f (x) = 1−x satisfy the conditions of Rolle’s theorem on the interval
[−1, 1]?
11). Prove that the equation 1 + 2x + 3x2 + 4x3 = 0 has a unique solution.
12). How many roots does the equation x3 + 6x2 + 15x − 25 = 0 have?
13). Prove that the equation 2x3 − 6x + 1 = 0 has only one real solution on the interval
(0, 1).
14). Can the mean value theorem be applied to f (x) = 4x2 − 5x + 1 on [0, 2]?
15). Can the mean value theorem be applied to f (x) = 1/x2 on [0, 2]?
16). In the segment of the parabola between the points A = (1, 1) and B = (3, 0), find a
point whose tangent is parallel to the chord.
17). Calculate a point on the interval [1, 3] in which the tangent to the curve y = x3 −x2+2
is parallel to the line determined by the points A = (1, 2) and B = (3, 20). What
theorem guarantees the existence of this point?

H.W-Kayondo & D.W-Ddumba, Engineering Math I- Lecture Notes 180


CHAPTER 4. DIFFERENTIATION

4.5.3 Second Derivative Test and Concavity


Definition 4.5.4 Let f be differentiable in (a, b), and let p ∈ (a, b).
(a) We call point p a stationary (or critical) point for f if f 0 (p) = 0,
(b) we call point p a point of inflection for f if f 00 (p) = 0,
(c) we call f (strictly) concave upwards if f 0 is (strictly) increasing on (a, b),
(d) we call f (strictly) concave downwards if f 0 is (strictly) decreasing on (a, b).
Concave up is also referred to as positive curvature and concave down is referred as
negative curvature.
Theorem 4.5.6 On Concavity: Let f be twice differentiable in (a, b).
(i) If f 00 (x) ≥ 0 ∀ x ∈ (a, b) then f is concave upwards,

A y B y

(ii) If f 00 (x) ≤ 0 ∀ x ∈ (a, b) then f is concave downwards,

C y D y
f (x)

x x
x

(iii) If f 00 (x) > 0 ∀ x ∈ (a, b) then f is strictly concave upwards,


(iv) If f 00 (x) < 0 ∀ x ∈ (a, b) then f is strictly concave downwards.
Theorem 4.5.7 Second derivative Test: Let f be defined on an open interval con-
taining point p. Suppose f 0 (p) = 0 and f 00 (p) exists.
(i) If f 00 (p) < 0 then f (p) is a relative maximum
(ii) If f 00 (p) > 0 then f (p) is relative minimum
(iii) If f 00 (p) = 0 then f (p),the test is not informative, there is no conclusion

H.W-Kayondo & D.W-Ddumba, Engineering Math I- Lecture Notes 181


CHAPTER 4. DIFFERENTIATION

Definition 4.5.5 Let f be differentiable in (a, b), and let p ∈ (a, b), a point p is a
point of inflection for f if
(i) f 00 (p) = 0, and
(ii) at point p, the concavity of a function f switches from up to down or down to up,
that is
f 00 (p − δ) · f 00 (c + δ) < 0
Example 4.5.19 Sketch the graph where
(i) f (x) is concave upwards [f 00 (x) > 0] where −1 < x ≤ 1, and f (x) is concave
downwards [f 00 (x) < 0] where 1 < x ≤ 3
(ii) f 00 (0) > 0[relative minima], f 00 (2) < 0 [local maxima]
(iii) f 0 (x) > 0[ f increasing], x ∈ (0, 2) and f 0 (x) < 0[f decreasing], x ∈ (2, 4)
The sketch after using the first derivative, second derivative tests and the concavity defi-
nitions is as shown [the curve can also be in the negative/lower side]

The graph can even be drawn


in the negative/lower side

Example 4.5.20 Given a function


f (x) = x3 − 3x2
(a) The Stationary or critical points
f 0 (x) = 0 ⇒ 3x2 − 6x = 0 ⇒ 3x(x − 2) = 0 ⇒ x = 0, x = 2

(b) Maxima or Minima.


Since
f 00 (x) = 6x − 6
x=0 a maxima point (x, y) = (0, 0)
x=2 a minima point (x, y) = (2, −4)
Thus f (x) = f (2) = −4 is a relative minima since f 00 (2) = 6(2) − 6 = 6 > 0.
Thus f (x) = f (0) = 0 is a relative maxima since f 00 (0) = 6(0) − 6 = −6 < 0.

H.W-Kayondo & D.W-Ddumba, Engineering Math I- Lecture Notes 182


CHAPTER 4. DIFFERENTIATION

Since f 00 (x) = 6x − 6

f 00 (x) < 0 if x < 1. f is concave down in the interval


f 00 (x) > 0 if x > 1. f is concave up in the interval

Figure 4.6: A curve f (x) = x3 − 3x2

Example 4.5.21 Show that for the function f (x) = x3

x=0 : is a point of inflection


x ∈ [0, ∞) : f is concave up in the interval
x ∈ (−∞, 0] : f is concave down in the interval

as could be visualised from the graph

Figure 4.7: A curve f (x) = x3

H.W-Kayondo & D.W-Ddumba, Engineering Math I- Lecture Notes 183


CHAPTER 4. DIFFERENTIATION

Summary of Derivatives

First Derivative f 0 (x) Second Derivative f 00 (x)


1. Concavity
0
Critical points f (c) = 0 Concave up: f 00 (x) ≥ 0, x ∈ (a, b)
Concave down: f 00 (x) ≤ 0, x ∈ (a, b)
2. Minima/Maxima Minima/Maxima
0 0
Maxima: f (c − δ, c) > 0&f (c, c + δ) < 0 Maxima: f 00 (p) < 0
Minima: f 0 (c − δ, c) < 0&f 0 (c, c + δ) > 0 Minima: f 00 (p) > 0
No conclusion: f (c − δ, c) < 0&f (c, c + δ) < 0 No conclusion: f 00 (p) = 0
0 0

f 0 (c − δ, c) > 0&f 0 (c, c + δ) > 0


3. Increasing/Decreasing
Increasing: f 0 (x) > 0, x ∈ I
Decreasing: f 0 (x) < 0, x ∈ I

There is always a minima and a maxima in any interval


A point of inflection is not an extremum
If f (k) = 0 ⇒ k is an x-intercept, where the curve cuts the x−axis

Absolute = Global : Domain D (everywhere)


Relative = Local : Interval I

Example 4.5.22 Given a graph of f 0 (x) below

f � (x)

f 0 (c) = 0, ⇒ c ≡ Critical points/Turning points/Stationary points: c = 0, 3

f 0 (0 − δ, 0) > 0, f 0 (0, 0 + δ) < 0 ⇒ f (0) is a relative maxima


f 0 (3 − δ, 3) < 0, f 0 (3, 3 + δ) > 0 ⇒ f (3) is a relative minima

H.W-Kayondo & D.W-Ddumba, Engineering Math I- Lecture Notes 184


CHAPTER 4. DIFFERENTIATION

Example 4.5.23 Let g be a function whose derivative g 0 is continuous and has the graph
shown below.

Figure 4.8: Curve of g 0 (x)

(i) State the turning points of g(x)? Turning point or critical points are when g 0 (x) = 0,
that is at x = 2 and x = 5

(ii) Which of the following values of g is largest?

(a) g(1) (b) g(2) (c) g(3) (d) g(4) (e) g(5)

Its g(2) by first derivative test, a maxima, before g 0 (x) > 0 and after g 0 (x) < 0. At
x = 5, it is a minima

(iii) Sketch the curve of g(x). Draw any sketch where there is maxima at x = 2, and a
minima at x = 5.

Example 4.5.24 Sketch the graph for the function f (x) on the interval −4 ≤ x ≤ 6
where

(i) f (−3) = f (−1) = f (5) = 0

(ii) f 0 (−2) = f 0 (3) = 0

(iii) f 0 (x) < 0, x ∈ (−4, −2) ∪ (3, 6) and f 0 (x) > 0, x ∈ (−2, 3)

(iv) f 00 (x) ≥ 0, x ∈ (−4, 2) and f 00 (x) ≤ 0, x ∈ (2, 4)

Example 4.5.25 Given



0 x, x<0
f (x) =
−x , x≥0

Determine the interval on which f (x) is increasing. [Region DNE]

H.W-Kayondo & D.W-Ddumba, Engineering Math I- Lecture Notes 185


CHAPTER 4. DIFFERENTIATION

Example 4.5.26 If f 0 (x) and g 0 (x) exist and f 0 (x) > g 0 (x) for all real x, then the graph
of y = f (x) and the graph of y = g(x)

(A) intersect exactly once

(B) intersect no more than once

(C) do not intersect

(D) could intersect more than once

Convince your self that the best answer is option (B). You might consider an example of
f = 2x, g = x

Example 4.5.27 If a function f is continuous for all x and if f has a relative maximum
at the point (−1, 4) and a relative minimum at the point (3, −2), which of the following
statements must be true

(A) The graph of f has a point of inflection somewhere between x = −1 and x = 3

(B) f 0 (−1) = 0

(C) The graph of f has a horizontal asymptote

(D) The graph of f has a horizontal line at x = 3

(E) The graph of f intersects both axes

The best option is (E) since we have the word ”must” in he question. Option (B) is not
correct since it might be f 0 (−1) = 0 or f 0 (−1) DNE as in Defintion (4.5.3)
0
Example 4.5.28
√ State the definition of a derivative at a point x0 . Hence find f (x0 ), x0 >
0 if f (x) = x + 7.

f (x) − f (a)
f 0 (a) = lim
x→a x−a
√ √  √ √
0 f (x) − f (x0 ) ( x + 7) − x0 + 7 x − x0
f (x0 ) = lim = lim = lim
x→x0 x − x0 x→x0 x − x0 x→x0 x − x0
1√
= x0
2

By rationalisation or LaHopital
To indicate that x0 > 0, because we cannot have a square root of a negative number.

H.W-Kayondo & D.W-Ddumba, Engineering Math I- Lecture Notes 186


CHAPTER 4. DIFFERENTIATION

Example 4.5.29 Find the equations of the tangent lines to the curve f (x) = x2 + 9
which pass through the origin (0, 0).

Where (x0 , y0 ) is a point on a curve and m is the gradient

f 0 (x) = 2x = 2x0
0 − y0
= 2x0 ⇒ y0 = 2x20
0 − x0
y0 = x20 + 9
⇒ x0 = 3, y0 = 18
y − y0
=m
x − x0
y−0
= 6 ⇒ y = 2x & y = −2x
x−0
dy
Exercise 4.23 For the circle (x − 2)2 + (y − 2)2 = 16, find dx . Also find the slope of
the horizontal and vertical tangent lines to the circle. At what points do the horizontal
tangents touch the circle?

dy 2−x
Tangent : =
dx y−2

dy
Horizontal : = 0 ⇒ 2 − x = 0 ⇒ (x, y) = (2, ±4 + 2)
dx
dy
Vertical : undefined ⇒ y − 2 = 0 ⇒ (x, y) = (±4 + 2, 2)
dx
√ √
Exercise 4.24 Use MVT to prove that: lim ( x + 2 − x ) = 0.
x→∞

Let f (t) = t on [x, x + 2], then ∃ c ∈ (x, x + 2) such that

f (x + 2) − f (x) 1
= f 0 (c) = √
(x + 2) − x 2 c

1 1
⇔ f (x + 2) − f (x) = √ [(x + 2) − x] = √
2 c c
√ √
but also f (x + 2) − f (x) = x+2− x
√ √ 1
T hus lim ( x + 2 − x ) = lim √ = 0
x→∞ c→∞ c

H.W-Kayondo & D.W-Ddumba, Engineering Math I- Lecture Notes 187


CHAPTER 4. DIFFERENTIATION

Example 4.5.30 Given the function g(x) = 2x3 +12x2 +18x+12, find stationary points
on the curve and determine their nature.

f 0 (x) = 6x2 + 24x + 18


⇒ for f 0 (x) = 0 ⇒ 6(x2 + 4x + 3) = 0
⇒ (x + 3)(x + 1) = 0 ⇒ x = −3 or x = −1
f 00 (x) = 6(2x + 4) = 12(x + 2)
but at x = −1, f 00 (x) = 12 > 0.

Hence x = −1 is a relative minimum At x = −3, f 00 (x) = −12 < 0, hence x = −3 is a


relative maximum. But when f 00 (x) = 0 ⇒ 12(x + 2) = 0 ⇒ x = −2, therefore x = −2
is an inflection point which is a stationary point .
Example 4.5.31 Find the turning points of the function y = x3 − 3x + 2 and distinguish
between them.
1 : minima , − 1 : maxima
Example 4.5.32 Suppose we wish to find points on the curve y(x) given by

y = x3 − 6x2 + x + 3

where the tangents are parallel to the line y = x + 5.

If the tangents have to be parallel to the line then they must have the same gradient. The
standard equation for a straight line is y = mx + c, where m is the gradient. So what
we gain from looking at this standard equation and comparing it with the straight line
y = x + 5 is that the gradient, m, is equal to 1. Thus the gradients of the tangents we
are trying to find must also have gradient 1.

We know that if we differentiate y(x) we will obtain an expression for the gradients of the
tangents to y(x) and we can set this equal to 1. Differentiating, and setting this equal to
1 we find
dy
= 3x2 − 12x + 1 = 1
dx

from which
3x2 − 12x = 0
This is a quadratic equation which we can solve by factorisation.

3x2 − 12x = 0
3x(x − 4) = 0

3x = 0 or x−4=0

H.W-Kayondo & D.W-Ddumba, Engineering Math I- Lecture Notes 188


CHAPTER 4. DIFFERENTIATION

x=0 or x=4
Now having found these two values of x we can calculate the corresponding y coordinates.
We do this from the equation of the curve: y = x3 − 6x2 + x + 3.

when x = 0: y = 03 − 6.02 + 0 + 3 = 3.

when x = 4: y = 43 − 6.42 + 4 + 3 = 64 − 96 + 4 + 3 = −25.

So the two points are (0, 3) and (4, −25)

These are the two points where the gradients of the tangent are equal to 1, and so where
the tangents are parallel to the line that we started out with, i.e. y = x + 5.
Exercise 4.25 For each of the functions given below determine the equation of the
tangent at the points indicated.
a) f (x) = 3x2 − 2x + 4 at x = 0 and 3.
y = −2x + 4, y = 16x − 23

b) f (x) = 5x3 + 12x2 − 7x at x = −1 and 1.


y = −16x − 2, y = 32x − 22

c) f (x) = xex at x = 0.
y=x
3
d) f (x) = (x2 + 1) at x = −2 and 1.
y = −300x − 0475, y = 24x − 16

e) f (x) = sin 2x at x = 0 and π6 .


3 π
y = 2x, y = x + −
2 6

f) f (x) = 1 − 2x at x = −3, 0 and 2.


y = 1 − 2x, y = 1 − 2x, y = 1 − 2x

Exercise 4.26 Find the equation of each tangent of the function f (x) = x3 −5x2 +5x−4
which is parallel to the line y = 2x + 1.

95
y = 2x − , y = 2x − 13
27

H.W-Kayondo & D.W-Ddumba, Engineering Math I- Lecture Notes 189


CHAPTER 4. DIFFERENTIATION

Exercise 4.27 Find the equation of each tangent of the function f (x) = x3 + x2 + x + 1
which is perpendicular to the line 2y + x + 5 = 0.

22
y = 2x + 2, y = 2x +
27

Example 4.5.33 Suppose we wish to find the equation of the tangent and the equation
of the normal to the curve
1
y =x+
x

at the point where x = 2.

First of all we shall calculate the y coordinate at the point on the curve where x = 2:

1 5
y =2+ =
2 2

Next we want the gradient of the curve at the point x = 2. We need to find dy/dx.

Noting that we can write y as y = x + x−1 then

dy 1
= 1 − x−2 = 1 − 2
dx x

Furthermore, when x = 2
dy 1 3
=1− =
dx 4 4

This is the gradient of the tangent to the curve at the point (2, 52 ). We know that the
standard equation for a straight line is

y − y1
=m
x − x1

With the given values we have


y − 52 3
=
x−2 4

H.W-Kayondo & D.W-Ddumba, Engineering Math I- Lecture Notes 190


CHAPTER 4. DIFFERENTIATION

Rearranging
5 3
y− 2
= 4
(x − 2)

5

4 y− 2
= 3(x − 2)

4y − 10 = 3x − 6
4y = 3x + 4

So the equation of the tangent to the curve at the point where x = 2 is 4y = 3x + 4.

Now we need to find the equation of the normal to the curve.

Let the gradient of the normal be m2 . Suppose the gradient of the tangent is m1 . Recall
that the normal and the tangent are perpendicular and hence m1 m2 = −1. We know
m1 = 43 . So
3
× m2 = −1
4

and so
4
m2 = −
3

So we know the gradient


 of the normal and we also know the point on the curve through
5
which it passes, 2, 2 .
As before,
y−y1
x−x1
= m
y− 52
x−2
= − 43

Rearranging
5

3 y− 2
= −4(x − 2)

15
3y − 2
= −4x + 8

15
3y + 4x = 8+ 2

6y + 8x = 31

This is the equation of the normal to the curve at the given point.

H.W-Kayondo & D.W-Ddumba, Engineering Math I- Lecture Notes 191


CHAPTER 4. DIFFERENTIATION

Example 4.5.34 Consider the curve xy = 4. Suppose we wish to find the equation of
the normal at the point x = 2. Further, suppose we wish to know where the normal meet
the curve again, if it does.

Notice that the equation of the given curve can be written in the alternative form y = x4 .
A graph of the function y = x4 is shown in Figure (4.9).

Figure 4.9: A graph of the curve xy = 4 showing the tangent and normal at x = 2.

From the graph we can see that the normal to the curve when x = 2 does indeed meet the
curve again (in the third quadrant). We shall determine the point of intersection. Note
that when x = 2, y = 42 = 2.

We first determine the gradient of the tangent at the point x = 2. Writing

4
y = = 4x−1
x

and differentiating, we find

dy 4
= −4x−2 = − 2
dx x

dy
Now, when x = 2 dx
= − 44 = −1.

H.W-Kayondo & D.W-Ddumba, Engineering Math I- Lecture Notes 192


CHAPTER 4. DIFFERENTIATION

So, we have the point (2, 2) and we know the gradient of the tangent there is −1. Re-
member that the tangent and normal are at right angles and for two lines at right angles
the product of their gradients is −1. Therefore we can deduce that the gradient of the
normal must be +1. So, the normal passes through the point (2, 2) and its gradient is 1.

As before, we use the equation of a straight line in the form:

y − y1
= m
x − x1

y−2
= 1 ⇒ y−2=x−2 ⇒ y =x
x−2

So the equation of the normal is y = x.

We can now find where the normal intersects the curve xy = 4. At any points of inter-
section both of the equations

xy = 4 and y=x

are true at the same time, so we solve these equations simultaneously. We can substitute
y = x from the equation of the normal into the equation of the curve:

xy = 4
x·x = 4
x2 = 4
x = ±2

So we have two values of x where the normal intersects the curve. Since y = x the
corresponding y values are also 2 and −2. So our two points are (2, 2), (−2, −2). These
are the two points where the normal meets the curve. Notice that the first of these is the
point we started off with.

Exercise 4.28 For each of the functions given below determine the equations of the
tangent and normal at each of the points indicated.

(a) f (x) = x2 + 3x + 1 at x = 0 and 4.

1
At x = 0: y = 3x + 1, y = − 31 x + 1, At x = 4: y = 11x − 15, y = − 11 x+ 323
11

(b) f (x) = 2x3 − 5x + 4 at x = −1 and 1.

At x = −1: y = x + 8, y = −x + 6, At x = 1: y = x, y = −x + 2

H.W-Kayondo & D.W-Ddumba, Engineering Math I- Lecture Notes 193


CHAPTER 4. DIFFERENTIATION

Exercise 4.29 Find the equation of each normal of the function f (x) = 13 x3 + x2 + x − 13
which is parallel to the line y = − 14 x + 31

Exercise 4.30 Find the x co-ordinate of the point where the normal to f (x) = x2 −3x+1
21
at x = −1 intersects the curve again. 5

Exercise 4.31 A total of x feet of fencing is to form three sides of a level rectangular
yard. What is the maximum possible area of the yard, in terms of x ?
x2 x2 x2
(a) 9
(b) 8
(c) 4
(d) x2 (e) 2x2

Exercise 4.32 Let f and g be twice-differentiable real-valued functions defined on R. If


f 0 (x) > g 0 (x) for all x > 0, which of the following inequalities must be true for all x > 0?

(A) f (x) > g(x) (D) f 0 (x) − f 0 (0) > g 0 (x) − g 0 (0)
(B) f 00 (x) > g 00 (x)
(C) f (x) − f (0) > g(x) − g(0) (E) f 00 (x) − f 00 (0) > g 00 (x) − g 00 (0)

Exercise 4.33 Find an equation of the line tangent to the graph of y = x + ex at x = 0.

Example 4.5.35 Find two non-negative numbers whose sum is 9 and so that the product
of one number and the square of the other number is a maximum.

Let variables x and y represent two non-negative numbers. The sum of the two numbers
is given to be 9 = x + y,so that y = 9 − x.

We wish to maximize the product

P = xy 2 = x(9 − x)2

Now differentiate this equation using the product rule and chain rule, getting

P 0 = x(2)(9 − x)(−1) + (1)(9 − x)2


= (9 − x)[−2x + (9 − x)]
= (9 − x)[9 − 3x] = (9 − x)(3)[3 − x] = 0

for x = 9 or x = 3.

Note that since both x and y are non-negative numbers and their sum is 9, it follows that
0 ≤ x ≤ 9.

If x = 3 and y = 6, then P = 108 is the largest possible product.

H.W-Kayondo & D.W-Ddumba, Engineering Math I- Lecture Notes 194


CHAPTER 4. DIFFERENTIATION

Example 4.5.36 Jesse is to build a rectangular pen with four parallel partitions using
500 feet of fencing. What dimensions will maximize the total area of the pen?

Let variable x be the width of the pen and variable y the length of the pen.

The total amount of fencing is given to be 500 = 5(width) + 2(length) = 5x + 2y, so


that 2y = 500 − 5x ⇒ y = 250 − 25 x

We wish to maximize the total area of the pen

A = (width)(length) = xy = x 250 − 52 x = 250x − 25 x2 .




Now differentiate this equation, getting

A0 = 250 − (5/2)2x = 250 − 5x = 5(50 − x) = 0, x = 50

Note that since there are 5 lengths of x in this construction and 500 feet of fencing, it
follows that 0 ≤ x ≤ 100. For x = 50 ft. then y = 125 ft.,and A = 6250ft2 is the largest
possible area of the pen.

Exercise 4.34 The daily profit, P , of an oil refinery is given by

P = 8x − 0.02x2 ,

where x is the number of barrels of oil refined. How many barrels will give maximum
profit and what is the maximum profit? x = 200, P = $800

Exercise 4.35 A rectangular storage area is to be constructed along the side of a tall
building. A security fence is required along the remaining 3 sides of the area. What is
the maximum area that can be enclosed with 800 m of fencing?

x = 200, y = 400, and A = 80000m2

Exercise 4.36 A rectangular box with a square base and no top is to have a volume
of 108 cubic inches. Find the dimensions for the box that require the least amount of
material. (x, y, h) = (6, 6, 3)

H.W-Kayondo & D.W-Ddumba, Engineering Math I- Lecture Notes 195


CHAPTER 4. DIFFERENTIATION

Example 4.5.37 An open rectangular box (no top) with square base is to be made from
48ft2 of material. What dimensions will result in a box with the largest possible volume?

Let the sizes of the square base be x, and the height h.

The total surface area S of the box is given to be

S = x2 + 4(xh)
48 = x2 + 4(xh)

48 − x2 48 x2 12 1
⇒ h = = − = − x
4x 4x 4x x 4

The total volume V of the box is given by

 
2 2 12 1 1
V = x h=x − x = 12x − x3
x 4 4

Maximizing the total volume V of the box, getting extremas

V0 = 0
3 3 3
12 − x2 = (16 − x2 ) = (4 − x)(4 + x) = 0
4 4 4
⇒ x = 4, −4

But x 6= −4 since variable x measures a distance and x > 0.


Thus to maximise the volume, x = 4 Or we can use second derivative test, to determine
which x value gives the maximum volume.

H.W-Kayondo & D.W-Ddumba, Engineering Math I- Lecture Notes 196


CHAPTER 4. DIFFERENTIATION

Example 4.5.38 A container in the shape of a right circular cylinder with no top has
surface area 3πft2 . What height h and base radius r will maximize the volume of the
cylinder ?

Let variable r be the radius of the circular base and variable h the height of the cylinder.

The total surface area of the cylinder is given to be

3π = (area of base) + (area of the curved side) = πr2 + (2πr)h

3π − πr2 3 1
so that h = = − r
2πr 2r 2

We wish to maximize the total volume of the cylinder

 
2 2 3 1 3 1
V = πr h = πr − r = πr − πr3
2r 2 2 2

Now differentiate this equation, getting

V 0 = (3/2)π − (1/2)π3r2 = (3/2)π(1 − r2 ) = (3/2)π(1 − r)(1 + r) = 0

r = 1 or r = −1. But r 6= −1 since variable r measures a distance and r > 0. Since √ the
2
base of the box is a circle and there are 3πft of material, it follows that 0 < r ≤ 3.

Alternatively,

3
V 00 = π(−2r) ⇒ Vr=−1
00 00
= 3π > 0 a minima, Vr=1 = −3π < 0 a maxima
2

For r = 1ft and h = 1ft,then V = πft3 is the largest possible volume of the cylinder.

H.W-Kayondo & D.W-Ddumba, Engineering Math I- Lecture Notes 197


CHAPTER 4. DIFFERENTIATION

Example 4.5.39 A sheet of cardboard 3ft by 4ft will be made into a box by cutting
equal-sized squares from each corner and folding up the four edges. What will be the
dimensions of the box with largest volume?

Let variable x be the length of one edge of the square cut from each corner of the sheet
of cardboard.

⇒ Af-

ter removing the corners and folding up the flaps, we have an ordinary rectangular box.

We wish to maximize the total volume of the box


V = (length) (width) (height) = (4 − 2x)(3 − 2x)(x). Now differentiate this equation
using the triple product rule, getting
V0 = (−2)(3 − 2x)(x) + (4 − 2x)(−2)(x) + (4 − 2x)(3 − 2x)(1)
= 12x2 − 28x + 12
= 4(3x2 − 7x + 3)
= 0
for (Use the quadratic formula).

p
−(−7) ± (−7)2 − 4(3)(3)
x =
2(3)

7± 13
=
6

i.e., for x ≈ 0.57 or x ≈ 1.77.

For x ≈ 0.57ft (use second derivative to show for maxima), then


V ≈ 3.03ft3
is largest possible volume of the box.

H.W-Kayondo & D.W-Ddumba, Engineering Math I- Lecture Notes 198


CHAPTER 4. DIFFERENTIATION

Example 4.5.40 Find the maximum and minimum value of A(x) = |2x| on the interval
[−1, 6].

Since

 2x, if x ≥ 0
A(x) =
−2x, if x < 0

by definition of absolute value, the graph of A(x) is

Figure 4.10: A curve of A(x) = |2x| on [-1,6]

A0 (x) is undefined at x = 0 but A(0) = 0. Therefore x = 0 is a critical point. the


minimum occurs at this point, x = 0 and is equal to zero. A(−1) = 2 and A(6) = 12.
Therefore the maximum occurs at the endpoint, x = 6 and is equal to 12.
Example 4.5.41 Suppose f (x) = x3 − 3x2 + x − 2. Let’s determine where the graph of
f is concave up and where it is concave down. Since f is twice-differentiable for all x, we
use the result given above and first determine that
f 00 (x) = 6(x − 1)
Thus, f 00 (x) > 0 if x > 1 and f (x) < 0 if x < 1. By the Concavity Theorem, the graph of
f is concave up for x > 1 and concave down for x < 1.

H.W-Kayondo & D.W-Ddumba, Engineering Math I- Lecture Notes 199


CHAPTER 4. DIFFERENTIATION

Example 4.5.42 An investor has 100 houses. When the rent of every house is $80 per
month, all houses are occupied. However, for every $4 increase in rent, one house becomes
vacant. Each occupied unit requires a monthly average of $8 for repairs. If there are no
other expenses, what rent should be charged to make the most profit?

Let the number of vacant houses be x. Let the new income (revenue R(x)) and cost
(Expenditure E(x)) and Profits P (x) are

R(x) = Price × Quantity = (80 + 4x)(100 − x)


E(x) = cost × Quantity = (8)(100 − x)
P (x) = R(x) − E(x) = (80 + 4x)(100 − x) − 8(100 − x)
= (100 − x)(72 + 4x)

dP
= 4(100 − x) − (72 + 4x) = 0 ⇒ x = 41
dx
d2 P
= −8 < 0
dx2

at x = 41. So P (x) is maximum when x = 41. Thus, the maximum rent is R(41) =
80 + 164 = 244. That is, the profit is maximum when the monthly rent is $244 with 59
units occupied.
Example 4.5.43 During Christmas time Jackie makes and sells necklaces on the beach.
Last Christmas she sold the necklaces for $10 each and her sales averaged 20 per day.
When she increased the price by $1, she found that the average decreased by two sales
per day. If the material of each necklace costs Jackie $6, what should the selling price be
to maximize her profit?
Let unsold number of necklaces be x,

R(x) = Price × Quantity = (10 + x)(20 − 2x)


E(x) = cost × Quantity = (6)(20 − 2x)
P (x) = R(x) − E(x)
= (10 + x)(20 − 2x) − 6(20 − 2x)

dP
= 12 − 4x = 0 ⇒ x = 3
dx
d2 P
= −4 < 0
dx2

at x = 3. So P (x) is maximum when x = 3. That is, the profit is maximum when x = 3.

H.W-Kayondo & D.W-Ddumba, Engineering Math I- Lecture Notes 200


CHAPTER 4. DIFFERENTIATION

Example 4.5.44 Given an equation y = f (x) = x2 , find the shortest distance between
the parabola and the point (6, 3).

Let D denote the distance between the parabola and the point (6, 3). If (x, y) is a point
on the parabola, then

D2 = (x − 6)2 + (y − 3)2 , y = x2
= (x − 6)2 + (x2 − 3)2

dD
2D = 2(x − 6) + 2(x2 − 3)(2x)
dx

= 2(x − 6) + 4x(x2 − 3)

dD
⇔ D = 2x3 − 5x − 6
dx

So dD/dx = 0 ⇒ 2x3 − 5x − 6 = 0 ⇔ x = 2 is a critical point of D. Now,

dD dD 2x3 − 5x − 6
D = 2x3 − 5x − 6 ⇒ =
dx dx D

d2 D D(6x2 − 5) − (2x3 − 5x − 6) dD
dx d2 D 19
= ⇒ = √ >0
dx2 D2 dx 2
17

at x = 2. √
Thus, D is minimum when x = 2 and y = 4, D2 = (−4)2 + (1)2 = 17 ⇒ min D = 17

H.W-Kayondo & D.W-Ddumba, Engineering Math I- Lecture Notes 201


CHAPTER 4. DIFFERENTIATION

4.5.4 Approximation of functions and Rates of change


From the definition of a derivative

f (x + h) − f (x)
f 0 (x) = lim
h→0 h

f (x0 + ∆x) − f (x0 )


f 0 (x0 ) = lim
∆x→0 ∆x
f (x + ∆x) − f (x0 )
f 0 (x0 ) ≈
∆x

f (x0 + ∆x) ≈ f (x0 ) + f 0 (x0 )∆x (4.12)

Where
x = x0 + ∆x

Example 4.5.45 Use differentials to approximate 65

f (x) = x

1
f 0 (x) = √
2 x
√ √
f (x) = 65 = 64 + 1

f (x0 + ∆x) ≈ f (x0 ) + f 0 (x0 )∆x


√ √
64 + 1 ≈ 64 + f 0 (64)(1)
√ √ 1
65 ≈ 64 + √ (1)
2 64
√ √ 1
65 ≈ 64 + (1)
16

65 ≈ 8 + 0.0625
≈ 8.0625

H.W-Kayondo & D.W-Ddumba, Engineering Math I- Lecture Notes 202


CHAPTER 4. DIFFERENTIATION

Example 4.5.46 Use differentials to find the value of (0.96)3

f (x) = x3
f 0 (x) = 3x2
f (x) = (0.96)3 = (1 − 0.04)3

f (x0 + ∆x) ≈ f (x0 ) + f 0 (x0 )∆x


(1 − 0.04)3 ≈ (1)3 + f 0 (1)(−0.04)
(0.96)3 ≈ (1)3 + 3(1)2 (−0.04)
(0.96)3 ≈ 1 − 0.12
≈ 0.88

Example 4.5.47 Find 36.01 using differentials

f (x) = x

1
f 0 (x) = √
2 x
√ √
f (x) = 36.01 = 36 + 0.01

f (x0 + ∆x) ≈ f (x0 ) + f 0 (x0 )∆x


√ √
36 + 0.01 ≈ 36 + f 0 (36)(0.01)
√ √ 1
36.01 ≈ 36 + √ (0.01)
2 36
√ √ 1
36.01 ≈ 36 + (0.01)
12

36.01 ≈ 6 + 0.0008333
≈ 6.0008333

Example 4.5.48 Show that


√ 1
82 = 9 +
18

using differentials

H.W-Kayondo & D.W-Ddumba, Engineering Math I- Lecture Notes 203


CHAPTER 4. DIFFERENTIATION

Example 4.5.49 Find sin 42◦ using differentials.

f (x) = sin x
f 0 (x) = cos x
f (x) = sin 42 = sin(45◦ − 3◦ )

−3π −π
∆x = −3◦ = =
180 60

into radians, since there will be no trigonometric function on ∆x, we change from degrees
to radians.

f (x0 + ∆x) ≈ f (x0 ) + f 0 (x0 )∆x

−π
 
0 ◦
sin(45 − 3) ≈ sin 45 + f (45 )
60
π
sin 42 ≈ sin 45 − cos 45
60
π
sin 42 ≈ sin 45 − cos 45
60
√ √  
2 2 π
sin 42 ≈ −
2 2 60
≈ 0.6701

Example 4.5.50 Show that


cos 62◦ = 0.46977
using differentials (linear approximations)

Example 4.5.51 Show that


cos 42◦ = 0.74314
using linear approximations

H.W-Kayondo & D.W-Ddumba, Engineering Math I- Lecture Notes 204


CHAPTER 4. DIFFERENTIATION


3
Example 4.5.52 Approximate 124 using differentials (without using calculators)

1
f (x) = x 3

1
f 0 (x) = 2
3x 3
1 1
f (x) = (124) 3 = (125 − 1) 3

f (x0 + ∆x) ≈ f (x0 ) + f 0 (x0 )∆x


1 1
(125 − 1) 3 ≈ (125) 3 + f 0 (125)(−1)

1 1 1
(124) 3 ≈ (125) 3 + 2 (−1)
3(125) 3

1 1 1
(124) 3 ≈ (125) 3 − (1)
75
1 1
(124) 3 ≈ 5 −
75
≈ 4.9867

Example 4.5.53 The position of a particle is given by the equation s(t) = t3 − 6t2 + 9t,
where t is measured in seconds and s in metres.

(a) Find the velocity at time t.

(b) What is the velocity after 5 seconds?

(c) When is the particle at rest?

Example 4.5.54 Air is being pumped into a spherical balloon such that its radius
increases at a rate of 0.75 in/min. Find the rate of change of its volume when the radius
is 5 inches (V = 34 πr3 ).

dV dV dr
=
dt dr dt
dr
= 4πr2
dt

= (4πr2 )(0.75) = (4π52 )(0.75)

H.W-Kayondo & D.W-Ddumba, Engineering Math I- Lecture Notes 205


CHAPTER 4. DIFFERENTIATION

Example 4.5.55 Gas is being pumped into a spherical balloon at the rate of 2 cm3 per
second. How fast is the surface area of the balloon increasing when the radius is 12 cm.

dS dS dr
=
dt dr dt
dS dS dV dr
=
dt dr dt dV

But a sphere of radius r has volume V = 43 πr3 and surface area S = 4πr2 . Thus

dV
= 4πr2
dr
dS
= 8πr
dr
dV
= 2
dt

dS dS dV dr
=
dt dr dt dV
dS 1 4
= (8πr)(2) 2
=
dt 4πr r

So when r = 12,
dS 4 4 1
= = = cm2
dt r 12 3

Example 4.5.56 Pressure (P) and volume (V) of air at room temperature are related
by the equation
P V 1.4 = C
Here C is a constant. At some instant t0 the pressure of the gas is 25 kg/cm2 and the
volume is 200 cm3 . Find the rate of change of P if the volume increases at a rate of
10 cm3 /min.

dP dP dV 1.4P dV
= · =− · = −1.75 Kg/cm2 sec
dt dV dt V dt

Since
dP 1.4
V + 1.4P V 0.4 = 0
dV

H.W-Kayondo & D.W-Ddumba, Engineering Math I- Lecture Notes 206


CHAPTER 4. DIFFERENTIATION

Exercise 4.37 Use differentials to approximate



(i) 64.2 (iii) (0.83)4

(ii) cos 91◦ (iv) 51

Exercise 4.38 Find the differential


√ √
(i) αx + β (ii) √x−1 (iii) sin x
x+1

Exercise 4.39 The radius of a sphere in creases from 10cm to 10.5cm. Use differentials
to approximate the relative and percentage change in its volume.

Exercise 4.40 Use derivatives to estimate 24. [4.9]

Exercise 4.41 Estimate 3 29 without using a calculator.

Exercise 4.42 Suppose the radius of a ball changes at a rate of 2 cm/min. At which
rate does its volume change when r = 20 cm?

3200π cm3 /min

Exercise 4.43 Suppose that a mountain climber ascends at a rate of 0.5 kilometer
per hour. The temperature is lower at higher elevations; suppose the rate by which it
decreases is 6◦ C per kilometer. To calculate the decrease in air temperature per hour that
the climber experiences, one multiplies 6◦ C per kilometer by 0.5 kilometer per hour, to
obtain 3◦ C per hour. This calculation is a typical chain rule application.
Exercise 4.44 State Intermediate Value Theorem and use it to show that the equation

2x3 − 3x2 = 12x + 6

has only one real root in the interval (−1, 0).


Exercise 4.45 State the Mean Value Theorem (M V T ) and the Roll’s theorem. Hence
find the constant c that satisfys the
(i) M V T for f (x) = x3 − 6x2 + 9x + 2 ; 0 ≤ x ≤ 4
(ii) Roll’s theorem for f (x) = 4 − x2 on [−2, 2]

√ 3
Exercise 4.46 Of the following, which is the best approximation of 1.5 (266) 2

(a) 1, 000 (b) 2, 700 (c) 3, 200 (d) 4, 100 (e) 5, 300

Exercise 4.47 Find the linear approximate value of y = 4 + sin x at x = 0.12 obtained
from the tangent to the graph at x = 0.

H.W-Kayondo & D.W-Ddumba, Engineering Math I- Lecture Notes 207


CHAPTER 4. DIFFERENTIATION

4.5.5 Curve Sketching


Among the steps involved in curve sketching are;
(i) x and y intercepts
(ii) where curve is increasing or decreasing
(iii) and classify the critical numbers
(iv) where function is concave up and concave down
(v) inflection points
(vi) horizontal, oblique and vertical asymptotes

Note 4.5.6 An oblique asymptote is an asymptote defined by a linear line.


Note 4.5.7 If you divide the polynomial into quotient and remainder, and then the
linear part will be your oblique asymptote.
Note 4.5.8 For a horizontal asymptote is a horizontal line and an oblique asymptote is
a line that is neither horizontal or vertical. In simple terms, it is slanted.
Example 4.5.57 Find the oblique asymptote of

(x2 + 2x + 2)
f (x) =
(x + 1)

The f (x) can be rewritten as

 
1
f (x) = (x + 1) +
(x + 1)

Hence the line y = x + 1 is the oblique asymptote.


Example 4.5.58 Find the slant asymptote of the following function:

x2 + 3x + 2
y=
x−2

The slant asymptote is y = x + 5 since

x2 + 3x + 2 12
y= = (x + 5) +
x−2 x−2

H.W-Kayondo & D.W-Ddumba, Engineering Math I- Lecture Notes 208


CHAPTER 4. DIFFERENTIATION

Example 4.5.59 Sketch the curve


y = x2 − 4
(i) x and y intercepts (where curve cuts the axes). i.e

2x2
f (x) = y = 0 ⇒ = 0 ⇒ (x − 2)(x + 2) = 0 ⇒ x = 2, x = −2
x2 − 1

x = 0 ⇒ f (x) = 02 − 4 ⇒ y = −4

(ii) where curve is increasing f 0 (x) > 0 or decreasing f 0 (x) < 0.

dy
f 0 (x) = = 2x
dx

Increasing:
f 0 (x) = 2x > 0 ⇔ x > 0
Decreasing:
f 0 (x) = 2x < 0 ⇔ x < 0

(iii) and classify the critical numbers (turning points)

dy
f 0 (x) = = 0 ⇒ 2x = 0
dx
⇒ 2x = 0 ⇒ x = 0

d2 y
f 00 (x) = =2
dx2

x = 0 is a local minima, since; f 00 (0) = 2 > 0


(iv) where the function is concave up f 00 (x) > 0 and concave down f 00 (x) < 0.
f 00 (x) = 2
concave up
f 00 (x) = 2 > 0 ∀ x
⇒ x ∈ (−∞, +∞)
⇒ x ∈ <

H.W-Kayondo & D.W-Ddumba, Engineering Math I- Lecture Notes 209


CHAPTER 4. DIFFERENTIATION

concave down

f 00 (x) = 2 6< 0 ∀ x
⇒ x ∈6 (−∞, +∞)
⇒ x ∈ 6 <

The curve is only concave up.

(v) Inflection points


An inflection point p is where the concavity of a function f switches from up to
down or down to up. That is where f 00 (x) = 0

f 00 (x) = 2 = 0
⇔ 2 = 0
⇔ a contradiction

The function f (x) = x2 − 4 has no point of inflection.

(vi) horizontal, oblique and vertical asymptotes:


There is no real horizontal asymptotes since

lim f (x) = lim x2 − 4 = ∞


x→+∞ x→+∞

& lim f (x) = lim x2 − 4 = −∞


x→−∞ x→−∞

For vertical asymptotes by computing one-sided limits at the zeroes of the denomi-
nator, i.e., No vertical asymptote as no denominator.
Thus the curve is

Figure 4.11: Curve y = x2 − 4

H.W-Kayondo & D.W-Ddumba, Engineering Math I- Lecture Notes 210


CHAPTER 4. DIFFERENTIATION

Example 4.5.60 Given the function

2x2
f (x) =
x2 − 1

find
(i) x and y intercepts
(ii) where curve is increasing or decreasing
(iii) and classify the critical numbers (turning points)
(iv) where function is concave up and concave down
(v) inflection points
(vi) horizontal, oblique and vertical asymptotes
Hence sketch the curve
2x2
f (x) = 2
x −1

(i) x and y intercepts (where curve cuts the axes). i.e

2x2
f (x) = y = 0 ⇒ = 0 ⇒ 2x2 = 0 ⇒ x = 0
x2 − 1

2x2
x = 0 ⇒ f (x) = ⇒ f (x) = y = 0
x2 − 1

(ii) where curve is increasing f 0 (x) > 0 or decreasing f 0 (x) < 0.

dy 4x(x2 − 1) − (2x)(2x2 ) −4x


f 0 (x) = = = 2
dx (x − 1)
2 2 (x − 1)2

Increasing:

−4x
f 0 (x) = >0 ⇔ x<0
(x2− 1)2

Decreasing:

−4x
f 0 (x) = <0 ⇔ x>0
(x2− 1)2

H.W-Kayondo & D.W-Ddumba, Engineering Math I- Lecture Notes 211


CHAPTER 4. DIFFERENTIATION

(iii) and classify the critical numbers (turning points)

dy −4x
f 0 (x) = =0 ⇒ =0
dx (x − 1)2
2

⇒ −4x = 0 ⇒ x = 0

d2 y −4(x2 − 1)2 + 4x · 2(x2 − 1) · 2x 12x2 + 4


f 00 (x) = = =
dx2 (x2 − 1)4 (x2 − 1)3

x = 0 is a local maxima, since; f 00 (0) = −4 < 0

(iv) where function is concave up f 00 (x) > 0 and concave down f 00 (x) < 0.

12x2 + 4
f 00 (x) =
(x2 − 1)3

concave up

12x2 + 4
f 00 (x) = > 0
(x2 − 1)3

⇔ (x2 − 1)3 > 0


⇔ (x2 − 1) > 0
⇔ x2 > 1
⇒ x ∈ (−∞, −1) ∪ (1, +∞)

concave down

00 12x2 + 4
f (x) = 2 < 0
(x − 1)3

⇔ (x2 − 1)3 < 0


⇔ (x2 − 1) < 0
⇔ x2 < 1
⇒ x ∈ (−1, 1)

(v) Inflection points


An inflection point p is where the concavity of a function f switches from up to

H.W-Kayondo & D.W-Ddumba, Engineering Math I- Lecture Notes 212


CHAPTER 4. DIFFERENTIATION

down or down to up. That is where f 00 (x) = 0

12x2 + 4
f 00 (x) = =0 ⇔ 12x2 + 4 = 0
(x2 − 1)3

1
⇔ x2 = −
3

2x2
The function f (x) = x2 −1
has no real point of inflection.

(vi) horizontal, oblique and vertical asymptotes:


There is a horizontal asymptote y = 2 since

2x2 2x2
lim f (x) = lim =2 & lim f (x) = lim =2
x→+∞ x→+∞ x2 − 1 x→−∞ x→−∞ x2 − 1

In this case, the oblique (slant) asymptote is also the horizontal asymptote y = 2. For
vertical asymptotes by computing one-sided limits at the zeroes of the denominator,
i.e.,
x2 − 1 = 0
at x = −1 and at x = 1.

f (x)

y=2

2x2
Figure 4.12: A curve f (x) = x2 −1

H.W-Kayondo & D.W-Ddumba, Engineering Math I- Lecture Notes 213


CHAPTER 4. DIFFERENTIATION

Example 4.5.61 Sketch the curve y = x3 − 3x2 − 13x + 15


Can you realise that
(i) x and y intercepts (where curve cuts the axes). i.e
f (x) = y = 0 ⇒ (x − 1)(x − 5)(x + 3) = 0 ⇒ x = 1, x = 5, x = −3
x = 0 ⇒ f (x) = x3 − 3x2 − 13x + 15 ⇒ f (x) = y = 15

(ii) where curve is increasing f 0 (x) > 0 or decreasing f 0 (x) < 0.

dy
f 0 (x) = = 3x2 − 6x − 13
dx

Increasing:
f 0 (x) = 3x2 − 6x − 13 > 0
⇔ x < −1.31, x > 3.31
⇒ x ∈ (−∞, −1.31) ∪ (3.31, +∞)
Decreasing:
f 0 (x) = 3x2 − 6x − 13 < 0
⇔ x > −1.31, x < 3.31
⇒ x ∈ (−1.31, 3.31)

(iii) and classify the critical numbers (turning points)

dy
f 0 (x) = = 0 ⇒ 3x2 − 6x − 13 = 0
dx
p
6 ± 62 − 4(3)(−13)
⇒ x=
2(3)

⇒ x = −1.31, 3.31

d2 y
f 00 (x) = = 6x − 6
dx2

x = −1.31 is a local maxima, since


f 00 (−1.31) = −13.86 < 0
x = 3.31 is a local minima, since
f 00 (3.31) = 13.86 > 0

H.W-Kayondo & D.W-Ddumba, Engineering Math I- Lecture Notes 214


CHAPTER 4. DIFFERENTIATION

(iv) where function is concave up f 00 (x) > 0 and concave down f 00 (x) < 0.

f 00 (x) = 6x − 6

concave up

f 00 (x) = 6x − 6 > 0
⇔ 6(x − 1) > 0 ⇔ x > 1 ⇔ x ∈ (1, +∞)

concave down

f 00 (x) = 6x − 6 < 0
⇔ 6(x − 1) < 0 ⇔ x < 1 ⇔ x ∈ (−∞, 1)

(v) Inflection points


An inflection point p is where the concavity of a function f switches from up to
down or down to up. That is where f 00 (x) = 0

f 00 (x) = 6x − 6 = 0 ⇔ x = 1

The function f (x) has a real point of inflection at x = 1.

(vi) horizontal, oblique and vertical asymptotes:


There are no horizontal asymptotes since

lim f (x) = lim (x3 − 3x2 − 13x + 15) = ∞ & lim f (x) = −∞
x→+∞ x→+∞ x→−∞

No vertical asymptotes since not a rational function

Figure 4.13: Curve y = x3 − 3x2 − 13x + 15

H.W-Kayondo & D.W-Ddumba, Engineering Math I- Lecture Notes 215


CHAPTER 4. DIFFERENTIATION

Example 4.5.62 Graph the curve

f (x) = x3 − 3x2

(i) x and y intercepts (where curve cuts the axes). i.e

y = 0 ⇒ x3 − 3x2 = 0 ⇒ x2 (x − 3) = 0 ⇒ x = 0, x = 3
x = 0 ⇒ f (x) = x3 − 3x2 ⇒ f (x) = y = 0

(ii) where the curve is increasing f 0 (x) > 0 or decreasing f 0 (x) < 0.

dy
f 0 (x) = = 3x2 − 6x
dx
= 3x(x − 2)

Increasing:

x>0&x>2 x>2
0
f (x) = 3x(x − 2) > 0 ⇔ or ⇔ or
x<0&x<2 x<0

Decreasing:

x>0&x<2 x ∈ (0, 2)
0
f (x) = 3x(x − 2) > 0 ⇔ or ⇔ or
x<0&x>2 x DNE

(iii) and classify the critical numbers (turning points)

dy
f 0 (x) = = 0 ⇒ 3x(x − 2) = 0
dx
⇒ x = 0, x = 2

d2 y
f 00 (x) = = 6x − 6
dx2

x = 0 is a local maxima, since; f 00 (0) = −6 < 0


x = 2 is a local minima, since; f 00 (2) = 6 > 0

H.W-Kayondo & D.W-Ddumba, Engineering Math I- Lecture Notes 216


CHAPTER 4. DIFFERENTIATION

(iv) where function is concave up f 00 (x) > 0 and concave down f 00 (x) < 0.

f 00 (x) = 6x − 6

concave up

f 00 (x) = 6x − 6 > 0
⇔ 6(x − 1) > 0 ⇔ x > 1 ⇔ x ∈ (1, +∞)

concave down

f 00 (x) = 6x − 6 < 0
⇔ 6(x − 1) < 0 ⇔ x < 1 ⇔ x ∈ (−∞, 1)

(v) Inflection points


An inflection point p is where the concavity of a function f switches from up to
down or down to up. That is where f 00 (x) = 0

f 00 (x) = 6x − 6 = 0 ⇔ x = 1

The function f (x) has a real point of inflection at x = 1.

(vi) horizontal, oblique and vertical asymptotes:


There are no horizontal asymptotes since

lim f (x) = lim (x3 − 3x2 ) = ∞ & lim f (x) = lim (x3 − 3x2 ) = −∞
x→+∞ x→+∞ x→−∞ x→−∞

No vertical asymptotes since not a rational function


The graph can be sketched as follows

Figure 4.14: A curve f (x) = x3 − 3x2

H.W-Kayondo & D.W-Ddumba, Engineering Math I- Lecture Notes 217


CHAPTER 4. DIFFERENTIATION

Example 4.5.63 Show that the curve


f (x) = x4 − 4x3
is given by

Figure 4.15: A curve f (x) = x4 − 4x3

Example 4.5.64 Graph the curve


f (x) = x3 (x − 2)2
(i) x and y intercepts (where curve cuts the axes). i.e
y = 0 ⇒ x3 (x − 2)2 = 0 ⇒ x = 0, x = 2
x = 0 ⇒ y = x3 (x − 2)2 = 0 ⇒ y = 0

(ii) where curve is increasing f 0 (x) > 0 or decreasing f 0 (x) < 0.

dy
f 0 (x) = = 3x2 (x − 2)2 + 2(x − 2)x3 = x2 (x − 2)[2x + 3(x − 2)] = x2 (x − 2)[5x − 6]
dx

Increasing:
 
6
x ∈ (−∞, 0) ∪ 0, ∪ (2, ∞)
5

Decreasing:
 
6
x ∈ , 2
5

H.W-Kayondo & D.W-Ddumba, Engineering Math I- Lecture Notes 218


CHAPTER 4. DIFFERENTIATION

(iii) and classify the critical numbers (turning points)

dy
= 0 ⇒ 3x2 (x − 2)2 + 2(x − 2)x3 = 0
dx

⇒ x2 (x − 2)[2x + 3(x − 2)] = x2 (x − 2)[5x − 6] = 0

6
⇒ x = 0, x = 2, x =
5

d2 y
f 00 (x) = = 4x[5x2 − 12x + 6]
dx2
6
x= is a local maxima
5
x = 0, 2 is a local minima

(iv) where function is concave up f 00 (x) > 0 and concave down f 00 (x) < 0.

(v) Inflection points

(vi) horizontal, oblique and vertical asymptotes: None

Figure 4.16: A curve f (x) = x3 (x − 2)2

H.W-Kayondo & D.W-Ddumba, Engineering Math I- Lecture Notes 219


CHAPTER 4. DIFFERENTIATION

Example 4.5.65 Graph the curve

2x2 − 3x
f (x) =
x−2

(i) x and y intercepts (where curve cuts the axes). i.e

3
y = 0 ⇒ 2x2 − 3x = x(2x − 3) = 0 ⇒ x = 0, x =
2
2x2 − 3x
x=0 ⇒ y= =0 ⇒ y=0
x−2

(ii) where curve is increasing f 0 (x) > 0 or decreasing f 0 (x) < 0.

dy 2x2 − 8x + 6 2(x − 1)(x − 3)


f 0 (x) = = =
dx (x − 2)2 (x − 2)2

Increasing:

2(x − 1)(x − 3)
f 0 (x) = > 0 ⇔ x ∈ (−∞, 1) ∪ (3, +∞)
(x − 2)2

Decreasing:

2(x − 1)(x − 3)
f 0 (x) = < 0 ⇔ x ∈ (1, 3)
(x − 2)2

The denominator is always positive.


(iii) and classify the critical numbers (turning points)

dy 2x2 − 8x + 6 2(x − 1)(x − 3)


=0 ⇒ = =0
dx (x − 2)2 (x − 2)2

⇒ 2(x − 1)(x − 3) = 0 ⇒ x = 1, x = 3

d2 y 4
f 00 (x) = =
dx 2 (x − 2)3

x=1 a maxima
x=3 a minima

H.W-Kayondo & D.W-Ddumba, Engineering Math I- Lecture Notes 220


CHAPTER 4. DIFFERENTIATION

(iv) where function is concave up f 00 (x) > 0 and concave down f 00 (x) < 0.

4
f 00 (x) =
(x − 2)3

concave up

4
f 00 (x) = > 0
(x − 2)3

⇒ x ∈ (2, ∞)
concave down

4
f 00 (x) = < 0
(x − 2)3

⇒ x ∈ (−∞, 2)

(v) Inflection points


An inflection point p is where the concavity of a function f switches from up to
down or down to up. That is where f 00 (x) = 0

4
f 00 (x) = = 0
(x − 2)3

The function f (x) has no real point of inflection.


(vi) horizontal, oblique and vertical asymptotes:

There is no horizontal asymptote since

2x2 − 3x
lim f (x) = lim =∞
x→+∞ x→+∞ x − 2

and

2x2 − 3x
lim f (x) = lim = −∞
x→−∞ x→−∞ x − 2

Remember, a horizontal asymptote exists only if the limit


lim f (x) = L exists
x→±∞

H.W-Kayondo & D.W-Ddumba, Engineering Math I- Lecture Notes 221


CHAPTER 4. DIFFERENTIATION

The oblique (slant) asymptote is y = 2x since

2x2 − 3x x
= 2x +
x−2 x−2

Now check for a vertical asymptote by computing one-sided limits at the zero of the
denominator, i.e., at x = 2. Thus,

2x2 − 3x 2
lim+ f (x) = lim+ = + = +∞
x→2 x→2 x−2 0

2x2 − 3x 2
lim− f (x) = lim− = − = −∞
x→2 x→2 x−2 0

This shows that the line x = 2 is a vertical asymptote for the graph of f . Remember,
if either of these one-sided limits is +∞ or −∞, a vertical asymptote exists.

See the adjoining detailed graph of f .

f (x)

y = 2x

x=2

2x2 −3x
Figure 4.17: A curve f (x) = x−2

H.W-Kayondo & D.W-Ddumba, Engineering Math I- Lecture Notes 222


CHAPTER 4. DIFFERENTIATION

Example 4.5.66 Graph the curve

(x − 4)2
f (x) =
x2 − 4

(i) Where curve cuts the axes. i.e x and y intercepts

(x − 4)2
y=0 ⇒ = (x − 4)2 = 0 ⇒ x = 4
x2 − 4

(x − 4)2
x=0 ⇒ y= = 0 ⇒ y = −4
x2 − 4

(ii) Increasing and decreasing function: Exercise

(iii) The turning points (critical or stationary points)

dy 8(x − 4)(x − 1)
=0 ⇒ =0
dx (x2 − 4)2

⇒ 8(x − 4)(x − 1) = 0 ⇒ x = 1, x = 4

d2 y 8 [−2x3 + 15x2 − 24x + 20]


=
dx2 (x2 − 4)3

x=1 a maxima; (x, y) = (1, −3)


x=4 a minima; (x, y) = (4, 0)

(iv) Concave up and concave down: Exercise

(v) Points of inflection: Exercise

(vi) asymptotes:

There is a horizontal asymptote since

(x − 4)2
lim f (x) = lim =1
x→+∞ x→+∞ x2 − 4

(x − 4)2
lim f (x) = lim =1
x→−∞ x→−∞ x2 − 4

H.W-Kayondo & D.W-Ddumba, Engineering Math I- Lecture Notes 223


CHAPTER 4. DIFFERENTIATION

Thus, the line y = 1 is a horizontal asymptote for the graph of f . The oblique
asymptote is also y = 1 since

(x − 4)2 (−8x + 20)


f (x) = y = = 1 +
x2 − 4 x2 − 4

Now check for vertical asymptotes by computing one-sided limits at the zeroes of
the denominator, i.e., at x = 2 and at x = −2. Thus,

(x − 4)2 4 (x − 4)2 4
lim+ f (x) = lim+ = + = +∞ & lim− f (x) = lim− = − = −∞
x→2 x→2 x −4
2 0 x→2 x→2 x −4
2 0

(x − 4)2 36 (x − 4)2 36
lim + f (x) = lim + = + = +∞ & lim − f (x) = lim − = − = −∞
x→−2 x→−2 x −4
2 0 x→−2 x→−2 x −4
2 0

This shows that the line x = −2 and x = 2 are vertical asymptote for the graph of
f . Remember, if either of these one-sided limits is +∞ or −∞, a vertical asymptote
exists.

f (x)

y=1

(x−4)2
Figure 4.18: A curve f (x) = x2 −4

H.W-Kayondo & D.W-Ddumba, Engineering Math I- Lecture Notes 224


CHAPTER 4. DIFFERENTIATION

Example 4.5.67 Graph the curve

x2 |x − 3|
f (x) =
(5 + x)

Here the point of trouble is at x = 3 above and below, so we are to use two graphs

x2 (3−x)


 (5+x)
, if x ≤ 3
f (x) =
 x2 (x−3)

(5+x)
, if x > 3

Example 4.5.68 Graph the curve



f (x) = x 4 − x2

(i) Where curve cuts the axes. i.e

√ p
y = 0 ⇒ x 4 − x2 = x (2 − x)(2 + x) = 0 ⇒ x = 0, x = 2, x = −2

x = 0 ⇒ y = x 4 − x2 = 0 ⇒ y = 0

(ii) The turning points and their nature.

√ √
dy 2(2 − x2 ) 2( 2 − x)( 2 + x)
=0 ⇒ √ =0 ⇒ √ =0
dx 4 − x2 4 − x2
√ √ √ √
⇒ 2( 2 − x)( 2 + x) = 0 ⇒ x = − 2, x = 2

and

d2 y 2x(x2 − 6) √ √ √
= ⇒ x = 2 absolute maxima (x, y) = ( 2, 2 2)
dx2 (4 − x2 )3/2
√ √ √
& x = − 2 absolute minima (x, y) = (− 2, −2 2)

(iii) Increasing and decreasing function: Exercise

(iv) Concave up and concave down: Exercise

(v) Points of inflection: Exercise

(vi) asymptotes: Exercise

H.W-Kayondo & D.W-Ddumba, Engineering Math I- Lecture Notes 225


CHAPTER 4. DIFFERENTIATION

Example 4.5.69 The curve for

x2 |x − 3|
f (x) =
(5 + x)

can be shown to be


Figure 4.19: A curve f (x) = x 4 − x2

H.W-Kayondo & D.W-Ddumba, Engineering Math I- Lecture Notes 226


CHAPTER 4. DIFFERENTIATION

Example 4.5.70 Graph the curve


1
f (x) = x − 3x 3

(i) Where curve cuts the axes. i.e

1 1 2 √ √
y = 0 ⇒ x − 3x 3 = x 3 (x 3 − 3) = 0 ⇒ x = 0, x = 27, x = − 27
1
x = 0 ⇒ y = x − 3x 3 = 0 ⇒ y = 0

(ii) The turning points

dy 1 x2/3 − 1
= 0 ⇒ 1 − 3(1/3)x 3 −1 = 0 ⇒ 1 − x−2/3 = = 0 ⇒ x = −1, x = 1
dx x2/3

d2 y 2
(iii) Maxima or Minima: Since dx2
= 3x5/3

x = −1 a maxima (x, y) = (−1, 2)


x=1 a minima (x, y) = (1, −2)

1
Figure 4.20: A curve f (x) = x − 3x 3

H.W-Kayondo & D.W-Ddumba, Engineering Math I- Lecture Notes 227


CHAPTER 4. DIFFERENTIATION

Example 4.5.71 Graph the curve

|x + 2|x2
f (x) =
|x|

Now its not only one point to think of, but now both x = −2 and x = 0. We need to
have different functions for

x < −2
−2 ≤ x ≤ 0
x>2

−(x+2)x2


 −(x)
, if x < −2




(x+2)x2
f (x) = −(x)
, if − 2 ≤ x ≤ 0




(x+2)x2

, if x > 0


(x)

and sketch those function in the different ranges.

Exercise 4.48 Plot the curves;


x−5
(i) y = x2 (ii) y = x2 −9
(iii) y 2 = x

Example 4.5.72 f is a function given by

f (x) = |x − 2|

Find the x and y intercepts of the graph of f . Find the domain and range of f . Sketch
the graph of f .

(a) The y intercept is given by

(0, f (0)) = (0, | − 2|) = (0, 2)

The x coordinate of the x intercepts is equal to the solution of the equation

|x − 2| = 0

which is x = 2

(b) The x intercepts is at the point (2, 0)

H.W-Kayondo & D.W-Ddumba, Engineering Math I- Lecture Notes 228


CHAPTER 4. DIFFERENTIATION

(c) The domain of f is the set of all real numbers


Since |x − 2| is either positive or zero for x = 2, the range of f is given by the interval
[0, +∞).

(d) To sketch the graph of f (x) = |x − 2|, we first sketch the graph of y = x − 2 and then
take the absolute value of y.

The graph of y = x − 2 is a line with x intercept (2, 0) and y intercept (0, −2).

Figure 4.21: A curve y = x − 2

(e) But


(x − 2) , x≥2
|x−2| =
−(x − 2) , x<2

whose curve is given on the


right hand side

H.W-Kayondo & D.W-Ddumba, Engineering Math I- Lecture Notes 229


CHAPTER 4. DIFFERENTIATION

Example 4.5.73 Plot the curve


f (x) = |(x − 2)2 − 4|

Figure 4.22: The curves y = (x − 2)2 − 4 and y = |(x − 2)2 − 4|

Example 4.5.74 Graph the curve


y = |x − 3| + 2
Example 4.5.75 Graph the function

2x2 − 11
y=
x2 + 9

Take note of the horizontal asymptote


Example 4.5.76 Find the slant (oblique) asymptote of the following function

2x3 + 4x2 − 9
f (x) =
3 − x2

y = −2x − 4
Exercise 4.49 Sketch the curve

x2 + 2
f (x) =
x−2

H.W-Kayondo & D.W-Ddumba, Engineering Math I- Lecture Notes 230


Chapter 5

The Definite Integral

5.1 Area of plane regions


We can approximate the area under a curve by counting the full squares covered.

Figure 5.1: Unit Squares covered

2 ≤ R ≤ 11
after counting where the figure falls.

231
CHAPTER 5. THE DEFINITE INTEGRAL

5.2 Approximating Sums: Riemann sums


5.2.1 Introduction
Georg Friedrich Bernhard Riemann (pronounced REE man or in IPA: September 17, 1826
July 20, 1866) was a German mathematician who made important contributions to anal-
ysis and differential geometry, some of them paving the way for the later development of
general relativity.

Early life

Riemann was born in Breselenz, a village near Dannenberg in the Kingdom of Hanover in
what is today Germany. His father, Friedrich Bernhard Riemann, was a poor Lutheran
pastor in Breselenz who fought in the Napoleonic Wars. His mother died before her
children were grown. Riemann was the second of six children, shy, and suffered from
numerous nervous breakdowns. Riemann exhibited exceptional mathematical skills, such
as fantastic calculation abilities, from an early age, but suffered from timidity and a fear
of speaking in public.

Middle life

In high school, Riemann studied the Bible intensively, but his mind often drifted back
to mathematics. To this end, he even tried to prove mathematically the correctness of
the Book of Genesis. His teachers were amazed by his genius and his ability to solve
extremely complicated mathematical operations. He often outstripped his instructor’s
knowledge. In 1840, Riemann went to Hanover to live with his grandmother and attend
lyceum (middle school). After the death of his grandmother in 1842, he attended high
school at the Johanneum Lneburg. In 1846, at the age of 19, he started studying philology
and theology in order to become a priest and help with his family’s finances.

In 1847, his father (Friedrich Riemann), after gathering enough money to send Riemann
to university, allowed him to stop studying theology and start studying mathematics. He
was sent to the renowned University of Gttingen, where he first met Carl Friedrich Gauss,
and attended his lectures on the method of least squares.

In 1847, Riemann moved to Berlin, where Jacobi, Dirichlet, and Steiner were teaching.
He stayed in Berlin for two years and returned to Gttingen in 1849.

Later life

Bernhard Riemann held his first lectures in 1854, which not only founded the field of
Riemannian geometry but set the stage for Einstein’s general relativity. In 1857, there
was an attempt to promote Riemann to extraordinary professor status at the Univer-

H.W-Kayondo & D.W-Ddumba, Engineering Math I- Lecture Notes 232


CHAPTER 5. THE DEFINITE INTEGRAL

sity of Gttingen. Although this attempt failed, it did result in Riemann finally being
granted a regular salary. In 1859, following Dirichlet’s death, he was promoted to head
the mathematics department at Gttingen. He was also the first to propose the theory of
higher dimensions[citation needed], which greatly simplified the laws of physics. In 1862
he married Elise Koch and had a daughter. He died of tuberculosis on his third journey
to Italy in Selasca (now a hamlet of Verbania on Lake Maggiore) where he was buried in
the cemetery in Biganzolo (Verbania).

He had to flee Gttingen in a hurry when the armies of Hanover and Prussia clashed there.
This haste for a sick man may have hastened his end. When she heard of his death,
his housekeeper at Gttingen started to throw out the papers in his study thus possibly
destroying a proof of the Riemann hypothesis. No one else has yet proved it and another
paper suggests that he had at least the bones of a proof[1].

5.2.2 Riemann sums


Assuming the function is continuous on [a, b] and f (x) ≥ 0 ∀ x  [a, b]

We try to find the area R between the curve and the y-axis.
We divide [a, b] into n equal sub-intervals such that

a = x0 ≤ x1 ≤ · · · ≤ xn−1 ≤ xn = b

xi = x0 + i∆x

b−a
Where ∆x =
n

H.W-Kayondo & D.W-Ddumba, Engineering Math I- Lecture Notes 233


CHAPTER 5. THE DEFINITE INTEGRAL

e.g x2 = x0 + 2∆x

Since the function is continuous it attains a maximum f (di ) and a minimum f (ci ) in every
interval: For example, take the interval.

Such that can get two areas

H.W-Kayondo & D.W-Ddumba, Engineering Math I- Lecture Notes 234


CHAPTER 5. THE DEFINITE INTEGRAL

The area can be (∆x)f (di ) or (∆x)f (ci ), but non of them is the accurate area AI
where
(∆x)f (ci ) < AI < (∆x)f (di )
with [c, d]  [xi−1 , xi ]

So now the total area

n
X
Sn = (∆x)f (ti ) where ti ∈ [xi−1 , xi ]
i=1

n
X n
X
S̄n = (∆x)f (di ) = (∆x)f (xi ) and
i=1 i=1

n
X n
X
Sn = (∆x)f (ci ) = (∆x)f (xi−1 ) such that,
¯ i=1 i=1

Sn ≤ AR ≤ S̄n
¯
Since both Sn (the lower sum) and S̄n (the upper sum) are increasing and decreasing both
¯ and above i.e Monotone and bounded sequences, then they converge to
bounded below
AR .

Note 5.2.1

The sum of the rectangles is an approximation of the area AR . Choosing more and more,
skinner and skinner rectangles gives better and better approximation. The limit of these
approximations as the number of rectangles → ∞ and their widthes → 0, gives precisely

H.W-Kayondo & D.W-Ddumba, Engineering Math I- Lecture Notes 235


CHAPTER 5. THE DEFINITE INTEGRAL

the area AR we need.

n
X
AR = lim Sn = lim (∆x)f (ti ) where ti ∈ [xi−1 , xi ]
n→∞ n→∞
i=1

n
X
AR = lim S̄n = lim (∆x)f (xi ) or
n→∞ n→∞
i=1

n
X
AR = lim Sn = lim (∆x)f (xi−1 ) such that,
n→∞ ¯ n→∞
i=1

AR = lim S̄n = lim Sn


n→∞ n→∞ ¯

Example 5.2.1 Find the area of the region between the curves y = x2 and the lines
y = 0, x = 0 and x = 1

then
b−a 1−0 1
∆x = = =
n n n

i i
xi = xo + i∆x = 0 + =
n n
i−1
⇒ xi−1 =
n

Sincef (x) = x2

H.W-Kayondo & D.W-Ddumba, Engineering Math I- Lecture Notes 236


CHAPTER 5. THE DEFINITE INTEGRAL

2
i−1

f (xi−1 ) = f (ci ) =
n

i2
f (xi ) = f (di ) =
n2

Since the series identities

n
X n
i = (n + 1)
i=1
2

n
X n
i2 = (n + 1)(2n + 1)
i=1
6

n
X n2
i3 = (n + 1)2
i=1
4

n
X n5 n4 n3 n
i4 = + + −
i=1
5 2 3 30

Approximating the lower sum

n
X n
X
Sn = (∆x)f (ci ) = (∆x)f (xi−1 )
¯ i=1 i=1

n 2 n n
1 i−1

X 1 X 2 1 X 2
= = 3 (i − 1) = 3 [i − 2i + 1]
i=1
n n n i=1
n i=1

n n n
!
1 X
2
X X
= i −2 i+ 1
n3 i=1 i=1 i=1

1 n n 
= (n + 1)(2n + 1) − 2 (n + 1) + n
n3 6 2
 
1 1 3 2 2
= (2n + 3n + n) − (n + n) + n
n3 6
 
1 3 2
 1 3 4
= 2n − 3n − 4n = 2− − 2
6n3 6 n n

H.W-Kayondo & D.W-Ddumba, Engineering Math I- Lecture Notes 237


CHAPTER 5. THE DEFINITE INTEGRAL

Approximating the upper sum

n
X n
X
S̄n = (∆x)f (di ) = (∆x)f (xi )
i=1 i=1

n  2
X 1 i
=
i=1
n n

n
1 X 2
= i
n3 i=1

n
1 X 2
= i
n3 i=1

1 n 
= (n + 1)(2n + 1)
n3 6
 
1 1 3 2
= (2n + 3n + n)
n3 6
 
1 1 1
S̄n = + + 2
3 2n 6n

Taking limits as n → ∞

1
AR = lim S̄n =
n→∞ 3
1
AR = lim Sn =
n→∞ ¯ 3

1
Thus AR = 3

H.W-Kayondo & D.W-Ddumba, Engineering Math I- Lecture Notes 238


CHAPTER 5. THE DEFINITE INTEGRAL

Example 5.2.2 Find the area under the graph f (x) = x4 from x = 0 to x = b > 0.

b
∆x =
n
ib
xi =
n
n
X
S̄n = (∆x)f (xi )
i=1

n
b5 X 4
= i
n i=1

b5
=
5

b5 b5
AR = lim =
n→∞ 5 5

Note 5.2.2 Since, the upper and lower will converge to the same value, we do not need
to bother with both.
8
Example 5.2.3 Find the area under x2 between [0, 2] AR ≈ 3

Definition 5.2.1 A partition of closed interval [a, b] is the set P = xo , x1 , ..., xn of points
on [a, b] such that
a = x0 < x1 < · · · < xn = b
Definition 5.2.2 The norm (mesh) of Pn denoted by k Pn k or m(Pn ) is defined as
k Pn k= max(xi − xi−1 )
i

For example, in the figure below

The partition is P = [1, 4.2] and the mesh k Pn k= 0.6

H.W-Kayondo & D.W-Ddumba, Engineering Math I- Lecture Notes 239


CHAPTER 5. THE DEFINITE INTEGRAL

Rb
Definition 5.2.3 The definite integral a
f (x)dx is defined as

Z b n
X
f (x)dx = lim (∆xi )f (ti )
a n→∞
i=1

where ti ∈ [xi−i , xi ] any of the two ends or mid point (no criterion is set for making the
selection). And if this limit exists, and k Pn k→ 0 then the sum → AR

Condition 5.1 The area under the curve will be equal to the Riemann sum if

(i) f (x) is continuous,

(ii) f (x) > 0.

Note 5.2.3 A sketch of the graph is always very important, before computing the Rie-
mann sums.

Example 5.2.4 Compute the area bounded by the curve f (x) = 2x − 4 and the lines
x = 0 and x = 3.

Computing the Riemann sum

n     
X 3 3i
S̄n = 2 −4
i=1
n n
 Xn  
3 6i
= −4
n i=1 n

 " X n n
#
3 6 X
= i− 4
n n i=1 i=1

  
3 6n
= (n + 1) − 4n
n n2
 
3
= [3 − n]
n
 
9
S̄n = −3
n

AR = lim S̄n = −3
n→∞

H.W-Kayondo & D.W-Ddumba, Engineering Math I- Lecture Notes 240


CHAPTER 5. THE DEFINITE INTEGRAL

Computing the exact value of the definite integral

Z 3
(2x − 4)dx = −3
0

Note 5.2.4 The value −3 in the example is not equal to the area of f (x), since f (x) <
0, ∀ x ∈ [0, 2]

Area cannot be negative, why the need to split the region into two, so that we compute
the Riemann sum of two regions and sum them. The negative sign imply region is below
the x-axis.
Z 3 Z 2 Z 3
(2x − 4)dx = (2x − 4)dx + (2x − 4)dx = 5
0 0 2

Repeat using the

(i) Lower point


ti = xi−1

(ii) mid point


xi + xi−1
ti =
2

H.W-Kayondo & D.W-Ddumba, Engineering Math I- Lecture Notes 241


CHAPTER 5. THE DEFINITE INTEGRAL

Example 5.2.5 Compute the area under the curve sin x and the lines y = 0, x = 0 and
x = 2π.
Realize that the integral
Z 2π
sin x dx = 0
0

but the area (Riemann sum) is not zero (After splitting the region), although the
definite integral equal to zero.
x x3
sin x ≈ −
1! 3!

Note 5.2.5 A definite integral is not necessarily equal to a Riemann sum. The integral
of a function represents the area between the graph of the function and the x-axis only
when the function is positive. If the function is negative, then the area is represented by
minus the integral.

It must be noted that not all functions are integrable. However the following theorem
holds for continuous functions

Theorem 5.2.1 If f is continuous on [a, b], then f is integrable on [a, b].

H.W-Kayondo & D.W-Ddumba, Engineering Math I- Lecture Notes 242


CHAPTER 5. THE DEFINITE INTEGRAL

Example 5.2.6 For the function y = f (x) = x + 2 on [1, 15],


by making x1 = 3, x2 = 5, x3 = 10, x4 = 14, x5 = 15, evaluate both S̄n and Sn the area
¯
obtained by taking inscribed and circumscribed rectangles respectively.

Since the curve is only above the x-axis and strictly increasing, xi is the upper limit
and xi−1 is the lower limit.

5
X
S̄n = (∆xi )f (xi )
i=1

= (2)f (3) + (2)f (5) + (5)f (10) + (4)f (14) + (1)f (15)
= 165
5
X
Sn = (∆xi−1 )f (xi−1 )
¯ i=1

= (2)f (1) + (2)f (3) + (5)f (5) + (4)f (10) + (1)f (14)
= 115
Note 5.2.6 To emphasis the same idea (approximating sums), but solutions too approx-
imated since number of subinterval are definite.
Rb
Example 5.2.7 Use Riemann sums to compute 0 x2 dx

b3
AR =
3

H.W-Kayondo & D.W-Ddumba, Engineering Math I- Lecture Notes 243


CHAPTER 5. THE DEFINITE INTEGRAL

Note 5.2.7 Geometrically, the Riemann sum is the sum of areas of the rectangles lying
above the x-axis plus the negative of the areas lying below the x-axis (algebraic sum)
Example 5.2.8 Using a regular partition, approximate
Z 1
x2 dx
0

using the left-end, midpoint and right-end Riemann sum with n = 10. Since the curve is
strictly increasing, left-end is the lower limit xi−1 , right-end is the upper limit xi .

The working can be best be handled in a table

LEFT MID RIGHT

i x?i = xi−1 f (x?i ) x?i f (x?i ) x?i = xi f (x?i )

1 0 0 0.05 0.0025 0.1 0.01

2 0.1 0.01 0.15 0.225 0.2 0.04

3 0.2 0.04 0.25 0.0625 0.3 0.09

4 0.3 0.09 0.35 0.1225 0.4 0.16

5 0.4 0.16 0.45 0.2025 0.5 0.25

6 0.5 0.25 0.55 0.3025 0.6 0.34

7 0.6 0.36 0.65 0.4225 0.7 0.49

8 0.7 0.49 0.75 0.5625 0.8 0.64

9 0.8 0.64 0.85 0.7225 0.9 0.81

10 0.9 0.81 0.95 0.9025 1.0 1.0

f (x? ) = 3.325
P P P
f (xi−1 ) = 2.85 f (xi ) = 3.85

Using the
(a) lower limit
A = 0.1 × 2.85 = 0.285

H.W-Kayondo & D.W-Ddumba, Engineering Math I- Lecture Notes 244


CHAPTER 5. THE DEFINITE INTEGRAL

(b) midpoint limit


A = 0.1 × 3.325 = 0.333

(c) upper limit


A = 0.1 × 3.85 = 0.385

Example 5.2.9 Use Example 5.2.8 to give the left-end, midpoint, and right-end Rie-
mann sums of
1
(a) f (x) = x
on [1, b]; n = 5.

(b) f (x) = x3 − 3x on [1, 4]; n = 5.



(c) f (x) = 1 + 2 x on [2, 3]; n = 5.

n
2i
− 1 n1
P 
Example 5.2.10 Evaluate the limit I = lim n
[1]
n→∞ i=1

n
i
P 1
Example 5.2.11 Evaluate the limit I = lim n2 2
n→∞ i=1

Example 5.2.12 Evaluate the integrals


R2
(a) 0 x dx

R4
(b) 0
x3 dx

using Riemann sums.

Note 5.2.8 The Riemann sums over [0, 1] and for regular partitions ∆x = n1 this allows
an integral to be evaluated from an expression of the limit of its Riemann sum.

Exercise 5.1 Simplify


n
X
(n2 − i2 )2
i=1

Exercise 5.2 Find the area under the graph from a to b. (A sketch is always necessary).

(a) f (x) = x2 , a = 0, b = 1.

(b) f (x) = 3x2 + 2 , a = 1, b = 5.

(c) f (x) = 10 − x2 , a = 1, b = 3.

H.W-Kayondo & D.W-Ddumba, Engineering Math I- Lecture Notes 245


CHAPTER 5. THE DEFINITE INTEGRAL

Exercise 5.3 Find the area A under the given functions on the given intervals

(i) f (x) = 3x2 − 5 , [1, 6].

(ii) f (x) = 3x2 − 5 , [1, b] b > 1.

Exercise 5.4 From the function y = x4 on [0, b]

(i) By using a regular partition, establish the width of each interval


when n = 100.

(ii) By making b = 50, x1 = 10, x2 = 15, x3 = 30, x4 = 38 and x5 = 50, evaluate


both A and Ā the area obtained by taking inscribed and circumscribed rectangles
¯
respectively.

Exercise 5.5 Compute the Riemann sum for the function f (x) = 2x − x2 over the
interval [0, 4] using

(a) x?i = xi the upper limit.

(b) x?i = xi−1 the lower limit.


xi +xi−1
(c) x?i = 2
the mid point.

Hint: A sketch is necessary since part of the area is below the x-axis. Whenever the width
is not given, we assume an equal width.

Exercise 5.6 Compute the Riemann sum for the function f (x) = 2x − x2 over the
interval [0, 4] for the partition [0, 1, 2.5, 3, 4] choosing

x?1 = 0.5, x?2 = 1.5, x?3 = 2.8, x?4 = 3.2

R3
Exercise 5.7 Let f (x) = x2 , evaluate 0
x2 dx by subdividing [0, 3] into n subintervals.

Exercise 5.8 Find the area A (in square units) of the region bounded by the function
f (x) = 6 − x, the line x = 1, the x-axis and the line x = 5 using circumscribed and
inscribed rectangles.

Exercise 5.9 Find the area of the region bounded by the x-axis, and the lines x = a,
and x = b where a < x < b using circumscribed rectangles. What is the geometric
interpretation.

Exercise 5.10 Use mathematical induction to prove the formula

n
X n
k= (n + 1)
k=1
2

H.W-Kayondo & D.W-Ddumba, Engineering Math I- Lecture Notes 246


CHAPTER 5. THE DEFINITE INTEGRAL

Exercise 5.11 Approximate


Z 2π
cos x dx
0

x2 x4
Using the Riemann sums. [Hint: cos x ≈ 1 − 2!
+ 4!
]
Exercise 5.12 Given f (x) = 2 + x2 with −1 ≤ x ≤ 2, find the Riemann sum for the
function f on [−1, 2].
For the partition P : x0 = −1, x1 = 0, x2 = 0.5, x3 = 1.5 and x4 = 2 and 1 = − 12 , 2 =
0.3, 3 = 1 and 4 = 1.8. What is the mesh of this partition.
Hint: The i = x? to use for each subinterval.
Theorem 5.2.2 For continuous functions f (x) and g(x).
Rb Rb Rb
(a) a
(f (x) ± g(x)) dx = a
f (x)dx ± a
g(x)dx

Rb Rb
(b) a
αf (x)dx = α a
f (x)dx

Rb Rc Rb
(c) a
f (x)dx = a
f (x)dx + c
f (x)dx where c ∈ [a, b]

Ra Rb
(d) b
f (x)dx = − a
f (x)dx
Exercise 5.13 Prove theorems 5.2.2 above.
Example 5.2.13 Find the definite integral represented by the Riemann sum
n  
X 2i 1
lim −1 on [0, 1]
n→∞
i=1
n n

Z b n
X
f (x) dx = lim ∆xf (xi )
a n→∞
i=1

n  
X 1 i 1 i
= lim f since ∆x = , xi =
n→∞
i=1
n n n n

n   n  
X 1 i X 2i 1
lim f = lim −1
n→∞
i=1
n n n→∞
i=1
n n
   
i 2i
⇒ f = −1
n n

⇒ f (x) = (2x − 1)

H.W-Kayondo & D.W-Ddumba, Engineering Math I- Lecture Notes 247


CHAPTER 5. THE DEFINITE INTEGRAL

Thus the definite integral that represents the Riemann sum is

n   Z 1
X 2i 1
lim −1 = 2x − 1 dx
n→∞
i=1
n n 0

Example 5.2.14 What if the interval is not [0, 1]?

n    
X 5i 5
lim 2 5+ +5 on [5, 10]
n→∞
i=1
n n

Since
5 5i
∆x = , xi = 5 +
n n
n    
X 5i 5
lim 2 5+ +5
n→∞
i=1
n n
|{z}
∆x

 
 
5i  5i 
f (xi ) = f 5 +
n
= 25+   + 5 ⇒ f (x) = 2x + 5
| {z n}

xi

n     Z 10
X 5i 5
lim 2 5+ +5 = 2x + 5 dx
n→∞
i=1
n n 5

Note 5.2.9 Whenever the interval is not given, we use [0, 1]. The trick is with first
pulling out n1 .
Example 5.2.15
n Z 1
X i2
lim 3
= x2 dx
n→∞
i=1
n 0

Example 5.2.16
n Z 1
X i3
lim 4
= x3 dx
n→∞
i=1
n 0

H.W-Kayondo & D.W-Ddumba, Engineering Math I- Lecture Notes 248


CHAPTER 5. THE DEFINITE INTEGRAL

Example 5.2.17

n √ Z 1
X i √
lim √ = x dx
n→∞
i=1
n n 0

Example 5.2.18
r r r r ! n √
1 2 3 n 1 X i
lim + + + ··· + = lim √
n→∞ n n n n n n→∞
i=1
n n
Z 1 √
= x dx
0

Example 5.2.19

√ √ √ √ ! n √
2 1 + 2 2 + 2 3 + ··· + 2 n X 2 i
lim √ = lim √
n→∞ n n n→∞
i=1
n n
Z 1 √
= 2 x dx
0

Example 5.2.20 Find the Riemann sum for f (x) = x3 , 0 ≤ x ≤ 2 with 4 terms (4
subintervals) using equal subintervals , taking the selection to be the right end points. (
Give your answer correct to 6 decimal places).
Example 5.2.21 Evaluate

n  2
X 6 3i 9
lim 2+ +
n→∞
i=1
n n n

By interpreting
n  2
X 6 3i 9
lim 2+ +
n→∞
i=1
n n n

as area (under a curve f (x)) , and given

Z 5 n  2
X 6 3i 9
f (x) dx = lim 2+ +
2 n→∞
i=1
n n n

H.W-Kayondo & D.W-Ddumba, Engineering Math I- Lecture Notes 249


CHAPTER 5. THE DEFINITE INTEGRAL

determine the function f (x).


Note 5.2.10 Whenever the value of n is not given, we use the analytical Riemann sum.
Example 5.2.22 Let f be a continuous function on a closed interval [a, b] and Pn be a
partition of [a, b]
(i) Define a partition Pn on [a, b].
(ii) Define a Riemann sum R(f, Pn ) of f over [a, b] with respect to the partition Pn .
(iii) Obtain the lower and upper Riemann sums of f (x) = x3 over [0, 1] for a partition
Pn of n subintervals. Hence show that
lim L(f, Pn ) = lim U (f, Pn )
n→∞ n→∞

where L(f, Pn ) and U (f, Pn ) are respectively the lower and upper Riemann sums of
f over [0, 1] with respect to the partition Pn .

Pn n2 (n+1)2
Hint: i=1 i3 = 4

Example 5.2.23 Compute the Riemann sum for the function f (x) = 2x + x2 over the
interval [0, 4] for
(i) x?i = xi the upper limit, using the circumscribed rectangles with n = 4.
(ii) x?i = xi−1 the lower limit, using the inscribed rectangles with n = 4.
(iii) the partition [0, 1, 2.5, 3, 4] choosing
x?1 = 0.5, x?2 = 1.5, x?3 = 2.8, x?4 = 3.2

Example 5.2.24 State the Fundermental Theorem(s) of Calculus. Find the derivative
of F if
Z sin x
3
F (x) = 1 − t2 2 dt
x

Example 5.2.25 Write the following expression as a definite integral

13 + 23 + 33 + · · · + n3
lim
n→∞ n4

Example 5.2.26 State the Mean Value Theorem for integral calculus. Find all numbers
c for which the function
f (x) = x2 − 4x + 5
defined on the closed interval [0, 3], satisfies the Mean Value Theorem.

H.W-Kayondo & D.W-Ddumba, Engineering Math I- Lecture Notes 250


CHAPTER 5. THE DEFINITE INTEGRAL

Example 5.2.27 Using the function f (x) = 2x − 4 on [0, 3], demonstrate that the
R3
Riemann sum is not necessarily equal to the definite integral 0 2x − 4 dx.

Example 5.2.28 Show that the sums

10
X
(4i − 3) = 190
1

10
X
(3t2 + 1) = 1165
1

8
X
(r − 1)(r + 2) == 224
1

Example 5.2.29 Using the Riemann Sum, find the sum of f (x) = x2 on [0, 1] with
n=5
0.44
Example 5.2.30 Using the Riemann Sum, find the sum of
f (x) = x1 on [1, 6) = [1, 5], n = 5
1.45
R3
Example 5.2.31 Evaluate 0 (2x + 1)dx using Riemann sums. Ans: 12
Example 5.2.32 Prove that
Z b
1
xdx = b2
0 2

using Riemann sums.

b−0 b
∆x = =
n n
b b
xi = 0 + i =i
n n
b
f (xi ) = i
n
Xb b
S̄n = i
n n
H.W-Kayondo & D.W-Ddumba, Engineering Math I- Lecture Notes 251
CHAPTER 5. THE DEFINITE INTEGRAL

b
b2 X
= i
n2 i=0

b2 h n i
= (n + 1)
n2 2

AR = lim S̄n
n→∞

b2 h n i
= lim (n + 1)
n→∞ n2 2

b2
=
2

Exercise 5.14 Prove that


Z b
cdx = c(b − a)
a

using Riemann sums.

Example 5.2.33 Find


Z 2
| x3 − x | dx
−1

Ans: 2.75

Example 5.2.34 Find the average value (mean value)

f (x) = x3 ; [0, 2]

Ans : 2

H.W-Kayondo & D.W-Ddumba, Engineering Math I- Lecture Notes 252


CHAPTER 5. THE DEFINITE INTEGRAL

5.3 Boundedness Theorem


Theorem 5.3.1 Suppose f is defined on [a, b], ∀ x ∈ [a, b] and m ≤ f (x) ≤ M then

Z b
m(b − a) ≤ f (x) dx ≤ M (b − a)
a

Proof 5.3.1

m ≤ f (x) ≤ M
Z b Rb
Z b
m dx ≤ a
f (x) dx ≤ M dx
a a

Rb
m(b − a) ≤ a
f (x) dx ≤ M (b − a)
1
Example 5.3.1 Let f (x) = x2 +3
on [1, 3]
Show that
Z 3
1 1 1
≤ dx ≤
6 1 x2 +3 2

1 1 1
Since ≤ x2 +3

2 4
Z 3 Z 3
1 R3 1 1
dx ≤ 1 x2 +3
dx ≤ dx
1 2 1 4

1 R3 1 1
(3 − 1) ≤ 1 x2 +3
dx ≤ (3 − 1)
12 4
1 R3 1 1
≤ 1 x3 +3
dx ≤
6 2

Example 5.3.2 Show that


Z 1
0≤ x2 dx ≤ 1
0

H.W-Kayondo & D.W-Ddumba, Engineering Math I- Lecture Notes 253


CHAPTER 5. THE DEFINITE INTEGRAL

5.4 Fundamental theorem of Integration


5.4.1 First Fundamental theorem of integral calculus
Theorem 5.4.1 Let f be continuous on [a, b], if F (x) is the antiderivative of f (x) on
[a, b], then

Z b
f (x) dx = [F (x)]ba = F (b) − F (a) (5.1)
a

Proof 5.4.1 Since the function f (x) is differentiable and continuous on [a, b], thus on
any interval [xi−1 , xi ] by MVT

F (xi ) − F (xi−1 )
F 0 (ti ) = , ti ∈ [xi−1 , xi ]
xi − xi−1

MVT: For the function f (x) differentiable and continuous on [a, b], there exists a c ∈ [a, b]

such that f 0 (c) = f (b)−f


b−a
(a)

Let the Riemann sum


n
X
Rn = (∆x)f (ti )
i=1

Then the definite integral (area under a curve)

Z b n
X
f (x)dx = lim (∆x)f (ti )
a n→∞
i=1

Z b n
X
f (x)dx = lim (∆x)F 0 (ti )
a n→∞
i=1

since f (x) = F 0 (x) ⇒ f (ti ) = F 0 (ti )


b n
F (xi ) − F (xi−1 )
Z X  
f (x)dx = lim (∆x)
a n→∞
i=1
xi − xi−1

n
F (xi ) − F (xi−1 )
X  
= lim (∆x)
n→∞
i=1
∆x

H.W-Kayondo & D.W-Ddumba, Engineering Math I- Lecture Notes 254


CHAPTER 5. THE DEFINITE INTEGRAL

n
X
= lim F (xi ) − F (xi−1 ) Telescoping sum
n→∞
i=1

= lim F (xn ) − F (x0 )


n→∞

= F (b) − F (a)

Example 5.4.1 Evaluate

2 2
bx3 8 −1
Z   
2
bx dx = =b − = 3b
−1 3 −1 3 3

5.4.2 Second Fundamental Theorem of the Integral Calculus


Theorem 5.4.2 If f is continuous on [a, b] then the anti-derivative F (x) of f on [a, b] is
given by

Z x
F (x) = f (t)dt ∀ x ∈ [a, b] (5.2)
a

then F 0 (x) = f (x) ∀ x ∈ [a, b] (5.3)

Note 5.4.1
From the theorem, it is not necessary to find F (x) in order to get F 0 (x) = f (x).

Example 5.4.2 Given


Z x
dt
F (x) =
1 t

find F 0 (x). Since the function 1


t
is continuous, then from Second Funder mental Theorem

1
F 0 (x) = f (x) =
x

Example 5.4.3 If
Z x √
F (x) = t sec(πt)dt
2


⇒ F 0 (x) = f (x) = x sec(πx)

H.W-Kayondo & D.W-Ddumba, Engineering Math I- Lecture Notes 255


CHAPTER 5. THE DEFINITE INTEGRAL

Note 5.4.2 In other words to use the 2nd FT, the lower limits should be a constant, and
the upper limit a variable, e.g x, u, t, · · · but not x2 , x3 + 2, t2 , ...
Example 5.4.4 Given
Z 4 √
F (x) = 1 + t2 dt
x

Z 4 √
F (x) = 1 + t2 dt
x
Z x √
F (x) = − 1 + t2 dt lower limit as a constant & the upper limit as a variable
4

⇒ F 0 (x) = f (x) = − 1 + x2
Example 5.4.5 Given an antiderivative F (x)

Z x3
dt
F (x) =
1 1 + t2
Ru
Find the function F 0 (x) = f (x) We need 1 dt
We use the substitution, let x3 = u ⇒ du dx
= 3x2
Z u
dt
F (u) = dt
1 1 + t2

1
⇒ F 0 (u) = f (u) =
1 + u2
dF dF du
F 0 (x) = =
dx du dx
 
1
= 3x2
1 + u2
 
1
= (3x2 )
1 + x6

3x2
=
1 + x6

Since we want F 0 (x) not F 0 (u) and still in x not u.

H.W-Kayondo & D.W-Ddumba, Engineering Math I- Lecture Notes 256


CHAPTER 5. THE DEFINITE INTEGRAL

Example 5.4.6 If
Z x2 +3
F (x) = √
cos2 t
x

Find F 0 (x) = f (x).

Z x3 +3 Z a Z x2 +3
2 2
F (x) = 1
cos t dt = 1
cos t dt + cos2 t dt
x2 x2 a

1
Z x2 Z x2 +3
2
F (x) = − cos t dt + cos2 t dt
a a

1
Z x2
For F1 (x) = − cos2 t dt
a

1 1
We use the substitution, let x 2 = u ⇒ du
dx
= 12 x− 2

Z u
F10 (u) =− cos2 t dt
a

⇒ F 0 (u) = f (u) = − cos2 u

dF dF du
F10 (x) = =
dx du dx
 
1 −1
F10 (x) = − cos u 2
x 2
2
 
1 1 −1
F10 (x) = − cos x 2 2 x 2
2

1 1 1
F10 (x) = − x− 2 cos2 x 2
2

Z x2 +3
For F2 (x) = cos2 t dt
a

H.W-Kayondo & D.W-Ddumba, Engineering Math I- Lecture Notes 257


CHAPTER 5. THE DEFINITE INTEGRAL

du
We use the substitution, let x2 + 3 = u ⇒ dx
= 2x

Z u
F20 (u) = cos2 t dt
a

⇒ F 0 (u) = f (u) = cos2 u

dF dF du
F20 (x) = =
dx du dx

F20 (x) = cos2 u (2x)


F20 (x) = 2x cos2 u
F20 (x) = 2x cos2 (x2 + 3)

F 0 (x) = f (x) = F10 (x) + F20 (x)

1 1 1
= − x− 2 cos2 x 2 + 2x cos2 (x2 + 3)
2

Example 5.4.7 Given that


Z x √
3
G(x) = t dt
3

Find G0 (x)

5.5 Mean Value Theorem (MVT)of Integration


Theorem 5.5.1 If a function f (x) is continuous on [a, b] then ∃ c ∈ [a, b] such that

Z b
1
f (c) = f (x) dx (5.4)
b−a a

Proof 5.5.1 R
b
Let F (x) = a f (x) dx be the antiderivative of f (x) on the interval [a, b].

H.W-Kayondo & D.W-Ddumba, Engineering Math I- Lecture Notes 258


CHAPTER 5. THE DEFINITE INTEGRAL

But from the MVT of differentiation

F (b) − F (a)
F 0 (c) =
b−a
1
F 0 (c) = F (b) − F (a)
b−a
1
F 0 (c) = [F (x)]ba
b−a
Z b
0 1
F (c) = f (x) dx
b−a a
Z b
1
f (c) = f (x) dx Since F 0 (c) = f (c)
b−a a

Thus the theorem


Z b
1
f (c) = f (x) dx
b−a a

Example 5.5.1 Find the c ∈ [−2, 2] that satisfy the MVT of integration for f (x) = x2 .
Z b
1
f (c) = f (x) dx
b−a a

Z 2
1
= x2 dx
2 −− 2 −2

 2
1 1 3
= x
4 3 −2
 
1 16 4
f (c) = =
4 3 3

but f (c) = c2 since f (x) = x2

4
c2 =
3
2
⇒ c = ±√
3

H.W-Kayondo & D.W-Ddumba, Engineering Math I- Lecture Notes 259


CHAPTER 5. THE DEFINITE INTEGRAL

Example 5.5.2 For the function f (x) = 3x + 2, find the value of c ∈ [2, 6] that satisfy
the MVT of integration.
c=4
Example 5.5.3 Suppose Temperature of a yam in an oven is given by H(t) in ◦ F for t
minutes since being put in an oven. The average value of the temperature over the first
hour is
Z 60
1
H(t) dt
60 0

1
Rb
Example 5.5.4 Geometrically, the MVT f (c) = b−a a
f (x) dx,

Z b
f (c) × b − a = f (x) dx
a
|{z} | {z }
1 Base | {z }
Height Area

Example 5.5.5 Population of Mexico is given by the model P (t) = 67.38(1.026)t where
t = 0 corresponds to 1980 and population measured in millions of people. What is the
average population going to be between 2000 and 2020?

Z 40
1
P (t) dt ≈ 147 million
40 − 20 20

R3
Example 5.5.6 Given 0
f (x) dx = 6, what is the average value of f (x) over 0 ≤ x ≤ 3?

R −1 2
Example 5.5.7 Why is −2
ex dx = −3 false?

Example 5.5.8 Find the average value of x(3x2 − 1)3 from 0 to 1.

Z 1
1
x(3x2 − 1)3 dx
1−0 0

Example 5.5.9 Find the x and y- intercepts if they exists for

Z −1
y(x) = (4t + 2) dt
x

(−1, 0) , (0, 0)

H.W-Kayondo & D.W-Ddumba, Engineering Math I- Lecture Notes 260


CHAPTER 5. THE DEFINITE INTEGRAL

Example 5.5.10 Use the Fundamental Theorem of Calculus to evaluate the tangent/
gradient of F (x), F 0 (x) given that

Z x2 p
F (x) = y y + 1 dy
3x−2

Evaluate f (5).
Example 5.5.11 Use the Fundamental Thereon of Calculus to find f (x) and f (1) given
that
Z 1
f (t) = − sin(x2 − 1)
x

5.6 Some applications of definite integrals


The definite integral of a rate gives total change
Total change in position = S(b) − S(a)
Z b
= S 0 (t) dt
a | {z }
velocity

F (b) − F (a) = Total change in F (t) between t = a & t = b


Z b
= F 0 (t) dt
a | {z }
velocity

Z b
S = v dt (5.5)
a

Example 5.6.1 An empty bucket is placed under a tap and filled with water. t minutes
after the bucket has been placed under the tap. The rate of flow of water into the bucket
is equal to 2.3 − 0.1t gallons per minute. How much water is in the bucket five minutes
after it has been placed under the tap?

Z 5 Z 5
dV
V (5) = V (5) − V (0) = dt = (2.3 − 0.1t)dt
0 dt 0

H.W-Kayondo & D.W-Ddumba, Engineering Math I- Lecture Notes 261


CHAPTER 5. THE DEFINITE INTEGRAL

Example 5.6.2 How does the result √ in the previous example change if water starts
leaking out of the bucket at a rate 0.2 t gallons per minute two minutes after the bucket
has been placed under the tap?

Example 5.6.3 A bee travels at a velocity of v(t) = 10−10e−t meters per second, where
t is measured in seconds from the moment it leaves the hive. How far does the bee travel
during the 2nd second (pun intended) of its flight?

Z 2 Z 2
D(2) − D(1) = v(t)dt = 10 − 10e−t dt
1 1

Example 5.6.4 Sap is oozing out of a shunt in a maple tree at a rate of R(t) gallons/hour
where t is measured in hours. Write down the definite integral that expresses the total
amount of Sap that is collected over a 24 hour period.

Z 24
R(t) dt
0

Note 5.6.1 In physics, work is done when a force acting upon an object causes a dis-
placement. (For example, riding a bicycle...).
If the force is not constant, we must use integration to find the work done.

Z b
W = F (x) dx (5.6)
a

where F (x) is the variable force



Example 5.6.5 Find the work done if a force F (x) = 2x − 1 is acting on an object
and moves it from x = 1 to x = 5.

5 √
Z
W = 2x − 1 dx
1

Example 5.6.6 Find the displacement of an object from t = 2 to t = 3, if the velocity


2 +1
of the object at time t is given by (t3t +3t)2

3
t2 + 1
Z
dt
2 (t3 + 3t)2

H.W-Kayondo & D.W-Ddumba, Engineering Math I- Lecture Notes 262


CHAPTER 5. THE DEFINITE INTEGRAL

Example 5.6.7 √ An experimental cholesterol treatment lowers a patients cholesterol level


at a rate of t 25 − t2 units per day, where t is the number of days since the beginning
of the treatment. Find the amount of change expected in the first three days after the
treatment. answer: Cholesterol is lowered by 61
3
units.
Example 5.6.8 Let f (x) = αx + β be a linear function on the interval [a, b], a, b ∈ <.
Show that the required number c implied in the Mean Value Theorem for integrals is
attained at the point c = a+b
c

Example 5.6.9 A fire fighting team is called in to stop a bush fire. The team realizes
that the rate at which the bush fire is spreading (in Km2 /Hr) from the time they start
work (when t = 0) is modeleled by the function

 √
 1 − t2 , 0≤t≤1
r(t) =
2(t − 1) , 1≤t≤3

Find the average rate at which the bush fire is burning the first 3 hours of their work.

R1√ R3
0
1 − t2 + 1
2(t − 1) 4.7854
= = 1.59513
3 3

(1) Let f be a continuous function on a closed interval [a, b] and Pn be a partition of


[a, b]

(i) Define a partition Pn on [a, b].


(ii) Define a Riemann sum R(f, Pn ) of f over [a, b] with respect to the partition
Pn .
(iii) Obtain the lower and upper Riemann sums of f (x) = x3 over [0, 1] for a
partition Pn of n subintervals. Hence show that

lim L(f, Pn ) = lim U (f, Pn )


n→∞ n→∞

where L(f, Pn ) and U (f, Pn ) are respectively the lower and upper Riemann
sums of f over [0, 1] with respect to the partition Pn .

Pn n2 (n+1)2
Hint: i=1 i3 = 4

(2) Compute the Riemann sum for the function f (x) = 2x − x2 over the interval [0, 4]
for

(i) x?i = xi the upper limit, using the circumscribed rectangles with n = 4.

H.W-Kayondo & D.W-Ddumba, Engineering Math I- Lecture Notes 263


CHAPTER 5. THE DEFINITE INTEGRAL

(ii) x?i = xi−1 the lower limit, using the inscribed rectangles with n = 4.
(iii) the partition [0, 1, 2.5, 3, 4] choosing

x?1 = 0.5, x?2 = 1.5, x?3 = 2.8, x?4 = 3.2

(3) State the Fundermental Theorem(s) of Calculus. Find the derivative of F if

Z sin x  23
F (x) = 1 − t2 dt
x

(4) Write the following expression as a definite integral on [0, 1]

13 + 23 + 33 + · · · + n3
lim
n→∞ n4

(5) State the Mean Value Theorem for integral calculus. Find all numbers c for which
the function
f (x) = x2 − 4x + 5
defined on the closed interval [0, 3], satisfies the Mean Value Theorem.

(6) Using the function f (x) = 2x − 4 on [0, 3], demonstrate that the Riemann sum is
R3
not necessarily equal to the definite integral 0 2x − 4 dx.

H.W-Kayondo & D.W-Ddumba, Engineering Math I- Lecture Notes 264


Chapter 6

Antiderivatives

So far, our method for finding anti-derivatives has been to guess, check (by taking the
derivative of our guess) and modify until the answer is correct. It would be better to
have a more systematic approach. Unfortunately, while differentiation requires only a
few rules to take the derivatives of most familiar functions, anti-differentiation is a much
more complicated game, involving many more rules (so many that at MIT they hold
2
an “Integration Bee”). Moreover, there are many functions, such as ex , which have no
simple anti-derivative.

6.1 The indefinite integral


We recall some facts about integration from first semester calculus.

Definition 6.1.1 A function y = F (x) is called an antiderivative of another function


y = f (x) if F 0 (x) = f (x) for all x.

Definition 6.1.2 Integration is the inverse of differentiation. It is the anti-derivative.

Example 6.1.1 F1 (x) = x2 is an antiderivative of f (x) = 2x.


F2 (x) = x2 + 2004 is also an antiderivative of f (x) = 2x.
G(t) = 21 sin(2t + 1) is an antiderivative of g(t) = cos(2t + 1).

The Fundamental Theorem of Calculus states that if a function y = f (x) is continuous


on an interval a ≤ x ≤ b, then there always exists an antiderivative F (x) of f , and one
has
Z b
f (x) dx = F (b) − F (a). (6.1)
a

The best way of computing an integral is often to find an antiderivative F of the given
function f , and then to use the Fundamental Theorem (6.1). How you go about finding
an antiderivative F for some given function f is the subject of this chapter.

265
CHAPTER 6. ANTIDERIVATIVES

The following notation is commonly used for antiderivates:


Z
F (x) = f (x) dx. (6.2)

The integral which appears here does not have the integration bounds a and b. It is
called an indefinite integral, as opposed to the integral in (6.1) which is called a definite
integral. It’s important to distinguish between the two kinds of integrals. Here is a list of
differences:

Indefinite integral Definite integral

R Rb
f (x) dx is a function of x. a
f (x) dx is a number.
R Rb
By definition f (x) dx is any a f (x) dx was defined in terms
function of x whose derivative of Riemann sums and can be
is f (x). interpreted as “area under the
graph of y = f (x)”, at least
when f (x) > 0.
x is not a Rdummy variable, for x is a dummyR1 variable, for
2
Rexample, 2 2x dx = x + C and example, 0 2x dx = 1, and
R1 R1
2t dt = t + C are functions of 2t dt = 1, so 2x dx =
different variables, so they are R01 0

0
2t dt.
not equal.

6.2 You can always check the answer


Suppose you want to find an antiderivative of a given function f (x) and after a long and
messy computation which you don’t really trust you get an “answer”, F (x). You can
then throw away the dubious computation and differentiate the F (x) you had found. If
F 0 (x) turns out to be equal to f (x), then your F (x) is indeed an antiderivative and your
computation isn’t important anymore.
R
Example 6.2.1 Suppose we want to find ln x dx. My cousin Jesse says it might be
F (x) = x ln x − x. Let’s see if he’s right:

d 1
(x ln x − x) = x · + 1 · ln x − 1 = ln x.
x x
R
Who knows how Jesse thought of this1 , but he’s right! We now know that ln x dx =
x ln x − x + C.
1
He integrated by parts.

H.W-Kayondo & D.W-Ddumba, Engineering Math I- Lecture Notes 266


CHAPTER 6. ANTIDERIVATIVES

6.3 About “+C”


Let f (x) be a function defined on some interval a ≤ x ≤ b. If F (x) is an antiderivative of
f (x) on this interval, then for any constant C the function F̃ (x) = F (x) + C will also be
an antiderivative of f (x). So one given function f (x) has many different antiderivatives,
obtained by adding different constants to one given antiderivative.
Example 6.3.1 For the functions

F1 (x) = x2
F2 (x) = x2 − 16
F3 (x) = x2 + 19

1
F4 (x) = x2 +
2
.. ..
. = .

are all anti-derivatives of f (x) = 2x


R
It follows that there is some
R ambiguity in the notation f (x) dx. Two functions F1 (x)
and F2 (x) can both equal f (x) dx without equaling each other. When this happens,
they (F1 and F2 ) differ by a constant. This can sometimes lead to confusing situations,
e.g. you can check that

Z
2 sin x cos x dx = sin2 x

Z
2 sin x cos x dx = − cos2 x

are both correct. (Just differentiate the two functions sin2 x and − cos2 x!) These two
answers look different until you realize that because of the trig identity sin2 x + cos2 x = 1
they really only differ by a constant: sin2 x = − cos2 x + 1.

To avoid this kind of confusion we will from


now on never forget to include the “arbi-
trary constant +C” in our answer when we
compute an antiderivative.

Theorem 6.3.1 If F1 (x) and F2 (x) are antiderivatives of the same function f (x) on
some interval a ≤ x ≤ b, then there is a constant C such that F1 (x) = F2 (x) + C.

H.W-Kayondo & D.W-Ddumba, Engineering Math I- Lecture Notes 267


CHAPTER 6. ANTIDERIVATIVES

Proof 6.3.1 Consider the difference G(x) = F1 (x) − F2 (x). Then G0 (x) = F10 (x) −
F20 (x) = f (x) − f (x) = 0, so that G(x) must be constant. Hence F1 (x) − F2 (x) = C for
some constant.

Theorem 6.3.2 The antiderivative of f (x) = 0 in an interval I are the constant func-
tions, then F (x) = C, x  I

Proof 6.3.2 From the Mean Value theorem,

F(b) − F(a)
F 0 (c) =
b−a

Since f (x) = F 0 (c) = 0, then

F (b) − F (a) = 0
F (b) = F (a) = c

a constant ∀a, b, c ∈ I

Another way of stating Theorem 6.3.1 is by

Theorem 6.3.3 If F (x) is an anti derivative of f (x) on xεI then F (x) + C is also an
anti-derivative of f (x) on I.

Proof 6.3.3 Suppose F (x) and G(x) are both ant-derivative off (x) on I and define h(x)
by h(x) = G(x) − F (x)

h0 (x) = (G(x) − f (x))0


= G0 (x) − F 0 (x)
= f (x) − f (x) Since both are anti-derivatives.
= 0
0
⇒ h (x) = 0
⇒ h(x) = C Theorem 6.3.2
⇒ G(x) = F (x) + C

H.W-Kayondo & D.W-Ddumba, Engineering Math I- Lecture Notes 268


Chapter 7

Techniques of Integration

7.1 Standard Integrals


Here is a list of the standard derivatives and hence the standard integrals everyone should
know.
Z
f (x) dx = F (x) + C

xn+1
Z
n
x dx = + C for all n 6= −1
n+1
Z
1
dx = ln |x| + C
x
Z
sin x dx = − cos x + C

Z
cos x dx = sin x + C

Z
tan x dx = − ln cos x + C

Z
1
dx = arctan x + C
1 + x2
Z
1 π
√ dx = arcsin x + C = − arccos x + C
1−x 2 2
Z
dx 1 1 + sin x π π
= ln + C for − < x < .
cos x 2 1 − sin x 2 2

269
CHAPTER 7. TECHNIQUES OF INTEGRATION

All of these integrals are familiar from High school calculus (like Pure Math I), except
for the last one. You can check the last one by differentiation (using ln ab = ln a − ln b
simplifies things a bit).

Example 7.1.1 Evaluate the following indefinite integrals:


R
(i) (2x3 − 5x + 3)dx.
1 4 5 2
Ans : x − x + 3x + C
2 2
R
(ii) e3x dx.
1 3x
Ans : e +C
3

[ x5 + 3
R
(iii) x2
]dx.
3
Ans : 5 ln x − +C
x
R
(iv) x(x − 2)2 dx.

Exercise 7.1 Evaluate the following integrals. They shouldn’t require fancy substitu-
tions, integration by parts, or partial fraction expansions.
R0
1. (6x5 −2x−4 −7x+3/x−5+4ex +7x ) dx9. −3 (5y 4 − 6y 2 + 14) dy
R

R x a 
2. a
+ x + xa + ax + ax dx R1
10. 0 (y 9 − 2y 5 + 3y) dy
R √ √3 7

x R4√
3. x − x4 + √ 3 2 − 6e + 1 dx
x 11. 0 x dx
R
4. 2x dx 12.
R1
x3/7 dx
0

R4
5. −2
(3x − 5) dx 13.
R3 1
− 1

dt
1 t2 t4

R2 R2
6. x−2 dx t6 −t2
1 14. 1 t4
dt

R1 R2 2 +1
x√
7. 0
(1 − 2x − 3x2 ) dx 15. 1 x
dx

R2 R2
8. 1
(5x2 − 4x + 3) dx 16. 0
(x3 − 1)2 dx

H.W-Kayondo & D.W-Ddumba, Engineering Math I- Lecture Notes 270


CHAPTER 7. TECHNIQUES OF INTEGRATION

R1 √ √ R4 √ √
17. 0
u( u + 3 u) du 22. 1
( t − 2/ t) dt

R1 3
R 8 √ 1

18. −1 t4
dt 23. 1
3
r+ √
3 r
dr

R2 R1
19. 1
(x + 1/x)2 dx 24. u
0 1+u
du

R3√ R0 2x−3
20. 3
x5 + 2 dx 25. −1 x+4
dx

R −1 R −21−σ
21. 1
(x − 1)(3x + 2) dx 26. −5 1+σ 2

Exercise 7.2
R0 R π/3
1. −1 (x + 1)3 dx 9. π/4
sin t dt

R −2x4 −1 R π/2
2. −5 x2 +1
dx 10. (cos θ + 2 sin θ) dθ
0

3.
R2
eat dt R √3 6
0 11. 1 1+x2
dx
R2
4. 2t dt R 0.5
0 12. √ dx
0 1−x2
Re x2 +x+1
5. 1 x
dx R8
13. 4
(1/x) dx
R 9 √ 2
6. x+ √1 dx R ln 6
4 x 14. ln 3
8ex dx

R 1 √
4
√ 
7. x 5 + 5 x4 dx R9
0 15. 8
2t dt

R8 x−1
R −e 3
8. 1

3 2
x
dx 16. −e2 x
dx

Exercise 7.3 Compute the following integrals


R3 R2
1. −2 |x2 − 1| dx 4. 0 (x2 − |x − 1|) dx

R2 R 2π
|x − x2 | dx

2. −1 5. 0
sin θ dθ

R2
3. −1
(x − 2|x|) dx
H.W-Kayondo & D.W-Ddumba, Engineering Math I- Lecture Notes 271
CHAPTER 7. TECHNIQUES OF INTEGRATION

R2
Exercise 7.4 0
f (x) dx where

(
x4 , if 0 ≤ x < 1,
f (x) =
x5 , if 1 ≤ x ≤ 2.


Exercise 7.5 f (x) dx where
−π
(
x, if −π ≤ x ≤ 0 ,
f (x) =
sin x, if 0 < x ≤ π.

Example 7.1.2

Z Z
2
(x − 1) dx = x2 + 2x + 1 dx

x3
= − x2 + x + c
3
R
Exercise 7.6 3 cos 3x dx
R
Exercise 7.7 sec2 4x dx

H.W-Kayondo & D.W-Ddumba, Engineering Math I- Lecture Notes 272


CHAPTER 7. TECHNIQUES OF INTEGRATION

7.2 Function and its derivative


Given an anti derivative
Z
f 0 (x)(f (x))n dx

Consider
d
(f (x))n+1 = (n + 1)f 0 (x) ((f (x))n
dx

Integrating both sides

Z Z
d n+1
⇒ ((f (x)) dx = (n + 1) f 0 (x) (f (x))n dx
dx
Z
1
⇒ f 0 (x) ((f (x))n = (f (x))n+1
n+1

d
R
Example 7.2.1 6x(3x2 + 2)4 dx since dx
(3x2 + 2) = 6x then technique apply

d
consider (3x2 + 2)5 = 5(3x2 + 2)4 6x
dx
1 d
⇒ (3x2 + 2)5 = 6x(3x2 + 2)4
5 dx
Z Z
1 d 2 5
⇒ (3x + 2) dx = 6x(3x2 + 2)4 dx
5 dx
Z
1 2 5
⇒ (3x + 2) + C = 6x(3x2 + 2)4 dx
5

R 1 1 3
Example 7.2.2 x(x2 + 1) 2 dx 3
(x2 + 1) 2 + C
1
R
Example 7.2.3 (3x − 1)5 dx 18
(3x − 1)6 + C
x
x(x2 + 1)−2 dx
R R
Example 7.2.4 (x2 +1)2
dx =

H.W-Kayondo & D.W-Ddumba, Engineering Math I- Lecture Notes 273


CHAPTER 7. TECHNIQUES OF INTEGRATION

R
Example 7.2.5 cos x sin xdx

d
consider sin2 x = 2 sin x cos x
dx
1 d
⇒ sin2 x = sin x cos x
2 dx
Z Z
1 d 2
⇒ sin x dx = sin x cos x dx
2 dx
Z
1 2
⇒ sin x cos x dx = sin x + C
2

− 61 cos6 x + C
R
Exercise 7.8 cos5 x sin x

− 13 cos3 x + C
R
Exercise 7.9 cos2 x sin x

sin4 x cos x 1
sin5 x + C
R
Exercise 7.10 5
1
sin2 (3x) + C
R
Example 7.2.6 sin 3x cos 3x 6
R R
Example 7.2.7 tan x sec3 x = (tan x sec x) sec2 xdx

d
consider sec3 x = 3 sec2 x tan x sec x = 3 sec3 x tan x
dx
1 d
⇒ sec3 x = sec3 x tan x
3 dx
Z Z
1 d 3
⇒ sec x dx = sec3 x tan x dx
3 dx
Z
1
⇒ sec3 x tan x = sec3 x + C
3

− 91 cos3 (3x) + C
R
Exercise 7.11 sin 3x cos2 3x dx
1
R
Example 7.2.8 sec2 x tan2 x dx 3
tan3 x + C

sec4 x(sec x tan x) dx = 15 sec5 x + C


R R
Exercise 7.12 sec5 x tan x dx =

1 3
2
R
Exercise 7.13 cos x sin 2 x dx 3
sin 2 x + C

H.W-Kayondo & D.W-Ddumba, Engineering Math I- Lecture Notes 274


CHAPTER 7. TECHNIQUES OF INTEGRATION

R
Example 7.2.9 csc3 x cot x dx

d
consider csc3 x = 3 csc2 x(csc x cot x)
dx
Z
1
⇒ csc3 x cot x dx = csc3 x + C
3

sin5 x cos2 xdx


R
Example 7.2.10

Z
sin x sin4 x cos2 x dx

Z
sin x(1 − cos2 x)2 cos2 x dx

Z Z Z
2 4
sin x cos x − 2 sin x cos x + sin x cos6 x

sec x − 31 sec3 x + C
R
Exercise 7.14 tan3 x sec xdx

H.W-Kayondo & D.W-Ddumba, Engineering Math I- Lecture Notes 275


CHAPTER 7. TECHNIQUES OF INTEGRATION

7.3 Trigonometric functions


7.3.1 Even powers of trigonometric functions
Express the even powers in double angles as given in section 7.3.4

sin2 xdx
R
Example 7.3.1
Since sin2 x = 21 (1 − cos 2x)

Z Z  
2 1 1 1
sin x dx = 1 − cos 2x dx = x − sin 2x + C
2 2 2

Example 7.3.2

Z Z  
2 1 1 1
cos x dx = (1 + cos 2x) dx = x + sin 2x + C
2 2 2

Example 7.3.3

Z Z
4
sin x dx = (sin2 x)2 dx

 2
1
= (1 − cos 2x) dx
2
Z
1
1 − 2 cos 2x + cos2 2x dx

=
4
Z  
1 1
= 1 − 2 cos 2x + (1 − cos 4x) dx
4 2
   
1 1 1
= x − sin 2x + x − sin x + C
4 2 4

R
Exercise 7.15 cos4 x dx

sin6 x dx
R
Exercise 7.16
R
Exercise 7.17 cos6 x dx

H.W-Kayondo & D.W-Ddumba, Engineering Math I- Lecture Notes 276


CHAPTER 7. TECHNIQUES OF INTEGRATION

7.3.2 Odd powers of trigonometric functions


Express the power into both the odd(1) and even and apply section 7.2.

Example 7.3.4

Z Z
5
cos x dx = cos x cos4 x dx

Z
= cos x(cos2 x)2 dx

Z
= cos x(1 − sin2 x)2 dx : apply section 7.2

Example 7.3.5

Z Z Z
3 2
sin x dx = (1 − cos x) sin x dx = − (1 − cos2 x)(− sin x) dx

7.3.3 Factor formula


Example 7.3.6

Z Z
1
sin 2x cos 5x dx = (sin 7x + sin 3x) dx
2
 
1 1 1
= − cos 7x − cos 3x + C
2 7 3

7.3.4 The double angle trick


If an integral contains sin2 x or cos2 x, then you can remove the squares by using the
double angle formulas from trigonometry.
Recall that

cos2 α − sin2 α = cos 2α and cos2 α + sin2 α = 1,

Adding these two equations gives

1
cos2 α = (cos 2α + 1)
2

H.W-Kayondo & D.W-Ddumba, Engineering Math I- Lecture Notes 277


CHAPTER 7. TECHNIQUES OF INTEGRATION

while subtracting them gives

1
sin2 α = (1 − cos 2α) .
2

Example 7.3.7 The following integral shows up in many contexts, so it is worth know-
ing:

Z Z
2 1
cos x dx = (1 + cos 2x) dx
2
 
1 1
= x + sin 2x + C
2 2

x 1
= + sin 2x + C.
2 4

Since sin 2x = 2 sin x cos x this result can also be written as


Z
x 1
cos2 x dx = + sin x cos x + C.
2 2

Example 7.3.8

x  12
Z Z 
1
(1 + cos x) dx =
2 2 cos2 dx
2
√ Z x
= 2 cos dx
2
√ 1
= 2 2 sin x + C
2

Example 7.3.9

Z Z
2
sin 2x sin x dx = 2 sin x cos x sin2 x dx

Z
= 2 cos x sin3 x dx : apply section 7.2

H.W-Kayondo & D.W-Ddumba, Engineering Math I- Lecture Notes 278


CHAPTER 7. TECHNIQUES OF INTEGRATION

7.4 Method of substitution


The chain rule says that
dF (G(x))
= F 0 (G(x)) · G0 (x),
dx

so that
Z
F 0 (G(x)) · G0 (x) dx = F (G(x)) + C.

Example 7.4.1 Consider the function f (x) = 2x sin(x2 + 3). It does not appear in the
list of standard integrals we know by heart. But we do notice1 that 2x = xd (x2 + 3). So
let’s call G(x) = x2 + 3, and F (u) = − cos u, then
F (G(x)) = − cos(x2 + 3)
and
d
x = sin(x2 + 3) · |{z}
2x = f (x),
F (G(x)) | {z } 0
F 0 (G(x)) G (x)

so that
Z
2x sin(x2 + 3) dx = − cos(x2 + 3) + C.

The most transparent way of computing an integral by substitution is by introducing


new variables. Thus to do the integral
Z
f (G(x))G0 (x) dx

where f (u) = F 0 (u), we introduce the substitution u = G(x), and agree to write du =
dG(x) = G0 (x) dx. Then we get
Z Z
0
f (G(x))G (x) dx = f (u) du = F (u) + C.

At the end of the integration we must remember that u really stands for G(x), so that
Z
f (G(x))G0 (x) dx = F (u) + C = F (G(x)) + C.

1
You will start noticing things like this after doing several examples.

H.W-Kayondo & D.W-Ddumba, Engineering Math I- Lecture Notes 279


CHAPTER 7. TECHNIQUES OF INTEGRATION

For definite integrals this implies

Z b
f (G(x))G0 (x) dx = F (G(b)) − F (G(a)).
a

which you can also write as

Z b Z G(b)
0
f (G(x))G (x) dx = f (u) du. (7.1)
a G(a)

Example 7.4.2 [Substitution in a definite integral. ] As an example we compute

Z 1
x
dx,
0 1 + x2

using the substitution u = G(x) = 1 + x2 . Since du = 2x dx, the associated indefinite


integral is
Z Z
1 1 1
2
x dx = du.
1 + x
| {z } 1
|{z} 2 u
2
du
1
u

To find the definite integral you must compute the new integration bounds G(0) and G(1)
(see equation (7.1).) If x runs between x = 0 and x = 1, then u = G(x) = 1 + x2 runs
between u = 1 + 02 = 1 and u = 1 + 12 = 2, so the definite integral we must compute is

Z 1 Z 2
x 1 1
2
dx = du,
0 1+x 2 1 u

which is in our list of memorable integrals. So we find

Z 1 Z 2
x 1 1 1 2 1
dx = du = ln u 1 = ln 2.
0 1 + x2 2 1 u 2 2

R
Example 7.4.3 Evaluate 20(2x − 3)10 dx.

10
(2x − 3)11 + C
11

H.W-Kayondo & D.W-Ddumba, Engineering Math I- Lecture Notes 280


CHAPTER 7. TECHNIQUES OF INTEGRATION

R R
Example 7.4.4 Evaluate 10x(x2 − 1)5 dx and x(x2 − 1)5 dx
Evaluate the following integrals:
Example 7.4.5 (x4 + 2x − 1)−5 (2x3 + 1) dx
R

R 3 −1
Example 7.4.6 ex x2 dx
1
R
Example 7.4.7 x2 −4
x dx

1
R
Example 7.4.8 x ln x
dx

x2 +2
R
Example 7.4.9 x+1
dx

R √
Example 7.4.10 x2 x3 + 5 dx
ex
R
Example 7.4.11 ex +1
dx

2
e−x dx
R
Example 7.4.12

R 1
Example 7.4.13 x(3x − 1) 2 dx

1
Let u = (3x − 1) 2 ⇒ u2 = 3x − 1

1 2 
⇒ x= u +1
3
dx 1 2
= (2u) = u
du 3 3
Z Z
1 1 2  2
x(3x − 1) 2 dx = u + 1 u u du
3 3
Z
2
= u2 (u2 + 1) du
9

2 u5 u3
 
= +
9 5 3
5 3
!
2 (3x − 1) 2 (3x − 1) 2
= +
9 5 3

2 2
= (3x − 1)5/2 + (3x − 1)3/2 + C
45 27

H.W-Kayondo & D.W-Ddumba, Engineering Math I- Lecture Notes 281


CHAPTER 7. TECHNIQUES OF INTEGRATION

Use a substitution to evaluate the following integrals.


R 2 u du
Exercise 7.18 1 1+u 2

R2 x dx
Exercise 7.19 1 1+x2

R π/3
Exercise 7.20 π/4
sin2 θ cos θ dθ

R3 1
Exercise 7.21 2 r ln r
dr
sin 2x
R
Exercise 7.22 1+cos2 x
dx

sin 2x
R
Exercise 7.23 1+sin x
dx

R1 √
Exercise 7.24 0
z 1 − z 2 dz

R2 ln 2x
Exercise 7.25 1 x
dx

R √2
Exercise 7.26 ξ=0
ξ(1 + 2ξ 2 )10 dξ

R3
Exercise 7.27 2
sin ρ cos 2ρ)4 dρ

2
αe−α dα
R
Exercise 7.28
R
Note 7.4.1 Given the integral [f (x)]p dx

(i) if p is a fraction, let u = [f (x)]p

(ii) if p is a whole number, let u = [f (x)]

R 1
Example 7.4.14 x(2x2 + 1) 2 dx

1 3
(2x2 + 1) 2
6

R √
1+ x √
Exercise 7.29 √ dx
x
; let u = x

H.W-Kayondo & D.W-Ddumba, Engineering Math I- Lecture Notes 282


CHAPTER 7. TECHNIQUES OF INTEGRATION

R
Example 7.4.15 Have a look for an integer power, x(2x − 1)6 dx ; u = (2x − 1)

1
Let u = (2x − 1) ⇒ x = (u + 1)
2
dx 1
⇒ = u
du 2
Z Z
1 1
x(2x − 1)6 dx = (u + 1) u6 du
2 2
Z
1
= u6 (u + 1) du
4
Z
1
= u7 + u6 du
4

1 u8 u6
 
= + +C
4 8 6

1 (2x − 1)8 (2x − 1)6


 
= + +C
4 8 6
R x 1
Exercise 7.30 1 dx ; u = (x − 2) 2
(x−2) 2

R
Exercise 7.31 (x + 2)(x − 1)4 dx ; u = (x − 1)
R x(x−4)
Example 7.4.16 (x−2)2
u = (x − 2)

4
x+
(x − 2)

1
√x−1
R
Exercise 7.32 2x+3
; u = (2x + 3) 2

Example 7.4.17 (with limits)


Z 3 Z 1
1 1 1
x(x − 2) dx = 2 (u + 2)u 2 du , since u = (x − 2) 2
2 0

Z 1 Z 0
4
x(x − 1) dx = u5 + u4 du , since u = (x − 1)
0 −1

H.W-Kayondo & D.W-Ddumba, Engineering Math I- Lecture Notes 283


CHAPTER 7. TECHNIQUES OF INTEGRATION

7.5 Logarithmic integration


Given an integral
f 0 (x)
Z
dx
f n (x)

We consider
d
ln(f n (x))
dx

1
R
Example 7.5.1 x
dx

d 1
consider ln x =
dx x
Z Z
d 1
⇒ ln x dx = dx
dx x
Z
1
⇒ = ln x + C
x

sin x
R R
Example 7.5.2 tan xdx = cos x
dx

d − sin x
consider ln cos x =
dx cos x
d sin x
⇒ − ln cos x =
dx cos x
Z Z
d sin x
⇒ − ln cos x dx = dx
dx cos x
Z
sin x
⇒ dx = −ln cos x + C
cos x

1
R
Example 7.5.3 1−x
dx = − ln(1 − x) + C

x2
dx = 91 ln(3x3 + 2) + C
R
Example 7.5.4 3x3 +2

H.W-Kayondo & D.W-Ddumba, Engineering Math I- Lecture Notes 284


CHAPTER 7. TECHNIQUES OF INTEGRATION

1
R
Example 7.5.5 3x+2
dx

d 3
consider ln(3x + 2) =
dx (3x + 2)

1 d 1
ln(3x + 2) =
3 dx (3x + 2)
Z Z
1 d 1
⇒ ln(3x + 2) = dx
3 dx (3x + 2)
Z
1 1
⇒ ln(3x + 2) + C = dx
3 (3x + 2)

H.W-Kayondo & D.W-Ddumba, Engineering Math I- Lecture Notes 285


CHAPTER 7. TECHNIQUES OF INTEGRATION

7.6 Integration by Parts


The product rule states

d
(f (x)g(x)) = f (x)g 0 (x) + g(x)f 0 (x)
x

and therefore, after rearranging terms,

d
f (x)g 0 (x) = (f (x)g(x)) − g(x)f 0 (x)
x

This implies the formula for integration by parts

Z Z Z
0 d
f (x)g (x) dx = (f (x)g(x)) dx − g(x)f 0 (x) dx
x
Z Z
f (x)g (x) dx = f (x)g(x) − g(x)f 0 (x) dx
0

Example 7.6.1 (Example – Integrating by parts once.)

Z Z
x x
x |{z}
|{z} e dx = |{z} e −
x |{z} |{z} 1 dx = xex − ex + C.
ex |{z}
f (x) g 0 (x) f (x) g(x) g(x) f 0 (x)

Observe that in this example ex was easy to integrate, while the factor x becomes an easier
function when you differentiate it. This is the usual state of affairs when integration by
parts works: differentiating one of the factors (f (x)) should simplify the integral, while
integrating the other (g 0 (x)) should not complicate things (too much).
d
Example 7.6.2 Another example: sin x = dx
(− cos x) so

Z Z
x sin x dx = x(− cos x) − (− cos x) · 1 dx

= −x cos x + sin x + C

Example 7.6.3 [Example – Repeated Integration by Parts.] Sometimes one integration


d 1 2x
by parts is not enough: since e2x = dx ( 2 e ) one has Be careful
with all the
minus signs
that appear
H.W-Kayondo & D.W-Ddumba, Engineering Math I- Lecture Notes 286 when you
integrate by
parts.
CHAPTER 7. TECHNIQUES OF INTEGRATION

2x
e2x
Z Z
2 2x 2e
x |{z}
e dx = x − 2x dx
|{z}
0
2 2
f (x) g (x)

2x
e2x e2x
 Z 
2e
= x − 2x − 2 dx
2 4 4
2x
e2x e2x
 
2e
= x − 2x − 2+C
2 4 8

1 2 2x 1 2x 1 2x
= x e − xe + e − C
2 2 4

df
Another way of differetiating by parts is to have a table, till f 0 (x) = dx
=0

df
g 0 (x)dx
R
dx

+ x2 e2x
1 2x
- 2x 2
e
1 2x
+ 2 4
e
1 2x
- 0 8
e

x2 2x 2x 2x 2 2x
Z
x2 e2x dx = e − e + e +C
2 4 8

x2 2x x 2x 1 2x
= e − e + e +C
2 2 4

The same procedure will work whenever you have to integrate


Z
P (x)eax dx

where P (x) is a polynomial, and a is a constant. Each time you integrate by parts, you
get this

eax eax 0
Z Z
ax
P (x)e dx = P (x) − P (x) dx
a a
Z
1 1
= ax
P (x)e − P 0 (x)eax dx.
a a

H.W-Kayondo & D.W-Ddumba, Engineering Math I- Lecture Notes 287


CHAPTER 7. TECHNIQUES OF INTEGRATION

You have replaced the integral P (x)eax dx with the integral P 0 (x)eax dx. This is the
R R

same kind of integral, but it is a little easier since the degree of the derivative P 0 (x) is
less than the degree of P (x).

Example 7.6.4 [Example – My cousin Jesse’s computation.] Sometimes the factor g 0 (x)
is “invisible”. Here is how you can get the antiderivative of ln x by integrating by parts:

Z Z
ln x dx = ln x · |{z}
|{z} 1 dx
f (x) g 0 (x)

Z
1
= ln x · x − · x dx
x
Z
= x ln x − 1 dx

= x ln x − x + C.
R
You can do P (x) ln x dx in the same way if P (x) is a polynomial.

Summary 7.6.1 The technique is extensively used to compute

Z Z Z Z Z Z Z Z Z
−1 −1 −1
tan x, sin x, cos x, 2
log x , 3
ln x +2, 2
x sin x, x cos x, x
xe , x tan−1 x

where

f (x) = x, x2 , tan−1 x, sin−1 x, ln that can be differentiated

g 0 (x) = e2x , ex , · · ·

2 loge x2 ln x2
where logxp = loge p
= ln p
all logs should be first put into base e the natural log.

What you can differentiate is the f (x), and can integrate is the g 0 (x).

Example 7.6.5
Z
x cos x dx = x sin x + cos x + C

Example 7.6.6
Z
x 1
x cos 2x dx = sin 2x + cos 2x + C
2 4

H.W-Kayondo & D.W-Ddumba, Engineering Math I- Lecture Notes 288


CHAPTER 7. TECHNIQUES OF INTEGRATION

Example 7.6.7
x2 x2
Z Z
x loge x dx = x ln x = ln x − +C
2 4

Example 7.6.8

Z Z
−1 −1
tan x dx = |tan{z x} · |{z}
1 dx
f (x) g 0 (x)

Z
−1 1
| {z x}[|{z}
= tan x ]− x dx
|{z} (1 + x2 )
f (x) g(x) g(x) | {z }
f 0 (x)

Z
−1 x
= x tan x− dx
(1 + x2 )

1
= x tan−1 x − ln(1 + x2 ) + C
2

But let us review how we differentiate tan−1 x

dy
let y = tan−1 x , we need
dx
⇒ tan y = x

dy
(diff w.r.t.x), sec2 y = 1
dx
dy 1 1 1
= 2
= 2
=
dx sec y 1 + tan y 1 + x2

Example 7.6.9

Z
x2 sin x dx = −x2 cos x + 2x sin x + 2 cos x + C

Example 7.6.10
Z
x2 ex dx = ex (x2 − 2x + 2) + C

H.W-Kayondo & D.W-Ddumba, Engineering Math I- Lecture Notes 289


CHAPTER 7. TECHNIQUES OF INTEGRATION

Example 7.6.11

Z Z
x x x
e sin x dx = −e cos x + e sin x − ex sin x dx

Z
⇒ 2 ex sin x dx = −ex cos x + ex sin x collecting like terms

Z
1
⇒ ex sin x dx = [−ex cos x + ex sin x + C]
2

Example 7.6.12
Z p
sin−1 x dx = x sin−1 x + (1 − x2 ) + C

Example 7.6.13

Z
1p
sin−1 3x dx = x sin−1 3x + (1 − 9x2 ) + C
3

R
Example 7.6.14 xex dx

df
g 0 (x)dx
R
dx
+ x ex
- 1 ex
+ 0 ex

Using cross multiplication with a minus:

xex − ex + C

Example 7.6.15

Z
1 1 1
x tan−1 xdx = x2 tan−1 x + tan−1 x − x + C
2 2 2

H.W-Kayondo & D.W-Ddumba, Engineering Math I- Lecture Notes 290


CHAPTER 7. TECHNIQUES OF INTEGRATION

Example 7.6.16 Find


Z
x ln xdx

x ln x dx and g 0 (x) = x
R
Let I =
Z
⇒ g(x) = xdx = x2

1
f (x) = ln x ⇒ f 0 (x) =
x
Z Z
0
But f (x)g (x)dx = f (x)g(x) − g(x)f 0 (x)dx

Z
2 1
therefore, I = x ln x − x2 · dx
x
Z
2
= x ln x − xdx

1
= x2 ln x − x2 + C
2

Note 7.6.1 Tables are not applicable if f (x) is not a polynomial, as in example 7.6.16
where f (x) = ln x not a polynomial, we dont apply tables.
Example 7.6.17 Compute,
Z
ex cos xdx

Let
Z
I = ex cos xdx and

f (x) = cos x ⇒ f 0 (x) = − sin x


g 0 (x) = ex ⇒ g(x) = ex .
Z Z
But g (x)f (x)dx = g(x)f (x) − g(x)f 0 (x)dx
0

Z
x
Therefore, I = e cos x + ex sin xdx

H.W-Kayondo & D.W-Ddumba, Engineering Math I- Lecture Notes 291


CHAPTER 7. TECHNIQUES OF INTEGRATION

7.7 Reduction Formulas


Consider the integral
Z
In = xn eax dx.

Integration by parts gives you

Z
n 1 ax 1
In = x e nxn−1 eax dx

a a
Z
1 n ax n
= x e − xn−1 eax dx.
a a

We haven’t computed the integral, and in fact the integral that we still have to do is of
the same kind as the one we started with (integral of xn−1 eax instead of xn eax ). What we
have derived is the following reduction formula

1 n
In = xn eax − In−1
a a

which holds for all n.


For n = 0 the reduction formula says

Z
1 1
I0 = eax , i.e. eax dx = eax + C.
a a

When n 6= 0 the reduction formula tells us that we have to compute In−1 if we want to
find In . The point of a reduction formula is that the same formula also applies to In−1 ,
and In−2 , etc., so that after repeated application of the formula we end up with I0 , i.e.,
an integral we know.
R
Example 7.7.1 To compute x3 e5x dx we use the reduction formula three times:

1 3 5x 3
I3 = x e − I2
5 5
 
1 3 5x 3 1 2 5x 2
= xe − x e − I1
5 5 5 5
  
1 3 5x 3 1 2 5x 2 1 5x 1
= xe − xe − xe − I0
5 5 5 5 5 5

H.W-Kayondo & D.W-Ddumba, Engineering Math I- Lecture Notes 292


CHAPTER 7. TECHNIQUES OF INTEGRATION

Insert the known integral I0 = 51 e5x + C and simplify the other terms and you get

Z
1 3 6 6
x3 e5x dx = x3 e5x − 2 x2 e5x + 3 xe5x − 4 e5x + C.
5 5 5 5

Example 7.7.2 [Reduction formula requiring two partial integrations.] Consider

Z
Sn = xn sin x dx.

Then for n ≥ 2 one has

Z
n
Sn = −x cos x + n xn−1 cos x dx

Z
n n−1
= −x cos x + nx sin x − n(n − 1) xn−2 sin x dx.

Thus we find the reduction formula

Sn = −xn cos x + nxn−1 sin x − n(n − 1)Sn−2 .

Each time you use this reduction, the exponent n drops by 2, so in the end you get either
S1 or S0 , depending on whether you started with an odd or even n.

Example 7.7.3 [A reduction formula where you have to solve for In .] We try to compute

Z
In = (sin x)n dx

by a reduction formula. Integrating by parts twice we get

Z
In = (sin x)n−1 sin x dx

Z
n−1
= −(sin x) cos x − (− cos x)(n − 1)(sin x)n−2 cos x dx

Z
n−1
= −(sin x) cos x + (n − 1) (sin x)n−2 cos2 x dx.

H.W-Kayondo & D.W-Ddumba, Engineering Math I- Lecture Notes 293


CHAPTER 7. TECHNIQUES OF INTEGRATION

We now use cos2 x = 1 − sin2 x, which gives


Z
n−1
sinn−2 x − sinn x

In = −(sin x) cos x + (n − 1) dx

= −(sin x)n−1 cos x + (n − 1)In−2 − (n − 1)In .


You can think of this as an equation for In , which, when you solve it tells you
nIn = −(sin x)n−1 cos x + (n − 1)In−2
and thus implies
1 n−1
In = − sinn−1 x cos x + In−2 .
n n

Since we know the integrals


Z Z Z
0
I0 = (sin x) dx = dx = x + C and I1 = sin x dx = − cos x + C

the reduction formula (S) allows us to calculate In for any n ≥ 0.


Example 7.7.4 [A reduction formula which will be handy later.] In the next section
you will see how the integral of any “rational function” can be transformed into integrals
of easier functions, the hardest of which turns out to be
Z
dx
In = .
(1 + x2 )n

When n = 1 this is a standard integral, namely


Z
dx
I1 = = arctan x + C.
1 + x2

When n > 1 integration by parts gives you a reduction formula. Here’s the computation:
Z
In = (1 + x2 )−n dx

Z
x −n−1
= − x (−n) 1 + x2 2x dx
(1 + x2 )n

x2
Z
x
= + 2n dx
(1 + x2 )n (1 + x2 )n+1

H.W-Kayondo & D.W-Ddumba, Engineering Math I- Lecture Notes 294


CHAPTER 7. TECHNIQUES OF INTEGRATION

Apply
x2 (1 + x2 ) − 1 1 1
2 n+1
= 2 n+1
= 2 n

(1 + x ) (1 + x ) (1 + x ) (1 + x2 )n+1

to get
x2
Z Z  
1 1
dx = − dx = In − In+1 .
(1 + x2 )n+1 2
(1 + x )n (1 + x2 )n+1

Our integration by parts therefore told us that

x 
In = + 2n In − In+1 ,
(1 + x2 )n

which you can solve for In+1 . You find the reduction formula

1 x 2n − 1
In+1 = 2 n
+ In .
2n (1 + x ) 2n

1 x 2n − 1
In+1 = 2 n
+ In
2n (1 + x ) 2n

For
Z
1
In+1 = dx and I1 = arctan x
(1 + x2 )n

As an example of how you can use it, we start with I1 = arctan x + C, and conclude that

Z
dx
= In+1 ⇒ n = 1
(1 + x2 )2

1 x 2·1−1
= + I1
2 · 1 (1 + x2 )1 2·1

1 x 1
= 2
+ arctan x + C.
21+x 2

H.W-Kayondo & D.W-Ddumba, Engineering Math I- Lecture Notes 295


CHAPTER 7. TECHNIQUES OF INTEGRATION

Apply the reduction formula again, now with n = 2, and you get

Z
dx
= In+1 ⇒ n = 2
(1 + x2 )3

1 x 2·2−1
= + I2
2 · 2 (1 + x )
2 2 2·2
 
1 x 3 1 x 1
= + + arctan x
4 (1 + x2 )2 4 2 1 + x2 2

1 x 3 x 3
= 2 2
+ 2
+ arctan x + C.
4 (1 + x ) 81+x 8

R
Problem 7.7.1 Evaluate xn ln x dx where n 6= −1.

xn+1 ln x xn+1
R
Answer 7.7.1 xn ln x dx = n+1
− (n+1)2
+ C.

Example 7.7.5 ?

Z Z
1 3 3x
e sin 2x dx = − e3x cos 2x +
3x
e cos 2x dx
2 2
Z Z
3x 1 3x 3 3x 9
e sin 2x dx = − e cos 2x + e sin 2x − e3x sin 2x dx
2 4 4
Z
13 1 3
e3x sin 2x dx = − e3x cos 2x + e3x sin 2x + C
4 2 4
Z
2 3
e3x sin 2x dx = − e3x cos 2x + e3x sin 2x + C
13 13

R
Problem 7.7.2 Evaluate eax sin bx dx where a2 + b2 6= 0. [Hint: Integrate by parts
twice.]
ax
Answer 7.7.2 eax sin bx dx = a2e+b2 (a sin bx − b cos bx) + C.
R

R
Problem 7.7.3 Evaluate eax cos bx dx where a2 + b2 6= 0.
ax
Answer 7.7.3 eax cos bx dx = a2e+b2 (a cos bx + b sin bx) + C.
R

H.W-Kayondo & D.W-Ddumba, Engineering Math I- Lecture Notes 296


CHAPTER 7. TECHNIQUES OF INTEGRATION

Problem 7.7.4 Prove the formula


Z Z
n x n x
x e dx = x e − n xn−1 ex dx

R
and use it to evaluate x2 ex dx.

Exercise 7.33 Prove the formula

n−1
Z Z
1
sin x dx = − cos x sinn−1 x +
n
sinn−2 x dx, n 6= 0
n n

Z Z
n
Hint : sin x dx = sin x sinn−1 x dx

Exercise 7.34 Evaluate sin2 x dx. Show that the answer is the same as the answer
R

you get using the half angle formula.



Exercise 7.35 Evaluate 0 sin14 x dx.

Problem 7.7.5 Prove the formula

n−1
Z Z
1
cos x dx = sin x cosn−1 x +
n
cosn−2 x dx, n 6= 0
n n

R π/4
and use it to evaluate 0
cos4 x dx.

Exercise 7.36 Prove the formula

xm+1 (ln x)n


Z Z
m n n
x (ln x) dx = − xm (ln x)n−1 dx, m 6= −1.
m+1 m+1

and use it to evaluate


R
(i) ln x dx,
R
(ii) (ln x)2 dx,
R
(iii) x3 (ln x)2 dx

x−1 ln x dx by another method. [Hint: the solution is short!]


R
Exercise 7.37 Ẽvaluate

H.W-Kayondo & D.W-Ddumba, Engineering Math I- Lecture Notes 297


CHAPTER 7. TECHNIQUES OF INTEGRATION

Problem 7.7.6 For an integer n > 1 derive the formula

Z Z
n 1
tan x dx = tann−1 x − tann−2 x dx
n−1

Z Z
n
Hint : tan x dx = tan2 x tann−2 x dx

R π/4
Using this, find 0
tan5 x dx by doing just one explicit integration.

Problem 7.7.7 Use the reduction formula from example 7.7.4 to compute

Z
dx
(1 + x2 )3
Z
dx
(1 + x2 )4
Z
x dx
(1 + x2 )4
Z
1+x
dx
(1 + x2 )2

x
R
[Hint: (1+x2 )n
dx is easy.]

Problem 7.7.8 The reduction formula from example 7.7.4 is valid for all n 6= 0. In
particular, n does not have to
R√ be an integer, and
R itdxdoes not have to be 1positive.
Find a relation between 1 + x dx and √1+x2 by setting n = − 2 .
2

H.W-Kayondo & D.W-Ddumba, Engineering Math I- Lecture Notes 298


CHAPTER 7. TECHNIQUES OF INTEGRATION

7.8 Partial Fraction Expansion


A rational function is one which is a ratio of polynomials,

P (x) pn xn + pn−1 xn−1 + · · · + p1 x + p0


f (x) = = .
Q(x) qd xd + qd−1 xd−1 + · · · + q1 x + q0

Such rational functions can always be integrated, and the trick which allows you to do this
is called a partial fraction expansion. The whole procedure consists of several steps which
are explained in this section. The procedure itself has nothing to do with integration: it’s
just a way of rewriting rational functions. It is in fact useful in other situations, such as
finding Taylor seriesand computing ”inverse Laplace transforms”

7.8.1 Reduce to a proper rational function


A proper rational function is a rational function P (x)/Q(x) where the degree of P (x)
is strictly less than the degree of Q(x). the method of partial fractions only applies to
proper rational functions. Fortunately there’s an additional trick for dealing with rational
functions that are not proper.
If P/Q isn’t proper, i.e. if degree(P ) ≥ degree(Q), then you divide P by Q, with result

P (x) R(x)
= S(x) +
Q(x) Q(x)

where S(x) is the quotient, and R(x) is the remainder after division. In practice you
would do a long division to find S(x) and R(x).

Example 7.8.1 Consider the rational function

x3 − 2x + 1
f (x) = .
x2 − 1

Here the numerator has degree 3 which is more than the degree of the denominator (which
is 2). To apply the method of partial fractions we must first do a division with remainder.
One has
x = S(x)
2
x −1 x3 −2x +1
−(x3 −x)
−x +1 = R(x)

H.W-Kayondo & D.W-Ddumba, Engineering Math I- Lecture Notes 299


CHAPTER 7. TECHNIQUES OF INTEGRATION

so that
x3 − 2x + 1 −x + 1
f (x) = =x+ 2
x −1
2 x −1

When we integrate we get

x3 − 2x + 1 −x + 1
Z Z  
dx = x+ 2 dx
x2 − 1 x −1

x2 −x + 1
Z
= + dx.
2 x2 − 1

−x+1
The rational function which still have to integrate, namely x2 −1
, is proper, i.e. its numer-
ator has lower degree than its denominator.

7.8.2 Partial Fraction Expansion: The Easy Case


P (x)
To compute the partial fraction expansion of a proper rational function Q(x)
you must

factor the denominator Q(x). Factoring the denominator is a problem as difficult as


finding all of its roots; in MTH 1201 we shall only do problems where the denominator is
already factored into linear and quadratic factors, or where this factorization is easy to
find.
In the easiest partial fractions problems, all the roots of Q(x) are real and distinct, so
the denominator is factored into distinct linear factors, say

P (x) P (x)
= .
Q(x) (x − a1 )(x − a2 ) · · · (x − an )

To integrate this function we find constants A1 , A2 , . . . , An so that

P (x) A1 A2 An
= + + ··· + . (#)
Q(x) x − a1 x − a2 x − an

Then the integral is

Z
P (x)
dx = A1 ln |x − a1 | + A2 ln |x − a2 | + · · · + An ln |x − an | + C.
Q(x)

H.W-Kayondo & D.W-Ddumba, Engineering Math I- Lecture Notes 300


CHAPTER 7. TECHNIQUES OF INTEGRATION

One way to find the coefficients Ai in (#) is called the method of equating coefficients.
In this method we multiply both sides of (#) with Q(x) = (x − a1 ) · · · (x − an ). The result
is a polynomial of degree n on both sides. Equating the coefficients of these polynomial
gives a system of n linear equations for A1 , . . . , An . You get the Ai by solving that system
of equations.
Another much faster way to find the coefficients Ai is the Heaviside Trick 2 . Multiply
equation (#) by x − ai and then plug in3 x = ai . On the right you are left with Ai so

P (x)(x − ai ) P (ai )
Ai = = .
Q(x)
x=ai (ai − a1 ) · · · (ai − ai−1 )(ai − ai+1 ) · · · (ai − an )

−x+2
R
Example 7.8.2 Evaluate x2 −1
dx, we factor the denominator,

x2 − 1 = (x − 1)(x + 1).
−x+2
The partial fraction expansion of x2 −1
then is

−x + 2 −x + 2 A B
= = + .
x −1
2 (x − 1)(x + 1) x−1 x+1

Multiply with (x − 1)(x + 1) to get


−x + 2 = A(x + 1) + B(x − 1) = (A + B)x + (A − B).
The functions of x on the left and right are equal only if the coefficient of x and the
constant term are equal. In other words we must have
A + B = −1 and A − B = 2.
These are two linear equations for two unknowns A and B, which we now proceed to
solve. Adding both equations gives 2A = 1, so that A = 21 ; from the first equation one
then finds B = −1 − A = − 23 . So

−x + 2 1/2 3/2
= − .
x −1
2 x−1 x+1

Instead, we could also use the Heaviside trick: multiply (†) with x − 1 to get

−x + 2 x−1
=A+B
x+1 x+1
2
Named after Oliver Heaviside, an electrical engineer in the late 19th and early 20th century.
3
More properly, you should take the limit x → ai . If you multiply both sides with x − ai and then
set x = ai , then you have effectively multiplied both sides of the equation with zero!

H.W-Kayondo & D.W-Ddumba, Engineering Math I- Lecture Notes 301


CHAPTER 7. TECHNIQUES OF INTEGRATION

Take the limit x → 1 and you find

−1 + 2 1
= A, i.e. A = .
1+1 2

Similarly, after multiplying (†) with x + 1 one gets

−x + 2 x+1
=A + B,
x−1 x−1

and letting x → −1 you find

−(−1) + 2 3
B= =− ,
(−1) − 1 2

as before.
Either way, the integral is now easily found, namely,

−x + 2
Z Z  
1/2 3/2
dx = − dx
x2 − 1 x−1 x+1

1 3
= ln |x − 1| − ln |x + 1| + C.
2 2

7.8.3 Partial Fraction Expansion: The General Case


Buckle up.
When the denominator Q(x) contains repeated factors or quadratic factors (or both)
the partial fraction decomposition is more complicated. In the most general case the
denominator Q(x) can be factored in the form

Q(x) = (x − a1 )k1 · · · (x − an )kn (x2 + b1 x + c1 )`1 · · · (x2 + bm x + cm )`m (7.2)

Here we assume that the factors x − a1 , . . . , x − an are all different, and we also assume
that the factors x2 + b1 x + c1 , . . . , x2 + bm x + cm are all different.
It is a theorem from advanced algebra that you can always write the rational function
P (x)/Q(x) as a sum of terms like this

P (x) A Bx + C
= ··· + + · · · + + ··· (7.3)
Q(x) (x − ai )k (x2 + bj x + cj )`

H.W-Kayondo & D.W-Ddumba, Engineering Math I- Lecture Notes 302


CHAPTER 7. TECHNIQUES OF INTEGRATION

How did this sum come about?


For each linear factor (x − a)k in the denominator (7.2) you get terms

A1 A2 Ak
+ + · · · +
x − a (x − a)2 (x − a)k

in the decomposition. There are as many terms as the exponent of the linear factor that
generated them.
For each quadratic factor (x2 + bx + c)` you get terms

B1 x + C1 B2 x + C2 Bm x + Cm
2
+ 2 2
+ ··· + 2 .
x + bx + c (x + bx + c) (x + bx + c)`

Again, there are as many terms as the exponent ` with which the quadratic factor appears
in the denominator (7.2).
In general, you find the constants A... , B... and C... by the method of equating coeffi-
cients.

Example 7.8.3 To do the integral

x2 + 3
Z
dx
x2 (x + 1)(x2 + 1)2

apply the method of equating coefficients to the form

x2 + 3 A1 A2 A3 B1 x + C1 B2 x + C2
2 2 2
= + 2 + + + 2 .
x (x + 1)(x + 1) x x x+1 x2 + 1 (x + 1)2

Solving this last problem will require solving a system of seven linear equations in the
seven unknowns A1 , A2 , A3 , B1 , C1 , B2 , C2 . A computer program like Maple can do this
easily, but it is a lot of work to do it by hand. In general, the method of equating
coefficients requires solving n linear equations in n unknowns where n is the degree of the
denominator Q(x).
See Problem 7.8.4 for a worked example where the coefficients are found.

Unfortunately, in the presence of quadratic factors or re-


!! peated linear factors the Heaviside trick does not give
the whole answer; you must use the method of equating
!!
coefficients.

H.W-Kayondo & D.W-Ddumba, Engineering Math I- Lecture Notes 303


CHAPTER 7. TECHNIQUES OF INTEGRATION

Once you have found the partial fraction decomposition (EX ) Ryou still have to integrate
the terms which appeared. The first three terms are of the form A(x − a)−p dx and they
are easy to integrate:
Z
A dx
= A ln |x − a| + C
x−a

and
Z
A dx A
= +C
(x − a)p (1 − p)(x − a)p−1

if p > 1. The next, fourth term in (EX ) can be written as


Z Z Z
B1 x + C1 x dx
dx = B1 dx + C1
x2 + 1 x2 + 1 x2 + 1

B1
= ln(x2 + 1) + C1 arctan x + Cintegration const.
2

While these integrals are already not very simple, the integrals
Z
Bx + C
dx with p > 1
(x2 + bx + c)p

which can appear are particularly unpleasant. If you really must compute one of these,
then complete the square in the denominator so that the integral takes the form
Z
Ax + B
dx.
((x + b)2 + a2 )p

After the change of variables u = x + b and factoring out constants you have to do the
integrals
Z Z
du u du
and .
(u2 + a2 )p (u2 + a2 )p

Use the reduction formula we found in example 7.7.4 to compute this integral.
An alternative approach is to use complex numbers (which are on the menu for this
semester.) If you allow complex numbers then the quadratic factors x+ bx + c can be
factored, and your partial fraction expansion only contains terms of the form A/(x − a)p ,
although A and a can now be complex numbers. The integrals are then easy, but the
answer has complex numbers in it, and rewriting the answer in terms of real numbers
again can be quite involved.

H.W-Kayondo & D.W-Ddumba, Engineering Math I- Lecture Notes 304


CHAPTER 7. TECHNIQUES OF INTEGRATION

Problem 7.8.1 Express each of the following rational functions as a polynomial plus a
proper rational function. (See §7.8.1 for definitions.)

x3
x3 − 4

x3 + 2x
x3 − 4

x3 − x2 − x − 5
x3 − 4

x3 − 1
x2 − 1

Problem 7.8.2 Factor the following rational functions f (x). [Hint: f (0) = f (1) =
f (2) = 0.]

Exercise 7.38 f (x) = 16x5 − 16x4 − 56x3 + 40x2 + 16x

Exercise 7.39 f (x) = 16x9 − 48x8 + 32x7 − 25x5 + 75x4 − 50x3

Problem 7.8.3 Write ax2 + bx + c in the form a(x + p)2 + q, i.e. find p and q in terms of
a, b, and c (this procedure, which you might remember from high school algebra is called
“completing the square.”). Then evaluate the integrals

Z
dx
x2 + 6x + 8
Z
dx
x2 + 6x + 10
Z
dx
5x2 + 20x + 25

Exercise 7.40 Use the method of equating coefficients to find numbers A, B, C such that

x2 + 3 A B C
= + +
x(x + 1)(x − 1) x x+1 x−1

x2 +3
R
and then evaluate the integral x(x+1)(x−1)
dx.

H.W-Kayondo & D.W-Ddumba, Engineering Math I- Lecture Notes 305


CHAPTER 7. TECHNIQUES OF INTEGRATION

Answer 7.8.1 We add

A B C A(x + 1)(x − 1) + Bx(x − 1) + Cx(x + 1)


+ + =
x x+1 x−1 x(x + 1)(x − 1)

(A + B + C)x2 + (C − B)x − A
= .
x(x + 1)(x − 1)

The numerators must be equal, i.e.

x2 + 3 = (A + B + C)x2 + (C − B)x − A

for all x, so equating coefficients gives a system of three linear equations in three unknowns
A, B, C:

A+B+C = 1
C −B = 0
−A = 3

so A = −3 and B = C = 2, i.e.

x2 + 3 3 2 2
=− + +
x(x + 1)(x − 1) x x+1 x−1

and hence

x2 + 3
Z
dx = −3 ln |x| + 2 ln |x + 1| + 2 ln |x − 1| + constant.
x(x + 1)(x − 1)

Exercise 7.41 Do this problem using the Heaviside trick.

Answer 7.8.2 To solve

x2 + 3 A B C
= + + ,
x(x + 1)(x − 1) x x+1 x−1

multiply by x:
x2 + 3 Bx Cx
=A+ +
(x + 1)(x − 1) x+1 x−1

H.W-Kayondo & D.W-Ddumba, Engineering Math I- Lecture Notes 306


CHAPTER 7. TECHNIQUES OF INTEGRATION

and plug in x = 0 to get A = −3; then multiply by x + 1:

x2 + 3 A(x + 1) C(x + 1)
= +B+
x(x − 1) x x−1

and plug in x = −1 to get B = 2; finally multiply by x − 1:

x2 + 3 A(x − 1) B(x − 1)
= + + C,
x(x + 1) x x+1

and plug in x = 1 to get C = 2.


x2 +3
R
Problem 7.8.4 Find the integral x2 (x−1)
dx.

Answer 7.8.3 Apply the method of equating coefficients to the form

x2 + 3 A B C
= + 2+ .
x (x − 1)
2 x x x−1

In this problem, the Heaviside trick can still be used to find C and B; we get B = −3 and
C = 4. Then
A 3 4 Ax(x − 1) + 3(x − 1) + 4x2
− 2+ =
x x x−1 x2 (x − 1)

so A = −3. Hence

x2 + 3
Z
3
dx = −3 ln x + + 4 ln(x − 1) + constant.
x (x − 1)
2 x

Evaluate the following integrals:


R 3 dx
Exercise 7.42 xx4 +1

x5 dx
R
Exercise 7.43 x2 −1

x5 dx
R
Exercise 7.44 x4 −1

e3x dx
R
Exercise 7.45 e4x −1

H.W-Kayondo & D.W-Ddumba, Engineering Math I- Lecture Notes 307


CHAPTER 7. TECHNIQUES OF INTEGRATION

x
√e dx
R
Exercise 7.46 1+e2x

ex dx
R
Exercise 7.47 e2x +2ex +2

dx
R
Exercise 7.48 x(x2 +1)

dx
R
Exercise 7.49 x(x2 +1)2

dx
R
Exercise 7.50 x2 (x−1)

1
R
Exercise 7.51 (x−1)(x−2)(x−3)
dx

x2 +1
R
Exercise 7.52 (x−1)(x−2)(x−3)
dx

x3 +1
R
Exercise 7.53 (x−1)(x−2)(x−3)
dx

R2 dx
Exercise 7.54 Compute 1 x(x−h)
where h is a small positive number.

Exercise 7.55 What happens to your answer to (i) when h → 0+ ?

Summary 7.8.1 Check whether

(i) Is it an improper fraction (if so 1st long divide)

(ii) Linear factors (do it by factorising, difference of two squares)

(iii) Are there repeated factors?

(iv) Are there quadratic factors?

H.W-Kayondo & D.W-Ddumba, Engineering Math I- Lecture Notes 308


CHAPTER 7. TECHNIQUES OF INTEGRATION

Example 7.8.4 ?

Z Z
1 1
dx = dx
(x − 4x − 5)
2 (x + 1)(x − 5)

1 A B
= +
(x + 1)(x − 5) x+1 x−5

1 A(x − 5) + B(x + 1)
=
(x + 1)(x − 5) (x + 1)(x − 5)

1 = A(x − 5) + B(x + 1)

1
let x = 5, ⇒ 1 = 6B ⇒ B=
6
1
let x = −1, ⇒ 1 = −6A ⇒ A = −
6
Z Z Z
1 A B
dx = dx + dx
(x + 1)(x − 5) x+1 x−5
Z Z
1 1 1 1
= − dx + dx
6 x+1 6 x−5

1 1
= − ln(x + 1) + ln(x − 5) + C
6 6

H.W-Kayondo & D.W-Ddumba, Engineering Math I- Lecture Notes 309


CHAPTER 7. TECHNIQUES OF INTEGRATION

Example 7.8.5 ?

x2 + 3
Z
dx
x2 (x − 1)

x2 + 3 A B C
= + 2+
x (x − 1)
2 x x x−1

x2 + 3 Ax(x − 1) + B(x − 1) + Cx2


=
x2 (x − 1) x2 (x − 1)

x2 + 3 = Ax(x − 1) + B(x − 1) + Cx2


let x = 1, ⇒ C = 4
let x = 0, ⇒ B = −3
let x = 2, ⇒ A = −3

x2 + 3
Z Z Z Z
A B C
dx = dx + dx + dx
x2 (x − 1) x x 2 x−1
Z Z Z
1 1 1
= −3 dx + −3 dx + 4 dx
x x2 x−1

3
= −3 ln x + + 4 ln(x − 1) + C
x

H.W-Kayondo & D.W-Ddumba, Engineering Math I- Lecture Notes 310


CHAPTER 7. TECHNIQUES OF INTEGRATION

Example 7.8.6 ?

Z
3x + 1
dx
(x − 1)(x2 + 1)

3x + 1 A Bx + C
= + 2
(x − 1)(x + 1)
2 x−1 x +1

3x + 1 A(x2 + 1) + (Bx + C)(x − 1)


=
(x − 1)(x2 + 1) (x − 1)(x2 + 1)

3x + 1 A(x2 + 1) + Bx + C(x − 1)
=
let x = 1, ⇒ A 2=
let x = 0, ⇒ C 1=
let x = 2, ⇒ B −2
=
Z Z Z
3x + 1 A (Bx + C)
dx = dx + dx
(x − 1)(x + 1)
2 x−1 x2 + 1

1 − 2x
Z Z
2
= dx + dx
x−1 x2 + 1

−2x
Z Z Z
2 1
= dx + dx + dx
x−1 2
x +1 x2 + 1

= 2 ln(x − 1) + tan−1 x − ln(x2 + 1) + C

Example 7.8.7 ?
Z
2x
dx
2x + 5

Since an improper fraction, we apply long-division, such that

2x −5
=1+
2x + 5 2x + 5

−5
Z Z Z
2x
dx = 1 dx + dx
2x + 5 2x + 5

5
= x− ln(2x + 5) + C
2

H.W-Kayondo & D.W-Ddumba, Engineering Math I- Lecture Notes 311


CHAPTER 7. TECHNIQUES OF INTEGRATION

Example 7.8.8

x x
=
25 − x 2 (x + 5)(x − 5)

x A B
= +
25 − x 2 (x + 5) (x − 5)

⇒ x = A(x − 5) + B(x + 5)

1 −1
⇒ A= , B=
2 2
Z Z Z
x dx dx
dx = −
25 − x 2 2(x + 5) 2(x − 5)

1 1
= ln(x + 5) − ln(x − 5) + C
2 2

Example 7.8.9

1 1
=
x(x2− 1) x(x − 1)(x + 1)

1 A B C
= + +
x(x − 1)(x + 1) x x−1 x+1

⇒ 1 = A(x − 1)(x + 1) + B(x)(x + 1) + C(x)(x − 1)


Example 7.8.10 Factor the following functions
6x2 − x − 25 = (3x + 7)(2x + 5)
(x + 5)(x2 − 9) = (x + 5)(x − 3)(x + 3)
(x2 − 1)2 = [(x − 1)(x + 1)]2 = (x − 1)2 (x + 1)2 repeated
Example 7.8.11 For repeated factors

1 A B C
= + +
(x + 2)(x − 1)2 (x + 2) x − 1 (x − 1)2

⇒ 1 = A(x − 1)2 + B(x + 2)(x − 1)C(x + 2)

1 A B C
= + +
(x + 2)3 (x + 2) (x + 2)2 (x + 2)3

⇒ 1 = A(x + 2)2 + B(x + 2) + C

H.W-Kayondo & D.W-Ddumba, Engineering Math I- Lecture Notes 312


CHAPTER 7. TECHNIQUES OF INTEGRATION

Example 7.8.12 For Quadratic factors

1 A Bx + C D E
= + 2 + +
(x − 1)(2x2 + 3)(x + 1) 2 (x − 1) 2x + 3 (x + 1) (x + 1)2

Example 7.8.13

x4 − 2x3 − x2 − 4x + 4 x2 − 2x + 7
= (x + 1) +
(x − 3)(x2 + 1) (x − 3)(x2 + 1)

Example 7.8.14

2x3 − x − 1 6x2 − 3x + 5
= 2+
(x − 3)(x2 + 1) (x − 3)(x2 + 1)

⇒ (A, B, C) = (5, 1, 0)

5 x
= 2+ + 2
(x − 3) (x + 1)

Example 7.8.15

2x2 − 5x + 7 5 3 4
= − −
(x − 2)(x − 1) 2 (x − 2) (x − 1) (x − 1)2

Example 7.8.16

11x + 12 2 2 1
= − +
(2x + 3)(x + 2)(x − 3) (2x + 3) (x + 2) (x − 3)

Example 7.8.17
x3 1
= (x2 + x + 1) +
x−1 (x − 1)

H.W-Kayondo & D.W-Ddumba, Engineering Math I- Lecture Notes 313


CHAPTER 7. TECHNIQUES OF INTEGRATION

7.9 Inverse Trigonometric Functions


Problem 7.9.1 The inverse sine function is the inverse function to the (restricted) sine
function, i.e. when π2 ≤ θ ≤ π2 we have

θ = sin−1 (y) ⇐⇒ y = sin θ.

Warning: sin−1 (y) 6= (sin y)−1 . The inverse sine function is sometimes called arcsine
function and denoted θ = arcsin(y).

Exercise 7.56 If y = sin θ, express sin θ, cos θ, and tan θ in terms of y when 0 ≤ θ < π/2.

Exercise 7.57 If y = sin θ, express sin θ, cos θ, and tan θ in terms of y when π/2 < θ ≤
π.

Exercise 7.58 If y = sin θ, express sin θ, cos θ, and tan θ in terms of y when −π/2 <
θ < 0.

Exercise 7.59 Evaluate √ dy 2 using the substitution y = sin θ, but give the final an-
R
1−y

swer in terms of y.

Problem 7.9.2 Express in simplest form:

Exercise 7.60 cos(sin−1 (x));

ln 14
n o
Exercise 7.61 tan arcsin ln 16 ;


Exercise 7.62 sin 2 arctan α

Problem 7.9.3 Draw the graph of y = f (x) = arcsin sin(x) , for −2π ≤ x ≤ +2π.
Make sure you get the same answer as your graphing calculator (or Maple, or Matlab, or
Octave, etc. if you use a computer.)
Given the antiderivative
kxn
Z
dx
(c ± dx2 )m

r
c 1
let x = sin u if ” − ”, m =
d 2
r
c
let x = tan u if ” + ”, m = 1
d

H.W-Kayondo & D.W-Ddumba, Engineering Math I- Lecture Notes 314


CHAPTER 7. TECHNIQUES OF INTEGRATION

Example 7.9.1
Z Z Z
3x + 2 3x 2
1 dx = 1 dx + 1 dx
(4 − 7x2 ) 2 (4 − 7x2 ) 2 (4 − 7x2 ) 2

to one term up.


Note 7.9.1 The denominator can be found by;
(i) Completing squares
(ii) Factorising

3 3
1 = 1
(2 − x4 ) 2 (2 − (x2 )2 ) 2

−x −x
=
(x2 − 2x + 5) (x − 1)2 + 4

Example 7.9.2
Z Z
1 1
√ dx = 1 dx
2 − x2 (2 − x2 ) 2
√ dx √
let x = 2 sin u ⇒ = 2 cos u
du
Z Z √
1 2 cos u
1 dx = 1 du
(2 − x2 ) 2 (2 − 2 sin2 u) 2
Z √
2 cos u
= 1 du
[2(1 − sin2 u)] 2
Z √
2 cos u
= √ 1 du
2(1 − sin2 u) 2
Z √
2 cos u
= √ 1 du
2(cos2 u) 2
Z
= 1 du

 
−1 x
= u = sin √ +c
2

H.W-Kayondo & D.W-Ddumba, Engineering Math I- Lecture Notes 315


CHAPTER 7. TECHNIQUES OF INTEGRATION

dx √1
R
Example 7.9.3 1 : let x = 3
sin u
(1−3x2 ) 2

Example 7.9.4

Z √
5 sec2 u du
Z
dx
=
5 + x2 5 + 5 tan2 u
√ √ !
5 5
= tan−1 x
5 5


By letting x = 5 tan u

Example 7.9.5
π
Z 1 Z
3 2 3 cos u du
1 dx = 1
1
2
(1 − x2 ) 2 π
6
(1 − sin2 u) 2

Example 7.9.6
Z
dx 1 −1 3
1 = sin x
(4 − 9x2 ) 2 3 2

Example 7.9.7

Z
dx
1
(4 − (x + 1)2 ) 2
√ dx √
let (x + 1) = 2 sin u ⇒ = 2 cos u
du
Z √ Z
2 cos udu 1 1
1 = √ du = √ u
(4 − 4 sin2 u) 2 2 2
 
1 x+1
= √ sin−1 √ +C
2 2

Example 7.9.8
Z  
dx 1 1
= arctan x−1 +C
9 + (x − 3) 2 3 3

H.W-Kayondo & D.W-Ddumba, Engineering Math I- Lecture Notes 316


CHAPTER 7. TECHNIQUES OF INTEGRATION

Example 7.9.9

Z    
dx 1 1 1 1 1 1
= arctan x− = tan−1 x− +C
x − 2x + 5
2 2 2 2 2 2 2

Example 7.9.10

1√ √
Z  
dx 1
= 2 arctan (4x + 4) 2 + C
2x2 + 4x + 11 6 12

Example 7.9.11
Z  
x 1 1 2
1 dx = arcsin x +C
(4 − x ) 2
4 2 2

Example 7.9.12
Z
1 x
3 =p +C
(1 − x2 ) 2 (1 − x2 )

Problem 7.9.4 Evaluate these indefinite integrals:

Z
dx
(i) √
= arcsin x
1 − x2
Z  
dx 1
(ii) √ = arcsin x
4−x 2 2
Z
x dx 1
(iii) √ = arcsin(2x2 )
1 − 4x 4 4

and these definite ones:


Z 1/2
dx
(iv) √ = 0.5053605103
−1/2 4 − x2
Z 1
dx 1
(v) √ = π
−1 4−x 2 3

Z 3/2
dx
(vi) √
0 1 − x2

H.W-Kayondo & D.W-Ddumba, Engineering Math I- Lecture Notes 317


CHAPTER 7. TECHNIQUES OF INTEGRATION

R √3/2
Problem 7.9.5 Use the change of variables formula to evaluate √ dx first using
1/2 1−x2

the substitution x = sin u and then using the substitution x = cos u.


Problem 7.9.6 The inverse tangent function is the inverse function to the (restricted)
tangent function, i.e. for π/2 < θ < π/2 we have
θ = tan−1 (w) ⇐⇒ w = tan θ.
Warning: tan−1 (w) 6= (tan w)−1 . The inverse tangent function is sometimes called
arctangent function and denoted θ = arctan(w).
Exercise 7.63 If w = tan θ, express sin θ and cos θ in terms of w when
(i)0 ≤ θ < π/2; (ii)π/2 < θ ≤ π; (iii) − π/2 < θ < 0.
R dw
Exercise 7.64 Evaluate 1+w 2 using the substitution w = tan θ, but give the final answer

in terms of w.
Problem 7.9.7 Evaluate these indefinite integrals:

(i) x2dx
R R dx
+1
(iii) 7+3x 2

dx
R
(ii) x2 +a2

and these two definite ones:


R √3 R a√3
(iv) 1 x2dx+1
(v) a
dx
x2 +a2

7.10 t-substitution
let

θ
tan = t (7.4)
2
2t
⇒ tan θ = (7.5)
1 − t2

1 − t2
⇒ cos θ = (7.6)
1 + t2
2t
⇒ sin θ = (7.7)
1 + t2

Exercise 7.65 Derive the expressions above.

H.W-Kayondo & D.W-Ddumba, Engineering Math I- Lecture Notes 318


CHAPTER 7. TECHNIQUES OF INTEGRATION

Note 7.10.1 The technique is mostly used to solve:


R R sin θ
(i) sec θdθ (iii) 1+cos θ

dx
R R
(ii) csc xdx (iv) a cos x+b sin x

For such substitution,

θ
since tan = t
2
dt 1 θ
⇒ = sec2
dx 2 2
 
1 2 θ
= 1 + tan
2 2

dt 1
1 + t2

=
dx 2

7.11 Trignometric Substitutions


Numerator = A(derivative of the denominator) + B(denominator)

Note 7.11.1 Only for trignometric integrands.

Example 7.11.1

Z
2 cos x + 9 sin x A(−3 sin x + cos x) + B(cos x + sin x)
=
3 cos x + sin x 3 cos x + sin x

⇒ 2 cos x + 9 sin x = A(−3 sin x + cos x) + B(cos x + sin x)

−5 3
⇒ A= , B=
2 2
Z
2 cos x + 9 sin x 5 3
= = − ln(3 cos x + sin x) + x + C
3 cos x + sin x 2 2

H.W-Kayondo & D.W-Ddumba, Engineering Math I- Lecture Notes 319


CHAPTER 7. TECHNIQUES OF INTEGRATION

7.12 Miscellaneous and Mixed Integrals


Problem 7.12.1 Find the area of the region bounded by the curves

2 x2 − 8x + 7
x = 1, x = 2, y= , y= .
x2 − 4x + 5 x2 − 8x + 16

Problem 7.12.2 Let P be the piece of the parabola y = x2 on which 0 ≤ x ≤ 1.

Exercise 7.66 Find the area between P, the x-axis and the line x = 1.

Exercise 7.67 Find the length of P.

Problem 7.12.3 Let a be a positive constant and

Z x
F (x) = sin(aθ) cos(θ) dθ.
0

[Hint: use a trig identity for sin A cos B, or wait until we have covered complex exponen-
tials and then come back to do this problem.]

Exercise 7.68 Find F (x) if a 6= 1.

Exercise 7.69 Find F (x) if a = 1. (Don’t divide by zero.)

Problem 7.12.4 Find


Z
dx
x(x − 1)(x − 2)(x − 3)

and
(x3 + 1) dx
Z

x(x − 1)(x − 2)(x − 3)

Problem 7.12.5 Evaluate the following integrals:


Ra
Exercise 7.70 0 x sin x dx
Ra
Exercise 7.71 0 x2 cos x dx

R4
Exercise 7.72 √x dx
3 x2 −1

H.W-Kayondo & D.W-Ddumba, Engineering Math I- Lecture Notes 320


CHAPTER 7. TECHNIQUES OF INTEGRATION

R 1/3
Exercise 7.73 √x dx
1/4 1−x2

R4
Exercise 7.74 √dx
3 x x2 −1

x dx
R
Exercise 7.75 x2 +2x+17

x4
R
Exercise 7.76 (x2 −36)1/2
dx

x4
R
Exercise 7.77 x2 −36
dx

x4
R
Exercise 7.78 36−x2
dx

x4
R
Exercise 7.79 (36−x2 )3/2
dx

R (x2 +1) dx
Exercise 7.80 x4 −x2

R (x2 +3) dx
Exercise 7.81 x4 −2x2

dx
R
Exercise 7.82 (x2 −3)1/2

R
Exercise 7.83 ex (x + cos(x)) dx
R
Exercise 7.84 (ex + ln(x)) dx
dx
R
Exercise 7.85 √
(x+5) x2 +5x

3x2 +2x−2
R
Exercise 7.86 x3 −1
dx

x4
R
Exercise 7.87 x4 −16
dx

x
R
Exercise 7.88 (x−1)3
dx

4
R
Exercise 7.89 (x−1)3 (x+1)
dx

1
R
Exercise 7.90 √
1−2x−x2
dx

H.W-Kayondo & D.W-Ddumba, Engineering Math I- Lecture Notes 321


CHAPTER 7. TECHNIQUES OF INTEGRATION

dx
R
Exercise 7.91 √
x2 +2x+3

Re
Exercise 7.92 1
x ln x dx

R e3
Exercise 7.93 e2
x2 ln x dx
Re
Exercise 7.94 x(ln x)3 dx
1
R √
Exercise 7.95 arctan( x) dx
R
Exercise 7.96 x(cos x)2 dx

Rπp
Exercise 7.97 0
1 + cos(6w) dw

Problem 7.12.6 You don’t always have to find the antiderivative to find a definite inte-
gral. This problem gives you two examples of how you can avoid finding the antiderivative.

Exercise 7.98 To find


Z π/2
sin x dx
I=
0 sin x + cos x

you use the substitution u = π/2 − x. The new integral you get must of course be equal
to the integral I you started with, so if you add the old and new integrals you get 2I. If
you actually do this you will see that the sum of the old and new integrals is very easy
to compute.

R π/2
Exercise 7.99 Use the same trick to find 0
sin2 x dx

2 2 2
Problem 7.12.7 Graph the equation x 3 + y 3 = a 3 . Compute the area bounded by this
curve.

Problem 7.12.8 The Bow-Tie Graph. Graph the equation y 2 = x4 − x6 . Compute


the area bounded by this curve.

Problem 7.12.9 The Fan-Tailed Fish. Graph the equation

a−x
 
2 2
y =x a > 0.
a+x

Find the area enclosed by the loop. (Hint: Rationalize the denominator of the integrand.
)

H.W-Kayondo & D.W-Ddumba, Engineering Math I- Lecture Notes 322


CHAPTER 7. TECHNIQUES OF INTEGRATION

Problem 7.12.10 Find the area of the region bounded by the curves

x
x = 2, y = 0, y = x ln
2

Problem 7.12.11 Find the volume of the solid of revolution obtained by rotating around
the x−axis the region bounded by the lines x = 5, x = 10, y = 0, and the curve

x
y=√ .
2
x + 25

1
Problem 7.12.12 How to find the integral of f (x) = cos x

Exercise 7.100 Verify the answer given in the table in the lecture notes.
Exercise 7.101 Note that

1 cos x cos x
= = ,
cos x 2
cos x 1 − sin2 x

and applyR the substitution s = sin x followed by a partial fraction decomposition to


dx
compute cos x
.

Some Basic integrals


Z
sin x dx = − cos x + C

Z
cos x dx = sin x + C

Z
sec2 x dx = tan x + C

Example 7.12.1 Since (by the chain rule)

d sin x
e = cos x esin x
dx

then we can anticipate that


Z
cos x esin x dx = esin x + C

H.W-Kayondo & D.W-Ddumba, Engineering Math I- Lecture Notes 323


CHAPTER 7. TECHNIQUES OF INTEGRATION

Example 7.12.2 Since (by the chain rule)

d√ 5 1
x + 3x = (x5 + 3x)−1/2 · (5x4 + 3)
dx 2

then we can anticipate that


Z
1 √
(5x4 + 3)(x5 + 3x)−1/2 dx = x5 + 3x + C
2

Example 7.12.3 Since (by the chain rule)

d
sin7 (3x + 1) = 7 · sin6 (3x + 1) · cos(3x + 1) · 3
dx

then we have
Z
cos(3x + 1) sin6 (3x + 1) dx

Z
1 1
= 7 · 3 · cos(3x + 1) sin6 (3x + 1) dx = sin7 (3x + 1) + C
21 21

Example 7.12.4 Evaluate


Z √
sin x
√ dx
x

we use substitution method, let u = x



Z
sin x
√ dx = −2 cos x + C
x

Example 7.12.5 Evaluate


Z
x2 sin x dx = −x2 cos x + 2 (x sin x + cos x) + C

Exercise 7.102 Evaluate


Z 2
1
(1 + x2 ) 2 dx
1

H.W-Kayondo & D.W-Ddumba, Engineering Math I- Lecture Notes 324


CHAPTER 7. TECHNIQUES OF INTEGRATION

Exercise 7.103 Evaluate


1
x3
Z
5 dx
0 (3 + x2 ) 2

Exercise 7.104 Evaluate


x−1
Z
√ √ dx
6 x (4 + 3 x )

R x +x
Example 7.12.6 Find ee dx

Z Z Z
ex +x x ex x x
e dx = e e dx = ee dex = ee + C

R
Example 7.12.7 Find tan x log(cos x) dx.
Put u = log(cos x), etc.

(log(cos x))2
Z Z
tan x log(cos x) dx = (log(cos x)) d(− log(cos(x))) = − +C
2

log log x
R
Example 7.12.8 Find x log x
dx
Put u = log log x, etc.

(log log x)2


Z
log log x
dx = +C
x log x 2

R 18 −1
Example 7.12.9 Find xx3 −1 dx.
Carry out the long division.

x18 − 1 x16 x13 x10 x7 x4


Z Z
dx = (x15 + x12 + x9 + x6 + x3 + 1) dx = + + + + +x+C
x3 − 1 16 13 10 7 4

Example 7.12.10 Find x81+x dx


R

After an algebraic trick, put u = 1 + x−7 , etc.

x−8 (1 + x−7 )
Z Z Z
1 1 1
8
dx = −7
dx = − −7
= − log |1 + x−7 | + C
x +x 1+x 7 1+x 7

H.W-Kayondo & D.W-Ddumba, Engineering Math I- Lecture Notes 325


CHAPTER 7. TECHNIQUES OF INTEGRATION

4x
R
Example 7.12.11 Find 2x +1
dx.

Put u = 2x + 1

2x 2x
Z
1 1
x
. = (u − log |u|) + C = (2x + 1 − log |2x + 1|) + C
2x + 1 log 2 log 2

x2
R
Example 7.12.12 Find (x+1)10
dx.

Put u = x + 1.

x2 (x + 1)−7 (x + 1)−8 (x + 1)−9


Z
dx = − + − +C
(x + 1)10 7 4 9

R 1
Example 7.12.13 Find 1+e x dx.

Algebraic trick, and then u = e−x + 1, etc.

Z
1
dx = − log |e−x + 1| + C
1 + ex

1
R
Example 7.12.14 Find 1−sin x
dx.

Z Z Z Z
1 1 + sin x 1 + sin x
dx = dx = dx = sec2 x+sec x tan xx. = tan x+sec x+C
1 − sin x 1 − sin2 x cos2 x

R√
Example 7.12.15 Find 1 + sin 2x dx.


Z
1 + sin 2x dx = − cos x + sin x + C or + cos x − sin x + C

√ x
R
Example 7.12.16 Find 1−x4
dx.

Put u = x2 , etc.
Z
x 1
p dx = arcsin x2 + C
1 − (x2 )2 2

H.W-Kayondo & D.W-Ddumba, Engineering Math I- Lecture Notes 326


CHAPTER 7. TECHNIQUES OF INTEGRATION

Example 7.12.17 Let a > 0, b > 0, and f a continuous strictly increasing function with
f (0) = 0. Prove that
Z a Z b
ab ≤ f (x)dx + f −1 (x) dx.
0 0

Prove, moreover, that equality occurs if and only if b = f (a).


R
Example 7.12.18 Find sec4 x dx

Z Z
4
sec x dx = sec2 x(tan2 x + 1) dx

Z Z
2 2
= sec x tan x dx + sec2 x dx

Z Z
2
= (tan x) d(tan x) + sec2 x dx

tan3 x
= + tan x + C.
3

R
Example 7.12.19 Find sec5 x dx.

Z Z
5
sec x dx = sec3 x sec2 x dx

Z
= sec3 xd(tan x)

Z
3
= sec x tan x − tan xd(sec3 x)

Z
3
= sec x tan x − 3 tan2 x sec2 x sec x dx

Z
3
= sec x tan x − 3 (sec2 x − 1) sec3 x dx

Z Z
3 5
= sec x tan x − 3 sec x dx + 3 sec3 x dx

H.W-Kayondo & D.W-Ddumba, Engineering Math I- Lecture Notes 327


CHAPTER 7. TECHNIQUES OF INTEGRATION

The above implies that

tan x sec3 x 3
Z Z
5
sec x dx = + sec3 x dx
4 4

tan x sec3 x 3 tan x sec x 3


= + + log | sec x + tan x| + C
4 8 8

upon recalling from class that

Z
tan x sec x 1
sec3 x dx = + log | sec x + tan x| + C
2 2

R 1/3
Example 7.12.20 Find ex dx.
First put t = x1/3 , then t3 = x ⇒ 3t2 dt = dx. Thus

Z Z
x1/3
e dx = 3t2 et dt

= 3t2 et − 6tet − 6et + C


1/3 1/3 1/3
= 3x2/3 ex − 6x1/3 ex − 6ex + C,

where the penultimate step results from tabular integration by parts.


R
Example 7.12.21 Find log(x2 + 1) dx.
We have
Z Z
2 2
log(x + 1) dx = x log(x + 1) − xd(log(x2 + 1))

x2
Z
2
= x log(x + 1) − 2 dx
x2 + 1

x2 + 1 − 1
Z
2
= x log(x + 1) − 2 dx
x2 + 1
Z  
2 1
= x log(x + 1) − 2 1− 2 dx
x +1

= x log(x2 + 1) − 2(x − arctan x) + C

H.W-Kayondo & D.W-Ddumba, Engineering Math I- Lecture Notes 328


CHAPTER 7. TECHNIQUES OF INTEGRATION

R
Example 7.12.22 Find xex cos x dx.
This method parallels the one in class of “solving for the integral.” Put

Z
I= xex cos x := (Ax + B)ex cos x + (Cx + D)ex sin x + K.

Differentiating both sides,

xex cos x = Aex cos x+(Ax+B)ex cos x−(Ax+B)ex sin x+Cex sin x+(Cx+D)ex sin x+(Cx+D)ex cos x.

Equating coefficients,

xex cos x : 1=A+C


xex sin x : 0 = −A + C
ex cos x : 0=A+B+D
ex sin x : 0 = −B + C + D

From the first two equations C = 21 , A = 12 . Then the third and fourth equations become
− 12 = B + D; − 21 = −B + D, Hence D = − 12 , and B = 0. We conclude that

x−1
Z  
x
xe cos x = ex cos x +
x
ex sin x + K.
2 2

R
Example 7.12.23 Find x2/3 log x dx.
We will do this one two ways: first, by making the substitution

t = log x ⇒ et = x ⇒ et dt = dx.

Observe also that x2/3 = e2t/3 . Then


Z Z
2/3
x log x dx = te2t/3 et dt

3t 5t/3 9
= e − e5t/3 + C
5 25
3(log x) 5/3 9
= x − x5/3 + C.
5 25

H.W-Kayondo & D.W-Ddumba, Engineering Math I- Lecture Notes 329


CHAPTER 7. TECHNIQUES OF INTEGRATION

Aliter: By directly integrating by parts,

3x5/3
Z Z  
2/3
x log x dx = log x d
5

3x5/3
Z
3
= log x − x5/3 d(log x)
5 5
Z
3(log x) 5/3 3
= x − x2/3 dx
5 5

3(log x) 5/3 9
= x − x5/3 + C,
5 25

as before.
R
Example 7.12.24 Find sin(log x) dx.
This integral can be done multiple ways. For example, you may integrate by parts
directly and then “solve” for the integral. Another way, which parallels a method shewn
in class is the following. Start by putting

t = log x ⇒ et = x ⇒ et dt = dx.

Then
Z Z
sin(log x) dx = et sin t dt,

an integral that we found in class. We will find it again, using a method similar of Example
7.12.22. Put
Z
I = et cos t dt := Aet cos t + Bet sin t + K.

Differentiating both sides

et cos t = Aet cos t − Aet sin t + Bet sin t + Bet cos t.

Equating coefficients,

et cos t : 1 = A + B
et sin t : 0 = −A + B

H.W-Kayondo & D.W-Ddumba, Engineering Math I- Lecture Notes 330


CHAPTER 7. TECHNIQUES OF INTEGRATION

and so A = B = 21 . We have thus

Z Z
sin(log x) dx = et sin t dt

1 t 1
= e cos t + et sin t + K
2 2
1 1
= x cos log x + x sin log x + K.
2 2

log log x
R
Example 7.12.25 Find x
dx.
t t
Put t = log log x ⇒ ee = x ⇒ et ee dt = dx. Hence

t
tet ee
Z Z
log log x
dx = dt
x eet

= tet − et + C
= (log x)(log log x) − (log x) + C,

where the penultimate equality follows from a tabular integration by parts.


1 cos x cos x
R
Example 7.12.26 Prove that cos x
= 2(1+sin x)
+ 2(1−sin x)
. Use this to find sec x dx.

Simple algebra will yield the identity. We have

Z Z Z
cos x cos x
sec x dx = dx + dx
2(1 + sin x) 2(1 − sin x)

1 1
= log |1 + sin x| − log |1 − sin x| + C
2 2
1 1 + sin x
= log +C

2 1 − sin x

Example 7.12.27 Using sin 2θ = 2 sin θ cos θ show that R csc x dx = log | tan x2 | + C.
R

Now use csc( π2 + x) = sec x to find yet another formula for sec x dx.

H.W-Kayondo & D.W-Ddumba, Engineering Math I- Lecture Notes 331


CHAPTER 7. TECHNIQUES OF INTEGRATION

We have
Z Z
1
csc x dx = dx
sin x
Z
1
= dx
2 sin cos x2
x
2

cos x2
Z
= dx
2 sin x2 cos2 x
2

sec2 x2
Z
= dx
2 tan x2
Z
u=tan x
2 du
=
u
x
= log | tan | + C.
2

Thus
Z Z Z
π π π π x
sec x dx = csc( + x) dx = csc( + x) d( + x) = log tan( + ) + C.

2 2 2 4 2

R
Example 7.12.28 Find (arcsin x)2 dx
Putting t = arcsin x we have

sin t = x ⇒ cos t dt = dx,

Hence
Z Z
2
(arcsin x) dx = t2 cos t dt

= t2 sin t + 2t cos t − 2 sin t + C


= (arcsin x)2 x + 2(arcsin x) cos(arcsin x) − 2x + C

= (arcsin x)2 x + 2(arcsin x) 1 − x2 − 2x + C
dx√
R
Example 7.12.29 Find √
x+1+ x−1
.

H.W-Kayondo & D.W-Ddumba, Engineering Math I- Lecture Notes 332


CHAPTER 7. TECHNIQUES OF INTEGRATION

We have

√ √
( x + 1 − x − 1) dx
Z Z
dx
√ √ =
x+1+ x−1 2

1 1
= (x + 1)3/2 − (x − 1)3/2 + C
3 3

R
Example 7.12.30 x arctan x dx.
We have

x2
Z Z  
x arctan x dx = arctan x d
2

x2 x2
Z
= arctan x − d(arctan x)
2 2

x2 1 x2
Z
= arctan x − dx
2 2 1 + x2

x2 1 x2 + 1 − 1
Z
= arctan x − dx
2 2 1 + x2

x2 x 1
= arctan x − + arctan x + C
2 2 2

R√
Example 7.12.31
√ Find tan x dx.
Put u = tan x and so u = tan x, 2u du = sec2 x dx = (tan2 x + 1) dx = (u4 + 1) dx.
2

Hence the integral becomes

Z √
u2
Z
tan x dx = 2 du.
u4 + 1

To decompose the above fraction into partial fractions observe (Sophie Germain’s trick)

H.W-Kayondo & D.W-Ddumba, Engineering Math I- Lecture Notes 333


CHAPTER 7. TECHNIQUES OF INTEGRATION

√ √
that u4 + 1 = u4 + 2u2 + 1 − 2u2 = (u2 + u 2 + 1)(u2 − u 2 + 1) and hence

Z √
u2
Z
tan x dx = 2 du
u4 + 1
√ Z √ Z
2 u 2 u
= − √ du + √ du
2 2
u +u 2+1 2 u −u 2+1
2

√ √
2 2
√ 2 √
= − log(u + u 2 + 1) + log(u2 − u 2 + 1) +
4 4
√ √
2 √ 2 √
arctan( 2u + 1) − arctan(− 2u + 1) + C
2 2
√ √
2 √ 2 √
= − log(tan x + 2 tan x + 1) + log(tan x − 2 tan x + 1)
4 4
√ √
2 √ 2 √
+ arctan( 2 tan x + 1) − arctan(− 2 tan x + 1) + C
2 2

2x+1
R
Example 7.12.32 Find x2 (x−1)
dx
Put

2x + 1 A B C
= + 2+ ⇒ 2x + 1 = Ax(x − 1) + B(x − 1) + Cx2 .
x2 (x − 1) x x x−1

Letting x = 1 we get 3 = C. Letting x = 0 we get 1 = −B ⇒ B = −1. To get


A observe that equating the coefficients of x2 on both sides we get 0 = A + C, whence
A = −3. Thus

Z Z Z Z
2x + 1 1 1 1
dx = −3 dx − dx + 3 dx
x (x − 1)
2 x x2 x−1

1
= −3 log |x| + + 3 log |x − 1| + C
x
x − 1 1
= 3 log + +C

x x

H.W-Kayondo & D.W-Ddumba, Engineering Math I- Lecture Notes 334


CHAPTER 7. TECHNIQUES OF INTEGRATION

R √
Example 7.12.33 Find log(x + x) dx
Integrating by parts,

√ √ √
Z Z
log(x + x) dx = x log(x + x) − x dlog(x + x)

Z x(1 + 1
√ √
2 x
)
= x log(x + x) − √ dx
x+ x


Z  
1 x
= x log(x + x) − 1− · √ dx
2 x+ x


Z
1 x
= x log(x + x) − x + √ dx
2 x+ x

√ u2
Z
u= x
= x log(x + x) − x + du
u2 + u


Z
u= x 1
= x log(x + x) − x + 1− du
u+1

= x log(x + x) − x + u − log(u + 1) + C
√ √ √
= x log(x + x) − x + x − log( x + 1) + C

Example 7.12.34 Find x41+1 dx.


R

We use Sophie Germain’s trick to factor


√ √
x4 + 1 = x4 + 2x2 + 1 − 2x2 = (x2 + 1)2 − 2x2 = (x2 − 2x + 1)(x2 + 2x + 1),

and seek the partial fraction decomposition

1 Ax + B Cx + D 2
√ 2

= √ + √ ⇒ 1 = (Ax+B)(x + 2x+1)+(Cx+D)(x − 2x+1).
x4 + 1 x2 − 2x + 1 x2 + 2x + 1

Equating coefficients

x3 : 0=A+C

x2 : 0 = B + D + 2(A − C)

x : 0 = A + C + 2(B − D)
x0 : 1=B+D

H.W-Kayondo & D.W-Ddumba, Engineering Math I- Lecture Notes 335


CHAPTER 7. TECHNIQUES OF INTEGRATION

From the first and third equation it follows that A = −C and that B = D. From the
fourth equation B = D = 12 and from the second equation A = − 2√1 2 = −C. Hence we
must integrate
√ √
2x − 2
Z Z Z
1 2x + 2
4
dx = √ dx − √ dx
x +1 2
4(x + 2x + 1) 4(x − 2x + 1)
2

√ Z √ Z
2 2x + 2 1 1
= √ dx + √ dx
8 x2 + 2x + 1 4 x2 + 2x + 1
√ Z √ Z
2 2x + 2 1 1
− √ dx + √ dx
8 x2 − 2x + 1 4 x2 − 2x + 1
√ √
2 2
√ 2 √
= log(x + x 2 + 1) − log(x2 − x 2 + 1)
8 8
Z Z
1 dx 1 dx
+ √ + √
2 (x 2 + 1)2 + 1 2 (−x 2 + 1)2 + 1
√ √
2 √ 2 √
= log(x2 + x 2 + 1) − log(x2 − x 2 + 1)
8 8
√ √
2 √ 2 √
+ arctan(x 2 + 1) − arctan(−x 2 + 1) + C
4 4

Example 7.12.35 Find x31+1 dx.


R

We begin by observing that

1 A Bx + C
= + 2 ⇒ 1 = A(x2 − x + 1) + (Bx + C)(x + 1).
x3 +1 x+1 x −x+1

Letting x = −1 we obtain 1 = 3A ⇒ A = 13 . Letting x = 0 we obtain 1 =


2
A + C ⇒ C = 1 − A = 3 . Finally, we must have A + B = 0, since the coefficient of x2
must be zero. thus B = − 31 . We must then integrate

x−2
Z Z
dx
⇒ − dx =
3(x + 1) 3(x2 − x + 1)

x − 12
Z Z
1 1 1
= log |x + 1| − +
3 3(x − 12 )2 + 3
4
2 (x − 1 2
2
) + 3
4
Z
1 1 1 3 2 1
= log |x + 1| − log |(x − )2 + | + 4
3 6 2 4 3 3
(x − 21 )2 + 1

H.W-Kayondo & D.W-Ddumba, Engineering Math I- Lecture Notes 336


CHAPTER 7. TECHNIQUES OF INTEGRATION

√  
1 1 1 2 3 2 3 2 1
= log |x + 1| − log |(x − ) + | + · arctan √ x −
3 6 2 4 3 2 3 2
√  
1 1 2 3 2 1
= log |x + 1| − log |x − x + 1| + arctan √ x −
3 6 3 3 2

Example 7.12.36 Verify that

Z
f (x) dx = F (x) + C

for f (x) given in the table below.

f (x) F (x)

1 1
1−√2 sin x
− 4 sin + log 1−sin
1 x
− 1
√ log 1+√2 sin x

sin x sin 4x x 8 1+sin x 2 2

tan x x
1+tan x 2
− 21 log | cos x + sin x|
√ √
sin x cos 2x 1

cos x cos 2x 2
+ √
2 2
arcsin( 2 sin x)

1 1
sin x+sin 2x 6
log(1 − cos x) + 21 log(1 + cos x) − 23 log |1 + 2 cos x|

1
√ √
cos x cos 2x
2arc tanh( 2 sin x) − arc tanh(sin x)
q √ q
√ 1
−2 1−sin x
sin x
+ 2 arctan 1−sin
2 sin x
x
(put u = 1/ sin x)
sin x sin x(1+sin x)

√ 
cos2 x−a2 sin2 x
√ a sin x
− arctan a
cos x cos2 x−a2 sin2 x

H.W-Kayondo & D.W-Ddumba, Engineering Math I- Lecture Notes 337


CHAPTER 7. TECHNIQUES OF INTEGRATION

R
Example 7.12.37 Verify that f (x) dx = F (x) + C.

f (x) F (x)
 
1 1 1 √1 2x+1
x3 −1 3log |x − 1| − 6 log(x2
+ x + 1) − arctan 3

3
 
1
(x3 −1)2
− 29 log |x − 1| + 19 log(x2 + x + 1) + 3√2
3
arctan 2x+1

3
− x
3(x3 −1)
h 2 i
1 −x+1
− 2x12 + 61 log x(x+1) − √13 arctan 2x−1
 

x3 (1+x3 ) 2
3

x2 +x+1 3 1
(x2 −1)2
− 4(x−1) − 4(x+1)

1 1
h √
1+x√2+x2
i
1
 √ √ 
1+x4

4 2
log 1−x 2+x2
+ √
2 2
arctan(1 + x 2) − arctan(1 − x 2)

x2 1
h √
1−x√2+x2
i
1
 √ √ 
1+x4

4 2
log 1+x 2+x2
+ √
2 2
arctan(1 + x 2) − arctan(1 − x 2)

x arctan x2 x2
(x4 +1)2 4 + 4(x4 +1)

x2 +x+1 7 3 1
x3 −2x−4 10 log |x − 2| + 20 log(x2 + 2x + 2) − 10 arctan(x + 1)

x2 −4 4 3x 11 x−1
x6 −2x4 +x2 x + 2(x2 −1)
+ 4 log x+1
P9 h i
1 1 1 x−cos kα 1
cos kα log(x2 − 2x cos kα + 1) − sin kα arctan + log x−1
x+1 , α= π

x20 −1 10 k=1 2 sin kα 20 10

1 1 x−b Pn−1 1
(x−a)n (x−b) (b−a)n log x−a + k=1 k(b−a)n−k (x−a)k
, n ≥ 2.

H.W-Kayondo & D.W-Ddumba, Engineering Math I- Lecture Notes 338


CHAPTER 7. TECHNIQUES OF INTEGRATION

R
Example 7.12.38 Verify that f (x) dx = F (x) + C.

f (x) F (x)
√ √
√ x+1 x2 − 3x + 2 + 52 log 2x − 3 + 2 x2 − 3x + 2

x2 −3x+2
4x−3


−4x2 +12x−5
− −4x2 + 12x − 5 + 23 arcsin(x − 3/2)

1√ 1− 2x−x2
2x−x2 + 2x−x2 x−1
√ √  
√ 1 √ 1 + x − 3 − x − arcsin x−1 (put x = 1 + 2 cos ϕ)
2+ 1+x+ 3−x 2

2+√x+3 √ √ √ √ √
1+ x+4
( x + 3 + 4)( x + 4 − 2) − 4 log(1 + x + 4 ) + log( x + 3 + x + 4 )
√ √ √
(x+ a2 +x2 )2 2
x+ a2 + x2 4 + a2 log(x + a2 + x2 )
√ √ √
(x+ a2 +x2 )n+1 a2 +x2 )n−1
(x + a2 + x2 )n 2(n+1) + a2 (x+ 2(n−1) (n 6= 1)
h i
1 1 u2 +u+1
− √13 arctan 2u+1
p

3 6 log (u−1)2
√ , u = 3 1 + 1/x3 (put v = 1/x3 )
1+x3 3

H.W-Kayondo & D.W-Ddumba, Engineering Math I- Lecture Notes 339


CHAPTER 7. TECHNIQUES OF INTEGRATION

R
Example 7.12.39 Verify that f (x) dx = F (x) + C.

f (x) F (x)

 
xk+1 1
xk log x k+1 log x − k+1

log(1 + x2 ) x log(1 + x2 ) − 2x + 2 arctan x

x2 +a 1
(2x + (a − 1) arctan x) arctan x − log(1 + x2 )

x2 +1
arctan x 2

 
1
1− x e1/x xe1/x

x
cos2 x
x tan x + log | cos x|

1

√ x
e −1
2 arctan ex − 1

q
x+1
q
x+1
√ √ 
arctan x+3 (x + 2) arctan x+3 − log x+1+ x+3

q
x
q
x √ √
arcsin x+1 x arcsin x+1 − x + arctan x


x+ 1−x2 arcsin x
earcsin x 2 e

e−x
x(cos2 x)e−x

50 (3 − 5x) cos 2x + (4 + 10x) sin 2x − 25(x + 1)

2 2
(x2 + x + 1)e2x cos x ( 2x5 + 4x
25 + 39 2x
125 )e cos x + ( x5 − 3x
25 + 27 2x
125 )e sin x

Example 7.12.40 Let f be twice continuously differentiable in [0; 2π] and convex. Prove
R 2π
that 0 f (x) cos x dx ≥ 0.
R 2π R 2π
Integrate by parts twice to obtain 0 f (x) cos x dx = 0 f 00 (x)(1 − cos x) dx. Since
f is convex, f 00 ≥ 0 and the assertion follows.

Example 7.12.41 Find q √dx √ .


R
1+ 1+ x


q p
Put u = 1 + 1 + x, then x = (u2 −2)2 u4 and dx = (4u3 (u2 −2)2 +4u5 (u2 −2))u..

H.W-Kayondo & D.W-Ddumba, Engineering Math I- Lecture Notes 340


CHAPTER 7. TECHNIQUES OF INTEGRATION

Hence

(4u3 (u2 −2)2 +4u5 (u2 −2)) du


√dx
R R
q
√ = u
1+ 1+ x

R R
= 4 u2 (u2 − 2)2 du + 4 u4 (u2 − 2) du

R R
= 4 (u6 − 4u4 + 4u2 ) du + 4 (u6 − 2u4 ) du

R R R
= 8 u6 du − 24 u4 du + 16 u2 du

8 7 24 5 16 3
= 7
u − 5
u + 3
u +C

√ 7 √ 5 √ 3
q  q  q 
8
p 24
p 16
p
= 7
1+ 1+ x − 5
1+ 1+ x + 3
1 + 1 + x + C.

Note 7.12.1 Maple X doesn’t quite know how to do this problem!

Example 7.12.42 Let f : [a; b] → < a bounded integrable function for which ∀x ∈ [a; b],
f (a + b − x) = f (x). Demonstrate that

Z b Z b
a+b
xf (x) dx = · f (x) dx.
a 2 a

Rπ x sin x dx
Rπ x dx
Use this to calculate 0 1+cos2 x
and 0 1+sin x
.

H.W-Kayondo & D.W-Ddumba, Engineering Math I- Lecture Notes 341


Chapter 8

Improper Integrals

8.1 Introduction
It is a very natural question to ask if the area under the graph of

1
f :<→<:x→ (8.1)
x2

from 1 to ∞ is finite or not. Looking at the graph doesn’t really help all that much:
Clearly x12 goes to 0 as x gets larger, but we kind of have a situation where we have to
measure the area of a surface that is infinitely long and infinitely thin, so which one wins?
Sometimes the area will turn out to be infinite, and sometimes it will turn out to be finite.
In this section we will develop a way to decide in which case we are and what the final
answer to this question is.

8.2 Improper Integrals of First Kind


Rt
Let F (t) = a
f (t)dt with a constant and t ≥ a then

Z ∞
lim F (t) = f (x)dx
t→∞ a

is called an improper integral of 1st kind.

if any of the limits or both are infinity, then its an improper integral of 1st kind.

Rb
Also lim G(u) = −∞
f (x)dx is an improper integral of first kind.
t→−∞

342
CHAPTER 8. IMPROPER INTEGRALS

R∞
Theorem 8.2.1 If lim F (t) or lim G(u) exists and is finite then we say a
f (t)dt or
t→∞ u→−∞
Rb
−∞
f (x) dx is converging and its value is this limit.
Note 8.2.1 The improper integral of a function over an infinite interval [a, +∞[ is
defined as
Z +∞ Z t
f (x)dx = lim f (x)dx
a t→+∞ a

if the series on the right hand side exists and converges. Otherwise, the improper integral
is undefined. This case is also referred to as divergent.
Example 8.2.1 Find

Z ∞ Z t
dx dx
I= = lim
1 xa t→∞ 1 xa
t
x1−a

= lim
t→∞ 1 − a
1

t1−a 11−a
 
= lim −
t→∞ 1 − a 1−a

t1−a − 1
 
= lim
t→∞ 1−a


1
1 − a < 0, then I = 1−a ,

for 1 − a > 0, then I = ∞,

α = 1, thenI = ∞.

Example 8.2.2

Z 2 Z 2
ax
I= e = lim eax dx
−∞ u→−∞ u

 2
1 ax
= lim e
u→−∞ a u
 
1 2a 1 ua
= lim e − e
u→−∞ a a

H.W-Kayondo & D.W-Ddumba, Engineering Math I- Lecture Notes 343


CHAPTER 8. IMPROPER INTEGRALS


1 2a
a > 0, then I = a e ,

for a < 0, then I = −∞,

a = 0, thenI = undefined.

Example 8.2.3 ?

Z ∞ Z 0 Z ∞
dx dx dx
= +
−∞ 1 + x2 −∞ 1 + x2 0 1 + x2
0  −1 t
tan−1 x

= lim u
+ lim tan x 0
u→−∞ t→∞

0 − tan−1 u + lim tan−1 t − 0


   
= lim
u→−∞ t→∞

h π i hπ i
= 0 −− + −0
2 2
= π

So the improper integral converges to π

8.3 Convergence of integrals


8.3.1 Comparison test
Let f ,g be continuous functions on [a, ∞] such that 0 ≤ f (x) ≤ g(x) and x ≥ a,if
R∞ R∞
(i) a g(x) converges, so does a f (x)dx
R∞ R∞
(ii) a f (x)dx diverges so does a g(x)dx

Example 8.3.1 Investigate the convergence of the following integrals.

(a) For
Z ∞
2
e−(1+x ) dx
1

2
Realise 1 + x2 ≥ x ⇒ e−1+x ≤ e−x

H.W-Kayondo & D.W-Ddumba, Engineering Math I- Lecture Notes 344


CHAPTER 8. IMPROPER INTEGRALS

But since
Z ∞ Z t
−x
e dx = lim e−x dx
1 t→∞ 1

t
= lim −e−x 1

t→∞

= lim −e−t −− e−1


 
t→∞

= lim e−1 − e−t


 
t→∞

= e−1
R∞ R∞ 2)
thus converges (exists, not infinity); then since 1
e−x converges, so does 1
e−(1+x
(b) For
Z ∞
2
ex dx
a

2
but since x2 > x then ex > ex , but
Z ∞ Z t
x
e dx = lim ex dx
a t→∞ a

= lim [ex ]ta


t→∞

= lim et − ea
 
t→∞

R∞ R∞ 2
for a > 0 → I = ∞ diverges, so since a
ex diverges so a
ex diverges

8.4 Improper Integrals of Second Kind


(a) let f be continuous on [a, b] except at b, when f is infinite then
Z b Z t
f (x)dx = lim− f (x)dx
a t→b a

(b) let f be continuous on [a, b] except at a where the function is infinite, then
Z b Z b
f (x)dx = lim+ f (x)dx
a t→a t

H.W-Kayondo & D.W-Ddumba, Engineering Math I- Lecture Notes 345


CHAPTER 8. IMPROPER INTEGRALS

(c) If instead f is infinite at c ∈ (a, b) then

Z b Z c Z b
f (x)dx = f (x)dx + f (x)dx
a a c

Z t1 Z b
= lim− f (x)dx + lim+ f (x)dx
t1 →c a t2 →c t2

Parts a, b, c, are all integrals of 2nd kind.

An integral is of 2nd kind Improper if a function f (x) is not defined on any point of
[a, b], or c ∈ (a, b)

Example 8.4.1 Evaluate the improper integral

Z 0
dx
1
−1 (1 + x) 2

Realise at x = −1 the function is not defined. i.e denominator = 0 ⇒ 2nd order

Z 0 Z 0
dx dx
1 = lim+ 1
−1 (1 + x) 2 t→−1 t (1 + x) 2

1 0
h i
= lim+ 2(1 + x) 2
t→−1 t

h 1
i
= lim 2 − 2(1 + t) 2
t→−1+

= 2

Example 8.4.2 Evaluate

Z 3 Z 3
− 43 1
(x − 2) = 4 dx
0 0 (x − 2) 3

H.W-Kayondo & D.W-Ddumba, Engineering Math I- Lecture Notes 346


CHAPTER 8. IMPROPER INTEGRALS

not defined at x = 2 yet 2 ∈ (0, 3)

Z 3 Z 2 Z 3
1 1 1
4 dx = 4 dx + 4 dx
0 (x − 2) 3 0 (x − 2) 3 2 (x − 2) 3
Z t Z 3
1 1
= lim− 4 dx + lim+ 4 dx
t→2 0 (x − 2) 3 t→2 t (x − 2) 3
" #t  3
3 3
= lim − 1 + lim+ −
t→2− (x − 2) 3 t→2 x−2 t
0

= ∞
R∞ 1
Example 8.4.3 1 x2
dx 1st order

R1
Example 8.4.4 √1 dx 2nd order at x = 0
0 x

R0 1
Example 8.4.5 −∞ (1−x) 12
dx 1st order

Example 8.4.6

Z 1 Z t
1 1
√ dx = lim− √ dx
0 1 − x2 t→1 0 1 − x2
t
lim− sin−1 x 0

=
t→1

 −1
sin (1) − sin−1 (0)

=
π
=
2
R∞ 1
Example 8.4.7 Our original example 1 x2
dx turns out to be a convergent improper
integral:

Z +∞  n
dx 1
= lim −
1 x2 n→∞ x 1

1
= 1 − lim
n→∞ n

= 1

H.W-Kayondo & D.W-Ddumba, Engineering Math I- Lecture Notes 347


CHAPTER 8. IMPROPER INTEGRALS

Example 8.4.8 On the other hand, f (x) = √1 integrated over [1, +∞[ will turn out to
x
diverge:

Z +∞ √
dx
√ = lim [2 x]n1
1 x n→∞


= lim 2 n − 2
n→∞

= +∞
1
Example 8.4.9 Our original example f (x) = x2
turns out to be a divergent over [0, 1]:

Z 1 Z 1
dx dx
= lim+
0 x2 t→0 t x2
 1
1
= lim+ −
t→0 x t

1
= lim+ −1
t→0 t
= +∞

Example 8.4.10 On the other hand, f (x) = √1 integrated over [1, +∞[ will turn out
x
to converge:

Z 1 Z 1
dx dx
√ = lim+ √
0 x t→0 t x
 √ 1
= lim+ 2 x t
t→0


= 2 − lim+ 2 t
t→0

= 2

Note 8.4.1 It is of course not the case that only rational functions have improper
integrals. In fact, improper integral show up whenever an asymptote shows up. Look
e.g.:

Z π Z π
2 2 sin(x)
tan(x)dx = dx
0 0 cos(x)

H.W-Kayondo & D.W-Ddumba, Engineering Math I- Lecture Notes 348


CHAPTER 8. IMPROPER INTEGRALS

Z 0
dy
= − with y = cos(x)
1 y

= [− ln(y)]10
= +∞

Note 8.4.2

(a) What makes an integral improper?

An integral is improper if the limits of integration describe an infinite interval or


the interval on which we are integrating contains a point for which the integrand is
not continuous.

(b) Geometrically, what does it mean for an improper integral to converge? How about
algebraically?

Geometrically, it means that the area bounded by the curve is finite. we can see
this algebraically by noting that all of the limits exist (and are finite).

(c) Geometrically, what does it mean for an improper integral to diverge? How about
algebraically?

Geometrically, it often means that the area bounded by the curve is infinite.
Algebraically this means that one or more of the limits does not exist (or is
infinite).

Exercise 8.1 evaluate the following improper integrals


R∞
(1) 1 x1 dx = limt→∞ ln t DNE

R1 1
(2) 0 x
dx = limt→0 −10 ln t DNE
1
Since limx→0 x
Does not exist

Exercise 8.2 Show that


Z ∞
x−p dx
1

converges if p > 1 and diverges if p ≤ 1

H.W-Kayondo & D.W-Ddumba, Engineering Math I- Lecture Notes 349


CHAPTER 8. IMPROPER INTEGRALS

1
Exercise 8.3 Consider the function f (x) = 2x2 −x
defined on [0, 1]. It is easy to see that

f (x) is unbounded at x = 0 and x = 21 . Therefore, in order to study the integral

Z 1
1
dx
0 2x2 −x

we will write

1
Z 1 Z 0.4 Z Z 1
1 1 2 1 1
dx = dx + dx + dx
0 2x − x
2
0 2x − x
2
0.4 2x − x
2 1
2
2x2−x
Z 0.4 Z t Z 1
1 1 1
= lim dx + lim1 dx + lim1 dx
t→0 t 2x − x
2
t→ 2 0.4 2x − x
2
t→ 2 t 2x2−x

R1 1
We have forced in 0.4 not to have a case 2
0 2x2 −x
dx where both limits will make the
integrand improper.
Exercise 8.4 Test the convergence of the integral
Z ∞
1
dx
1 x3 +x

it would be time-consuming to try and evaluate this by anti differentiation. However, we


can determine whether the integral converges or not in a simpler way. Notice that, for
positive x , we have x > 0 which means that x3 + x > x3 . This means that x31+x < x13 .
Consequently,
Z ∞ Z ∞
1 1 1
3
dx < 3
dx =
1 x +x 1 x 2
R∞
because the integral 1 x13 dx was computed
R ∞ 1 (exist) in the example above. This means
that the integral we are interested in, 1 x3 +x dx , must converge as well.
Exercise 8.5
Z ∞
1 1
3
=
3 x 18

Exercise 8.6
Z −1
2 1
5
=−
−∞ x 2

H.W-Kayondo & D.W-Ddumba, Engineering Math I- Lecture Notes 350


CHAPTER 8. IMPROPER INTEGRALS

Exercise 8.7
Z ∞ Z 0 Z ∞
x x x
dx = dx + dx
−∞ (1 + x2 )2 −∞ (1 + x2 )2 0 (1 + x2 )2
Z 0 Z b
x x
= lim dx + lim dx
t→−∞ t (1 + x2 )2 b→∞ 0 (1 + x2 )2
0 b
−1 −1
 
= lim + lim
t→−∞ 2(1 + x2 ) t
b→∞ 2(1 + x2 )
0
   
1 1 1 1
= lim − + + lim − +
t→−∞ 2 2(1 + t2 ) b→∞ 2(1 + b2 ) 2

1 1
= − +
2 2
= 0
Exercise 8.8 Test whether the following improper integral converge? if so to what.
Z ∞
1
dx
0 1 + x2

Since x2 < 1 + x2
Z ∞ Z ∞
1 1
dx < dx
0 1 + x2 0 x2

and it converges
Z ∞
1 π
2
dx =
0 1+x 2

Exercise 8.9 It is also possible for an improper integral to diverge to infinity. In that
case, one may assign the value of 8 (or -8) to the integral. For instance
Z ∞
1
dx = ∞
1 x

However, other improper integrals may simply diverge in no particular direction, such as
Z ∞
x sin xdx
1

H.W-Kayondo & D.W-Ddumba, Engineering Math I- Lecture Notes 351


CHAPTER 8. IMPROPER INTEGRALS

which does not exist, even as an extended real number.


Exercise 8.10
Z 0
1 1
dx =
−∞ (5 − x) 3 50

Exercise 8.11
Z ∞
2 1
xe−x dx = −
−∞ 2

Exercise 8.12
Z 1
1
1 dx = 2
0 x2

Exercise 8.13
Z 1 Z 0 Z 1
1 1 1
dx = dx + dx
−1 x2 −1 x2 0 x2

Does not exist, thus does not converge.


Exercise 8.14
Z 4
1 √
dx = 2 5
2 (x − 3)3

Exercise 8.15
Z 1
xdx 3

3
=
0 1−x 2 4

Exercise 8.16 Test the improper integral for convergence

Z ∞
x
√ dx
0 x4+1
R∞ 1
diverges, since 0 x
dx diverges.
Exercise 8.17 Decide on the convergence or divergence of

Z ∞
1
dx
1 x ln x

H.W-Kayondo & D.W-Ddumba, Engineering Math I- Lecture Notes 352


CHAPTER 8. IMPROPER INTEGRALS

First notice that the denominator is equal to 0 when x = 1. Then the function inside the
integral sign is unbounded at x = 1. Hence we have two bad points 1 and ∞. So we must
split the integral and write

Z ∞ Z 2 Z ∞
1 1 1
dx = dx + dx
1 x ln x 1 x ln x 2 x ln x

Exercise 8.18 Since


Z ∞ Z ∞
sin x 1
dx < dx
1 x2 1 x2
R∞ sin x
1 x2
dx converges.
Exercise 8.19
Z 2
1
2 dx
0 (x − 1) 3
Z 1 Z 2
1 1
= 2 dx + 2 dx
0 (x − 1) 3 1 (x − 1) 3

= 3+3
= 6

Exercise 8.20
Z 1 Z 0 Z 1
x x x
dx = dx + dx
−1 (x − 1)2
2
−1 (x − 1)2
2
0 (x2 − 1)2
 0  s
1 2x x+1 1 2x x+1
= lim + ln + lim + ln
t→−1 4 1−x 2 x−1 t
s→1 4 1−x 2 x−1 0

Exercise 8.21

π
Z π Z Z π
sin x 2 sin x sin x
4 dx = 4 dx + 4 dx
0 (cos x) 3 0 (cos x) 3 π
2
(cos x) 3
h −1
it h −1

= limπ 3(cos x) 3 + limπ 3(cos x) 3
t→ 2 0 s→ 2 s

= DN E

H.W-Kayondo & D.W-Ddumba, Engineering Math I- Lecture Notes 353


CHAPTER 8. IMPROPER INTEGRALS

Exercise 8.22 Evaluate

Z ∞  t
ln x 1 + ln x
dx = lim −
1 x2 t→∞ x 1

= 1

Exercise 8.23 Evaluate

Z 1 Z 0 Z 1
1 1 1
p dx = √ dx + √ dx
−1 |x| −1 −x 0 x
Z 0 Z 1
−1 −1
= (−x) 2 dx + (x) 2

−1 0

t 1
= lim −(−x)1/2 −1 + lim (x)1/2 s
 
t→0 s→0

= 2

Exercise 8.24 Use the comparison test to determine the convergence of the improper
integral
Z ∞
1
dx
1 x + ex
3

We know that for x > 0,


1 1
0 < <
x3 + e x x3

So
Z ∞ Z ∞ Z ∞
1 1
0 dx < dx < dx
1 1 x + ex
3
1 x3

Since the latter integral converges by the p-test, the original integral must also converge
by the comparison test.

Exercise 8.25 Use the comparison test to determine the convergence of the improper
integral
Z ∞
1
dx
2 x − ln x

H.W-Kayondo & D.W-Ddumba, Engineering Math I- Lecture Notes 354


CHAPTER 8. IMPROPER INTEGRALS

We know that for x > 0,


1 1
0 < <
x x − ln x
Z ∞ Z ∞ Z ∞
1 1
0 dx < dx < dx
2 2 x 2 x − ln x

Since the latter integral diverges, the original integral must also diverge by the comparison
test.
Exercise 8.26
Z ∞ Z ∞
ln x 1
dx < dx
2 x3 2 x3
converges
Example 8.4.11 The following are good examples of improper integrals of the first kind.
R∞
1. −2 sin xdx DNE

R∞ 1
2. 1 x
dx

R0 1
3. −∞ x−2
dx

R∞ 1
4. −∞ 1+x2
dx
Example 8.4.12 The following are good examples of improper integrals of the second
kind.
R 4 dx 1
1. 0 x−3 dx, since f (3) = 3−3 , hence unbounded at x = 3 ∈ [0, 4].

R41 1
2. 0 x
dx, since f (0) = 0
is unbounded.

R1 1 1
3. 0 x−1
dx, since f (1) = 1−1
is unbounded.

Exercise 8.27 Evaluate the improper integral


Z ∞
1
dx
2 (x + 1)2
R∞
Since lim F (t) exists and is equal to 13 , we say the improper integral, 2
1
(x+1)2
dx con-
t→∞

verges to 13 .

H.W-Kayondo & D.W-Ddumba, Engineering Math I- Lecture Notes 355


CHAPTER 8. IMPROPER INTEGRALS

R∞
Example 8.4.13 Check for convergence of the improper integral 1
e−3x dx
Since
Z ∞ Z t
−3x
e dx = lim e−3x dx
1 t→∞ 1

 t
1 −3x
= lim − e
t→∞ 3 1

 t
−1 1
= lim 3x
3 t→∞ e 1
 
1 1 1 1
= − lim − =
3 t→∞ e3 t e3 3e3

1
Thus, the improper integral converges to 3e3
.
R∞ dx
Example 8.4.14 Using comparison test, show that, 0 ex +1
converges.

R∞ R∞
Comparing withRthe convergent integral 1
e−x dx. Thus by comparison, since 0
e−x dx

converges, so is 0 ex1+1 dx.
Example 8.4.15 Determine if the following integral is convergent or divergent.


cos2 x
Z
dx
2 x2

Lets take a second and think about how the Comparison Test works. If this integral
is convergent then well need to find a larger function that also converges on the same
interval. Likewise, if this integral is divergent then well need to find a smaller function
that also diverges.

So, it seems like it would be nice to have some idea as to whether the integral converges
or diverges ahead of time so we will know whether we will need to look for a larger (and
convergent) function or a smaller (and divergent) function.

To get the guess for this function lets notice that the numerator is nice and bounded and
simply wont get too large. Therefore, it seems likely that the denominator will determine
the convergence/divergence of this integral and we know that
Z ∞
1
dx
2 x2

H.W-Kayondo & D.W-Ddumba, Engineering Math I- Lecture Notes 356


CHAPTER 8. IMPROPER INTEGRALS

converges since p ≥ 2. So we now know that we need to find a function that is larger than

cos2 x
x2

and also converges. Making a fraction larger is actually a fairly simple process. We can
either make the numerator larger or we can make the denominator smaller. In this case
cant do a lot about the denominator. However we can use the fact that 0 ≤ cos2 x ≤ 1
to make the numerator larger (i.e. well replace the cosine with something we know to be
larger, namely 1). So,
cos2 x 1
2
≤ 2
x x

Now, as weve already noted


Z ∞
1
dx
2 x2

converges and so by the Comparison Test we know that


cos2 x
Z
dx
2 x2

must also converge.


Example 8.4.16 Determine if the following integral is convergent or divergent.

Z ∞
1
dx
3 x + ex

Lets first take a guess about the convergence of this integral. As noted after the fact in
the last section about
Z ∞
1
dx
a xp

if the integrand goes to zero faster than x1 then the integral will probably converge.
Now, weve got an exponential in the denominator which is approaching infinity much
faster than the x and so it looks like this integral should probably converge.

So, we need a larger function that will also converge. In this case we cant really make the
numerator larger and so well need to make the denominator smaller in order to make the

H.W-Kayondo & D.W-Ddumba, Engineering Math I- Lecture Notes 357


CHAPTER 8. IMPROPER INTEGRALS

function larger as a whole. We will need to be careful however. There are two ways to do
this and only one, in this case only one, of them will work for us.

First, notice that since the lower limit of integration is 3 we can say that x ≥ 3 > 0 and
we know that exponentials are always positive. So, the denominator is the sum of two
positive terms and if we were to drop one of them the denominator would get smaller.
This would in turn make the function larger.

The question then is which one to drop? Lets first drop the exponential. Doing this gives,

1 1
x
<
x+e x

This is a problem however, since


Z ∞
1
dx
3 x

diverges by the fact. We’ve got a larger function that is divergent. This doesnt say
anything about the smaller function. Therefore, we chose the wrong one to drop.

Lets try it again and this time lets drop the x.

1 1
x
< x = e−x
x+e e

Also,
Z ∞
e−x dx = lim −e−t + e−3 = e−3
 
3 t→∞

R∞
So, 3
e−x dx is convergent. Therefore, by the Comparison test
Z ∞
1
dx
3 x + ex

is also convergent.
Example 8.4.17 Determine if the following integral is convergent or divergent.
Z ∞
1
dx
3 x + e−x

Diverges

H.W-Kayondo & D.W-Ddumba, Engineering Math I- Lecture Notes 358


CHAPTER 8. IMPROPER INTEGRALS

Example 8.4.18 Determine whether the improper integral

0
ex
Z
dx
−∞ 1 + ex

converges or diverges. If it converges, find its value.

0 0
ex ex
Z Z
dx = lim dx
−∞ 1 + ex t→−∞ t 1 + ex

= lim [ln(1 + ex )]0t function and its derivative


t→−∞

ln 2 − ln(1 + et )
 
= lim
t→−∞

= ln 2 − ln 1
= ln 2

So the integral converges.


R∞
Example 8.4.19 Use the comparison
R∞ test to determine whether 3
x3 ln x dx converges
or diverges. It diverges since 3 x dx divereges.

8.5 Applications of improper integrals.


Improper integrals can be used to find areas, volumes and lateral surface areas.

Definition 8.5.1 The area of the region above by the graph of y = f (x) and below by the
X-axis and extending to the right of a is defined by, Area

Z ∞
A= f (x)dx
a

(if the integral converges).

Example 8.5.1 Find the area of the region bounded above by the graph of

H.W-Kayondo & D.W-Ddumba, Engineering Math I- Lecture Notes 359


CHAPTER 8. IMPROPER INTEGRALS

1
y = f (x) = x3
and below by the x-axis and extending to the right of x = 1.

Z ∞
1
A = dx
1 x3
t  t
−1
Z
1 1
= lim dx = lim
t→∞ 1 x3 2 t→∞ x2 1
 
1 1 1
= lim 1 − 2 = sq units.
2 t→∞ t 2

Note 8.5.1 Area problem with definite integrals is considered in Chapter 9


Example 8.5.2 Find the volume of the solid obtained by revolving the region R, about
the X-axis
Z ∞  2 Z ∞
1 1
Volume, V = π dx = π dx.
1 x3 1 x6
Z t  t
1 1
= π lim dx = π lim − 5
t→∞ 1 x6 →∞ 5x 1
 
π 1 π
= lim 1 − 5 = cubic units.
5 t→∞ t 5

Note 8.5.2 Volume problems with a definite integral are considered in chapter 9.

Example 8.5.3 [Putnam, 1995] For what pairs (a, b) of positive real numbers does the
improper integral

∞ √ √ √ √
Z q q 
x+a− x− x − x − b dx
b

converge?
The integral converges iff a = b. Use the fact that (1 + x)1/2 = 1 + x/2 + O(x2 ) for
|x| < 1.
Now,
√ √ p
x + a − x = x1/2 ( 1 + a/x − 1) = x1/2 (1 + a/2x + O(x−2 ))
hence
q√

x + a − x = x1/4 (1 + a/4x + O(x−2 )

H.W-Kayondo & D.W-Ddumba, Engineering Math I- Lecture Notes 360


CHAPTER 8. IMPROPER INTEGRALS

and similarly
q
√ √
x− x − b = x1/4 (1 + b/4x + O(x−2 ).
Hence the integral we’re looking at is

Z ∞
x1/4 ((a − b)/4x + O(x−2 ) dx.
b

The term x1/4 O(x−2 ) is bounded by a constant times x−7/4 , whose integral converges.
Thus we only have to decide whether x−3/4 (a − b)/4 converges. But x−3/4 has divergent
integral, so we get convergence if and only if a = b (in which case the integral telescopes
anyway).

Example 8.5.4 [Putnam, 1997]


Evaluate


x3 x5 x7 x2 x4 x6
Z   
x− + − + ··· 1 + 2 + 2 2 + 2 2 2 + · · · x..
0 2 2·4 2·4·6 2 2 ·4 2 ·4 ·6

Note that the series on the left is simply x exp(−x2 /2). By integration by parts,

Z ∞ Z ∞
2n+1 −x2 /2 2 /2
x e dx = 2n x2n−1 e−x dx
0 0

and so by induction,
Z ∞
2 /2
x2n+1 e−x dx = 2 × 4 × · · · × 2n.
0

Thus the desired integral is simply


X 1 √
n n!
= e.
n=0
2

Example 8.5.5 [Putnam, 2000] Show that the improper integral

Z B
lim sin(x) sin(x2 ) dx
B→+∞ 0

H.W-Kayondo & D.W-Ddumba, Engineering Math I- Lecture Notes 361


CHAPTER 8. IMPROPER INTEGRALS

converges.
We use integration by parts:
Z B Z B
2 sin x
sin x sin x dx = sin x2 (2x dx)
0 0 2x
B Z B  
sin x
2 cos x sin x
= − cos x + − 2
cos x2 dx.
2x 0 0 2x 2x

Now sin
2x
x
cos x2 tends to 0 as B → +∞, and the integral of sin x
2x2
cos x2 converges absolutely
by comparison with 1/x2 . Thus it suffices to note that

Z B
cos x cos x
cos x2 dx = cos x2 (2x dx)
0 2x 4x2

cos x B Z B 2x cos x − sin x


2
= sin x − sin x2 dx,
4x2 0 0 4x 3

and that the final integral converges absolutely by comparison to 1/x3 .


An alternate approach is to first rewrite sin x sin x2 as 21 (cos(x2 − x) − cos(x2 + x).
Then

B
B Z B
2 sin(x2 + x)
Z
2 2x + 1
cos(x + x) dx = − − dx
0 sin(x2 + x) 0 0 (2x + 1)2

RB
converges absolutely, and cos(x2 − x) can be treated similarly.
0

Example 8.5.6 Prove that improper integral I = 0 log sin x dx converges and that
I = −π log 2.
From sin x = 2 sin x2 cos x2 we get
Z π Z π Z π
x x
I = log 2 dx + log sin dx + log cos dx
0 0 2 0 2
Z π/2 Z π/2
= π log 2 + 2 log sin y dy + 2 log cos y dy.
0 0

π π

Setting y = 2
− u and using sin(π − u) = sin u = cos 2
− u we see that

Z π/2 Z π/2
log sin y dy = log cos y dy
0 0

H.W-Kayondo & D.W-Ddumba, Engineering Math I- Lecture Notes 362


CHAPTER 8. IMPROPER INTEGRALS

Z π/2 Z π/2 Z π
⇒ 2 log sin y dy = (log sin u + log sin(π − x)) du = log sin u du = I,
0 0 0

from where
I = π log 2 + 2I ⇒ I = −π log 2.

H.W-Kayondo & D.W-Ddumba, Engineering Math I- Lecture Notes 363


Chapter 9

Applications of definite integrals

9.1 Area Between Curves


The are between two curves is given by

Z x=b
A = (y1 − y2 ) dx (9.1)
x=a

Z y=d
A = (x1 − x2 ) dy (9.2)
y=c

Example 9.1.1 Find the area between y = x and y = −x(x − 4)

? Graph both functions in your calculator or on graph paper to see what the area
looks like.

? The area between the functions can be found by summing rectangular areas whose
heights are given by the upper function minus the lower function, in this case,
−x(x − 4) − x. The widths of the rectangles are ∆x.

? Find the intersection of the two functions to find the limits of integration.

x = −x(x − 4)
x = −x2 + 4x
0 = −x2 + 3x
0 = −x(x − 3)
x = 0, 3

364
CHAPTER 9. APPLICATIONS OF DEFINITE INTEGRALS

Z 3 Z 3
[−x(x − 4) − x] dx = (−x2 + 3x)dx
0 0

9
= square units
2

Example 9.1.2 Find the area enclosed by the curve y = x2 − 2x the x- axis and the
lines x = 0 and x = 4

H.W-Kayondo & D.W-Ddumba, Engineering Math I- Lecture Notes 365


CHAPTER 9. APPLICATIONS OF DEFINITE INTEGRALS

Z 2 Z 4
= (y2 − y1 )dx + (y1 − y2 )dx
0 2

Z 2 Z 4
0 − (x2 − 2x) dx + (x2 − 2x) − 0 dx
   
=
0 2

Z 2 Z 4
2
= (2x − x )dx + (x2 − 2x)dx
0 2

40
=
3

1
Example 9.1.3 ? Find the area enclosed by the curve y = x(x2 − 1) 2 , the x-axis and
the lines x = ±2

Z −1 Z 0 Z 1 Z 2
= (y2 − y1 )dx + (y1 − y2 )dx + (y2 − y1 )dx + (y1 − y2 )dx
−2 −1 0 1

Z −1 Z 0 Z 1 Z 2
1 1 1 1
2 2 2
= −x(x − 1) dx +
2 x(x − 1) dx + 2 −x(x − 1) + 2 x(x2 − 1) 2 dx
−2 −1 0 1

9√ 3 3 9√
= 3+ + + 3
8 8 8 8
3 √
= (3 3 + 1)
4

H.W-Kayondo & D.W-Ddumba, Engineering Math I- Lecture Notes 366


CHAPTER 9. APPLICATIONS OF DEFINITE INTEGRALS

Example 9.1.4 Find the area between y = (x − 2)2 and y = 4

32
square units
3

Example 9.1.5 Find the area between x = −4y − y 2 and x = 0

32
square units
3

Example 9.1.6 Find the area enclosed by the line y = x−1 and the parabola y 2 = 2x+6.

Example 9.1.7 Estimate the area enclosed by the loop of the curve with parametric
equations
x = t2 + t + 1 , y = 3t4 − 8t3 − 18t2 + 25

Example 9.1.8 Find the area underneath the curve y = x2 + 2 from x = 1 to x = 2.

13
square units
3

A sketch is very important

H.W-Kayondo & D.W-Ddumba, Engineering Math I- Lecture Notes 367


CHAPTER 9. APPLICATIONS OF DEFINITE INTEGRALS

Example 9.1.9 Find the area bounded by y = x2 − 4, the x-axis and the lines x = −1
and x = 2
| − 9| = 9

1
Example 9.1.10 Find the area between y = x2 and y = x 2

Z 1 √
x − x2 dx = 0.3333335436 square units
0

Note 9.1.1 Before moving on to the next example, there are a couple of important
things to note.

First, in almost all of these problems a graph is pretty much required. Often the bounding
region, which will give the limits of integration, is difficult to determine without a graph.

Also, it can often be difficult to determine which of the functions is the upper function
and with is the lower function without a graph. This is especially true in cases like the
last example where the answer to that question actually depended upon the range of xs
that we were using.

Finally, unlike the area under a curve that we looked at in the previous chapter the area
between two curves will always be positive. If we get a negative number or zero we can
be sure that weve made a mistake somewhere and will need to go back and find it.

H.W-Kayondo & D.W-Ddumba, Engineering Math I- Lecture Notes 368


CHAPTER 9. APPLICATIONS OF DEFINITE INTEGRALS

Note as well that sometimes instead of saying region enclosed by we will say region
bounded by. They mean the same thing.

Example 9.1.11 What is the area bounded by the curve y = x3 , x = −2 and x = 1?

Z 0 Z 1
3 1
x dx + x3 dx = 4 +
−2 0 4


Example 9.1.12 Find the area of the region bounded by the curve y = x − 1 the
y-axis and the lines y = 1 and y = 5.

Z 5
1
y 2 + 1 dy = 45
1 3

2
Example 9.1.13 Determine the area of the region bounded by y = xe−x , y = x+1, x =
2 , and the y-axis.

Z 2
2
[(x + 1) − xe−x ] dx = 3.5092
0

Example 9.1.14 Find the area between the curves y = x2 + 5x and y = 3 − x2 between
x = −2 and x = 0.
2
A = 10 square units
3

H.W-Kayondo & D.W-Ddumba, Engineering Math I- Lecture Notes 369


CHAPTER 9. APPLICATIONS OF DEFINITE INTEGRALS

Example 9.1.15 Find the area bounded by y = x3 , x = 0 and y = 3

A = 3.245 square units

Exercise 9.1 Find the area bounded by the curves y = x2 + 5x and y = 3 − x2 .

A = 14.29 square units

Exercise 9.2 Find the area bounded by the curves y = x2 , y = 2 − x and y = 1

19
A= square units
6

H.W-Kayondo & D.W-Ddumba, Engineering Math I- Lecture Notes 370


CHAPTER 9. APPLICATIONS OF DEFINITE INTEGRALS

Example 9.1.16 Determine the area of the region bounded by y = 2x2 + 10 and y =
4x + 16

Z 3
64
[(4x + 16) − (2x2 + 10)] dx =
−1 3

Example 9.1.17 since y = x2 is certainly always positive (or at least non-negative,


which is really enough), the area ‘under the curve’ (and, implicitly, above the x-axis)
between x = 0 and x = 1 is just

1
x3 1 13 − 03
Z
1
x2 dx = [ ]0 = =
0 3 3 3

More generally, the area below y = f (x), above y = g(x), and between x = a and x = b
is
Z b
area... = f (x) − g(x) dx
a

Z right limit
= (upper curve - lower curve) dx
left limit

It is important that f (x) ≥ g(x) throughout the interval [a, b].

H.W-Kayondo & D.W-Ddumba, Engineering Math I- Lecture Notes 371


CHAPTER 9. APPLICATIONS OF DEFINITE INTEGRALS

For example, the area below y = ex and above y = x, and between x = 0 and x = 2 is

2
x2 2
Z
ex − x dx = [ex − ] = (e2 − 2) − (e0 − 0) = e2 + 1
0 2 0

since it really is true that ex ≥ x on the interval [0, 2].

As a person might be wondering, in general it may be not so easy to tell whether the
graph of one curve is above or below another. The procedure to examine the situation is
as follows: given two functions f, g, to find the intervals where f (x) ≤ g(x) and vice-versa:

• Find where the graphs cross by solving f (x) = g(x) for x to find the x-coordinates of
the points of intersection.
• Between any two solutions x1 , x2 of f (x) = g(x) (and also to the left and right of the
left-most and right-most solutions!), plug in one auxiliary point of your choosing to see
which function is larger.

Of course, this procedure works for a similar reason that the first derivative test for local
minima and maxima worked: we implicitly assume that the f and g are continuous, so
if the graph of one is above the graph of the other, then the situation can’t reverse itself
without the graphs actually crossing.

As an example, and as an example of a certain delicacy of wording, consider the problem


to find the area between y = x and y = x2 with 0 ≤ x ≤ 2. To find where y = x and y = x2
cross, solve x = x2 : we find solutions x = 0, 1. In the present problem we don’t care what
is happening to the left of 0. Plugging in the value 1/2 as auxiliary point between 0 and
1, we get 21 ≥ ( 12 )2 , so we see that in [0, 1] the curve y = x is the higher. To the right of 1
we plug in the auxiliary point 2, obtaining 22 ≥ 2, so the curve y = x2 is higher there.

Therefore, the area between the two curves has to be broken into two parts:
Z 1 Z 2
2
area = (x − x ) dx + (x2 − x) dx
0 1

since we must always be integrating in the form

Z right
higher - lower dx
left

In some cases the ’side’ boundaries are redundant or only implied. For example, the
question might be to find the area between the curves y = 2 − x and y = x2 . What is

H.W-Kayondo & D.W-Ddumba, Engineering Math I- Lecture Notes 372


CHAPTER 9. APPLICATIONS OF DEFINITE INTEGRALS

implied here is that these two curves themselves enclose one or more finite pieces of area,
without the need of any ‘side’ boundaries of the form x = a. First, we need to see where
the two curves intersect, by solving 2 − x = x2 : the solutions are x = −2, 1. So we infer
that we are supposed to find the area from x = −2 to x = 1, and that the two curves close
up around this chunk of area without any need of assistance from vertical lines x = a.
We need to find which curve is higher: plugging in the point 0 between −2 and 1, we see
that y = 2 − x is higher. Thus, the desired integral is

Z 1
area... = (2 − x) − x2 dx
−2

Exercise 9.3 Find the area between the curves y = x2 and y = 2x + 3.

Exercise 9.4 Find the area of the region bounded vertically by y = x2 and y = x + 2
and bounded horizontally by x = −1 and x = 3.

Exercise 9.5 Find the area between the curves y = x2 and y = 8 + 6x − x2 .

Exercise 9.6 Find the area between the curves y = x2 + 5 and y = x + 7.

H.W-Kayondo & D.W-Ddumba, Engineering Math I- Lecture Notes 373


CHAPTER 9. APPLICATIONS OF DEFINITE INTEGRALS

Example 9.1.18 Determine the area of the region bounded by y = 2x2 +10 , y = 4x+16
, x = −2 and x = 5

Z −1 Z 3 Z 5
2 2
A = (2x + 10) − (4x + 16) dx + (4x + 16) − (2x + 10) dx + (2x2 + 10) − (4x + 16) dx
−2 −1 3

14 64 64
= + +
3 3 3
142
=
3

Example 9.1.19 Determine the area of the region enclosed by y = sin x, y = cos x, x =
π
2
, and the y-axis.

Z π Z π
4 2
A = [cos x − sin x]dx + [sin x − cos x] dx
π
0 4


= 2 2−2

H.W-Kayondo & D.W-Ddumba, Engineering Math I- Lecture Notes 374


CHAPTER 9. APPLICATIONS OF DEFINITE INTEGRALS

1 2
Example 9.1.20 ? Determine the area of the region enclosed by x = 2
y − 3, and
y = x − 1.

Z −1 h√ √ i Z 5 h√ i
A = 2x + 6 − (− 2x + 6) dx + 2x + 6 − (x − 1) dx
−3 −1

= 18

While these integrals arent terribly difficult they are more difficult than they need to be.
Recall that there is another formula for determining the area. It is,

Z  4  
1 2
A= (y + 1) − y −3 dy = 18
−2 2

This is the same that we got using the first formula and this was definitely easier than
the first method.

So, in this last example weve seen a case where we could use either formula to find the
area. However, the second was definitely easier.

Students often come into a calculus class with the idea that the only easy way to work
with functions is to use them in the form y = f (x). However, as weve seen in this previous
example there are definitely times when it will be easier to work with functions in the
form x = f (y). In fact, there are going to be occasions when this will be the only way
in which a problem can be worked so make sure that you can deal with functions in this
form.

H.W-Kayondo & D.W-Ddumba, Engineering Math I- Lecture Notes 375


CHAPTER 9. APPLICATIONS OF DEFINITE INTEGRALS

Example 9.1.21 Determine the area of the region bounded by x = −y 2 + 10 and


x = (y − 2)3 .

Z 3
64
A= (−y 2 + 10) − (y − 2)3 dy =
−1 3

Definition 9.1.1 ?? Let f be a function which is continuous on the closed interval [a, b]
which represents force. The work in moving a particle from x = a to x = b along a
straight line is the integral
Z b
f dx
a

9.2 Volumes of Solids of revolution


In this section we consider applications of the definite integral to calculating geometric
quantities such as volumes. The idea will be to dissect the three dimensional objects
into pieces that resemble disks or shells, whose volumes we can approximate with simple
formulae. The volume of the entire object is obtained by summing up volumes of a stack
of disks or a set of embedded shells, and considering the limit as the thickness of the
dissection cuts gets thinner.

1.The volume of a cylinder of height h having circular base of radius r, is

Vcylinder = πr2 h

H.W-Kayondo & D.W-Ddumba, Engineering Math I- Lecture Notes 376


CHAPTER 9. APPLICATIONS OF DEFINITE INTEGRALS

2.The volume of a circular disk of thickness t , and radius r as a special case of the above,
is
Vdisk = πr2 t
3. The volume of a cylindrical shell of height h, with circular radius r and small thickness
t is
Vshell = 2πrht
(This approximation holds for t << r.)
Definition 9.2.1 If we rotate the plane region described by f (x) ≤ y ≤ g(x) and
a ≤ x ≤ b around the x-axis, the volume of the resulting solid is
Z b Z b
π g(x)2 − f (x)2 = π y12 − y22 dx
   
V = (9.3)
a a

Z right limit
= π(upper curve2 − lower curve2 ) dx
left limit

It is necessary to suppose that f (x) ≥ 0 for this to be right.


This formula comes from viewing the whole thing as sliced up into slices of thickness dx,
so that each slice is a disk of radius g(x) with a smaller disk of radius f (x) removed from
it. Then we use the formula
area of disk = π radius2
and ‘add them all up’. The hypothesis that f (x) ≥ 0 is necessary to avoid different pieces
of the solid ‘overlap’ each other by accident, thus counting the same chunk of volume
twice.
Definition 9.2.2 If we rotate the plane region described by f (x) ≤ y ≤ g(x) and
a ≤ x ≤ b around the y-axis (instead of the x-axis), the volume of the resulting solid is

Z x=b Z x=b
V = 2πx [g(x) − f (x)] dx = 2πx [y1 − y2 ] dx (9.4)
x=a x=a

Z y=d Z y=d
π g(y)2 − f (y)2 dy = π x21 − x22 dy
   
or V = (9.5)
y=c y=c

This first formula comes from viewing the whole thing as sliced up into thin cylindrical
shells of thickness dx encircling the y-axis, of radius x and of height g(x) − f (x). The
volume of each one is
(area of cylinder of height g(x) − f (x) and radius x) · dx = 2πx(g(x) − f (x)) dx
and ‘add them all up’ in the integral.

H.W-Kayondo & D.W-Ddumba, Engineering Math I- Lecture Notes 377


CHAPTER 9. APPLICATIONS OF DEFINITE INTEGRALS

Example 9.2.1 [Cone]


Take the line y = mx on [0, h] and spin it round the x-axis so as to produce a cone of
height h and ‘semi-angle’ α, where tanα = m.
The volume of this cone is, by our formula,
Z h  h
2 2 2 21 3
V = π[m x − 0 ]dx = πm 3 x = 13 πm2 h3
0 0

R
If R is the radius of the base of the cone then m = tan α = h
so, with a bit of
rearranging, we get
V = 13 πR2 h = 13 base height
Example 9.2.2 [Sphere] √
Take the semicircle y = r2 − x2 on [−r, r] and spin it round the x-axis. We get, as
our solid of revolution, a Sphere of radius r.
Our formula tells us that the volume of this sphere is

Z r  r
π (r2 − x2 ) − 02 dx = 2 1
πx3 = 43 πr3
 
V = πr x − 3
−r −r

So the volume of a sphere of radius r is

V = 34 πr3
1
Example 9.2.3 ? Consider the funnel formed by taking the curve y = x
and rotating
it round the x-axis on the interval [1, a],
The volume of this funnel is

Z a    
1 2
h π ia 1
V = π 2 − 0 dx = − =π 1−
1 x x 1 a

π
Now notice that, as a → ∞, this volume tends to the finite value π (because a
→ 0).
We write

Z ∞ Z a
π π  π 
dx = lim dx = lim π − =π
1 x2 a→∞ 1 x2 a→∞ a

Example 9.2.4 ? For the region bounded by f (x) = 1 − x2 and the x-axis on 0 ≤
x ≤ 1, find the volume of revolution.

Z 1

(1 − x2 ) − 02 dx =
 
V =
0 3

H.W-Kayondo & D.W-Ddumba, Engineering Math I- Lecture Notes 378


CHAPTER 9. APPLICATIONS OF DEFINITE INTEGRALS

Example 9.2.5 let’s consider the region 0 ≤ x ≤ 1 and x2 ≤ y ≤ x. Note that for
0 ≤ x ≤ 1 it really is the case that x2 ≤ y ≤ x, so y = x is the upper curve of the two,
and y = x2 is the lower curve of the two. Invoking the formula above, the volume of the
solid obtained by rotating this plane region around the x-axis is

Z right
volume = π(y12 − y22 ) dx
left

Z 1 1
π (x)2 − (x2 )2 dx = π x3 /3 − x5 /5 0 = π(1/3 − 1/5)
 
=
0

On the other hand, if we rotate this around the y-axis instead, then

Z right
volume = 2πx(y1 − y2 ) dx
left

1 1 1
2x3 2x4
 3
2x4
Z Z  
2 2x 2 1 π
= 2πx(x − x ) dx = π − dx = π − =π − =
0 0 3 4 3 4 0 3 2 6

H.W-Kayondo & D.W-Ddumba, Engineering Math I- Lecture Notes 379


CHAPTER 9. APPLICATIONS OF DEFINITE INTEGRALS

Simillary we can use

Z y=d
V = π(x21 − x22 ) dy
y=c

Z y=1 h√ i
= ( y)2 − y 2 dy
y=0

π
=
6

Example 9.2.6 ? The region bounded by y = x2 , the y-axis, and y = 4 is rotated about
the y-axis. Find the volume of the resulting solid.

H.W-Kayondo & D.W-Ddumba, Engineering Math I- Lecture Notes 380


CHAPTER 9. APPLICATIONS OF DEFINITE INTEGRALS

Z y=d
V = π(x21 − x22 ) dy
y=c

Z y=4 h√ i
= ( y)2 − 02 dy
y=0

= 8π
Z x=b
or V = = 2πx(y1 − y2 ) dx
x=a

Z 2
2πx 4 − x2 dx
 
=
0

Z 2
8x − 2x3 dx

= π
0

= 8π

Exercise 9.7 Find the volume of the solid obtained by rotating the region 0 ≤ x ≤
1, 0 ≤ y ≤ x around the y-axis.

Exercise 9.8 Find the volume of the solid obtained by rotating the region 0 ≤ x ≤
1, 0 ≤ y ≤ x around the x-axis.

Exercise 9.9 Set up the integral which expresses the volume of the doughnut obtained
by rotating the region (x − 2)2 + y 2 ≤ 1 around the y-axis.

9.3 Lengths of Curves

H.W-Kayondo & D.W-Ddumba, Engineering Math I- Lecture Notes 381


CHAPTER 9. APPLICATIONS OF DEFINITE INTEGRALS

In the triangle shown, by the Pythagorean theorem we have the length of the sloped
side given as follows:

∆l2 = ∆x2 + ∆y 2
p
∆l = ∆x2 + ∆y 2
r !
∆y 2
= 1+ ∆x
∆x2
s  2
∆y
= 1+ ∆x
∆x

We now consider a curve given by some function

y = f (x) a < x < b

We will approximate this curve by a set of line segments. To obtain these, we have selected
some step size ∆x along the x axis, and placed points on the curve at each of these x
values. We connect the points with straight line segments, and determine the length of
those segments. (The total length of the segments is only an approximation of the length
of the curve, but as the subdivision gets finer and finer, we will arrive at the true total
length of the curve by summing up the lengths of all the small line segments fit to it.)
According to our remarks, above, the length of this segment is given by

s  2
∆y
∆l = 1+ ∆x
∆x

As the step size is made smaller and smaller ∆x → dx, ∆y → dy and

s  2
dy
∆l = 1+ dx
dx

dy
We recognize the ratio in this root as the derivative, dx
. If our curve is given by a function
y = f (x) then we can rewrite this as

q
dl = 1 + (f 0 (x))2 dx

H.W-Kayondo & D.W-Ddumba, Engineering Math I- Lecture Notes 382


CHAPTER 9. APPLICATIONS OF DEFINITE INTEGRALS

Thus, the length of the entire curve is obtained from summing (i.e. adding up) these
small pieces, i.e.

Z x=b q
L = 1 + (f 0 (x))2 dx (9.6)
x=a

Z y=d q
or L = 1 + (f 0 (y))2 dy (9.7)
y=c

π
Example 9.3.1 Determine the length of y = ln(sec x) between 0 ≤ x ≤ 4

dy sec x tan x
= = tan x
dx sec x
Z πp
4
L = 1 + tan2 x dx
0

π
Z
4 √
= sec2 x dx
0

Z π
4
= sec x dx
0

π
= ln [sec x + tan x]04

= ln( 2 + 1)

3
Example 9.3.2 ? Determine the length of x = 23 (y − 1) 2 between 1 ≤ y ≤ 4

There is a very common mistake that students make in problems of this type. Many
students see that the function is in the form x = h(y) and they immediately decide that
it will be too difficult to work with it in that form so they solve for y to get the function
into the form y = f (x). While that can be done here it will lead to a messier integral for
us to deal with.

Sometimes its just easier to work with functions in the form x = h(y) . In fact, if you
can work with functions in the form y = f (x) then you can work with functions in the
form x = h(y). There really isnt a difference between the two so dont get excited about

H.W-Kayondo & D.W-Ddumba, Engineering Math I- Lecture Notes 383


CHAPTER 9. APPLICATIONS OF DEFINITE INTEGRALS

functions in the form x = h(y).

dx 1
f 0 (y) = = (y − 1) 2
dy
s  2
Z b
dx
L = 1+ dy
a dy
Z 4 p
L = 1 + (y − 1) dy
1

4

Z
= y dy
1

14
=
3

Exercise 9.10 Redo the previous example 9.3.2 using the function in the form y = f (x)
instead.
Example 9.3.3 Find the length of a line whose slope is −2 given that the line extends
from x = 1 to x = 5.

We could find the equation of the line, but that is not necessary: we are given that the
slope f 0 (x) is −2. The integral in question is

Z bq
2
Z 5 q √
L= 1 + (f 0 (x)) dx = 1 + (−2)2 dx = 4 5
a 1

Example 9.3.4 ? Find an integral that represents the length of the curve that forms
the graph of the function y = f (x) = x3 ; 1 < x < 2.
dy
We find that dx
= f 0 (x) = 3x2 Thus, the integral is

Z bq
2
Z 2 q Z 2 √
L= 0
1 + (f (x)) dx = 1+ (3x2 )2 dx = 1 + 9x4 dx
a 1 1

Example 9.3.5 First, we’ll find the circumference of a circle of radius R. Of course, we
expect to find the result that C = 2πR , but here we can verify the result.
The circle can be described as x2 + y 2 = R2 which means that, as a graph, the upper
semi-circle is

H.W-Kayondo & D.W-Ddumba, Engineering Math I- Lecture Notes 384


CHAPTER 9. APPLICATIONS OF DEFINITE INTEGRALS


y = f (x) = R 2 − x2
We will find the length along this upper semi-circle which is half the circumference of
the circle.
Notice that

x
f 0 (x) = − √
R − x2
2

x2 R 2 − x2 + x2 R2
1 + f 0 (x)2 = 1 + = =
R 2 − x2 R 2 − x2 R 2 − x2
p R
1 + f 0 (x)2 = √
R − x2
2

Then we can evaluate the arc length along the upper semi-circle as

Z R p
s = 1 + f 0 (x)2 dx
−R

Z R
R
= 2 √ dx
0 R − x2
2

x
= 2R sin−1 ( )|R
R 0
= πR
This means that the circumference of the circle, which is twice the length along the
upper semi-circle, is C = 2πR as expected.
Example 9.3.6 Now we’ll compute the length along the parabola y = x2 between x = 0
and x = 1 . This is, in fact, the curve shown in the demonstration above. Through our
approximation there, we expect that the arc length is approximately 1.479 . . .
Let’s begin:

f (x) = x2
f 0 (x) = 2x
1 + f 0 (x) = 1 + 4x2

R1√
This means that the arc length is given by s = 0 1 + 4x2 dx . This is a difficult
integral to evaluate using the Fundamental Theorem of Calculus: an antiderivative for

H.W-Kayondo & D.W-Ddumba, Engineering Math I- Lecture Notes 385


CHAPTER 9. APPLICATIONS OF DEFINITE INTEGRALS

the integrand is not immediately clear through one of the techniques we’ve discussed.
However, if we look on a table of integrals, we can really save ourselves some work. In
particular, there you will notice that

Z √
u√ 2 a2 √
a2 + u2 du = a + u2 + ln(u + a2 + u2 ) + C
2 2

This is not exactly the integral we want to evaluate, but it is pretty close. In fact, if
we use the substitution u = 2x , we find that

Z 1 √
s = 1 + 4x2 dx
0

1
Z 2 √
= 1 + u2 du
2 0

1 u√ √
 2
2
1 2
= 1 + u + ln(u + 1 + u )
2 2 2 0

1 √ 1 √ 
= 5 + ln(2 + 5)
2 2
= 1.479

Example 9.3.7 As a check on whether we are making sense, let’s work out the length
of a part of a straight line. Let y = mx + c be a straight line. Let us use the formula to
find its length between x = a and x = b.
y 0 = m so

s

 2
dy
1+ = 1 + m2
dx

So
Z b√ √ b−a
s= 1 + m2 dx = (b - a) 1 + m2 = where φ is the angle that the line makes
a cos φ
with the x-axis ( m = tanφ). You can easily check that this is indeed the right answer.

Example 9.3.8 What is the length of the graph y = cosh(x) between x = 0 and x = a?
dy
In this case dx = sinh(x) so

H.W-Kayondo & D.W-Ddumba, Engineering Math I- Lecture Notes 386


CHAPTER 9. APPLICATIONS OF DEFINITE INTEGRALS

s  2
dy
q
1+ = 1 + sinh2 (x) = cosh x
dx

So the length is
Z a
s= cosh x dx = [sinh x]a0 = sinh a
0


Exercise 9.11 Find the length of the curve y = 1 − x2 from x = 0 to x = 1.
Exercise 9.12 Find the length of the curve y = 41 (e2x + e−2x ) from x = 0 to x = 1.
Exercise 9.13 Set up (but do not evaluate) the integral to find the length of the piece
of the parabola y = x2 from x = 3 to x = 4.

9.4 Area of Surface of Revolution


Now we would like to find the surface area of a surface obtained by revolving the portion
of the graph y = f (x) between x = a and x = b about the x axis. Again, this is difficult
to understand directly, but we can approximate the surface area by small pieces whose
surface area can be computed.
To that end, let’s again approximate the curve y = f (x) by dividing up the interval
[a, b] into little pieces of width dx . The picture is the same one we saw above.
When we revolve this straight line about the x axis, we have a portion of a cone
whose base radius is f (x) . Now imagine that we cut this cone and lay it out flat. What
we would have would p almost be rectangular and so the area would be approximately
2πf (x) ds = 2πf (x) 1 + f 0 (x)2 dx .
Again, if we add up all the areas, we find that the surface area is
Z x=b p
S = 2πf (x) 1 + f 0 (x)2 dx (9.8)
x=a

Z y=d p
or S = 2πf (y) 1 + f 0 (y)2 dy (9.9)
y=c

This formula comes from extending the ideas of the previous section the length of a little
piece of the curve is p
dx2 + dy 2
This gets rotated around
p the perimeter of a circle of radius y = f (x), so approximately
give a band of width dx2 + dy 2 and length 2πf (x), which has area
s  2
p dy
2πf (x) dx2 + dy 2 = 2πf (x) 1+ dx
dx

H.W-Kayondo & D.W-Ddumba, Engineering Math I- Lecture Notes 387


CHAPTER 9. APPLICATIONS OF DEFINITE INTEGRALS

Integrating this (as if it were a sum!) gives the formula.

Similarly, we might rotate the curve y = f (x) around the y-axis instead. The same general
ideaspapply to compute the area of the resulting surface. The width of each little band is
still dx2 + dy 2 , but now the length is 2πx instead. So the band has area

p
width × length = 2πx dx2 + dy 2

Therefore, in this case the surface area is obtained by integrating this, yielding the formula

s 2
Z b 
dy
area = S = 2πy 1+ dx
a dx

Example
√ 9.4.1 ? Determine the surface area of the solid obtained by rotating
y = 9 − x2 , −2 ≤ x ≤ 2 about the x-axis

dy x
= −p
dx (9 − x2 )
s 2
Z 2 
dy
S = 2πy 1+ dx
−2 dx
v !2
u
Z 2 u x
= 2πy t1 + −p dx
−2 (9 − x2 )
Z 2
3
= 2πy √ dx
−2 9 − x2
Z 2 √ 3
= 2π 9 − x2 √ dx
−2 9 − x2
Z 2
= 6π dx
−2

= 24π

Example
√ 9.4.2 Determine the surface area of the solid obtained by rotating
y = x, 1 ≤ y ≤ 2 about the y-axis.
3

H.W-Kayondo & D.W-Ddumba, Engineering Math I- Lecture Notes 388


CHAPTER 9. APPLICATIONS OF DEFINITE INTEGRALS

Note 9.4.1 To say y-axis only indicates the given interval. But with such complications
in integrating it is better to apply

s 2
Z b 
dx
area = S = 2πx 1 + dy (9.10)
a dy

x = y3

dx
= 3y 2
dy
s 2
Z 2 
dx
S = 2πx 1 + dy
1 dy
Z 2 p
= 2πx 1 + 9y 4 dy
1

Z 2 p
= 2πy 3 1 + 9y 4 dy
1

= 199.48

Example 9.4.3 First, we’ll find the surface√ area of the sphere of radius R which can be
found by revolving thepgraph y = f (x) = R2 − x2 about the x axis. We have already
computed above that 1 + f 0 (x)2 = √RR 2 −x2 .
This means that

Z R √ R
Z R
S= 2π R2 − x2 dx = 2π R dx = 4πR2
−R R − x2
2
−R

This may be a familiar formula but now you verified it for yourself.

Example
√ 9.4.4 Now we’ll compute the surface area obtained by revolving the graph of
y = x between x = 0 and x = 1 about the y axis.

H.W-Kayondo & D.W-Ddumba, Engineering Math I- Lecture Notes 389


CHAPTER 9. APPLICATIONS OF DEFINITE INTEGRALS

1
r

Z
11
S = 2π x 1+ dx
0 4x
Z 1
r
1
= 2π x+ dx
0 4

2 1
= 2π (x + )3/2 |10
3 4
 3/2  3/2 !
4π 7 1
= −
3 4 4

Example 9.4.5 Sphere


As before, we obtain our sphere by rotating the semicircle

y = R 2 − x2 − R ≤ x ≤ R
around the x-axis.
For this curve

−x
 
dy
=√
dx R 2 − x2

So

s  2 r
dy x2 R
1+ = 1+ = √
dx R 2 − x2 R 2 − x2

So the area is given by

Z √
R
R
Z R
Area = 2π R − x √
2 2 dx = 2πR dx = 4πR2
R 2 − x2
−R −R

So a sphere of radius R has area 4πR2 .

Exercise 9.14 Find the area of the surface obtained by rotating the curve y = 14 (e2x +
e−2x ) with 0 ≤ x ≤ 1 around the x-axis.
Exercise 9.15 Just set up the integral for the surface obtained by rotating the curve
y = 14 (e2x + e−2x ) with 0 ≤ x ≤ 1 around the y-axis.

H.W-Kayondo & D.W-Ddumba, Engineering Math I- Lecture Notes 390


CHAPTER 9. APPLICATIONS OF DEFINITE INTEGRALS

Exercise 9.16 Set up the integral for the area of the surface obtained by rotating the
curve y = x2 with 0 ≤ x ≤ 1 around the x-axis.

Exercise 9.17 Set up the integral for the area of the surface obtained by rotating the
curve y = x2 with 0 ≤ x ≤ 1 around the y-axis.

9.5 Examples
Example 9.5.1 Population density measures the number of people per square mile
inhabiting a given living area. The population density of a certain city decreases as you
move away from the center of the city. The density at a distance r miles from the city
center can be approximated by the function 10000(2 − r).

(a) If the population density tails off to zero at the edges of the city, what is the city
radius?

(b) A thin ring of area around the center of the city has thickness ∆r and radius r. If
the ring is straightened out, it becomes a rectangular strip. What is the area of this
region?

(c) Explain why the population of the ring in part (b) is given by 10000(2 − r)(2πr)∆r.

(d) Find the total population of the city by setting up and evaluating a definite integral.

Example 9.5.2 Let R be the shaded region in the first quadrant enclosed by the y axis
and the graphs of y = sin x and y = cos x.

(a) Find the area of the region R.

(b) Find the volume of the solid when R is revolved about the x axis.

(c) Find the volume of the solid whose base is R and whose cross sections cut by planes
perpendicular to the x axis are squares.

H.W-Kayondo & D.W-Ddumba, Engineering Math I- Lecture Notes 391


CHAPTER 9. APPLICATIONS OF DEFINITE INTEGRALS

Example 9.5.3 Consider the region R bounded by y = sin x, y = 0, x = 0 and x = π.


Find the volume generated when R is rotated about

(i) x-axis
π π
π2
Z 
2 1
V = π sin x dx = π (x − sin x cos x) =
0 2 0 2

(ii) y-axis
2π 2

Example 9.5.4 Consider the region R bounded by the circle (x−4)2 +y 2 = 4. Compute
the volume V generated when R is rotated around y = 0.
Since the area crosses the x-axis, it is sufficient to rotate the top half to get the required
solid.

Z 6
π y12 − y22 dx

V =
2

Z 6
π y12 − 02 dx

=
2

Z 6
π 4 − (x − 4)2 dx
 
=
2

32
= π
3

Example 9.5.5 Consider the region R bounded by y = x and y = x2 . Find the volume
generated when R is rotated around the x- axis.

Z 1
2
π x2 − x4 dx = π
 
V =
0 15

n  o
2 23
Example 9.5.6 Let C = x, 3
x : 0≤x≤4

(i) compute the arc length L of the curve C.

2h √
4
4 √
Z 
2 3
i
L= 1 + x dx = (1 + x) 2 = 5 5−1
0 3 0 3

H.W-Kayondo & D.W-Ddumba, Engineering Math I- Lecture Notes 392


CHAPTER 9. APPLICATIONS OF DEFINITE INTEGRALS

(ii) compute the area S of solid of revolution of the region covered by C along the x-axis.

Example 9.5.7 Given the curve C represented by a parabola

1
y = x2 ; 0 ≤ x ≤ 1
2

(i) compute the arc length L of the curve C.

(ii) compute the area S of solid of revolution of the region covered by C along the x-axis.

(iii) compute the area S of solid of revolution of the region covered by C along the y-axis.

Example 9.5.8 Find the area under the function




x+2 , 0≤x≤1








f (x) =







 3, 1<x≤2

Example 9.5.9 Find the area between the line f (x) = x − 1, the x-axis and the y-axis.

1
A=
2

Example 9.5.10 Find the area bounded by the curves y = 2 − x2 and y = x2 .

Z 1
8
(2 − x2 ) − (x2 ) dx =
 
A=
−1 3

Example 9.5.11 Let R be the region bounded by the curves x = y 2 and x = y + 2, find
the area of R.
9
A=
2

Example 9.5.12 Find the area bounded by the curves x − y = 1 and x + 2y = 7

A=6

H.W-Kayondo & D.W-Ddumba, Engineering Math I- Lecture Notes 393


CHAPTER 9. APPLICATIONS OF DEFINITE INTEGRALS

Example 9.5.13 Find the area of the region bounded by the graphs f (x) = x3 and
g(x) = x
1
A=
2

Example 9.5.14 Find the volume generated by revolving about the x-axis the region
bounded by y 2 = 4x and x2 = 4y.
96π
V =
5

Example 9.5.15 Find the volume generated by revolving about the y-axis the region
bounded by x = y 2 and y = x − 2.
72π
V =
5

Example 9.5.16 Compute the length of the straight line segment joining the two points
(2, 4) and (5, 13)

f (x) = 3x − 2

L = 3 10
3
Example 9.5.17 Find the arc length of the graph y = x 2 between (1, 1) and (4, 8)

"   23 #
8 3 13
L= 10 2 −
27 4

Example 9.5.18 Find the area of the surface of revolution obtained by rotating y =
x3
3
, 0 ≤ x ≤ 3 about the x-axis.

πh 3
i
S= (82) 2 − 1
9


Example 9.5.19 Determine the surface area of a solid obtained by rotating y = 3
x, 1 ≤
y ≤ 2 about the y-axis.

Z 2 q
S = 2π y 3
1 + (3y 2 )2 dy = 199.48
1

H.W-Kayondo & D.W-Ddumba, Engineering Math I- Lecture Notes 394


CHAPTER 9. APPLICATIONS OF DEFINITE INTEGRALS

Example 9.5.20 Determine the area of a solid obtained by rotating y 2 + x2 = 9, − 2 ≤


x ≤ 2 about the x-axis.
1
f (x) = y = (9 − x2 ) 2
S = 24π

Example 9.5.21 Find the area of the surface of revolution obtained by rotating x =

y, 2 ≤ y ≤ 20 about the y-axis.
S = 117π

Example 9.5.22 Find the area of the region bounded by y = x2 − 2x − 8 and y = 2x − 8

108
A= = 36
3

Example 9.5.23 Find the area of the region bounded by f (x) = x and g(x) = x3 − 8x

81 81
A= + = 40.5
4 4

H.W-Kayondo & D.W-Ddumba, Engineering Math I- Lecture Notes 395


Chapter 10

Matrix

10.1 Definitions
Definition 10.1.1
An m × n matrix is a rectangular array of m × n numbers arranged in horizontal rows
and vertical columns.
A matrix could be generally written as
 
 a11 a12 · · · a1n 
 
 
 
 
 a21 a22 · · · a2n 
 
 
 
 

 · 

A=



 

 · 

 
 
 

 · 

 
 
 
am1 am2 . . . amn

Or more briefly we have that A = (aij ) where aij is an entry in the ith row and j th column.
Definition 10.1.2 Square Matrix;
A(m × n) is a square matrix if m = n.
Definition 10.1.3 Equality of matrices;
If matrices A = (aij ) and B = (bij ) both m × n are said to be equal if and only if all
corresponding entries of A and B are the same i.e aij = bij ∀ i, j

396
CHAPTER 10. MATRIX

Definition 10.1.4 Matrix transpose;


Given A = (aij ), an m × n matrix, then the transpose of A denoted by At or AT or A0 is
an n × m matrix.
AT = (aij )T where aTij = aji ∀ i, j. We can also denote AT = (aji ).

Note 10.1.1 The transpose of matrix A is determined by interchanging the rows with
columns of A.

Example 10.1.1 Given

   
 2 3 7   2 5 0 
   
   
   
A= 5 6  ⇒ AT =  3 6 1 
   
 8   
   
   
   
0 1 −1 7 8 1

Definition 10.1.5 Scalar multiplication of a matrix;


Given A = (aij ) then multiplying A by a scalar α, you get αA = (αaij ).

Definition 10.1.6 Identity or Unit matrix;


An identity matrix is a square matrix i.e m = n matrix of the form

 

 1 0 ... 0 

 
 
 

 0 1 ... 0 

 
 
 

 · 

In = 



 

 · 

 
 
 

 · 

 
 
 
0 0 ... 1

In usually denoted as I has a nice property that IA = AI = A where A is an n × n


matrix.

H.W-Kayondo & D.W-Ddumba, Engineering Math I- Lecture Notes 397


CHAPTER 10. MATRIX

Definition 10.1.7 Symmetry of matrices;


A square matrix is said to be symmetric if AT = A i.e if the matrix is equal to its
transpose.
And Anti-symmetric or sometimes called skew-symmetric if AT = −A.

Example 10.1.2 If

   
 1 2 3   1 2 3 
   
   
   
T
A =  2 3 4  Then A =  2 3 4 
   
   
   
   
   
3 4 5 3 4 5

i.e AT = A then A is symmetric.

Example 10.1.3 Given

   
 0 −2 −3   0 2 3 
   
   
   
T
B= 2  Then B =  −2
   
 0 −5   0 5 

   
   
   
3 5 0 −3 −5 0

i.e B T = −B then B is anti-symmetric.

10.2 Special Types of matrices


10.2.1 Diagonal Matrix
A square matrix A = (aij ) is said to be a diagonal matrix if aij = 0 for i 6= j, i.e off
diagonal elements are equal to zero.

H.W-Kayondo & D.W-Ddumba, Engineering Math I- Lecture Notes 398


CHAPTER 10. MATRIX

Example 10.2.1
   
 2 0 0   6 0 0 
   
   
   
 and  0 0 0 
   
 0 3 0
   
   
   
   
0 0 −1 0 0 0

are diagonal matrices.

10.2.2 Tri diagonal Matrix


A square matrix A = (aij ) is said to be a Tri-diagonal matrix if aij = 0 for |i − j| ≥ 2.
i.e if every element in the ith row and j th column is zero when the absolute difference
between i and j is greater or equal to two. A 3 × 3 tri-diagonal matrix can generally be
written as
 
 a11 a12 0 
 
 
 
A= a  For example
 
 21 a22 a23 
 
 
 
0 a32 a33

Example 10.2.2
 
 1 3 0 
 
 
 
A= 2 1
 
 5 

 
 
 
0 1 −1

is a tri-diagonal matrix.

10.2.3 Triangular matrix


It is a square matrix whose non-zero elements lie on the diagonal, and all are either
above or below the diagonal (but not both).

H.W-Kayondo & D.W-Ddumba, Engineering Math I- Lecture Notes 399


CHAPTER 10. MATRIX

Example 10.2.3

   
 1 0 0   0 0 1 
   
   
   
A= 2 3  and B =  0 3
   
 0   2 

   
   
   
4 2 −1 0 0 −1

are triangular matrices.

Upper triangular matrix


This is a type of triangular matrix A = (aij ) where aij = 0 for i > j for example

 
 a11 a12 . . . a1n 
 
 
 
 
 0 a22 . . . a2n 
 
 
 
 
 · 
 
 
 
 
 · 
 
 
 
 
 · 
 
 
 
 
0 0 . . . ann

such as matrix A in the above example.

H.W-Kayondo & D.W-Ddumba, Engineering Math I- Lecture Notes 400


CHAPTER 10. MATRIX

Lower triangular matrix


A = aij where A is lower triangular if aij = 0 for i < j they take the form

 
 a11 0 ... 0 
 
 
 
 
 a21 a22 0 . . . 0 
 
 
 
 
 · 
 
 
 
 
 · 
 
 
 
 
 · 
 
 
 
 
an1 an2 . . . ann

such as matrix B in the above example.

10.2.4 Idempotent matrix


A = (aij ) is an idempotent matrix if A2 = AA = A

 
 1 0 
 
Example 10.2.4 A = 

 is an idempotent matrix.

 
0 1

Exercise 10.1 Is the matrix


 
 1 1 
 
B=



 
0 0

idempotent? Defend your answer.

H.W-Kayondo & D.W-Ddumba, Engineering Math I- Lecture Notes 401


CHAPTER 10. MATRIX

10.2.5 Invertible or non-singular matrix


A square matrix A = (aij ) is non-singular or invertible if and only if ∃ an n × n matrix
denoted by A−1 such that
AA−1 = A−1 A = I.
(I the identity matrix, and A−1 will later be called the inverse of A). If A−1 does not
exist, then we say that A is singular or non invertible.

10.3 Operations of Matrices


10.3.1 Addition and Scalar Multiplication of matrices
Definition 10.3.1 Let A and B be matrices of the same size. The sum of A and B,
written A + B is the matrix whose ij-th entry is aij + bij .
Let A = (aij ), B = (bij ). Then
C = A + B = (cij = aij + bij )
Note that cij = bij + aij and so A + B = B + A.
Also if 0 is the zero matrix, then
0+A=A+0=A
for any matrix A.
Definition 10.3.2 Let k be a scalar. Then kA is a matrix whose ij-entry is kaij
i.e
kA = (kaij )
Thus, for example, (−1)A = (−aij ) and
A + (−1)A = A − A = 0
   
 1 2 3   −1 5 −2 
   
Example 10.3.1 Let A = 

 and B = 
 


   
−1 0 2 2 2 −1

Find (i) A + 2B (ii) 3A0 − B 0 (iii) (3A − B)0


Solution 10.3.1
     
 −1 2 3   −2 10 −4   −3 12 −1 
     
(i) A + 2B = 

+
 
=
 


     
−1 0 2 4 4 −2 3 4 0

H.W-Kayondo & D.W-Ddumba, Engineering Math I- Lecture Notes 402


CHAPTER 10. MATRIX

     
 −3 3   −1 2   −2 1 
     
     
     
(ii) 3A0 − B 0 =  6 0  −  5 =
     
2   1 −2 
     
     
     
     
9 6 −2 −1 11 7

(iii)
   0
 −3 6 9   −1 5 −2 
0
   
(3A − B) = 

 −
 


   
3 0 6 2 2 −1

 0
 −2 1 11 
 
= 
 

 
1 −2 7

 
 −2 1 
 
 
 
=  1 −2 
 
 
 
 
 
11 7

Note that 3A0 − B 0 = (3A − B)0 .


Theorem 10.3.1 Let A and B be matrices of the same size whose entries are in a field
F and s, t be scalars.Then (sA + tB)0 = sA0 + tB 0 .
Proof: Let
D = sA + tB = (saij + tbij )
Then D0 = (sA + tB)0 = (dji ) where
dji = saij + tbij
Let A0 = (a∗ji ), a∗ji = aij
and B 0 = (bji )∗ , b∗ji = bij
Then sA0 + tB 0 = (sa∗ji + tb∗ji ) = (saij + tbij )

H.W-Kayondo & D.W-Ddumba, Engineering Math I- Lecture Notes 403


CHAPTER 10. MATRIX

Thus the ji-entry of D0 is equal to the ji-entry of SA0 + tB 0 for all i and j. Therefore
D0 = (SA + tB)0 = SA0 + tB 0 .

Exercise 10.2

(1) EER (Easy Exercise for the Reader)


Solve the following matrix equation for a, b, c and d.

   
 a−b b+c   8 1 
   
 = 
   
   
3d + c 2a − 4d 7 6

(Answer: a = 5, b = −3, c = 4, d = 1)

   
 1 −3   7 4 
   
(2) Let A = 

 and B = 
 
.

   
2 6 −5 8

Find A + B + (A + B)0

Solution 10.3.2
   

 2 −1 

 14 −1 
 
0 0
Note that A + A = 

 and B + B = 
 


   
−1 12 −1 16

So that
 
 16 −2 
0 0 0
 
A + B = (A + B) = A + A + B + B = 



 
−2 28

H.W-Kayondo & D.W-Ddumba, Engineering Math I- Lecture Notes 404


CHAPTER 10. MATRIX

 

 8 1 

Alternatively use A + B = 



 
−3 14

   

8   8 −3 
1 
  
0

So that (A + B) + (A + B ) = 

+
 


   
−3 14 1 14

 
 16 −2 
 
= 



 
−2 28

Note that A + A0 , B + B 0 and their sum are symmetric matrices.


Theorem 10.3.2 Let A be a square matrix. Then A + A0 is symmetric.
Proof:
Let A = (aij ). Then A0 = (aji = aij )
Now A + A0 = (aij + a∗ij ) = (aij + aji )
Clearly aij + aji = bji = aji + aji for all i and j it follows that A + A0 is symmetric.
Alternatively (Shorter)
A + A0 is symmetric if it is equal to its transpose. Note that A + A0 is a square matrix
for A a square matrix.
Now (A + A0 ) = A0 + A (using Theorem (d))
= A + A0 (addition of matrices commutative).
Theorem 10.3.3 Let A, B and C be matrices and s and t scalars in a field F. Then
(a) A + B = B + A (Addition commutative law)
(b) (A + B) + C = A + (B + C) (Addition Associative law) Let A = (aij ) and B =
(bij ), C = (cij ). Then A + B = (aij + bij ) by definition
Also (A + B) + C = ((aij + bij ) + cij )
But (aij + bij ) + cij = aij + (bij + cij ) using associativity of scalars
So (A + B) + C = (aij + (bij + cij ))
= (aij ) + (bij + cij )
= A + (B + C)

H.W-Kayondo & D.W-Ddumba, Engineering Math I- Lecture Notes 405


CHAPTER 10. MATRIX

(c) A + 0 = A (Additive identity)

(d) A + (−A) = 0 (Additive inverse)

(e) (st)A = s(tA) (commutative law)

(f) (s + t)A = sA + tA (distributive law)

(g) t(A + B) = tA + tB (distributive law)

(h) 1.A = A (multiplicative identity)

Proof: EER (Easy Exercise for the Reader)

10.3.2 Matrix Multiplication


Definition 10.3.3

Let A be an m × r matrix and B an r × n matrix in a field F. The product of A and B


is the matrix whose ij-entry is the dot product of the ith row of A and j th column of B.
Let C = AB = (cij ).
r
P
Then cij = aik bkj . Note that cij ∈ F.
k=1

Thus matrix multiplication requires that the number of columns of A is the same as the
number of rows of B.

Example 10.3.2 (Please verify)

H.W-Kayondo & D.W-Ddumba, Engineering Math I- Lecture Notes 406


CHAPTER 10. MATRIX

(1)

   
 4 −1   1 4 2 
   
Let A = 


 and B = 



   
0 2 3 1 5

    
 4 −1   1 4 2   16 15 3 
    
Then AB = 



=
 


    
0 2 3 1 5 6 2 10

 

 1 3 
    
 1 4 2    21 17
  
0
  
and BB =   = .
 
  4 1 

  
    
3 1 5   17 35
 
2 5

H.W-Kayondo & D.W-Ddumba, Engineering Math I- Lecture Notes 407


CHAPTER 10. MATRIX

(2)

 
 3 + 2i 0   
 



  −i 2 
 
Let A =  , B=
 
−i 2

   
 
0 i
 
 
 
1+i 1−i

 
 −1 − i 0 −i 
 
and C = 

.

 
3 2i −5

 
 5 + i 4i − 11 
 
Then BC = 



 
3i − 2 −5

 
 −23 − 25i −1 − i 
 
CA = 



 
69 − 5i −5 + 9i

 
 25 − 7i 57 + 36i 
 
 
 
and (1 + i)AB + (3 − 4i)C 0 =  −1 − i
 
 −8 − 6i 

 
 
 
6 + 3i −15 + 26i

Theorem 10.3.4

Let A, B and C be conformable matrices over a field F and k a scalar.


Then
1. (AB)C = A(BC)

H.W-Kayondo & D.W-Ddumba, Engineering Math I- Lecture Notes 408


CHAPTER 10. MATRIX

2. A(B + C) = AB + AC
3. (B + C)A = BA + CA
4. k(AB) = (kA)B = A(kB)
Proof: EER see lips chapter 3 page 50

1. Let D = AB and E = BC
Then by definition,
X
D = (dij ) where dij = aik bkj
k

X
and E = (eij ) where eij = bik0 ckj
kj

Thus F = DC = (fij )
" #
X X X
where fij = dek ckj = ali bik ckj
k k i

X X
= aii bik Ckj
i k

X
= ali eij
i

Therefore F = (fli ) = AE = A(BC)

Example 10.3.3

   
 1 2   2 0 
   
(1) Let A = 
  and B = 
 
.

   
3 −1 1 1
   
 4 2   2 4 
   
Then AB = 
  and BA = 
 


   
5 −1 4 1

Note that AB 6= BA. In general, matrices A and B are said to commute if AB = BA

H.W-Kayondo & D.W-Ddumba, Engineering Math I- Lecture Notes 409


CHAPTER 10. MATRIX

(2) For conformable matrices A and B, consider

(A − B)(A + B) = A2 − B 2 + AB − BA

Then (A − B)(A + B) = A2 − B 2
if and only if A and B commute, i.e AB = BA. Note that the equality is always true
for scalars.

(3) Let

 
   1 5 2 
 
1 4 2 
   
  
X =   and Y =  
   −1 0 1 
   
3 1 5
 
 
 
3 2 4

 
 25 1 19 
0
 
Then XY = 



 
18 2 31

Find directly Y 0 X.

(4) Let A be a square matrix. The powers of A are defined as

A0 = I, A = A, A2 = AA, A2 A, . . .

Let f (x) be a polynomial. Then

f (x) = a0 + a1 x + a2 x2 + . . . + an xn
and f (A) = a0 I + a1 A + a2 A2 + . . . + an An

If f (A) = 0, A is called the root or zero of the polynomial f (x).


 
 2 2 
 
For example, Let A   and f (x) = x2 − x − 8.
 
 
3 −1

H.W-Kayondo & D.W-Ddumba, Engineering Math I- Lecture Notes 410


CHAPTER 10. MATRIX

   
 10 2   26 18 
2
  3
 
Then A = 
 
 and A = 

.

   
3 7 27 −1

(Verify)
So, f (A) = A2 − A − 8I
     
 10 2   2 2   8 0 
     
=  − − 
     
     
3 7 3 −1 0 8

 
 0 0 
 
= 

.

 
0 0

Therefore A is the zero of f (x).


Exercise 10.3 Prove that (AB)0 = B 0 A0 for A, B conformable matrices.
Definition 10.3.4 Trace of a square Matrix
Let A be an n × n square matrix, the trace of A denoted by tr(A) is the sum of all
n
P
diagonal elements of A. That is, trace(A) = aii
i

Example 10.3.4
 
 1 2 0 
 
Let A = 



 
3 −1 4
 
 5 1 
 0

Then AA =   and tr(AA0 ) = 31
 
 
1 26

H.W-Kayondo & D.W-Ddumba, Engineering Math I- Lecture Notes 411


CHAPTER 10. MATRIX

 
 10 −1 12 
 
 
 
Also A0 A =  −1 and tr(A0 A) = 31
 
 5 −4 

 
 
 
12 −4 16

Theorem 10.3.5

Let A and B be conformable matrices. Then tr(AB) = tr(BA)


Proof:
X
Let C = AB = (Cij ) where Cii = aik bkj
k

X XX
Then tr(C) = tr(AB) = Cii = aik bki
i i k

Similarly let D = BA
XX
Then tr(D) = bik aki (why?)
i k

XX
= aki bik (Why?)
i k

= tr(D)
Thus tr(AB) = tr(BA)

Home Work 10.1

Read Anton Section 1.4 pg 23-30


Section 1.5 pg 31-35
Lips Chapter 3

Exercise 10.4

Anton Section 1.4 (All questions)


Section 1.5 (1-3)
Let B be obtained from A by

H.W-Kayondo & D.W-Ddumba, Engineering Math I- Lecture Notes 412


CHAPTER 10. MATRIX

(i) multiplying a row (column) of A by a scalar k; then |B| = k|A|.


Proof:
If Ri ← kRj on A
Then every term in |A| is multiplied by k is

X
|B| = (sgnσ)a11 a2i2 . . . (kaj ij ) . . . an in
σ

X
= k (sgnσ)a1i1 a2i2 . . . ajij . . . ani1
σ

= k|A|.

(ii) adding a multiple of a row (column) of A to another; then |B| = |A|

Rj ← Rj + cRk
X
|B| = (sgnσ)a1 δ(1)a2σ(2) . . . (cak σ(k) + aj σ(j) − an σ(m))
σ

am (σk )
= cσσ sgna1 σ(1)ak σ(2) . . . . . . an σ(n)
j th

because j th row + k th row are the same.


X
+ sgna1 σ(1)a2 (σ2 ) . . . an σ(n)
σ

(iii) interchanging two rows (column) of A then |B| = −|A|


Proof: omitted but results can be demonstrated in case of 3 × 3 and 4 × 4 matrix

H.W-Kayondo & D.W-Ddumba, Engineering Math I- Lecture Notes 413


CHAPTER 10. MATRIX

10.4 Properties of matrices


Let A = aij , B = bij . C = cij then,

1. A + B = B + A. i.e Matrix addition is commutative

2. A + (B + C) = (A + B) + C. i.e Associativity of matrix addition

3. A + 0 = 0 + A = A. where 0 is a zero matrix, a matrix with zero entries.

4. A + (−A) = 0 where −A = (−1)A

5. A(BC) = (AB)C. Associativity of matrix multiplication.

6. A(B + C) = AB + AC. Distributivity from the left.

7. (A + B)C = AC + BC. Distributivity from the right.

8. α(βA) = αβ(A). where α, β are scalars.

9. α(A + B) = αA + αB.

10. A(αB) = α(AB) = (αA)B.

11. (AT )T = A.

12. (A + B)T = AT + B T .

13. (AB)T = B T AT .

14. (αA)T = αAT .

15. AB = 0. it does not mean that A = 0 or B = 0 or both.

Classwork 10.1 Some of the fundamental definitions in linear algebra include:-

(i) A matrix

(ii) A square matrix

(iii) A Diagonal matrix

(iv) A unit or Identity matrix

(v) A transpose of a matrix

H.W-Kayondo & D.W-Ddumba, Engineering Math I- Lecture Notes 414


CHAPTER 10. MATRIX

10.5 Determinants
To compute the determinant of any matrix, we can use two methods, namely

• Permutations and inversion of a permutation.

• Cofactors,minors and adjoint.

10.5.1 Permutations and inversion of a permutation


Definition 10.5.1 Permutation
Let S = {1, 2, . . . , n} containing n numbers. An ordered arrangement < i1 i2 . . . in > of
elements of set S is called a permutation of S.

Example 10.5.1 The permutations for S = {1, 2, 3} are

< 1, 2, 3 > < 1, 3, 2 > < 2, 1, 3 > < 2, 3, 1 > < 3, 1, 2 > < 3, 2, 1 >

Exercise 10.5 State the permutations for S = {1, 2, 3, 4}

Note 10.5.1 A set of n elements has n! permutations.

Note 10.5.2 We denote the set of all permutations of set S by Sn where n is the number
of elements of S, and thus

S1 = {< 1 >}
S2 = {< 1, 2 > < 2, 1 >}
S3 = {< 1, 2, 3 > < 1, 3, 2 > < 2, 1, 3 > < 2, 3, 1 > < 3, 1, 2 > < 3, 2, 1 >}

Definition 10.5.2 Inversion of Permutation


A permutation < i1 i2 . . . in > of set S is said to have an inversion if a larger integer it
preceeds (comes before) a smaller integer is
For example < 1, 2 > has no inversion, < 2, 1 > has one inversion, < 3, 2, 1 > has three
inversions, < 2, 3, 1 > has two inversions, and < 1, 2, 3 > has no inversions,

Definition 10.5.3 Even and Odd Permutation


A permutation is even or odd depending on the total number of inversions, either even or
odd.(Here we do consider zero to be even).
For example < 1, 2 > is an even permutation since it has no inversion,
where as < 1, 3, 2 > is an odd permutation since it has one inversion.

H.W-Kayondo & D.W-Ddumba, Engineering Math I- Lecture Notes 415


CHAPTER 10. MATRIX

Definition 10.5.4 Determinant


If A = (aij ) is an n × n matrix, the determinant of A denoted as |A| or det(A) is defined
by
X
|A| = ±a1i1 a2i2 . . . anin
σ

where the σ denotes all permutations < i1 i2 . . . in > in the set S = {1, 2, . . . , n} (i.e the
ij to be substituted in the formula should be [in their order] got from each permutation).
Note 10.5.3 Also that the + sign in the summation is taken when permutation is even,
or − the permutation is odd. For example:
 
 a11 a12 
 
For n = 2 where A = 

 and

 
a21 a22

S2 = {< 1, 2 > < 2, 1 >} then


|A| = Σ ± a1i1 a2i2 = +a11 a22 − a12 a21
 
 a11 a12 a13 
 
 
 
For n = 3 where A =  a  and
 
 21 a22 a23 
 
 
 
a31 a32 a33

S3 = {< 1, 2, 3 > < 1, 3, 2 > < 2, 1, 3 > < 2, 3, 1 > < 3, 1, 2 >
< 3, 2, 1 >} then
|A| = Σ ± a1i1 a2i2 a3i3 = +a11 a22 a33 + a12 a23 a31
+a13 a21 a32 − a12 a21 a33 − a13 a22 a31
Example 10.5.2
 
 1 2 
 
Find the determinant of A = 



 
3 7

⇒ |A| = +a11 a22 − a12 a21 = 1(7) − 2(3) = 1

H.W-Kayondo & D.W-Ddumba, Engineering Math I- Lecture Notes 416


CHAPTER 10. MATRIX

 
 1 2 0 
 
 
 
Example 10.5.3 Given A =  3 0 1  compute |A|. But
 
 
 
 
 
3 3 4

= |A| = Σ ± a1i1 a2i2 a3i3 = +a11 a22 a33 − a11 a23 a32 + a12 a21 a33
−a12 a23 a31 + a13 a21 a32 − a13 a22 a31
⇒ |A| = 1.0.4 + 2.1.3 + 0.3.3 − 1.1.3 − 2.3.4 − 0.0.3 = −21

10.5.2 Matrix Adjoint,Minors,Cofactors


Definition 10.5.5 If A = (aij ) is an n × n then Mij will denote (n − 1) × (n − 1) matrix
obtained from A by deleting its ith row and j th column.
Its determinant which we denote by |Mij | is called the minor of the element aij of A.

Definition 10.5.6 The Cofactor of aij denoted by Aij is given by

Aij = (−1)i+j |Mij |

Example 10.5.4 Given


 
 1 2 3 
 
 
 
A= 3 2 1 
 
 
 
 
 
2 0 2

Find all the cofactors of the matrix A.





2 1
= (−1)1+1

A11 = (1)(4) = 4


0 2

H.W-Kayondo & D.W-Ddumba, Engineering Math I- Lecture Notes 417


CHAPTER 10. MATRIX




3 1
= (−1)1+2

A12 = (−1)(4) = −4


2 2




3 2
= (−1)1+3

A13 = (1)(−4) = −4


2 0




2 3
= (−1)2+1

A21 = (−1)(4) = −4


0 2




1 3
= (−1)2+2

A22 = (1)(−4) = −4


2 2




1 2
= (−1)2+3

A23 = (−1)(−4) = 4


2 0




2 3
3+1

A31 = (−1) = (1)(−4) = −4


2 1




1 3
= (−1)3+2

A32 = (−1)(8) = 8


3 1

H.W-Kayondo & D.W-Ddumba, Engineering Math I- Lecture Notes 418


CHAPTER 10. MATRIX




1 2
= (−1)3+3

A33 = (1)(−4) = −4


3 2

Then the cofactor matrix of A is


   
 A11 A12 A13   4 −4 −4 
   
   
   
 =  −4 −4
   
 A21 A22 A23 4 
   
   
   
   
A31 A32 A33 −4 8 −4

Definition 10.5.7 Determinant


Suppose that A = aij is an n × n matrix, and let Aij denote the cofactor of the element
aij with i, j = 1, 2, . . . , n then
1.
n
X
|A| = akj Akj
k=1

That is summing along the j th column.


2.
n
X
|A| = aik Aik
k=1

That is summing along the ith row.


Example 10.5.5 Compute the determinant of the matrix
 
 1 2 3 
 
 
 
A= 3 2 1 
 
 
 
 
 
2 0 2

H.W-Kayondo & D.W-Ddumba, Engineering Math I- Lecture Notes 419


CHAPTER 10. MATRIX

But we know (last example) that the cofactor matrix of A is

 
 4 −4 −4 
 
 
 
=  −4 −4
 
 4 

 
 
 
−4 8 −4

Summing along the

1st row ⇒ |A| = (1)(4) + (3)(−4) + (2)(−4) = −16


2st row ⇒ |A| = (2)(−4) + (2)(−4) + (0)(8) = −16
3st row ⇒ |A| = (3)(−4) + (1)(−4) + (2)(−4) = −16
1st column ⇒ |A| = (1)(4) + (2)(−4) + (3)(−4) = −16
2st column ⇒ |A| = (3)(−4) + (2)(−4) + (1)(4) = −16
3st column ⇒ |A| = (2)(−4) + (0)(8) + (2)(−4) = −16

Same value of determinant no matter which row or column you consider. But preferably
a row or column with more zeros is better.

Definition 10.5.8 Adjoint


The transpose of the cofactor matrix is the adjoint of the matrix. It is usually denoted
by adj(A).

 
 a11 a12 . . . a1n 
 
 
 
 
 a21 a22 . . . a2n 
 
 
 
 
 · 
 
For A = 



 
 · 
 
 
 
 
 · 
 
 
 
 
an1 an2 . . . ann

H.W-Kayondo & D.W-Ddumba, Engineering Math I- Lecture Notes 420


CHAPTER 10. MATRIX

 
 A11 A12 . . . A1n 
 
 
 
 
 A21 a22 . . . A2n 
 
 
 
 

 · 

The Cofactor Matrix is 



 

 · 

 
 
 

 · 

 
 
 
An1 An2 . . . Ann

 
 A11 A21 . . . An1 
 
 
 
 
 A12 A22 . . . An2 
 
 
 
 

 · 

Thus Adj(A) = 
 

 

 · 

 
 
 

 · 

 
 
 
A1n A2n · Ann

 
 1 2 3 
 
 
 
Example 10.5.6 Compute the adjoint of the matrix A =  3 2 1 
 
 
 
 
 
2 0 2

H.W-Kayondo & D.W-Ddumba, Engineering Math I- Lecture Notes 421


CHAPTER 10. MATRIX

The cofactor matrix of A is


   
 4 −4 −4   4 −4 −4 
   
   
   
 ⇒ adj(A) =  −4 −4
   
 −4 −4 4 8 
   
   
   
   
−4 8 −4 −4 4 −4

10.5.3 Properties of determinants


(i) |A| = |AT |

(ii) Interchanging two rows or columns in a matrix gives the negative (determinant) of
the previous matrix.

Example 10.5.7 Compute the determinants of

     
 1 2 0   3 0 1   0 3 1 
     
     
     
A =  3 0 1 , B =  1 2 0 , C= 2 1 0 
     
     
     
     
     
3 3 4 3 3 4 3 3 4

(iii) If two rows or columns of a matrix are equal, then its determinant is equal to zero.

(iv) If any row or column in a matrix A is zero, then |A| = 0.

(v) A scalar α multiplying through a row or column of a matrix gives the determinant
as α|A|.

(vi) The value of the determinant remains unchanged if any row or column is replaced
by a linear combination of any two rows or columns

(vii) The determinant of a triangular or diagonal matrix is given by the product of the
diagonal elements.

(viii) |AB| = |A||B|

(ix) |A−1 | = 1
|A|
; 6 0 & A−1 is called the inverse of A.
|A| =

H.W-Kayondo & D.W-Ddumba, Engineering Math I- Lecture Notes 422


CHAPTER 10. MATRIX

Example 10.5.8 Using the properties of determinants explain why determinants of

 

3 1 0 0 0 
  
 
  



   1 0 0 0 
   
0 0 0  0 1 0 0 0  3 1 4 
     
      
 











  0 2 0 0 
 
= 0,  = −24, = 27,  = 24
     
2 3 0  0 0 4 −1 0  0 3 4 
   
      
 











  0 0 3 0 
       
−1 −2 4 0 0 0 −1 0  0 0 3
  
   
 
0 0 0 4
 
 
 
0 0 0 0 2

10.6 Test your self


Example 10.6.1 Use the method of permutation to compute |A|

 
 −2 1 4 
 
 
 
A =  3 5 −7 
 
 
 
 
 
1 6 2

|A| = +a11 a22 a33 − a11 a23 a32 + a12 a21 a33 − a12 a23 a31 + a13 a21 a32 − a13 a22 a31
= (−2)(5)(2) − (−2)(−7)(6) + (1)(−7)(1) − (1)(3)(2) + (4)(3)(6) − (4)(5)(1)
= −137 + 72 = −65.

Example 10.6.2 Compute the determinants using its properties

H.W-Kayondo & D.W-Ddumba, Engineering Math I- Lecture Notes 423


CHAPTER 10. MATRIX

(i)
   
 1 −3 0   1 −−3 0 
   
   
   
 =  0  using R2 ← 2R1 + R2
   
 −2 4 1 −2 1
   
   
   
   
5 −2 2 5 −2 2

 
 1 −3 0 
 
 
 
=  −2 −2 1  using R3 ← −5R1 + R3
 
 
 
 
 
0 13 2

 
 1 −3 0 
 
 
  13
=  0 −2 Using R3 ← R2 + R3
 
 1 
 2
 
 
 17

0 0 2

= −17

(ii)




2 1 3 1

2 1
3 1




1 0 1 1 0 1 1 −1
= −1





2


0 2 1 0

0 2
1 0





0 1 2 3

0 1
2 3

Using R2 ← R1 − 2R2

H.W-Kayondo & D.W-Ddumba, Engineering Math I- Lecture Notes 424


CHAPTER 10. MATRIX





2 1 1 3





1 0 −1 1 1

= −
2


0 0 2 1





0 3 1 2

(two column interchanges)






2 1 1 3




1 0 −1 1 1
= −
2


0 0 2 1




0 0 4 5

(R4 ← 3R2 + R4 )



2 1 1 3




1 0 −1 1 1


= − = 6 (why?)
2

0 0 2 1




0 0 0 3

Example 10.6.3 We can combine row-reduction and cofactor expansion to calculate

H.W-Kayondo & D.W-Ddumba, Engineering Math I- Lecture Notes 425


CHAPTER 10. MATRIX

determinants of large matrices.




2 1 3 1



0 1 1 1 3 1



1 0 1 1


= 2 −

2 10 2 1 0



0 2 1 0



1 2 3 1 2 3




0 1 2 3

expanding along column 1





0 1 1 1 0 1



Now 2 2 1 0 = 2 0 2 1 (2 column interchange)





1 2 3 3 1 2

 
 
 
 2 1 0 1 

 

= 2
+ 3



 
 1 2 2 1 

 

expanding along column 1


= 2(3 − 6) = −6.



1 3 1 1 3 1



Also 2 1 0 = 0 −5 −2 by row reduction





1 2 3 0 −1 2

H.W-Kayondo & D.W-Ddumba, Engineering Math I- Lecture Notes 426


CHAPTER 10. MATRIX




−5 −2

= expanding along column 1


−1 2

= −12
Therefore the required determinant = −6 + 12 = 6.
 
 1 0 −2 
 
 
 
Example 10.6.4 Given that A =  3 1 , find the adj(A).
 
 4 

 
 
 
5 2 −3

Solution
Since cofactors of A are



1 4

1+1

A11 = (−1) = −11


2 −3




3 4

A12 = (−1)1+2 = 29


5 −3




3 1

A13 = (−1)1+3 =1


5 2




0 −2

A21 = (−1)2+1 = −4


2 −3

H.W-Kayondo & D.W-Ddumba, Engineering Math I- Lecture Notes 427


CHAPTER 10. MATRIX




1 −2

A22 = (−1)2+2 =7


5 −3




1 0

A23 = (−1)2+3 = −2


5 2




0 −2

A31 = (−1)3+1 =2


4 1




1 −2

A32 = (−1)3+2 = −10


3 4




1 0

A33 = (−1)3+3 =1


3 1

The Matrix of cofactors is


 
 −11 29 1 
 
 
 
 
 −4 7 −2 
 
 
 
 
2 −10 1

H.W-Kayondo & D.W-Ddumba, Engineering Math I- Lecture Notes 428


CHAPTER 10. MATRIX

and thus the adjoint of A is


 
 −11 −4 2 
 
 
 
adj(A) =  29
 
 7 −10 

 
 
 
1 −2 1

The Matrix adjoint is useful in finding the inverse of a non-singular matrix.


Classwork 10.2 Given the matrix
 
 4 3 2 
 
 
 
A= 1 3 1 
 
 
 
 
 
2 0 2

Using the Permutation-inversion technique, and the cofactor minor technique.


The cofactors of A are



3 1

A11 = (−1)2 =6


0 2




1 1

A12 = (−1)3 =0


2 2




1 3

A13 = (−1)4 = −6


2 0

H.W-Kayondo & D.W-Ddumba, Engineering Math I- Lecture Notes 429


CHAPTER 10. MATRIX




3 2

A21 = (−1)3 = −6


0 2




4 2

A22 = (−1)4 =4


2 2




4 3

A23 = (−1)5 =6


2 0




3 2

A31 = (−1)4 = −3


3 1




4 2

A32 = (−1)5 = −2


1 1




4 3

6

A33 = (−1) =9


1 3

H.W-Kayondo & D.W-Ddumba, Engineering Math I- Lecture Notes 430


CHAPTER 10. MATRIX

The Matrix of cofactors is


 
 6
 0 −6 

 
 
 
 −6 4 6 
 
 
 
 
−3 −2 9

|A| = 4(6) + 3(0) + 2(−6) = 12

Classwork 10.3 Using properties of determinants, state the determinants of the follow-
ing matrices

 
 2 1 6 
 
(a) A =  : |A| Does not exist (DNE)
 
 
7 4 2

 
 2 0 0 
 
 
 
(b) A =  0 4 0 : |A| = 24
 
 
 
 
 
0 0 3

 
 9 8 6 
 
 
 
(c) A =  7 4 0 : |A| = 0
 
 
 
 
 
9 8 6

H.W-Kayondo & D.W-Ddumba, Engineering Math I- Lecture Notes 431


CHAPTER 10. MATRIX

10.7 Matrix Inverses


Definition 10.7.1 Inverse of a matrix
The matrix B is said to be the inverse of matrix A if AB = BA = I the identity matrix.
We denote the inverse of A by A−1 .
Note 10.7.1 If matrix A has an inverse we say that A is invertible .

The inverse of a matrix could be got by


(a) Direct method
(b) Adjoint method
(c) Method of elementary row operation

10.7.1 Direct method


   

 1 2 

 a b 
 
−1
For example A = 

 , If A = B = 
 
.

   
−1 1 c d
    

 1 2 
 a b
   1 0 
  
But AB = I, ⇒ 



=
 


    
−1 1 c d 0 1

Solving the system


 
1

 3
− 23 

1 −1
a= 3
,b = − 23 , c = 1
3
,d = 1
3
⇒ B=A =



 1 1

3 3

10.7.2 Adjoint method


Theorem 10.7.1 adj(A).A = det(A)I I the identity matrix .
adjA
Corollary 10.7.1 If |A| =
6 0 then A−1 = |A|

adj(A)
Proof : Since adj(A).A = det(A)I, dividing through by |A|, |A|
A =I .

H.W-Kayondo & D.W-Ddumba, Engineering Math I- Lecture Notes 432


CHAPTER 10. MATRIX

adj(A)
6 0 ⇒ A−1 exists .Multiplying through by A−1 we get
since |A| = |A|
AA−1 = IA−1
adj(A) adjA
⇒ |A|
= IA−1 = A−1 ⇒ A−1 = |A|
for |A| =
6 0

Example 10.7.1 Find the inverse of

 

 3 2 1 

 
 
A=
 
 0 1 1 

 
 
 
−1 1 0

1 1
Finding the co-factors A11 = (−1) 1+1
| | = −1
1 0

0 1
A12 = (−1)1+2 | | = −1
−1 0

0 1
A13 = (−1)1+3 | |=1
−1 1

2 1
A21 = (−1)2+1 | | = −1
1 0

3 1
A22 = (−1)2+2 | |=1
−1 0

H.W-Kayondo & D.W-Ddumba, Engineering Math I- Lecture Notes 433


CHAPTER 10. MATRIX

3 2
A23 = (−1)2+3
| |=5
−1 1

2 1
A31 = (−1)3+1
| |=1
1 1

3 2
A32 = (−1)3+2
| | = −3
0 1

3 1
A33 = (−1)3+3
| |=3
0 1
   
 −1 −1 1   −1 −1 1 
   
   
   
The cofactor matrix is  −1 ⇒ adj(A) =  −1
   
 1 5 
  1 −3 

   
   
   
1 −3 3 1 5 3

Then |A| = (3)(−1) + (0)(−1) + (−1)(1) = −4


But
   
 −1 −1 1   1 1
− 14 
   4 4 
   
adjA 1 
   
A−1 = = =
  
 −1 1 1 3
|A| −4  1 −3   4
− 4

4 
   
   
   1 5 3

1 5 3 −4 −4 −4

Theorem 10.7.2 An n × n matrix is non singular if and only if |A| =


6 0.

H.W-Kayondo & D.W-Ddumba, Engineering Math I- Lecture Notes 434


CHAPTER 10. MATRIX

Proof : Suppose that A is non singular, then A−1 exists such that
AA−1 = A−1 A = I
But |AA−1 | = |A−1 ||A| = |I| = 1
then |A| =
6 0.

Example 10.7.2 Find the inverse of

    
 4 −2 2    0 8 8 
    
    
  
−1 1  
A= 8 ; |A| = 96 , A =
    
 2 −6   96  −72 16 40
 

    
    
    
4 −2 6 −24 0 24

10.7.3 Method of elementary row operation ; Gauss-Jordan elim-


ination method
An elementary row operation on A = (aij ) is anyone of the following :

(a) Interchanging any two rows of a matrix .

(b) Multiplying any row of A by a non zero constant .

(c) Replacing any row by a linear combination of the row itself and any other row
of A :(Linear combination of rows involve summing and difference of rows But not
their product or quotient).

Process of computing the inverse .


For an n × n matrix

(i) Form the n × 2n matrix.i.e (A : I)

(ii) Apply elementary row operations to (A : I).

Apply operations to a full (all rows of a) column first before considering the next
column

(iii) Reduce (A : I) to a matrix of the form (I : B), then B will be the inverse matrix A.

H.W-Kayondo & D.W-Ddumba, Engineering Math I- Lecture Notes 435


CHAPTER 10. MATRIX

 

 3 2 1 

 
 
Example 10.7.3 Compute A−1 given A = 
 
 0 1 1 

 
 
 
−1 1 0

 

 3 2 1 1 0 0 

 
 
(A : I) = 
 
 0 1 1 0 1 0 

 
 
 
−1 1 0 0 0 1

Now the aim is to shift the identity matrix to side of A.


•We begin with first column (whole) and we need a21 = 0, a31 = 0
[Operations for first column we only use R1 ] .
R1 → R1 , R2 → R2 , R1 + 3R3 → R3
 
 3 2 1 1 0 0 
 
 
 
(A : I) =  0 1 1 0 1 0 
 
 
 
 
 
0 5 1 1 0 3

•In second column, have to make a12 = 0, a32 = 0


[But when looking for operations for second column,only use R2 ]
Let R1 − 2R2 → R1 , R2 → R2 , 5R2 − R3 → R3

 
 3 0 −1 1 −2 0 
 
 
 
(A : I) =  0 1
 
 1 0 1 0 

 
 
 
0 0 4 −1 5 −3

H.W-Kayondo & D.W-Ddumba, Engineering Math I- Lecture Notes 436


CHAPTER 10. MATRIX

•In third column, have to make a13 = 0, a23 = 0


[But when looking for operations for third column, only use R3 ]
Let 4R1 + R3 → R1 , 4R2 − R3 → R2 , R3 → R3

 
 12 0 0
 3 −3 −3 

 
 
(A : I) =  0 4 0
 
 1 −1 3 

 
 
 
0 0 4 −1 5 −3

1 1 1
• To have (I, B) by R
12 1
→ R1 , R
4 2
→ R2 , R
4 3
→ R3 to have

 
3 3 3
 1 0 0
 12
− 12 − 12 

 
 
(I : B) =  0 1 0
 
1
 4
− 14 3
4


 
 
 
0 0 1 − 14 5
4
− 34

   
3 3 3   1 1 1 

 12
− 12 − 12   4 −4 −4 
   
   
⇒ B = A−1 = 3  = 
   
1 1 1 1 3 
 4
−4 4   4 − 4 4 
   
   
   
− 14 5
4
− 3
4
− 1
4
5
4
− 3
4

Always prove your self by getting AA−1 = I

H.W-Kayondo & D.W-Ddumba, Engineering Math I- Lecture Notes 437


CHAPTER 10. MATRIX

 
 2 2 2 
 
 
 
Example 10.7.4 Compute A−1 given A =  0 2 0 
 
 
 
 
 
0 2 2

 
 2 2 2 1 0 0 
 
 
 
(A : I) =  0 2 0 0 1 0 
 
 
 
 
 
0 2 2 0 0 1

•The first column looks okay since the non-zero term is only in a11
•In second column, have to make a12 = 0, a32 = 0
[But when looking for operations for second column,only use R2 ]
Let R1 − R2 → R1 , R2 → R2 , R3 − R2 → R3
 
 2 0 2 1 −1 0 
 
 
 
(A : I) =  0 2 0 0
 
 1 0 

 
 
 
0 0 2 0 −1 1

•In third column, have to make a13 = 0, a23 = 0


[But when looking for operations for third column,only use R3 ]
Let R1 − R3 → R1 , R2 → R2 , R3 → R3
 
 2 0 0 1
 0 −1 

 
 
(A : I) =  0 2 0 0
 
 1 0 

 
 
 
0 0 2 0 −1 1

H.W-Kayondo & D.W-Ddumba, Engineering Math I- Lecture Notes 438


CHAPTER 10. MATRIX

1 1 1
• To have (I, B) we use R
2 1
→ R1 , R
2 2
→ R2 , R
2 3
→ R3 to have

 
 1 0 0 1 0 − 12 
 2 
 
 
(I : B) =  0 1 0 0
 
1
 2
0 
 
 
 1 1

0 0 1 0 −2 2

 
 1 0 − 12 
 2 
 
 
⇒ B = A−1 = 0
 
1
 2
0 

 
 
 1 1

0 −2 2

Make sure you prove your answer by AA−1 = I

 
 1 0 2 
 
 
 
Example 10.7.5 Compute A−1 given A =  2 −1 3 
 
 
 
 
 
4 1 8

 
 1 0 2 1 0 0 
 
 
 
(A : I) =  2 −1 3 0 1 0 
 
 
 
 
 
4 1 8 0 0 1

H.W-Kayondo & D.W-Ddumba, Engineering Math I- Lecture Notes 439


CHAPTER 10. MATRIX

•The first column : R1 → R1 , R2 − 2R1 → R2 , R3 − 4R1 → R3


 
 1 0 2 1 0 0 
 
 
 
(A : I) =  0 −1 −1 −2 1 0 
 
 
 
 
 
0 1 0 −4 0 1

•In second column, have to make a12 = 0, a32 = 0


Let R1 → R1 since a12 = 0, R2 → R2 since a22 6= 0, R3 + R2 → R3
 
 1 0 2 1 0 0 
 
 
 
(A : I) =  0 −1 −1 −2 1 0 
 
 
 
 
 
0 0 −1 −6 1 1

•In third column, have to make a13 = 0, a23 = 0[Always to use R3 and the row itself]
2R3 + R1 → R1 , R3 − R2 → R2 , R3 → R3 thus
 
 1 0
 0 −11 2 2 

 
 
(A : I) =  0 1
 
 0 −4 0 1 

 
 
 
0 0 −1 −6 1 1

• Having (I, B) by R1 → R1 , R2 → R2 , (−1)R3 → R3 to have


 
 1 0 0 −11 2 2 
 
 
 
(I : B) =  0 1 0 −4
 
 0 1 

 
 
 
0 0 1 6 −1 −1

H.W-Kayondo & D.W-Ddumba, Engineering Math I- Lecture Notes 440


CHAPTER 10. MATRIX

 
 −11 2 2 
 
 
 
⇒ B = A−1 =  −4
 
 0 1 

 
 
 
6 −1 −1

    
 4 −2 2    0 8 8 
    
    
    
 −1 1 
Example 10.7.6 Find the inverse of A =  8 ; A =
  
 2 −6 
   −72 16 40
96  

    
    
    
4 −2 6 −24 0 24

    
1 1

 3 2 1 




 2
−1 2


    
    
 −1 
Example 10.7.7 Find the inverse of A =  A = 
  
1 3
 0 1 1 
   2
1 2


    
    
    
−1 1 0 − 12 2 − 32

10.7.4 Properties of inverses


(i) (A−1 )−1 = A
(ii) (AB)−1 = B −1 A−1

Proof :

Let x = (AB)−1 then (AB)x = A(Bx)


But also (AB)x = (AB)(AB)−1 = I
Then A(Bx) = I multiplying both sides by A−1
A−1 A(Bx) = A−1 I
I(Bx) = A−1
Bx = A−1 multiplying both sides by B −1
(B −1 B) = B −1 A−1
Ix = B −1 A−1
x = B −1 A−1

H.W-Kayondo & D.W-Ddumba, Engineering Math I- Lecture Notes 441


CHAPTER 10. MATRIX

(iii) (AT )−1 = (A−1 )T

(iv) The inverse of a diagonal matrix is obtained by inverting the diagonal elements .

Example 10.7.8 If

   
 2 0 0   1 0 0 
   2 
   
   
A= 0 3  ⇒ A−1 = 0 1
   
 0   3
0 

   
   
   
0 0 −4 0 0 − 14

H.W-Kayondo & D.W-Ddumba, Engineering Math I- Lecture Notes 442


CHAPTER 10. MATRIX

Exercise 10.6 For the matrix

 
 1 1 2 
 
 
 
A =  3 5 −1 
 
 
 
 
 
2 3 0

Compute

(a) |A|, using

(i) Permutation-inversions technique


(ii) Cofactors.
|A| = −1

(b) A−1 , using

(i) A−1 = 1
|A|
adj(A)

(ii) Row reduction.


 
 −3 −6 11 
 
 
 
A−1 = 2
 
 4 −7 

 
 
 
1 1 −2

(c) Hence or otherwise solve the system

x1 + x2 + 2x3 = −2
3x1 + 5x2 − x3 = 1
2x1 + 3x2 = 0

Using

(i) X = A−1 b

H.W-Kayondo & D.W-Ddumba, Engineering Math I- Lecture Notes 443


CHAPTER 10. MATRIX

(ii) Row reduction


(iii) Crammer’s rule

 

 0 

 
 
X=
 
 0 

 
 
 
−1

Exercise 10.7 For the matrix


 
 1 2 3 
 
 
 
A= 3 2 1 
 
 
 
 
 
2 0 2

Compute
(a) |A|, using
(i) Permutation-inversions scheme
(ii) Cofactors.
|A| = −16
(b) A−1 , using
(i) A−1 = 1
|A|
adj(A)

(ii) Row reduction.


 
 −1 1 1 
 4 4 4 
 
 
A−1 = 1
 
1 1 
 4 4
−2 
 
 
 1 
4
− 41 1
4

H.W-Kayondo & D.W-Ddumba, Engineering Math I- Lecture Notes 444


CHAPTER 10. MATRIX

Classwork 10.4 Find the inverse A−1 for the matrix

 
 4 3 2 
 
 
 
A= 1 3 1 
 
 
 
 
 
2 0 2

Using the Cofactor-Adjoint technique.

Recall from Classwork10.2, the cofactor matrix of A was given as

 
 6
 0 −6 

 
 
 
 −6 4 6 
 
 
 
 
−3 −2 9

H.W-Kayondo & D.W-Ddumba, Engineering Math I- Lecture Notes 445


CHAPTER 10. MATRIX

 

 6 −6 −3 

 
 
⇒ adj(A) = 
 
 0 4 −2 

 
 
 
−6 6 9

 

 6 −6 −3 

 
1 1  
⇒ A−1 = adj(A) =
 
|A| 12 
 0 4 −2 

 
 
 
−6 6 9

 
1

 2
− 21 − 14 

 
 
= 
 
1 1 
 0 3
− 6 
 
 
 1 1 3

−2 2 4

Note 10.7.2 Check your answer with the property

AA−1 = A−1 A = I

H.W-Kayondo & D.W-Ddumba, Engineering Math I- Lecture Notes 446


CHAPTER 10. MATRIX

Finding the inverse using row-reduction

 
 4 3 2 1 0 0 
  R1 → R1
 
 
 
 1 3 1 0 1 0  R1 − 4R2 → R2
 
 
 
 
2 0 2 0 0 1 R1 − 2R3 → R3

 
 4 3 2 1 0 0  3R1 + R2 → R1
 
 
 
 
 0 −9 −2 1 −4 0  R2 → R2
 
 
 
 
0 3 −2 1 0 −2 3R3 + R2 → R3

 
 12
 0 4 4 −4 0  2R1 + R3 → R1

 
 
 
 0 −9 −2 1 −4 0  4R2 − R3 → R2
 
 
 
 
0 0 −8 4 −4 −6 R3 → R3

 
1
 24
 0 0 12 −12 −6 
 R
24 1
→ R1
 
 
 
 0 −36 1
 0 0 −12 6 
 R
−36 2
→ R2
 
 
  1
0 0 −8 4 −4 −6 R
−8 3
→ R3

H.W-Kayondo & D.W-Ddumba, Engineering Math I- Lecture Notes 447


CHAPTER 10. MATRIX

   
1
 1 0 0
 2
− 12 − 14 



1
2
− 12 − 14 

   
   
⇒ A−1 =
   
1 1 1 1 
 0 1 0
 0 3
−6    0 3
−6 
   
   
 1 1 3
  1 1 3

0 0 1 −2 2 4
−2 2 4

Note 10.7.3 The value for an inverse is always the same for both methods.

H.W-Kayondo & D.W-Ddumba, Engineering Math I- Lecture Notes 448


Chapter 11

Linear System of Equations

The general form of a linear system is

a11 x11 + a12 x12 + a13 x13 + · · · + a1n x1n = b1


a21 x21 + a22 x22 + a23 x23 + · · · + a2n x2n = b2
............................................................. = ..
............................................................. = ..
am1 xm1 + am2 xm2 + am3 xm3 + · · · + amn xmn = bm

This could be written as


Ax = b
 
 a11 a12 · · · a1n 
 
 
 
 
 a21 a22 · · · a2n 
 
where A = 

 called the Coefficient matrix ,

 .. 

 . 

 
 
 
am1 am2 · · · amn

x = (x1 , x2 , ..., xn )T the unknown solution , b = (b1 , b2 , ..., bn )T the RH vector , where as

449
CHAPTER 11. LINEAR SYSTEM OF EQUATIONS

 
 a11 a12 · · · a1n b1 
 
 
 
 
 a21 a22 · · · a2n b2 
 
Ā = (A : b) = 

 is called the augmented matrix of the

 .. 

 . 

 
 
 
am1 am2 · · · amn bm

system .

There are two ways of solving a system of linearequations

(i) Row reduction to Echelon form

(ii) Crammer’s rule.

11.1 Row reduction to Echelon form


A matrix A = (aij ) is said to be of row Echelon form if it satisfies the following conditions

(1) The non-zero entry in each row is a 1 .

(2) All rows of A whose entries are zero appear below the rows whose entries are not
all zeros .

(3) The first non-zero entry in any row appears in a column to the right of the first
non-zero entry in any preceding column .

Note : Given any m × n matrix we may change it to row Echelon form by applying
elementary row operations .
To solve a linear system Ax = b we ,

(i) Form the augmented matrix Ā = (A : b) .

(ii) Reduce Ā = (A : b) to row Echelon form matrix .

(iii) Use back substitution to get the solution .

11.2 Existence of a solution to a linear system


For a linear system Ax = b will either have

(i) No solution .

H.W-Kayondo & D.W-Ddumba, Engineering Math I- Lecture Notes 450


CHAPTER 11. LINEAR SYSTEM OF EQUATIONS

(ii) Unique solution (one solution) .

(iii) Infinitely many solutions .

Let our Ā be the row reduced form (Echelon) of the augmented matrix of the given
system. Let r be the number of non zero rows of the reduced augmented matrix and n
be the number of unknowns of the linear system .

11.2.1 No solution
When n = r and there is a row whose only non zero element is in the last column, then
the system has no solution. Indeed in the row reduced form, such a system is said to be
inconsistent .

Example 11.2.1 Solve the linear system

x1 − 3x2 + 5x3 = 3
x2 + 3x3 = 2
x2 + 3x3 = 5

   
 1 −3 5 3   1 −3 5 3 
   
   
   
⇒ Ā =  0  ⇒ Ā =  0
   
 1 3 2   1 3 2 

   
   
   
0 1 3 5 0 0 0 3

n = r = 3 But 0x3 = 1 ⇔ is algebraically undefined, thus the linear system is


inconsistent.

Classwork 11.1 Solve the linear system

x + 2y + z = 2
2x + 3y + 3z = 3
−3x − 4y − 5z = −5

H.W-Kayondo & D.W-Ddumba, Engineering Math I- Lecture Notes 451


CHAPTER 11. LINEAR SYSTEM OF EQUATIONS

     

 1 2 1
2 

 1 2
 1 2 

 1 2
 1 2 

     
     
⇒ (A, b) =  ⇒ (A, b) = ⇒ (A, b) =
     
2 3 3 3   0 1 −1 1   0 1 −1 1 
     
     
     
     
−3 −4 −5 −5 0 2 −2 1 0 0 0 1

thus inconsistent, no solutions

11.2.2 Unique solution


This occurs when n = r in the row reduced form of the augmented matrix Ā.

Classwork 11.2 Solve the linear system

x1 + x2 + x3 = 6
3x1 − 2x2 + 3x3 = 8
−2x1 + 4x2 − 3x3 = −3

 



 1 1 1 6 

 
 
⇒ Ā =   By R1 → R1 , 3R1 − R2 → R2 , 2R1 + R3 → R3
 
 3 −2 3 8 
 
 
 
−2 4 −3 −3

 


 1 1 1 6 
 
 
 
⇒ Ā =  0 5 ; By R1 → R1 , R2 → R2 , 6R2 − 5R3 → R3
 
 0 10 

 
 
 
0 6 −1 9

H.W-Kayondo & D.W-Ddumba, Engineering Math I- Lecture Notes 452


CHAPTER 11. LINEAR SYSTEM OF EQUATIONS

 



 1 1 1 6 
 
 
⇒ Ā = 
 
 0 5 0 10 

 
 
 
0 0 5 15

Thus x3 = 3, x2 = 2, x1 + x2 + x3 = 6 ⇒ (x1 , x2 , x3 ) = (1, 2, 3)

 



 1 0 0 1 

 
 
Alternatively can reduce by Gauss Jordan elimination to have Ā = 
 
 0 1 0 2 


 
 
 
0 0 1 3

Thus x3 = 3, x2 = 2, x1 + x2 + x3 = 6 ⇒ (x1 , x2 , x3 ) = (1, 2, 3)


Example 11.2.2 Solve the linear system

x1 + x2 + x 3 = 6
3x1 − 2x2 + 3x3 = 8
−2x1 + 4x2 − 3x3 = −3

   

 1 1 1
6 

 1 1 1 6 
 
   
   
⇒ Ā =  ⇒ Ā =  0 1 0 2 
   
 3 −2 3 8 
  
   
   
   
−2 4 −3 −3 0 0 1 3

n = r = 3 Thus x3 = 3, x2 = 2, x1 + x2 + x3 = 6 ⇒ (x1 , x2 , x3 ) = (1, 2, 3) a unique


solution .

11.2.3 Infinitely many solutions


If in the row reduced form of Ā, we have n > r, then the system has more than one
solution. i.e it has infinitely many solutions. Such a linear system is consistent.

H.W-Kayondo & D.W-Ddumba, Engineering Math I- Lecture Notes 453


CHAPTER 11. LINEAR SYSTEM OF EQUATIONS

Example 11.2.3 Solve the linear system


x1 − x2 + x3 = 1
4x1 − 2x2 + 8x3 = 1
3x1 − x2 + 7x3 = 0
6x1 − 4x2 + 10x3 = 3

 
 1 −1 1 1  R1 → R1
 
 
 
 
 4 −2 8 1  4R1 − R2 → R2
 
⇒ Ā = 


 ⇒
 
 3 −1 7 0  3R1 − R3 → R3
 
 
 
 
6 −4 10 3 6R1 − R4 → R4
   
 1 −1 1 1  R1 → R1  1 −1 1 1 
   
   
   
   
 0 −2 −4 3  R2 → R2  0 −2 −4 3 
   
Ā = 


 ⇒ Ā = 



   
 0 −2 −4 3  R2 − R3 → R3  0 0 0 0 
   
   
   
   
0 −2 −4 3 R2 − R4 → R4 0 0 0 0

Thus n = 3, r = 2 and x1 − x2 + x3 = 1, −2x2 − 4x3 = 3 ,


So let x3 = p, ⇒ − 2x2 − 4p = 3 ⇒ x2 = 3+4p −2
,

Substituting in x1 − x2 + x3 = 1 ⇒ x1 = − 12 − 3p.
       
 x1   − 1 − 3p   −1   −3 
   2   2   
       
       
 x2  =  − 3+4p  =  − 3  + p  − 4  Since p can take on many different
       
   2   2   2 
       
       
       
x3 p 0 1

values , then the system has infinitely many solutions .

H.W-Kayondo & D.W-Ddumba, Engineering Math I- Lecture Notes 454


CHAPTER 11. LINEAR SYSTEM OF EQUATIONS

Example 11.2.4 Solve the linear system


x1 − 3x2 + 5x4 = 4
−x1 + 3x2 + x3 − 3x4 = −11
−x1 + 3x2 − 5x3 + x5 = −3
3x1 − 9x2 + 15x4 = 12

   
 1 −3 0 5 0 4   1 −3 0 5 0 4 
   
   
   
   
 −1 3 1 −3 0 −11   0 0 1 2 0 −7 
   
⇒ Ā = 
 
 ⇒ Ā = 



   
 −1 3 0 −5 1 −3   0 0 0 0 1 1 
   
   
   
   
3 −9 0 15 0 12 0 0 0 0 0 0

Thus x5 = 1, x3 + 2x4 = −7, x1 − 3x2 + 5x4 = 4, You will realize that the number of
unknowns (n) exceeds the number of equations (r) after row reducing .

So let x4 = s, x2 = p ⇒ x3 = −7 − 2s, x1 = 4 + 3p − 5s
To get a particular equation, we assign values to s & p .
You could have realized that s & p could take on many values, thus giving us many
answers.
Note 11.2.1 So there are many types of Equation,Some have infinitely many solu-
tions,others one (unique solution), yet others do not have any solution .
Example 11.2.5 Solve
x1 + 3x2 + x3 = 4
2x1 + 2x2 + x3 = −1
4x1 + 8x2 + 3x3 = 2
The augmented matrix is
     
 1 3 1 4 
  1 3 1 4
   1 3 1 4 
   
     
     
 2 2 1 −1  ∼  0 −4 −1 −4  ∼  0 −4 −1 −9
     

     
     
     
     
4 8 3 2 0 −4 −1 −14 0 0 0 −5

H.W-Kayondo & D.W-Ddumba, Engineering Math I- Lecture Notes 455


CHAPTER 11. LINEAR SYSTEM OF EQUATIONS

The system is inconsistent. It has no solution

Example 11.2.6 Solve

2x1 + 2x2 + x3 − 4x4 = 1


2x1 + 3x2 − x4 = −1
x1 − 6x2 + 3x3 − 8x4 = 7

The augmented matrix is

   


 3 2 1 −4 1  2 1 −4 1
  1 3
 
 3 3 3
   
   
(A : b) =  2 ∼ 0 1
   
−2
 3 0 −1
−1   5
1 −1 

   
   
   
1 −6 3 −8
7 0 0 0 0 0

Observe that r(A) = 2 and 2 variables are free, i.e, are independent variables in the 1st
and 2nd equations.

Let x3 = s and x4 = t, s, t ∈ R, then

2
x2 = −1 − t + s
5
1 4 1 2
x1 = + t − s − x2
3 3 3 3
1 4 1 2 2 4
= + t− s+ + t− s
3 3 3 3 3 15
3
= 1 + 2t − s
5

So the general solution is

3
x1 = 1 + 2t − s
5
2
x2 = −1 − t + s s, t ∈ R
5

H.W-Kayondo & D.W-Ddumba, Engineering Math I- Lecture Notes 456


CHAPTER 11. LINEAR SYSTEM OF EQUATIONS

or in parametric vector form

       
 x1   1 −3
  



 2 


 5




=
 
 + t
 
 + s
 

 s, t ∈ R
       2

x2 −1 −1 5
,

Example 11.2.7 Find conditions that must satisfy for the system to be consistent

x1 − 2x2 + 5x3 = b1
4x1 − 5x2 + 8x3 = b2
−3x1 + 3x2 − 3x3 = b3

The augmented matrix is

   

 1 −2   1 −2
5 b1   5 b1 

   
   
∼
   

 4 −5 8 b2 
  0 3 −12 b2 − 4b1 

   
   
   
−3 3 −3 b3 0 −3 12 b3 + 3b1

   
 1 −2 5 b1   1 −2 5 b1 
   
   
   
∼ 0 = 0
   
 3 −12 b2 − 4b1   3 −12 b2 − 4b1 

   
   
   
0 0 0 b3 + 3b1 + b2 − 4b1 0 0 0 b3 + b2 − b 1

Thus the system is consistent only if b1 = b2 + b3 . In which case the solution is

1
x2 = (b2 − 4b1 + 12s), s ∈ R
3
2 8 −5 2
x1 = b1 − 5s + 2x2 = b1 − 5s + b2 − b1 + 8s = b1 + b2 + 3s
3 3 3 3

H.W-Kayondo & D.W-Ddumba, Engineering Math I- Lecture Notes 457


CHAPTER 11. LINEAR SYSTEM OF EQUATIONS

or equivalently

       
 x1   −5   2   3 
   3   3   
 =  b1 +   b2 +   s, s ∈ R
       
   −4
  1
  
x2 3 3
12

Go through Example 36, pages 56 of Anton

11.3 Crammer’s rule


To use Crammer’s rule introduced by Gabriel Crammer (1704 − 1754) for solving a linear
system Ax = b, for the matrix A,

(i) A has to be an n × n square matrix,

(ii) |A| =
6 0

Crammer’s rule states that:


1
xi = |Ai |
|A|

Where Ai is the matrix A whose ith column of the coefficient matrix has been replaced
by b .

Example 11.3.1 Solve the linear system using Crammer’s rule .

x1 + x2 + x3 = 1
2x1 + 3x2 + 4x3 = 1
x1 + 2x2 + x3 = 2

   
 1 1 1   1 
   
   
   
So A =  2 3 4  ; b= 1 
   
   
   
   
   
1 2 1 2

H.W-Kayondo & D.W-Ddumba, Engineering Math I- Lecture Notes 458


CHAPTER 11. LINEAR SYSTEM OF EQUATIONS

   
 1 1 1   1 1 1 
   
   
   
Then A1 =  1 3 4  ⇒ |A1 | = −2, and A2 =  2 1 4  ⇒ |A2 | = −2
   
   
   
   
   
2 2 1 1 2 1
 
 1 1 1 
 
 
 
1
A3 =  2 3 1  ⇒ |A3 | = 2 Then Using the formula xi = |Ai |
 
  |A|
 
 
 
1 2 2

1 −2
x1 = |A1 | = =1
|A| −2

1 −2
x2 = |A2 | = =1
|A| −2

1 2
x3 = |A3 | = = −1
|A| −2

Exercise 11.1 Use Crammer’s rule t solve the system .

2x1 + 5x2 − x3 = 1
x1 + 10x2 + x3 = 2
10x1 + x2 − 2x3 = 8

[(x1 , x2 , x3 ) = (1, 0, 1)]

11.4 Homogeneous systems of linear equations


For Ax = b, if b = 0 then the system Ax = 0 is said to be a homogeneous system.

11.4.1 Solution of a homogeneous system


For Ax = 0

H.W-Kayondo & D.W-Ddumba, Engineering Math I- Lecture Notes 459


CHAPTER 11. LINEAR SYSTEM OF EQUATIONS

(i) Form the augmented matrix Ā = (A : 0) .


¯
(ii) Reduce Ā = (A : 0) to row Echelon form matrix using elementary row operations.
¯
(iii) Use back substitution to get the solution .

11.4.2 Nature of Solution of a homogeneous system Ax = 0


¯
The homogeneous equation of the form Ax = 0
¯
(i) If n = r, after reducing, it always has a trivial solution.

(ii) n > r, before and after reducing the system, it has non trivial solutions .

(iii) If the coefficient matrix (A) is a square matrix, and |A| = 0, the system has non
trivial solutions .

Example 11.4.1 Solve the linear system

x1 + x2 + x3 = 0
2x1 + 3x2 + 2x3 = 0
−x1 + 4x2 + 3x3 = 0

   

 1 1 1 0 

 1
 1 1 0 

   
   
⇒ Ā =  ⇒ Ā =
   
 2 3 2 0 
 
 0 −1 0 0 

   
   
   
−1 4 3 0 0 0 4 0

n = r = 3 Thus 4x3 = 0, −x2 = 0, x1 + x2 + x3 = 0 ⇒ (x1 , x2 , x3 ) = (0, 0, 0) a unique


trivial (all zeros) solution .

Example 11.4.2 Solve the linear system

x1 + 2x2 + x3 = 0
3x1 + x2 + x3 = 0
2x1 + 2x2 − 3x3 = 0

H.W-Kayondo & D.W-Ddumba, Engineering Math I- Lecture Notes 460


CHAPTER 11. LINEAR SYSTEM OF EQUATIONS

   
 1 2 1 0   1 2 1 0 
   
   
   
n = r = 3 before and Ā =  3 1 ⇒ Ā =
   
 1 0 

 0 5
 2 0 

   
   
   
2 2 −3 0 0 0 −21 0

n = r = 3 after Thus a trivial solution (x1 , x2 , x3 ) = (0, 0, 0)


Example 11.4.3
Solve the linear system

x1 + x2 − 2x3 + x4 = 0
x1 + 2x2 + x3 + 3x4 = 0

 
 1 1 −2 1 0 
 
n = 4, r = 2 before, so already non trivial solution and Ā = 



 
0 1 3 2 0
 
 1 1 −2 1 0 
 
⇒ Ā = 



 
0 1 3 2 0

Thus n = 4, r = 2 after. To solve


So let x3 = p, x4 = s ⇒ x2 = −3p − 2s ,and x1 = 5p + 2s
   
 x1   5p + 2s 
   
   
   
   
 x2   −3p − 2s 
   
 =  Thus non trivial .
   
   
 x3   p 
   
   
   
   
x4 s

Exercise 11.2

H.W-Kayondo & D.W-Ddumba, Engineering Math I- Lecture Notes 461


CHAPTER 11. LINEAR SYSTEM OF EQUATIONS

Show that the system has a non trivial solution .

x1 − 4x2 + 2x3 = 0
x1 + x2 + 2x3 = 0
2x1 − 3x2 + 4x3 = 0

11.5 Sets of linear systems


Suppose we wish to solve Ax = b1 , Ax = b2 and Ax = b3 i.e sets of linear equations with
a common matrix coefficients.
Instead of solving each system we combine the calculations by row reducing the augmented
matrix
(A : b1 : b2 : b3 ).

Example 11.5.1

Solve the set of linear system

    
 1 3 5   x1   1 0 −1 
    
    
    
  x2  =  0 1 −1 
    
 −1 −2 0
    
    
    
    
2 5 4 x3 −1 1 0

H.W-Kayondo & D.W-Ddumba, Engineering Math I- Lecture Notes 462


CHAPTER 11. LINEAR SYSTEM OF EQUATIONS

Row reducing the augmented matrix


 
 1
 3 5 1 0 −1 

 
 
 
 −1 −2 0 0 1 −1 
 
 
 
 
2 5 4 −1 1 0

   
 1 3 5 1
 0 −1 
  1 3
 5 1 0 −1 

   
   
∼ 0 1 5 1 ∼ 0 1
   
 1 −2   5 1 1 −2 

   
   
   
0 1 6 3 −1 −2 0 0 −1 −2 2 0

For b1 , (x1 , x2 , x3 ) = (18, −9, 2)


For b2 , (x1 , x2 , x3 ) = (−23, 11, −2)
For b3 , (x1 , x2 , x3 ) = (5, −2, 0)
 
 1 0 0 
 
 
 
Example 11.5.2 Solve AX =  0 1 0 
 
 
 
 
 
0 0 1
 
 1 3 5 
 
 
 
where A =  −1 −2 0  as in the Example 11.5.1
 
 
 
 
 
2 5 4

Note that solving AX = I for invertible A gives X = A−1 which is the same as method
we have been using Example 37 page 57 of Anton to compute the inverse.

H.W-Kayondo & D.W-Ddumba, Engineering Math I- Lecture Notes 463


CHAPTER 11. LINEAR SYSTEM OF EQUATIONS

Exercise 11.3 Given the linear system below

x1 + 3x2 + 5x3 = α
−2x1 − x3 = 1
x1 + x2 + 2x3 = −1

(a) State the conditions on α for the system to have

(i) No solution : [<]


(ii) Infinitely many solutions : [−2]

(b) Can this system have a unique solution ?

Exercise 11.4 Describe the solution to the linear system

x1 + x2 − x3 = 1
2x1 + 2x2 − 2x3 = 2
3x1 + 2x2 + x3 = 5

H.W-Kayondo & D.W-Ddumba, Engineering Math I- Lecture Notes 464


Chapter 12

Eigen Values and Eigen Vectors

Definition 12.0.1
Given A(n × n) square matrix, the number λ is called an eigen value of A if ∃ a non zero
vector X such that
AX = λX . . . . . . . . . . . . (i)
X = eigen vector of this matrix
λ = eigen value
X is called an eigen vector of the matrix A associated with eigen value λ or (latent/characteristic
values)

Example 12.0.3 Let A = In , λ = 1 ∀ X ∈ Rn

 
 0 1 
 2 
Example 12.0.4 AX = λX A =   has two eigen values ± 1 with correspond-
  2
 1 
2
0

ing eigen vectors (1, 1) and (1, −1)

12.1 Finding eigen values & corresponding eigen vec-


tors.
Steps:
Given AX = λX, A(n × n)
⇒ we need to solve a homogeneous system by

Ax = λX
AX − λX = 0
(A − λIn )X = 0

465
CHAPTER 12. EIGEN VALUES AND EIGEN VECTORS

For a non trivial solution to the homogeneous system,


|A − λIn | = 0
or det(A − λIn ) ≡ P (λ)
P (λ) is called the characteristic polynomial of A. Equation P (λ) = 0 is called the char-
acteristic equation of A. Indeed
⇒ P (λ) = λn + a1 λn−1 λ + . . . + an
(A − λIn )X = 0 a1 , . . . an ∈ R
P (λ) = 0 has n roots (n ∈ R or n ∈ C) i.e the roots may be all real or all complex or
both.
Note 12.1.1
For complex roots, they occur in pairs i.e a complex root together with its conjugate.
     
 1 1   1 1   2 −1 
     
Example 12.1.1 Take A =  ,P =   , P −1 =  
     
     
−2 4 1 2 −1 1
 
 2 0 
−1
 
B=P AP = 

 is a diagonal matrix .

 
0 3

Try to get eigen values of A.


   


 1 1   1 0  1−λ 1
   
  − λ  = =0
   
   
−2 4 0 1 −2 4 − λ

Det = (1 − λ)(4 − λ) − (−2)(1) = 0


4 − λ − 4λ + λ2 + 2 = 0
λ2 − 5λ + 6 = 0 = P (λ)
(λ − 3)(λ − 2) = 0

Then λ1 = 2 and λ2 = 3. The eigen values are the values in the leading diagonal in the
diagonal matrix .
Then finding eigen vectors

H.W-Kayondo & D.W-Ddumba, Engineering Math I- Lecture Notes 466


CHAPTER 12. EIGEN VALUES AND EIGEN VECTORS

(a) for λ1 = 2

   

 1 1 

 1 0



=   − 2  X = 0
   
   
−2 4 0 1

    
 −1 1   x1   0 
    
  = 
    
    
−2 2 x2 0

   
 −1 1 0   −1 1 0 
   
 = 
   
   
−2 2 0 0 0 0

−x1 + x2 = 0 , let x2 = r then x1 = r


X = (x1 , x2 ) = (r, r) = r(1, 1)
if r = 1 X = (x1 , x2 ) = = (1, 1)

H.W-Kayondo & D.W-Ddumba, Engineering Math I- Lecture Notes 467


CHAPTER 12. EIGEN VALUES AND EIGEN VECTORS

(b) For λ2 = 3

   

 1 1 

 1 0



=   − 3  X = 0
   
   
−2 4 0 1

    
 −2 1   x1   0 
    
  = 
    
    
−2 1 x2 0

   
 −2 1 0   −2 1 0 
   
Augmented 







   
−2 1 0 0 0 0

s
−2x1 + x2 = 0 let x2 = s , x1 =
2
s 1
eigen Vectors (x1 , x2 ) = ( , s) = s( , 1)
2 2
Take s = 2 , X = (x1 , x1 ) = (1, 2)

 
 1
 2 −1 

 
 
Example 12.1.2 For A =  1 find the eigen values & their corresponding
 
 0 1 

 
 
 
4 −4 5

eigen vectors.

(i) 1st the characteristic polynomial.

|A − λIn | = 0
P (λ) = |A − λIn |
here P (λ) = |A − λI3 |

H.W-Kayondo & D.W-Ddumba, Engineering Math I- Lecture Notes 468


CHAPTER 12. EIGEN VALUES AND EIGEN VECTORS

   


 1
 2 −1 

 1 0 0 
 
   
   
=  1 − λ
   
 0 1 

 0 1 0 
 
   
   
   
4 −4 5 0 0 1




1−λ 2 −1





1 −λ 1





4 −4 5 − λ

|A| = (1 − λ)A11 + 2A12 + −1A13





−λ 1
A11 = (−1)2




−4 5 − λ




1 1
3

A12 = (−1)


4 5−λ




1 −λ
4

A13 = (−1)


4 −4




−λ 1 1 1 1 −λ

≡ (1 − λ)
− 2



=0


−4 5 − λ 4 5−λ 4 −4

H.W-Kayondo & D.W-Ddumba, Engineering Math I- Lecture Notes 469


CHAPTER 12. EIGEN VALUES AND EIGEN VECTORS

−(λ3 − 6λ2 + 11λ − 6) = 0 Characteristic equation of the system.


= (λ − 1)(λ − 2)(λ − 3) = 0
λ1 = 1, λ2 = 2, λ3 = 3

As the eigen values of A. for each eigen value we need to get the corresponding
eigen vector by solving the homogeneous equation (A − λi I3 )X = 0
For λ1 = 1,

       


 1
 2 −1 

 1 0 0   x1
     0 
  
       
       
− 1 = 0 
       
 1 0 1   0 1 0   x2
   
   
       
       
       
4 −4 5 0 0 1 x3 0

    
 0
 2 −1 
  x1   0 
   
    
    
= 0 
    
 1 −1 1   x 2
    
    
    
    
4 −4 4 x3 0

Solving the homogeneous equation X = (x1 , x2 , x3 ) = r( −1


2
, 1
2
, 1) general eigen
vectors with infinitely many solutions
Try it for λ2 = 2 , X = (x1 , x2 , x3 ) = (−2, 1, 4)
For λ3 = 3 , X = (x1 , x2 , x3 ) = (−1, 1, 4)

12.2 Similar Matrices


The matrix B is said to be similar to another matrix A if ∃ a non singular matrix P such
that B = P −1 AP
Thus given any matrix A, we can find a matrix B similar to A using B = P −1 AP .

How can we find P , or else, can we be able to find B without using the formula. See
example below

H.W-Kayondo & D.W-Ddumba, Engineering Math I- Lecture Notes 470


CHAPTER 12. EIGEN VALUES AND EIGEN VECTORS

     
 1 1   1 1   2 −1 
     
Classwork 12.1 Take A =  ,P =   , P −1 =  
     
     
−2 4 1 2 −1 1
 
 2 0 
 
B = P −1 AP = 

 a diagonal matrix .

 
0 3

Note 12.2.1 Try to get eigen values of A.


   


 1 1   1 0  1−λ 1
   
  − λ  = =0
   
   
−2 4 0 1 −2 4 − λ

Det = (1 − λ)(4 − λ) − (−2)(1) = 0


4 − λ − 4λ + λ2 + 2 = 0
λ2 − 5λ + 6 = 0 = P (λ)
(λ − 3)(λ − 2) = 0

Then λ1 = 2 and λ2 = 3. The eigen values are the values in the leading diagonal in the
diagonal matrix .
Then finding eigen vectors
(a) for λ1 = 2

   

 1 1 

 1 0



=   − 2  X = 0
   
   
−2 4 0 1

    
 −1 1   x1   0 
    
  = 
    
    
−2 2 x2 0

H.W-Kayondo & D.W-Ddumba, Engineering Math I- Lecture Notes 471


CHAPTER 12. EIGEN VALUES AND EIGEN VECTORS

   
 −1 1 0   −1 1 0 
   
 = 
   
   
−2 2 0 0 0 0

−x1 + x2 = 0 , let x2 = r then x1 = r


X = (x1 , x2 ) = (r, r) = r(1, 1)
if r = 1 X = (x1 , x2 ) = = (1, 1)

(b) For λ2 = 3

   

 1 1 

 1 0



=   − 3  X = 0
   
   
−2 4 0 1

    
 −2 1   x1   0 
    
  = 
    
    
−2 1 x2 0

   
 −2 1 0   −2 1 0 
   
Augmented 







   
−2 1 0 0 0 0

s
−2x1 + x2 = 0 let x2 = s , x1 =
2
s 1
eigen Vectors (x1 , x2 ) = ( , s) = s( , 1)
2 2
Take s = 2 , X = (x1 , x1 ) = (1, 2)

The eigen vectors form the columns of matrix P .

Note 12.2.2
You can get a similar matrix, even when not given P or can know P easily. (as 1st
asked the eigen vectors & values.

H.W-Kayondo & D.W-Ddumba, Engineering Math I- Lecture Notes 472


CHAPTER 12. EIGEN VALUES AND EIGEN VECTORS

12.2.1 Properties of similarity


(i) A is similar to A
(ii) If B is similar to A, then A is similar to B.
(iii) If A is similar to B and B is similar to C, then A is similar to C.

12.3 Diagonisable matrix


Note 12.3.1 We have realized, we can always get a similar matrix (easier to use- Diag-
onal matrices) for any matrix. But the problem is that, are the similar matrices always
diagonal? NO, some matrices can have similar matrices that are always diagonal (diago-
nisable matrices), some not.

Definition 12.3.1 Diagonalizable matrix


A matrix ”A” is diagonalizable, if
• it is similar to diagonal matrix. That is ”If there exists an invertible matrix P such
that P −1 AP is a diagonal matrix”

 

 1 1 

Example 12.3.1 A = 

 is diagonalizable, since its similar to the diagonal

 
−2 4
 
 2 0 
 
matrix 



 
0 3

Theorem 12.3.1
•The A(n × n) matrix is diagonalizable iff it has n linearly independent eigen vectors. In
this case, A is similar to diagonal matrix D with D = P −1 AP
where D is the matrix formed with eigen values of A while P is made up of the columns
of linearly independent eigen vectors of A.

Corollary 12.3.1
L : Rn → Rn defined by bx = Ax ∀ ∈ Rn . Then A is diagonalizable with n-linearly
independent vectors (eigen vectors) and the matrix of L with respect to S is diagonal.

Theorem 12.3.2
•A matrix A is diagonalizable if all its eigen values are real & distinct (different).
However, if eigen values of A are real but not distinct (polynomial with repeated roots).

H.W-Kayondo & D.W-Ddumba, Engineering Math I- Lecture Notes 473


CHAPTER 12. EIGEN VALUES AND EIGEN VECTORS

A may or may not be diagonalizable and


• if the eigen values of A are real but not distinct, A is diagonalizable
if and only if for each eigen value of a multiplicity k,
we can find k linearly independent non-zero eigen vectors.

Example 12.3.2 Refer to Example 12.1.1, is diagonisable since λ1 = 2, λ2 = 3 are real


and distinct.
 
 0 0 0 
 
 
 
Example 12.3.3 A =  0 1 0  is it diagonalizable ?
 
 
 
 
 
0 0 1

P (λ) = λ(λ − 1)2 = 0


λ1 = 0 λ2 = λ3 = 1 repeated thus Computing eigen vectors
    
 −λ 0 0   x1   0 
    
    
    
(A − λIn )X = 0 ⇒   x1  =  0 
    
 0 1−λ 0
    
    
    
    
0 0 1−λ x3 0

For λ2 = 1
    
 −1 0 0   x1   0 
    
    
    
 0 0 0   x1  =  0  x1 = 0 so x2 & x3 are variables.
    
    
    
    
    
0 0 0 x3 0

H.W-Kayondo & D.W-Ddumba, Engineering Math I- Lecture Notes 474


CHAPTER 12. EIGEN VALUES AND EIGEN VECTORS

Let x2 = r & x3 = s

       
 x1   0   0   0 
       
       
       
 x2  =  r  = r  1  + s 0 
       
       
       
       
       
x3 s 0 1

When λ1 = 0,
 
 −1 0 0 
 
 
 
 
 0 0 0 
 
 
 
 
1 0 0

X1 = (−r, 0, r) = r(−1, 0, 1)
Since 3 eigen vectors [(0, 1, 0), (0, 0, 1), (−1, 0, 1)] for 3 eigen values thus diagonalizable .

 
 3 −2 0 
 
 
 
Example 12.3.4 If A =  −2  is it diagonalizable ?
 
 3 0 
 
 
 
0 0 5

λ1 = 1 λ2 = λ3 = 5 repeated thus eigen vectors are

           
 x1   1   y1   −1   z1   0 
           
           
           
 x2  =  1  ,  y2  =  1  ,  z2  =  0 
           
           
           
           
           
x3 0 y3 0 z3 1

H.W-Kayondo & D.W-Ddumba, Engineering Math I- Lecture Notes 475


CHAPTER 12. EIGEN VALUES AND EIGEN VECTORS

Thus its diagonalizable since

(i) the eigen values and eigen vectors have the same multiplicity = 3 since real eigen
values and repeated or

   
 1 −1 0   5 0 0 
   
   
   
−1
(ii) we can find a matrix P =  1 such that P AP =  a
   
 1 0 


 0 5 0 
   
   
   
0 0 1 0 0 5

diagonal matrix.

 
 −3 2 
 
Example 12.3.5 If A = 

 is it diagonalizable ?

 
−2 1

λ1 = −1 λ2 = −1 real and repeated eigen values. Thus the eigen vectors are

 
 x1     
 
  t  1 
   
   
 x2  =   = t 
 
     
    
t 1

 
 
x3

Thus its not diagonalizable since

(i) the eigen values and eigen vectors do not have the same multiplicity 2 6= 1

(ii) we cant find a matrix P such that P −1 AP ) a diagonal matrix.

H.W-Kayondo & D.W-Ddumba, Engineering Math I- Lecture Notes 476


CHAPTER 12. EIGEN VALUES AND EIGEN VECTORS

 
 0 1 0 
 
 
 
Example 12.3.6 If A =  0 is it diagonalizable ?
 
 0 1 

 
 
 
4 −17 8
√ √
λ1 = 4 , λ2 = 2 + 3 , λ3 = 2 − 3 Since the eigen values are real and distinct, its
diagonalizable.

(i) Here we cant use the idea of multiplicity since no repeated eigen values.

 
 2 0 0 
 
 
(ii) But we can find a matrix P such that P AP =  0 2 + √3
 
−1
 a diagonal
 
 0 
 

 
 
0 0 2− 3

matrix. This method is sort of lengthy.

Note 12.3.2

1. A matrix A(n × n) may fail to be diagonalizable if

(i) Its eigen values are not all real


(ii) It does not have n linearly independent eigen vectors.
(iii) it does not have the same multiplicity of eigen values and vectors for real and
repeated eigen values.

2. If A is triangular, its eigen values are the elements on its main diagonal.

3. Let λ be a fixed real eigen value of A. The set S consisting of all eigen vectors
associated with λ together with O vector form a space of Rn called the eigen space
associated with λ.

Exercise 12.1

1. Show that the following matrices are not diagonalizable

H.W-Kayondo & D.W-Ddumba, Engineering Math I- Lecture Notes 477


CHAPTER 12. EIGEN VALUES AND EIGEN VECTORS

 
 2 −3 
 
(i) 



 
1 −1

 
 3 0 0 
 
 
 
(ii)  0 2 0 
 
 
 
 
 
0 1 2

 
 2 0 
 
(iii) 



 
1 2

2. Find which of the matrices are diagonalizable. For those which are find P , and D
such that P −1 AP = D (D is a diagonal matrix).
 
 1 1 
 
(i) 
 

 
1 −2

 
 1 1 −2 
 
(ii) 



 
4 0 4

 
 1 2 3 
 
 
 
(iii)  0 −1 −2 
 
 
 
 
 
0 0 2

H.W-Kayondo & D.W-Ddumba, Engineering Math I- Lecture Notes 478


CHAPTER 12. EIGEN VALUES AND EIGEN VECTORS

 
 3 1 0 
 
 
 
(iv)  0 3 1 
 
 
 
 
 
0 0 3

3. Find basis for eigen spaces associated with each eigen value of the following matrices.

 
 2 3 0 
 
 
 
(i)  0 1 0 
 
 
 
 
 
0 0 2

 
 2 2 3 4 
 
 
 
 
 0 2 3 2 
 
(ii) 



 
 0 0 1 0 
 
 
 
 
0 0 0 1

 
 3 −5 
 
4. Let A =   find A9 [Hint P such that P −1 AP = D) A9 = P D9 P −1 ]
 
 
1 −3

H.W-Kayondo & D.W-Ddumba, Engineering Math I- Lecture Notes 479


CHAPTER 12. EIGEN VALUES AND EIGEN VECTORS

     
 2 0 −2   3 0 0   −2 0 1 
     
     
     
5. Given A =  0 3 , Show that if B = and P =
     
 0 

 0 3 0 
 
 0 1 0



     
     
     
0 0 3 0 0 2 1 0 0

then B = P −1 AP
 
 5 0 0 
 
 
 
6. Show that A =  1 5 0  is not diagonalizable .
 
 
 
 
 
0 1 5

[λ1 = λ2 = λ3 = 5 and eigen vector of only one multiplicity i.e


     
 x1   0   0 
     
     
     
 x2  =  0  = s  0
     

     
     
     
     
x3 s 1

 
 1 1 2 
 
 
 
7. Given A =  0 1 0 
 
 
 
 
 
0 1 3

(i) Find the eigen values and their corresponding vectors .


[ λ1 = λ2 = 1, λ3 = 3.

H.W-Kayondo & D.W-Ddumba, Engineering Math I- Lecture Notes 480


CHAPTER 12. EIGEN VALUES AND EIGEN VECTORS

       
 x1   s   0   1 
       
       
       
= = r + s  0  for λ1 = λ2 = 1
       
 x2   −2r   −2 
       
       
       
       
x3 r 1 0
     
 x1   s   1 
     
     
     
and  x  =  0  = s  0  for λ3 = 3
     
 2     
     
     
     
x3 s 1

thus (0, −2, 1), (1, 0, 0), (1, 0, 1) are the eigen vectors]
(ii) Determine the diagonal matrix.
 
 1 0 0 
 
 
 
[D =  0 1 0 ]
 
 
 
 
 
0 0 3

(iii) Determine the invertible matrix P which diagonizes A .


 
 1 0 1 
 
 
 
[P =  0 −2 0 ]
 
 
 
 
 
0 1 1

(iv) Use the result to compute A2

H.W-Kayondo & D.W-Ddumba, Engineering Math I- Lecture Notes 481


CHAPTER 12. EIGEN VALUES AND EIGEN VECTORS

 
 1 4 8 
 
 
 
[Hint : Use the principle An = P Dn P −1 ⇒ A2 = P D2 P −1 =  0 1 0 ]
 
 
 
 
 
0 4 9

An = P Dn P −1 not An = P −1 Dn P
Its applied whenever to compute An , n > 2

 
 3 1 
 
Example 12.3.7 Given A = 



 
2 2

(i) Find the matrix B similar to A?


   
1

 1 1 


 3
− 31 

−1
Since P = 

, then P = 
 
 which imply that

   2 1

−2 1 3 3

 
 1 0 
−1
 
B=P AP = 

 Here realize that 1 and 4 are the eigen values of matrix

 
0 4

A.

(ii) Compute A4
     
1

 1 1 
 1
 0 
 3
− 13 
  171 85 
 
A4 = P B 4 P −1 =





=
 


   2 1
  
−2 1 0 256 3 3
170 86

H.W-Kayondo & D.W-Ddumba, Engineering Math I- Lecture Notes 482


CHAPTER 12. EIGEN VALUES AND EIGEN VECTORS

 
 4 2 
 
Example 12.3.8 Given A = 
 

 
3 −1

Find the matrix B similar to A?


   
 2 −1   3 1 
   7 7 
Since P =  , then P −1 =   which imply that
   
   1 2

1 3 −7 7

 
 5 0 
−1
 
B=P AP = 
 

 
0 −2

 

 2 −2 

Example 12.3.9 Given the matrix A = 

 , find an orthogonal matrix P

 
−2 5

such that P −1 AP is a diagonal matrix?


 
 √1 √2 
 5 5 
P =



 
− √25 √1
5

H.W-Kayondo & D.W-Ddumba, Engineering Math I- Lecture Notes 483


CHAPTER 12. EIGEN VALUES AND EIGEN VECTORS

Example 12.3.10 Which of the following matrices are diagonalizable?, In case of diag-
onalizable matrices, compute its eigenvalues and their corresponding eigenvectors.

 
 3 −2 0 
 
 
 
(i)  −2
 
 3 0 

 
 
 
0 0 5

(λ − 1)(λ − 5)2 = 0.

 
 5 0 0 
 
 
 
(ii)  1 5 0 ...Not diagonalizable.
 
 
 
 
 
0 1 5

H.W-Kayondo & D.W-Ddumba, Engineering Math I- Lecture Notes 484


Chapter 13

Complex Variable Algebra

13.1 Complex Numbers


Equations without real solutions, such as x2 = −1 or x2 − 10x + 40 = 0, were observed
early in history and led to the introduction of complex numbers. By definition, a complex
number z is an ordered pair (x, y) of real numbers x and y, written as;

z = (x, y)

x is called the real part and y the imaginary part of z, written as;

x = Rez, y = Imz

By definition, two complex numbers are equal if and only if their real parts are equal and
their imaginary parts are equal.
(0, 1) is called the imaginary unit and is denoted by i.

i = (0, 1) (13.1)

13.1.1 Addition and Multiplication


Notation z = x + iy
Addition of two complex numbers z1 = (x1 , y1 ) and z2 = (x2 , y2 ) is defined by as;

z1 + z2 = (x1 , y1 ) + (x2 , y2 ) = (x1 + x2 , y1 + y2 ) (13.2)

Multiplication is defined as;

z1 z2 = (x1 , y1 )(x2 , y2 ) = (x1 x2 − y1 y2 , x1 y2 + x2 y1 ) (13.3)

Set of complex numbers are defined by symbol C.


In practice, a complex number z = (x, y) is written as;

z = x + iy (13.4)

485
CHAPTER 13. COMPLEX VARIABLE ALGEBRA

If x = 0, then z = iy and is called pure imaginary. Also,

i2 = −1 (13.5)

For addition the standard notation gives

(x1 + iy1 ) + (x2 + iy2 ) = (x1 + x2 ) + i(y1 + y2 )

For multiplication it gives the following recipe.

(x1 + iy1 )(x2 + iy2 ) = x1 x2 + ix1 y2 + iy1 x2 + i2 y1 y2


(13.6)
= (x1 x2 − y1 y2 ) + i(x1 y2 + x2 y1 )

Example 13.1.1 a) Express (4 − 7i)(−2 + 3i) in the form x + iy.


b) Let z1 = 2+2i and z2 = 1+3i, find z1 +z2 and z1 z2 , giving your answer in the form x+iy.

Solution:
a) (4−7i)(−2+3i) = 4(−2)−(−7)3+i(4·3+(−2)(−7)) = −8+21+i(12+14) = 13+26i

b) z1 + z2 = 2 + 2i + 1 + 3i = 3 + 5i
z1 z2 = 2 · 1 − 2 · 3 + i(2 · 3 + 1 · 2) = −4 + 8i

13.1.2 Subtraction and Division


Subtraction and division are defined as the inverse operations of addition and multipli-
cation. Thus the difference z = z1 − z2 is the complex number z for which z1 = z + z2 .
Hence by (2)
z1 − z2 = (x1 − x2 ) + i(y1 − y2 ) (13.7)
The quotient z = z1 /z2 (z2 6= 0) is the complex number z for which z1 = zz2 . If we equate
the real and the imaginary parts on both sides of this equation, setting z = x + iy, we
obtain x1 = x2 x − y2 y, y1 = y2 x + x2 y. The solution is

z1
z= = x + iy (13.8)
z2

x1 x2 +y1 y2 x2 y1 −x1 y2
where x = x22 +y22
, y= x22 +y22
.

The practical rule used to get this is by multiplying numerator and denominator of
z1 /z2 by x2 − iy2 and simplifying.

H.W-Kayondo & D.W-Ddumba, Engineering Math I- Lecture Notes 486


CHAPTER 13. COMPLEX VARIABLE ALGEBRA

Example 13.1.2 For z1 = 4 + 3i and z2 = 2 − 5i, find a)z1 − z2 b) zz12


Solution:
a) z1 − z2 = 4 − 2 + i(3 − (−5)) = 2 + 8i

z1 4+3i (4+3i)(2+5i) 8+6i+20i−15 −7+26i −7 26


b) z2
= 2−5i
= (2−5i)(2+5i)
= 4+25
= 29
= 29
+ 29
i

13.1.3 Complex plane


The geometrical representation of complex numbers as points in the plane, has a great
practical importance. The idea is quite simple:
• x− axis is called the real axis
• y− axis is called the imaginary axis
• Both axis has the same unit length
This is called a Cartesian coordinate system. The xy-plane where the complex num-
bers are represented in this way is called the complex plane.

13.1.4 Complex Conjugate Numbers


The complex conjugate z̄ of a complex number z = x + iy is defined by
z̄ = x − iy (13.9)
It is obtained geometrically by reflecting the point z in the real axis as shown in the figure
below.

It is important because it permits us to switch from complex to real. Indeed, by


multiplication, z z̄ = x2 + y 2 . By addition and subtraction, z + z̄ = 2x, z − z̄ = 2iy.
We thus obtain for the real part x and the imaginary part y of z = x+iy the important
formulas.
1 1
Rez = x = (z + z̄), Imz = y = (z − z̄) (13.10)
2 2i
H.W-Kayondo & D.W-Ddumba, Engineering Math I- Lecture Notes 487
CHAPTER 13. COMPLEX VARIABLE ALGEBRA

If z is real, z = x, then z̄ = z by the definition of z̄, and conversely.


For conjugate numbers following rules also apply:

z1 + z2 = z1 + z2
z1 − z2 = z1 − z2
z1 z2 = z¯1 z¯2 (13.11)
 
z1 z1
=
z2 z2

z=z , |z| = |z|

Example 13.1.3 Let z1 = 8 + 3i, find Imz1 using conjugate.

Solution:

1 1 6i
Imz1 = (z1 − z1 ) = (8 + 3i − (8 − 3i)) = =3
2i 2i 2i

Example 13.1.4 Verify (11a) and (11c) for z1 = 8 + 3i, z2 = 9 − 2i.


a) Solution:

z1 + z2 = 8 + 3i + 9 − 2i = 17 + i = 17 − i
On the other hand:

z1 + z2 = 8 + 3i + 9 − 2i = 8 − 3i + 9 + 2i = 17 − i

b)
z1 z2 = (8 + 3i)(9 − 2i) = 8 · 9 − 3(−2) + i(8(−2) + 3 · 9)
= 72 + 6 + i(−16 + 27) = 78 + 11i = 78 − 11i
z¯1 z¯2 = (8 − 3i)(9 + 2i) = 8 · 9 − (−3)2 + i(8 · 2 − 3 · 9)
= 72 + 6 + i(16 − 27) = 78 − 11i

13.2 Polar Form of Complex Numbers, Powers and


Roots
We can substantially increase the usefulness of the complex plane and gain further insight
into the nature of complex numbers if besides the xy-coordinates we also employ the usual
polar coordinates r, θ defined by

x = r cos θ y = r sin θ (13.12)

H.W-Kayondo & D.W-Ddumba, Engineering Math I- Lecture Notes 488


CHAPTER 13. COMPLEX VARIABLE ALGEBRA

Then z = x + iy takes the so-called polar form

z = r(cos θ + i sin θ) (13.13)

r is called the absolute value or modulus of z and is denoted by |z| and defined as;

p
|z| = r = x2 + y 2 (13.14)

Geometrically, |z| is the distance of the point z from the origin. Similarly |z1 − z2 | is the
distance between z1 and z2 .
θ is called the argument of z and is denoted by argz. Thus

y
θ = argz = arc tan + nπ (13.15)
x

where n = 0 if x > 0 and n = 1 if x < 0.


Geometrically θ is the directed angle from the positive x-axis to OP. (where O denotes
the origin and P the point z = x + iy)
• All angles are measured in radians and positive in the counter clockwise sense.
• For z = 0, θ is undefined.
• For z 6= 0 θ is determined only up to integer multiples of 2π.
• The value of θ that lies in the interval −π < θ ≤ π is called the principle value
of the argument of z and is denoted by Argz. In otherwords
−π < Argz ≤ π.
Principle value is mainly used in connection with roots, the complex logarithm and certain
integrals.
Example 13.2.1 z = −4 + 4i. Represent z in polar form, find |z| and the principle value
of z.
Solution
4
θ = arctan −4 = 3π 4
and Argz = 3π
4
p √ √
|z| =√r = x2 + y 2 = 16 + 16 = 32
z = 32(cos 3π4
+ i sin 3π
4
)

13.2.1 Triangle inequality


For any complex numbers we have the important triangle inequality

|z1 + z2 | ≤ |z1 | + |z2 | (13.16)

H.W-Kayondo & D.W-Ddumba, Engineering Math I- Lecture Notes 489


CHAPTER 13. COMPLEX VARIABLE ALGEBRA

13.2.2 Generalized triangle inequality:


|z1 + z2 + ... + zn | ≤ |z1 | + |z2 | + ... + |zn | (13.17)
The absolute value of a sum cannot exceed the sum of the absolute values of the terms.

Example 13.2.2 Let z1 = −1 + i, z2 = 2 + 2i. Find |z1 + z2 | and |z1 | + |z2 | and verify
triangle inequality for z1 and z2 .

Solution: √ √
|z1 + z2 | = | − 1 + i + 2 + 2i| = |1 + p3i| = 1 + 3√= 10 ≈ 3.16
2 2
√ √
|z1 | + |z2 | = | − 1 + i| + |2 + 2i| = (−1)2 + 12 + 22 + 22 = 2 + 8 ≈ 4.24
√ √ √
10 < 2 + 8 ⇒ triangle inequality holds.
It directly follows that
|Rez| ≤ |z| |Imz| ≤ |z|

13.2.3 Multiplication and Division in Polar Form


This will give us ”geometrical” understanding of multiplication and division. Let

z1 = r1 (cos θ1 + i sin θ1 ) and z2 = r2 (cos θ2 + i sin θ2 )

Multiplication, the product is at first:

z1 z2 = r1 r2 ((cos θ1 cos θ2 − sin θ1 sin θ2 ) + i(sin θ1 cos θ2 + cos θ1 sin θ2 ))

The addition rules for the sine and cosine now yield

z1 z2 = r1 r2 (cos(θ1 + θ2 ) + i sin(θ1 + θ2 )) (13.18)

Taking absolute values and arguments on both sides of (13.18), we thus obtain the im-
portant rules
|z1 z2 | = |z1 ||z2 | (13.19)
arg(z1 z2 ) = argz1 + argz2 up to integer multiples of 2π (13.20)
Division
The quotient z = z1 /z2 is the number z satisfying zz2 = z1 . Hence |zz2 | = |z||z2 | = |z1 |,
arg(zz2 ) = argz + argz2 = argz1 . This yields

z1 |z1 |
| |= (13.21)
z2 |z2 |

and
z1
arg = argz1 − argz2 up to integer multiples of 2π (13.22)
z2

H.W-Kayondo & D.W-Ddumba, Engineering Math I- Lecture Notes 490


CHAPTER 13. COMPLEX VARIABLE ALGEBRA

By combining formulas (13.21) (13.22) we also have

z1 r1
= (cos(θ1 − θ2 ) + i sin(θ1 − θ2 )) (13.23)
z2 r2

Example 13.2.3 Let z1 = −1 + i, z2 = 2 + 2i. Find the polar representation of z1 and


z2 and calculate z1 z2 and zz12

Solution: √ √
|z1 | = r1 = 2, |z2 | = r2 = 8, θ1 = arctan xy = arctan −1
1


= 4
, θ2 = arctan 22 = π
4

√ √ 3π π 3π π
z1 z2 = 2 8(cos( + ) + i sin( + )) = 4(cos π + i sin π) = −4
4 4 4 4

|z1 | 2 1 3π π π
=√ = Arg(z1 /z2 ) = Argz1 − Argz2 = − =
|z2 | 8 2 4 4 2

z1 1 π π  i
= cos( ) + i sin( ) =
z2 2 2 2 2

13.2.4 Integer powers and De Moivre’s formula


From (13.19) and (13.20) with z1 = z2 = z we obtain by induction for n = 0, 1, 2, ...

z n = rn (cos nθ + i sin nθ) (13.24)

similarly, (13.23) with z1 = 1 and z2 = z n gives (13.24) for n = −1, −2, ....
For |z| = r = 1, formula (13.24) becomes De Moivre’s formula:

(cos θ + i sin θ)n = cos nθ + i sin nθ (13.25)

We can use De Moivre’s formula to express cos nθ and sin nθ in terms of powers of cos θ and
sin θ. For instance, for n = 2 we have on the left (cos θ + i sin θ)2 = cos2 θ + 2i cos θ sin θ −
sin2 θ. On the other hand according to De Moivre’s formula
cos2 θ + 2i cos θ sin θ − sin2 θ = cos 2θ + i sin 2θ
Taking the real and imaginary parts on both sides with n = 2 gives for real part:
cos 2θ = cos2 θ − sin2 θ and for imaginary part sin 2θ = 2 cos θ sin θ
As this example shows complex methods often also simplify the derivation of real formulas.

H.W-Kayondo & D.W-Ddumba, Engineering Math I- Lecture Notes 491


CHAPTER 13. COMPLEX VARIABLE ALGEBRA

Roots
If z = wn , n = 1, 2, ..., then to each value of w there corresponds one value of z. We shall
immediately see that, conversely, to a given z 6= 0 there corresponds precisely n distinct
values of w. Each of these values is called an nth root of z, and we write

w= nz (13.26)


n

n
θ + 2kπ θ + 2kπ
z= r(cos + i sin ) (13.27)
n n

where k = 0, 1, ..., n − 1. These n values lie on a circle of radius n r with center√at
the origin and constitute the vertices of a regular polygon of n sides. The value of n z
obtained
√ by taking the principle value of argz and k = 0 is called the principal value of
w = z.
n

In particular, taking z = 1, we have |z| = r = 1 and Argz = 0. Then (13.27) gives


n 2kπ 2kπ
1 = cos + i sin , (13.28)
n n

where k = 0, 1, ..., n − 1. These n values are called the nth roots of unity. They lie on
the circle of radius 1 and center 0, briefly called the unit circle.

Example 13.2.4 Find all the roots of 3 216

Solution:

√ √
 
3 3 2kπ 2kπ
216 = 216 cos + i sin
3 3


k = 0 ⇒ 6(cos 0 + i sin 0) = 6

√ √
−1 3
k = 1 ⇒ 6(cos 2π + i sin 2π
) = 6( + i ) = −3 + 3 3i
3 3 2 2√ √
) = 6( −1 + i −2 3 ) = −3 − 3 3i

k = 2 ⇒ 6(cos 4π + i sin 4π

3 3 2

Example 13.2.5 Solve the equation



z 4 + 8(1 − i 3) = 0

p
4

(In other words find all the roots of −8 + 8i 3)

H.W-Kayondo & D.W-Ddumba, Engineering Math I- Lecture Notes 492


CHAPTER 13. COMPLEX VARIABLE ALGEBRA

Solution:

For inside the root: (z1 )

√ √
 
2π 2π
q
z1 = −8 + 8i 3 = 8 (−1) + ( 3) cos
2 2 + i sin
3 3
 
2π 2π
= 16 cos + i sin
3 3

On the otherhand

√ √
q
θ + 2kπ θ + 2kπ 4
zk = n r(cos + i sin )= −8 + 8i 3
n n


 
4 2π π 2π π
= 16 cos( + k ) + i sin( +k )
3·4 2 3·4 2
 π π π π 
= 2 cos( + k ) + i sin( + k )
6 2 6 2

 
 π π  kπ kπ
= 2 cos( ) + i sin( ) cos( ) + i sin( )
6 6 2 2

Using De Moivres formula we get:

 π π  π π k
= 2 cos( ) + i sin( ) cos( ) + i sin( )
6 6 2 2
√ !
3 1
=2 +i ik , k = 0, 1, 2, 3
2 2

 √
z0 = 3+i




z
1 = −1 + i 3
⇒ √
z2 =− 3−i

 √
=1−i 3

z3

H.W-Kayondo & D.W-Ddumba, Engineering Math I- Lecture Notes 493


CHAPTER 13. COMPLEX VARIABLE ALGEBRA

13.3 The basic Transcendental Functions


13.3.1 Exponential Function
One of the most important analytic functions in complex analysis is the complex expo-
nential function ez also written sometimes expz.

The definition of ez in terms of the real functions, ex , cos y and sin y is

ez = ex (cos y + i sin y) (13.29)

wherez = x + iy
This definition is motivated by the requirements that make ez natural extension of the
real exponential function ex , namely

• ez should reduce to the latter when z = x is real;

• ez should be an entire function, that is, analytic for all z;

• similar to calculus, its derivative should be

(ez )0 = ez (13.30)

Further properties:
ez1 +z2 = ez1 ez2 (13.31)

1 ez1
e−z = , = ez1 −z2 and (ez )n = enz (13.32)
ez ez2

ez = ex eiy (13.33)
So called Euler formula:
eiy = cos y + i sin y (13.34)
From Euler formula we get:

(1◦ ) De Moivre formula


n
(cos y + i sin y)n = eiy = einy = cos ny + i sin ny, (13.35)

(2◦ ) and
z = x + iy = r(cos θ + i sin θ) = reiθ , (13.36)
p
where r = |z| = x2 + y 2 and θ = arg z = arctan(y/x).

H.W-Kayondo & D.W-Ddumba, Engineering Math I- Lecture Notes 494


CHAPTER 13. COMPLEX VARIABLE ALGEBRA

From this we get

ez = ex (cos y + i sin y) = ex cos y +i ex sin y ,


| {z } | {z }
u(x,y) v(x,y)

From which follows


(
|ez | = ex = eRez ,
arg ez = y = Imz.

Periodicity of ez with period 2πi,

ez+2πi = ez for all z (13.37)


Hence all the values that w = ez can assume are already in the horizontal strip of
width 2π

−π < y ≤ π (13.38)
This infinite strip is called a fundamental region of ez .

Example 13.3.1 Find ez in the form u + iv and |ez | if z = 2 + 3πi

Solution:

e2+3πi = e2 e3πi = e2 (cos 3π + i sin 3π) = −e2


|ez | = | − e2 | = e2 .

Example 13.3.2 Represent 1 + i in the exponential polar form.

Solution:

z = x + iy = r(cos θ + i sin θ) = reiθ


√ √
Now we have z = 1 + i, hence r = 12 + 12 = 2 and θ = arctan xy = arctan 11 = π
4
we have
√ iπ/4
z= 2e

Example 13.3.3 Find the solution of e3z = 3.

Solution:

ln3
|e3z | = e3x = 3 ⇒ 3x = ln3 ⇒ x =
3
H.W-Kayondo & D.W-Ddumba, Engineering Math I- Lecture Notes 495
CHAPTER 13. COMPLEX VARIABLE ALGEBRA

which is the real part of the solution.

e3z = e3x (cos 3y + i sin 3y) = 3


Separating real and imaginary parts we have that

e3x cos 3y = 3 e3x sin y = 0


⇒ cos 3y = 1 and sin 3y = 0 which holds when y = (0 ± 2πn)/3
So we get for z:
z = 31 (ln3 ± 2nπi)

13.3.2 Trigonometric Functions


Just as ez extends ex to complex, we want the complex trigonometric functions to extend
the familiar real trigonometric functions.
Series
∞ ∞
X z 2n X z 2n+1
(−1)n and (−1)n
n=0
(2n)! n=0
(2n + 1)!

are finite in all complex numbers z ∈ C and if z = x ∈ R, their sums are cos x and
sin x.

Definition 13.3.1 Let z ∈ C be any complex number. Then


X z 2n+1 z3 z5
sin z = (−1)n =z− + − +... (13.39)
n=0
(2n + 1)! 3! 5!


X z 2n z2 z4
cos z = (−1)n =1− + − +... (13.40)
n=0
(2n)! 2! 4!

By differentiating we notice that

D sin z = cos z and D cos z = − sin z. (13.41)

Similar manner as we got Euler formula, we get the result: When z ∈ C, is

eiz = cos z + i sin z

e−iz = cos(−z) + i sin(−z) = cos z − i sin z

H.W-Kayondo & D.W-Ddumba, Engineering Math I- Lecture Notes 496


CHAPTER 13. COMPLEX VARIABLE ALGEBRA

When we solve cos z and sin z from this equation pair, we get
( iz −iz
cos z = e +e 2
,
iz −iz z ∈ C.
sin z = e −e2i
,

Further more, as in calculus we define


Definition 13.3.2 Tangent and cotangent are defined

sin z 1
tan z = , cot z = . (13.42)
cos z tan z

Also for sec z and csc z


1 1
sec z = , csc z = . (13.43)
cos z sin z

tan z and sec z are analytic elsewhere except in point z = π/2 + nπ, where cos z = 0
(n ∈ Z). cot z and csc z are analytic elsewhere except in point z = nπ, where tan z = 0
(n ∈ Z).
Tangents and cotangents period is π.
Formulas for the derivatives follow readily from (ez )0 = ez as in calculus,
(cos z)0 = − sin z, (sin z)0 = cos z, (tan z)0 = sec2 z (13.44)
etc. Equations 13.39 and 13.40 also shows that Euler’s formula is valid in complex
eiz = cos z + i sin z (13.45)
for all z.
Real and imaginary parts of cos z and sin z are needed in computing values, and they
also help in displaying properties of our functions.

13.4 Real, imaginary parts and Absolute value.


sin z = sin(x + iy) = sin x cos iy + cos x sin iy

ei·iy + e−i·iy ei·iy − e−i·iy


= sin x + cos x
2 2i
ey + e−y ey − e−y
= sin x + i cos x
2 2

= sin x cosh y +i cos x sinh y (13.46)


| {z } | {z }
u(x,y) v(x,y)

H.W-Kayondo & D.W-Ddumba, Engineering Math I- Lecture Notes 497


CHAPTER 13. COMPLEX VARIABLE ALGEBRA

| sin z|2 = sin2 x cosh2 y + cos2 x sinh2 y

= sin2 x + sinh2 y = cosh2 y − cos2 x. (13.47)

From this follows


| sinh y| ≤ | sin z| ≤ | cosh y|.
When |y| → ∞, then | sinh y| → ∞, and we see that then | sin z| → ∞. If x ∈ R, then
| sin x| ≤ 1. Similarly for cosine function we get results that

cos z = cos x cosh y − i sin x sinh y (13.48)

and

| cos z|2 = cos2 x + sinh2 y (13.49)

From equations follows that 2π is periodicity of sine and cosine. We show that all periods
of sine ω are 2π:n multiples. Assume that ∀ z ∈ C, sin(z + ω) = sin z.
• We choose z = 0 =⇒ sin ω = 0,

• We choose z = π/2 =⇒ cos ω = 1.


From this follows cos ω + i sin ω = 1 =⇒ eiω = 1 = e0 =⇒ iω = n · 2πi =⇒
ω = n · 2π (n ∈ N). Same applies for cosine.
General formulas for the real trigonometric functions continue to hold for complex
values. We mention particular the addition rules

cos(z1 ± z2 ) = cos z1 cos z2 ∓ sin z1 sin z2

sin(z1 ± z2 ) = sin z1 cos z2 ± sin z2 cos z1 (13.50)


and the formula
cos2 z + sin2 z = 1 (13.51)
Example 13.4.1 Compute in the form u + iv
a)cos(π + iπ) b) cos 10i
Solution:

a)
cos(π + πi) = cos π cosh π − i sin π sinh π = − cosh π
b)
cos 10i = cosh 10 = 11013

Example 13.4.2 Solve the equation;cos z = 5

H.W-Kayondo & D.W-Ddumba, Engineering Math I- Lecture Notes 498


CHAPTER 13. COMPLEX VARIABLE ALGEBRA

Solution:

1 iz
e + e−iz = 5

cos z =
2

⇒ e2iz + 1 = 10eiz


iz 10 ± 2 100 − 4 √
e = = 5 ± 25 − 1 = 9.899 and 0.101
2

On the otherhand eiz = e−y+ix , thus e−y = 9.899 or 0.101 and eix = 1

⇒ y = ±2.292, x = 2nπ, z = ±2nπ ± 2.292i (n = 0, 1, 2, ...)

13.5 Hyperbolic Functions


The complex hyperbolic cosine and sine are defined by the formulas

1 1
cosh z = (ez + e−z ) sinh z = (ez − e−z ) (13.52)
2 2

These functions are entire, with derivatives

(cosh z)0 = sinh z (sinh z)0 = cosh z (13.53)

as in calculus. The other hyperbolic functions are defined by

sinh z cosh z
tanh z = coth z =
cosh z sinh z

1 1
sec hz = csc hz = (13.54)
cosh z sinh z

Complex trigonometric and hyperbolic functions are related.

cosh iz = cos z sinh iz = i sin z (13.55)

and conversely
cos iz = cosh z sin iz = i sinh z (13.56)

H.W-Kayondo & D.W-Ddumba, Engineering Math I- Lecture Notes 499


CHAPTER 13. COMPLEX VARIABLE ALGEBRA

Here we have another case of unrelated real functions that have related complex analogs,
pointing again to the advantage of working in complex in order to get both a more unified
formalism and a deeper understanding of special functions. This is one of the main reasons
for the importance of complex analysis.
For hyperbolic functions we also have

cosh z = cosh x cos y + i sinh x sin y (13.57)


sinh z = sinh x cos y + i cosh x sin y (13.58)

Example 13.5.1 Find all solutions of a) sin z = 1000 b) cosh z = 0 c) sinh(4 − 3i)

Solution:
a)
sin z = sin x cosh y + i cos x sinh y = 1000

⇒ sin x cosh y = 100 and cos x sinh y = 0

⇒ x = ±π/2 ± 2nπ cosh y = 1000


Now cosh y ≈ ey/2 when y is large, so ey ≈ 2000, y ≈ 7600902 (correct with six
desimals)
π
z= ± 2nπ ± 7600902i
2

b)
cosh z = cosh x cos y + i sinh x sin y = 0

⇒ cosh x cos y = 0 and sinh x sin y = 0


cosh x cos y = 0 holds when cos y = 0 ⇒ y = ±(2n + 1) π2

Now for sinh x sin y = 0 sin y 6= 0 for those y values hence sinh x = 0 which is true
when x = 0.
Answer: z = ±(2n + 1) π2 i, n = 0, 1, 2, ...
c)
sinh(4 − 3i) = sinh 4 cos(−3) + i cosh 4 sin(−3) = −27.017 − 3.854i

H.W-Kayondo & D.W-Ddumba, Engineering Math I- Lecture Notes 500


CHAPTER 13. COMPLEX VARIABLE ALGEBRA

Revision Questions

1. Write in form x + iy following;


3+6i
a) 2−3i
2+i
b) 3−4i + 2+i
5i
c) Im ((1 + 5i)3 )

2. Represent in polar form;


a) z = 3 + 6i

i 2
b)z = 4+4i

−1−i
√ )25 .
3. Find |z| and Arg(z) and write z in form x + iy, when z = ( 1−i 3


4. Find all values of −7 − 24i.

5. Solve the equation; z 2 + (2i − 3)z + 5 − i = 0

6. Solve equation; z 4 − 3(1 + 2i)z 2 − 8 + 6i = 0

7. Solve the equation; z 2 + z + 1 − i = 0

8. Solve the equation; cos z = 0

9. Let w = ez , compute |w|, Arg(w) and w when;


a) z = −1 + πi/6
b)z = −3πi

10. Find all the solutions of;


a) cos z = 3i
b) tan z = −1 + i

H.W-Kayondo & D.W-Ddumba, Engineering Math I- Lecture Notes 501

Das könnte Ihnen auch gefallen